You are on page 1of 236

CONTENTS

Section A . 1

Introduction to Behavioural Sciences 1


Holistic vs. Traditional Allopathic Medicine 2
Health Care Models and their Clinical Applications 3
1. Bio-Psycho-Social (BPS) model of health and disease 3
2. The Integrated ModeL of Health Care: Correlation of Body, Brain,
Mind, Spirit and Behavioural Sciences 5
3. The Public Health Care Model g

Non-pharmacologicaL Interventions (NPIs) in Clinical Practice 11


1. Communication Skills 11
2 Counselling 14

3. Informational Care (IC) 16


. Handling Difficult Patients and their Families i8
5. Breaking Bad News 20
6. Crisis Intervention and Disaster Management 27

7. Confticl. Resolution 29
Empathy 32

Sample MCQs and Essay Questions 33

Section B 35
Medical Ethics and Professionatism 36
Relevance of Ethics in the Life of a Doctor 37
;. Scope and Meaning of Medical Ethics 37
2. Guiding Principles of Medical Ethics 38
3. Common Ethical Issues in Medical Practice 39
4. Common Ethical Dilemmas in a Health Professonat’s Life 43
. Doctor-Patient Relationship 48

Rights and Responsibilities of Patients and Doctors 49


a. Rights of the Patient 49
b. Responsibilities of the Patients 50
c. Rights cf the Doctor 50
d. esponsibilities of the Doctor 50
Psychological Reactions in Doctor-Patient Relationship 52
a. Social bonding 52
b. Dependence 53
c. Transference 53
U. Counter-transference 54
e. Resistance 55
f, Unwell Physician / Burn-out 56

Professionalism in Heatth Care 57


- a. Knowledge 57
b.Skills 57
c. Attitudes 58

Sampte MCQs and Essay Questions 61

Section C 63
Psychotogy in Medicat Practice 63
a. Role of psychologicaL factors in the aetiology of health probLems 63
b. Role of psychological factors in the precipitation (triggering) of iltnesse 63
c. Role of psychological factors in the management of illnesses 64
U. Role of psychological and social factors in diseases causing disability.
handicap and stigma 64
e. Role of psychological factors in patients reactions to illness 64
f. Medicat[y Unexplained Physical Symptoms (MUPS) 64

Principles of Psychology 65
1. Learning 65
2. Metacognition 72

3. Memory 74
4. Perception 81
5. Thinking 85
6. Emotions 92

7. Motivation 94
8. Intelligence 97
9. Personality Development 101

NeurobiologicaL Basis of Behaviour 108


Emotion 109
Language 114
Memory . 116
ArousaL . 117
Sleep 118

Sample MCQs and Essay Questions 123

Section D 125
Socio[ogy and Anthropology 125
Introduction 125
1. Sociology and Health 127
2. Anthropology and Health 135

Sample MCQs and Essay Questions 141

Section E 143
Psychosociat Aspects of Health and Disease 143
Health and NormaLity 143
Defence Mechanisms 145
Psychosocial Assessment in Health Care 14$
ClinicaL Situations Demanding a Comprehensive PsychosociaL Assessmer 148
Psychological reactions to IlLness and Hospitalization 149

Psychosociat Assessment ... 153

Psychosocial Issues in SpeciaL Hospital Settings 157


a. Coronary Care Unit 157
b. Intensive Care Unit 158
c. The Emergency Department 159
d. Psychosocial Aspects of Organ Transplantation 159
e. The Dialysis Unit i6o
f. Reproductive Health 161
g. Paediatrics Ward 163
h. Oncology 167
i. Operating Theatre 168

PsychosociaL Peculiarities of Dentistry 170


PsychosociaL Aspects of Atternative Medicine 174

Common Psychiatric Disorders in General Health Settings 175


a. Mixed Anxiety and Depression in
b. Panic Disorder 179
c. Unexplained Somatic Complaints: Persistent Complainers 181
d. Dissociative and Possession States 182
e. Drug Abuse, ALcohol & Tobacco Use 184
f. Suicide and Deliberate SeLf harm (DSH) 188
g. Delirium 189

PsychosociaL Aspects of Gender and SexuaLity 192


Sexual Identity 192
Gender Identity 193
Sexual Behaviour 194
Gender differences in Sexual Behaviour 194
Masturbation 195
Sexual orientation 195
Psychiatric morbidity ig6
SexuaL Disorders 196
SexuaL Dysfunction 196
Disorders of SexuaL Preterence/ Paraphilias 197
Gender Dysphoria (DSM V) or Gender Identity Disorder (lCD io) 198
Management of Gender and Sexuality Issues 199

PsychosociaL Aspects of Pain 201

Psychosocial. Aspects of Aging 207

Psychosocial. Aspects of Death and Dying 210

Psychotrauma 211

Psychosocial Aspects of Terrorism 214

Stress and its Management 220


Job-related Stress & Burnout 222
Response to stress 222
Stress Management 225

Sample MCQs and Essay Questions 228


Appendix 230

Suggested Reading 232


£4P t’
F

ECTIONA
èhaviourai Sciences and their Relevance to Healthcare

— OUTLINE
Introduction to Behavioural Sciences
Holistic vs. Traditional Medicine
Models of Health Care
Non-pharmacological Interventions

hitroduction to Behavioura’ Sciences


As the name implies, behavioural sciences deal with the study of human
behaviour through an integrated knowledge of psychology. neuroscience,
sociology and anthropology. It is now widely recognised that the
psychological and social sciences play a role equal to biotogical sciences
in determining states of health and disease.

Amongst the behavioural sciences, psychology and neuroscience


contribute to the study of the human mind and the roLes played by its
various functions. They examine the role of functions such as emotions.
thoughts, cognitions. motivations, perceptions, and intelligence in
maintaining health or causing disease. Psychology also seeks to
understand how the development of personality takes place.

Another major influence on human behaviour is the role ptayed by the


family, the society and the community. The study of sociology helps a
doctor understand the influence of society and its various units and
institutions on the processes of heaLth and how they can change to cause
disease. The role of family. gender issues, social classes, socioeconomic
circumstances, housing, employment, social supports and social policies in
maintaining health or causing disease is studied in this domain.

Medical anthropology is the study of the effects of the evolutionary


history of human beings. It highlights their cultural history, racial
classification, geographic distribution of human races, and effects on
health and signs and symptoms of disease. It also involves the study of
cultural methods of deating with diseases and other distressing events
of human life. What disease is to be stigmatised, which symptom is to be
kept secret, what is to be handed over to the doctors and what is to be
dealt with by the faith healers is determined largely by anthropological
influences on a culture. Understanding the health belief model, attitudes
of a society and the rote culture assigns to a sick person can highlight the
importance of anthropology for a health professionaL

The behavioural sciences add to the disciplines of anatomy,


physiology, and biochemistry to support the study of holistic medicine.
t

While the former three teach about the body, psychotogy and
neuroscience educate the physician about the mind, sociology and
anthropology illustrate the evolution of human spirit and the factors that
constantly inftuence it.

Chapter 1
Holistic vs. Traditional Allopathic Medicine
Holistic medicine is inspired from the theory of Holism, which states that
reality (including all living matter) is made up of unified wholes that are
greater than the sum of their parts. Each sub-part is linked with the other in
a dynamic way. Holistic medicine considers mind, body and spirit sub-parts
that form the person; a whole that is greater than the sum of its parts. It
denies the separation of mind and body advocated in traditional atlopathic
medicine.

Traditional allopathic medicine works on a biomedical model that aims to


treat the diseased part of the human being. Holistic medicine on the other
hand is committed to the restoration of health and wellness to the
person as a whole, rather than focusing on the diseased part alone. A
health professional committed to holistic medicine is expected to
understand the following elements of this approach: Person, Environment, T
Health and Physician. I.:
Person: A human being who has the well-integrated etements of mind,
TraditionalADopathic Medicine
body and spirit held in a dynamic balance. methodology: Ju5t x the pmbLem
Artist Laura Zomhie
Environment: A set of external forces that can inftuence our experience
of health and disease such as family, community, culture, socioeconomic
resources, access to health care and quality of heaLth care. These external
factors help shape our attitudes and health beliefs, Attitudes and beliefs
that we learn from our environment have the capacity to either support or
disrupt the dynamic balance of our mind, body and spirit.

HeaLth: A dynamic state of well-being achieved through a mind-body-spirit


balance that hetps an individual realise their full potential.

Physician: A person who supports health (as defined above) rather than
one who merety treats disease. A practitioner of holistic medicine,
therefore, believes that health results from a dynamic and interactive
reLationship between the person, his environment and the physician.

4
Holistic medicine demands that a physician must be a person who has the
following characteristics:

• BeLief in the potential of the heating act


• Capacity to listen and empathise
Respect for the dignity of human beings
• Tolerance for difference of opinion 4’
• A gentte spirit
• Ability to mix creative thinking and intuition with scientific thought
• Will to never give up hope even against heavy odds

The knowledge of physical sciences and anatomy, physiology, and


biochemistry provide adequate basis for the practice of traditional
allopathic medicine. The practice of holistic medicine, however, demands
the knowledge of behavioural sciences as welt as natural sciences.

Chapter 2
Health Care Models and their Clinical
Applications
;. Bio-Psycho-Sociat (BPS) modeL of heatth and disease
In 1977, George Enget theorised the importance of integrating the
traditional biological (pathophysiological or structural) aspects of
medicine with the behavioural sciences (psychology, sociology and
anthropology). He put forward the concept of the Bio-Psycho-Sociat
(BPS) perspective of health and disease. Engels BPS model was based
on three principles:

a) Disease is a result of multiple factors that interact to make an


individual feel ilL Illness and disease are not a consequence of
biologicaL factors alone.

b) An individual is composed of a complex, integrated system


composed of interacting subsystem elements of mind, body, spirit
and social relationships, alt having feedback loops. Any change in
one will result to changes in other systems.

c) Biological, psychological, and social factors form a triad to interact


and serve as determinants of disease.

BIOLOGICAL

Biopsychosociat Modet
He proposed that the biological, psychological and social systems work
together to cause disease. The biological system ensures a structural,
biochemical and a molecutar study of a disease. The psychological
system provides insight into the role of personality, attitudes, attributes and
motivation in the genesis of the illness. The social system emphasises the
impact of family, society, social forces and culture on the aetiotogy,
presentation and the management of a given illness.

The biopsychosocial model stresses that understanding and manipulation


of the psychosocial environment of a patient is just as important to
recovery as the study of pathophysiological processes and methods of
treatment. Engel proposed that death of a significant other, grief, loss of
self-esteem, a threat to one’s life, property or integrity, even victories and
reunions were events that can trigger a medical, surgical or a psychiatric
condition. The biopsychosocial model, therefore, provides a
comprehensive clinical approach towards the practice of holistic medicine.
This approach lays great emphasis on the doctor-patient relationship. This
involves psychosociat assessment, the use of communication skills, infor
mational care, counselling crisis intervention and extension of care to the
family. One of the significant contributions of the BPS model in health care
is the emphasis it assigns to the use of interventions that do not
involve surgery or drus: the non-pharmacological interventions.

Ctinicat AppLication of BPS Model


It is useful for a health professional committed to holistic medicine to
approach patients using the BPS model. Research shows that biomedical
and behavioural factors come into play in infectious as well as non-infec
tious disordets. A patient of dengue fever is suffering at a biological level
on account c breakdown of the body’s reticuloendotheLial system. Social
issues related to drainage of fresh water, poor disposal of waste, however,
are also contributing factors. Psychological and anthropologicalfactors
such as risk taking behavior and inappropriate dressing in high risk settings
are equally important in the spread of this infectious disease. Sexually
transmitted diseases, HIV-AIDS, and hepatitis epidemics may atl occur due
to risk taking behavior and poor protection strategies.
Non-infectious disorders ar- also affected by biopsychosocial factors. This
includes heart disease, di&’• tes mellitus, cancer, and depression. This is
because changes n hormones, immune factors, metabolism and
neurotransmitters re alt associated with socioeconomic stressors.
Occupational hazards, dietary habits, child rearing practices, personality
development, exposure to childhood trauma are alt governed by culture
and geography. Many metabolic disorders are now called ‘life-style
disorders’ due to the socio-cultural and psychological factors that work
hand in hand with biological factors. Another example of the BPS model
determining disease is seen in road traffic accidents due to drug and
alcohol abuse.
‘U -

college together Hamid decided to stay in the hostel as he belonged to a ‘1

dtstant village while Hassan preferred to come to cottege ftam home eveiy
day Soon the stress ofmedical studies started to mount Hatndpraposed p

that they should try smoking a cigarette to qchieve ‘better concent,r%ion


white studying Hassan readily agreed and they both started to iridutge ip
smoking white studying togetherin the evenings. Hamid soon developed a
cough, but continued to smoke HassdAsparet7ts found out and discussed
the dangerous consequences ofhis habit Hassan opted out ofsmoking,
joined a gym and started to exercise regularly He consulted his
behavioural sciences teacher to learn some innovative methods oftudyIng
and techniques to give up smoking This helped him feel healthier and
conceritmte befterin his studies He tned to convince Hamid tojoin him in
these newly learnt techniques btHamtd did not Usten Within a year Hamid
went on to start use ofcannabis and a few months laterbecame addicted
to a stimuLant Msgiades as welt as his physicat health detenorated and he
failed his annual exams. He started to develop repeated episodes of chest
infections Repeated absence fann classes and poorperfoiwance in the
academics ted to his eventuat wfthdrawat from medicat college, while Hassan
went on to’ continue his medicqt studies enjoying good fi7ysicat health.

The story of Hamid and Hasan illustrates an interplay of bioLogicaL


psychologicaL and social factors resulting in contrasting outcomes on
account of the differences between the two friends in these domains. The
story highlights how homeostatic mechanisms failed Hamid. On the other
hand, restorative and predictive atlostatic behavior (discussed below) such
as parental concern, joining a gym and counselling by the behavioural
sciences teacher helped Hasan overcome a stress they both shared. The
outcome of disease in Hamid and health in Hasan was decided by an
integrative interplay of all three domains, Social support and allostatic
mechanisms were in place for Hasan, but none of these were avaiLable for
Hamid. Hasan managed to effectively turn the stress of studies into
eustress, He was, thus, able to achieve a better state of heaLth.
a The Integrated ModeL of HeaLth Care: CorreLation of Body,
Grain, Mind, Spirit and BehaviouraL Sciences
The integrated model of health care is a step ahead of the
biopsychosocial modeL It suggests a dynamic functional link between five
domains of human beings: biological, cognitive, behavioural, sociocu[tural,
and environmental. In this model, health is a state of a harmonious
equilibrium between these domains which occurs in response to eustress
or distress. This state is achieved through processes called homeostasis
and attostasis.
Homeostasis is a reactive state that ensures harmony within the body
systems through adaptive negative feedback loops. It also uses reactive
behavioural adjustments in domains operating outside the body.
-*
BEHAViOURAl.

SOCIO- ENVIRON
cucll.rnAL MENTAl.

Integrated Modet of Heatth Care

Attostasis is an adaptive mechanism in which the individual makes the


adaptations by predicting changes in advance, rather than in reaction.
These adaptations are creative and organised multisystem changes made
in anticipation of a possible challenge to health.

A typical example of homeostasis is the increased intake of fluids and salts


while working on a hot summer day. Allostasis on the other hand would be
to organise your work schedule in advance to be undertaken at the time
of the day when it is Least hot, so that you may not need the extra salt and
fluids.

In the Integrated Model, an optimum degree of stress called Eustress is


considered appropriate and necessary for a person to function and stay
healthy.

Eustress is seen as moderate, motivating and inspiring. It ensures optimum


functioning of homeostatic and atlostatic mechanisms that keep alt five
domains (biological, cognitive, behavioural, socioculturat, and
environmental) working in synergy.

Distress is a state in which the homeostatic and atlostatic mechanisms of


biological, behavioural, cognitive, environmental and sociocultural
domains are challenged by extrinsic or intrinsic factors. Challenge to any
one domain influences alL the other domains and sets up a restorative
feedback loop. If the systems respond with effective homeostatic and
allostatic responses health is restored. If the stressor worsens to result
in distress, a failure of homeostatic and allostatic mechanisms resuLts in
disease and illness.
Clinical Application of Integrated Heatth Care Model
Separating Disease from Sickness, Distress and Stress
Alt patients who develop symptoms and report to hospitals are not
suffering from disease. The body and mind respond to any disturbance
in biological, sociaL cognitive, behavioural and environmental domains
through unpleasant experiences which can be called symptoms. Most
of the time, symptoms serve as a stimulus for adaptive mechanisms and
homeostasis is restored through changes in the body, mind, social support
and environmental manipulation. Not all individuals reporting to hospitals
are, therefore, ‘patients’ in the biomedical sense. They may not require
PERFORMANCE

FATIGUE EXHAUSTION

RELAXED ANGER/FRUSTRATION/
PANIC

INACTIVE FAILUREI
‘,, BREAKDOWN

- I.
STRESS EUSTRESS STRESS BURNOUT
UNDERLOAD OVERLOAD
STRESS
Stress-Performance (Yerkes-Dodson) curve

laboratory and radiological tests or treatment with medication. It is, thus,


important to separate disease from distress, sickness and iltness.

Distress: This is the earliest unpteasant departure from a state of


happiness and health. This state appears when homeostatic and allostatic
mechanisms in the body and mind are challenged by stress. This sets into
motion immediate restorative mechanisms in the body in order to attain a
feeling of health through physiologicat means. No structural or
psychologicat damage takes place at the level of the body and mind.
Changes are visible, however, in the individuaL’s behaviour and social roles
as functioning of the individual may be affected. S/he can readity return
to normal following the restoration of homeostasis without any biological
interventions in the form of medication or surgery. Minor environmental
manipulation, mobilisation of social support and adjustments in cognitive
and behavioural domains may be all that is required.
Distressful states may present with the same symptoms as that of a
disease. Common distress symptoms include headache, backache, vague
bodily discomforts, feelings of indigestion, heaviness in abdomen, lack of
sleep, appetite, lethargy, fatigue. weakness, dizziness and Light
headedness. Individuals may also experience an urge to remain silent,
avoid responsibility at home or work and have a general feeling of inability
to cope.
These feelings in a state of distress usually last for a few hours, or a day or
two, but never beyond a week in one go. They are self-limiting, and
improve with pleasant occurrences such as meeting friends, sharing
feelings, indulging in a hobby or joyful pursuit or even a couple of
paracetamot tablets.

Sickness: The state of distress can sometimes give way to or be replaced


by a feeting of being sick’ or unwell, or nauseous. This unpleasant state can
appear without any disease or any pathological change. On the other hand
one may have a disease and not appear to feel sick at all (as in the case of
some diseases in early stages, like cancer).
Sick Rote: This is a state that an individual may assume at home or in office
settings to show his inability to perform his routine roles or duties. This role
may succeed in freeing the individual from their routine duties. S/he is
expected to seek medical help and follow the advice of his well-wisher. If
they do not do so, they may be seen as a malingerer.
Malingering is a derogatory term used to describe a frauduLent sick role
that an individual assumes to avoid responsibitity or gain a social or a
financial advantage. A competent doctor is hesitant to jump to this
‘diagnosis’ and always seeks a more experienced colleague’s opinion
before labelling a patient as a ‘malingerer’. Many patients who are seen
initiatly to be feigning an illness have been known to develop the same or
some other serious disease,

Ittness: is an overall view that an individual, the family and the society take
of a person who is feeling sick or unwell. The explanation that each has of
the sickness decides the course of actions and health care plan that wilt
follow. If the family and the community have no obvious or known
explanation of the symptoms experienced by the sick individual, the
likelihood of a medical consultation is rare. The patient wilt, instead, be
taken to a spiritual healer, an aamil, or a charlatan. This is especially true of
patients suffering from psychiatric disorders, epilepsy, and many
behavioural disorders. Most patients suffering from anxiety and depressive
disorders experience physical symptoms for which they prefer to undergo
tab tests and consultations with physicians and neurologists rather than
psychiatrists.

Disease: The diagnosis of disease is made when the symptoms of an


individual are attributed to a cause or aetiology. This can be in the form
of injury, an organism, a substance, a pathological or structural change or
a defect leading to changes in functioning in biologicat, behavioural, and
social spheres. These factors are severe enough to not only challenge but
disrupt and even destroy homeostatic and allostatic mechanisms. They
have the capacity to change the restorative negative physiological loops in
the body so that pathological processes begin to worsen the state of the
individual, instead of initiating repair and equilibrium. In a diseased state, it
is assumed that a reversaL of the causative pathoLogy would result in heal
ing of the disease. Typical examples would be enteric fever, a fracture, or
insulin dependent diabetes mellitus, Here, a complete return to health and
reversal of disease is guaranteed through a medicat or surgical
intervention.

It is important for health professionals to note that all of the above states
may or may not co-exist in the same patient at a given point in time. A
person may feel distressed and sick without any disease. S/he may move
around performing routine roles and duties even while harbouring a
serious disease.
WelL trained health professionals, clear about distress, sickness and
disease should not call for unnecessary lab and radiological tests, They
should also not prescribe ptacebos in the form of pain killers,
muttivitamins, intravenous drips, ‘brain tonics’ and ‘high energy pills’ to
individuals who report to hospitals in a state of distress. All medical and
surgical interventions are, thus, only to be used once the diagnosis of a
disease has been made.

I
li
I
Integrated Model of HeaLth Care Cilnicat Scenario
Mr Xis aiar oLd cterkk the tnxatianoffica HepresenL5 to the physician
with increased thirst and appetite toss ofsexualfeetings al?d weightgain.
His Ibstiog bLoQd UgatwaSfeufltb3OO mg/dL He has a (omityhistory
ofdibete& He is nqsedas Iiawng Type II Diabetes MeUitus The treat
ment arms are maintenance ofasgar- dIeL j%xstrng blood glucose levels j

or infections He is p1aced on.z gram ofmetlbnntn/day Hers ciskedto have .4

F tyhome ‘. meats waik to his offIce in the mornings andtakea3


km walk with die irithe averii#
Biotogis.trzsulin istc dIstUrbedarbohydratemetaboLisnz
encfzctors
Magee,tMetfi2rmii7. This helped in ave gtheb7sutin ies&dnce
and knpxove the carbohrate metqkoffstn

8ebavawoiiPeferencef& h Ca esugarithfaO a sedentary


tifestyte shssft(jØb and unhappy mari& Life.
ManngenentAttreversed with a change inea& habits. shait bursts of
pIcaLe’re dLiring working hours andin7pioved interaction with hIs wife
dJrinre9uIarevenfrg walks

CDislk..frpersonai physique and thoughts ofgrowing


oLd, overweight and ugly. rnaritatstress anda satIsed maritaL life
Management Information andunderstandingr ofhazards of overeating ond
.entanj Lifestyle, and cQmrnitment toa healthier way of thinking about self
future, his fqrnily and his work! in generaL

sociocuLturaLjactoxs Clerks in such oie regularly receive sweets and


unheatthy food and eat unhealthy high calQrle and carbohydrate nch food a
the office canteen, cuture of working long hOurs

Enviro)metaLfactois Colleagues wiihsfmllar unhealthy eating hat’its and


lifestyles Availability ofunhealthy food at the canteen absence of opportuni
ties for exercise and tack of access to healthy food Minorchonges in lifestyte
t work and home reversed the environnentatkl&1ence,
Management Mr X’s refusot to accept swAts bip lunch from home,.
simple physical exercises during ofñce hours for5 to 10 minutes instead of
constantty sitting on the choir alt helped in improving the culture in the office
and other workers soon started to copy Mr. X

3. The PubLic HeaLth Care Modet


Treatment at Primary Care Levet, Prevention of ILlness, Promotion
and Protection of Health
Hospital based health care models work primarily to emphasise treatment
of disease, This kind of health care approach is one of damage control
Public health care models on the other hand, work not only to treat disease
but also to prevent it.
The World Health Organisation (WHO), a premier stakeho[der in the field of
heatth care, promotes a public health care approach in addition to hospital
based care. This model is committed to treatment of common diseas
es and basic health issues through primary health care centres. Primary
heatth care centres are estabLished at the grassroot level, where the
maximum rural, and semi-urban population resides. In Pakistan these are
called Basic Health Units (BHU), and Rural Health Centres (RHCs). These
Centres work towards prevention of illness and promotion and protection
of health by working with the community in the delivery of heaLth care.
The strategies in place include immunization campaigns, mother and child
health programmes, reproductive health, HIV-AIDS programmes, nationaL
programme of mental heaLth, narcotics control, antimaLarial and dengue
controL programmes. These run in collaboration with national and
international governmental and non-governmentaL organisations. Health
legislations on smoking, healthy diets, seat belts, helmets, safe sex,
population welfare, and reduction in mentaL health gap are some of the
initiatives undertaken to achieve the promotion of health.

SUMMARY
KnowLedge, skills and attitudes rooted in Behavioural Sciences are an
essential component of alt the models of health care currently in practice.
A comprehensive understanding of psychology, sociology and
anthropology as well as biological determinants of health and disease is
cruciaL for the practice of scientiIc medicine.

The traditional biomedical model of reversing biological causes of disease


has proven to be inadequate. An integrated model of health care in which
the psychosocial, cognitive, behaviouraL and environmental stressors are
considered as important as biological causes of disease is the future of
modern medicine. This approach aims at restoration of homeostasis, and
stress reduction to optimise functioning. It also helps attain equilibrium
between the internal and external world through allostatic processes.

Interventions that go beyond medication and surgery to include


non-pharmacological measures heLp achieve health in a far more effective
and lasting way. This includes measures such as mobilising social
support, influencing existing health belief modeLs, ensuring a healthy and
safe environment, providing informational care, conflict resolution and early
handling of psychotrauma.

A public health approach of primary and secondary prevention which


emphasise treatment of disorders and promotion of health as cLose to
the community as possible helps to achieve a more global perspective of
health.
Chapter 3
Non-pharmacological Interventions fNPIs) in
Clinical Practice

The use of these interventions is advocated in the BPS model for their
established efficacy (as seen by extensive research) in augmenting the
impact of drug treatment and surgical procedures. Non-pharmacological
interventions (NPIs) enhance patient satisfaction, improve adherence to
treatment, and strengthen the bond between the doctor and his patients
as well as the community.

The NPI5 in particular that a medical or a dental student can use to


diagnostic and therapeutic advantage include the following:

1. Communication Skitts
While communication seems like the most basic and innate part of being
human, effective communication is a vital toot in clinical settings as it forms
the basis of the doctor-patient interaction. The doctor and patient
undertake a joint voyage, many a times into an unknown territory of
disease. Problems may arise when the two travelers 9nd it difficult to
communicate or understand each other. While the physician is expected
to know the patient’s language, the patient is often unaware of medical
jargon. As the service provider, the responsibility for effective
communication ties with the physician. The tools that can be employed to
make this communication effective and skillful are:
i) Attending and listening: Attending is the act of truly focusing on the
patient. It involves a conscious effort by the doctor to be aware of
what the other person is saying and trying to imply. This may only
be possible if the interaction with the patient is done in a setting of
exclusivity
Standing on a patient’s bedside with fellow students, amidst the
traffic in a ward, attending to mobile calls simultaneously. or
eating/drinking while talking to the patient may signal that you are
not exclusively attending to the patient and/or his family member. A
screen next to the bed, or a relatively quiet corner of the ward meant
for interaction of patients with the students may provide a setting
that allows for more effective communication.

ii) Active listening: This is a process that goes beyond merely hearing
and making notes of what the patient says. It involves a simultaneous
focus on the linguistic and the paralinguistic aspects of speech. The
linguistic aspect refers to the words and verbal aspect of the speech
Paralinguistics refers to nonverbal features of speech such as timing.
votume, pitch, accent, fluency, pauses and ums’ and ‘errs’. These are
important as they indicate how the person is feeling beyond just the
spoken word. An understanding of body language of the patient is
important for a doctor to communicate with the patient. Body
language refers to the way a patient expresses himself through the
use of non-verbal cues such as facial expressions, proximity to the
doctor, use of gestures. body position, movements and eye contact.

Li
Fadorsthatrmptøv. cotrimintcaUon

Use of minimal prompts Lark of exclusivity

Sit squarely in relation to the patient Preoccupied oranoious health professionals

Open body position In relation to the patient Uncomfortable seating

- - Lack of attention to non-verbal cues


Leanmg shghtly towards the chont
during active listening

Maintaining reasonable eye contact Offensive remarks orjudgmcnt


by the health professional

Pelaxed attentive health professional Frequent interruptions

Listen and respond to feelings Selective iintening

- Oay dreaming or dosing off during


Note all pamlrnguist,c and nonverbal cues
- -

the communication

It shoutd be borne in mind that body language expressions are only


cues and not ‘ctinical signs. These cues should be pointed out to the
patient to draw his attention to them, to understand his feelings or
their meaning to him, e.g. “I notice that you took angry, how are you
feeling at the moment?’, or ‘your eyes filled up with tears when you
told me the name of your father.” This is more rational than making
the wrong assumption about his gestures or body language.
This is essential as methods of non-verbal communication vary in
patients and their family members, according to their upbringing,
culture and background. Active listening also involves customizing
your style and language to match that of your patients or anybody
you are listening to. This can be done by using the same language
as the patient wherever possible. Another important aspect of active
listening is respecting the pauses and silences of the patient. This
would mean not immediatety jumping in and talking whenever the
patient pauses for breath or reflects silently.

iii) Verbat techniques.’ These are pivotal in making the communication


effective and thus contribute towards the therapeutic process. These
are vital skitls for the doctors and can be mastered through practice.
Any verbal communication in a clinical setting involves the following
components:
Questions: these can be closed or open ended.
Ctose ended questions elicit a yes/no or a fixed response e.g. 4What
is your name?” “Are you married?” 5Do you get nausea after taking
your meals”? These questions are vital at the start of an interaction
both, to collect data as well as establish familiarity and comfort with
the patient.
The open ended questions do not elicit a particular answer. They are
intended to encourage patients to talk more about their story or
to expand more upon their issues. Questions are usually used for
exploration of a particular aspect, for obtaining further information,
to clarify any details and to encourage a patient to talk. E.g. “What
brings you to the hospital today” or 5Kaisay aana hua?° or even
simply 5Jee, kohiye.”
It is important to start an interaction with the patient or his family
members with an open ended query, such as What brings you
to the hospitaL?” What can I do for you”? This gives the patient a
chance to open the conversation, with what s/he considers most
significant.

Leading questions are those that prompt the patient to answer in a


certain way. These lead to skewed information as we tend to give
the answer that we feet the person is looking for. These should be
avoided as should value laden ones. Some examples of these are
e.g. Don’t you think your pain radiates into the left arm?” or “Do you
feet ashamed of your short stature?”

Moreover ‘why’ questions should be used sparingly e.g. “Why do


you think you have developed shortness of breath?” An effective
communication therefore revolves around questions starting with
what, when, where and how

Funneling: This refers to the use of questions to guide the


conversation from a broader area to a more specific one. These
should follow open ended questions. This technique hetps the
interviewer move from general statements by the patient to
specific areas of clinical relevance e.g. “Now that you have
described your complaint of feeling weak and lethargic. can you
describe which specific part of the body you were referring to?”
Paraphrasing: It refers to the process of repeating the last few
words the patient said and summarising what the patient has
communicated so far, in your own words, and then ask him or her
to validate if you have understood it correctly, e.g. “you have told
me about the weakness in your legs and lethargy that you feel after
walking for only few yards. Is that right?” ‘Aap ne bataya k aap kal
maiday mal 2 haftay sejatan ho rahi haijo khanoy k baud barhjaati
hai, kya also he hal?”
Setective reflection: Reflection is a technique to bring out the
feelings attached to various symptoms and problems that a patient
has stated. It refers to the method of repeating back to the client a
part of something s/he said that was emphasised in some way or
which seemed emotionally charged. e.g. How does it feel when
you start to feel fatigued only walking for a few minutes? You told
me earlier, that you were once an athlete who could easily run a
mile.”
Empathy buitding: This refers to statements made by the doctor
that make the patient see that his or her feelings have been well
understood. It helps the patient understand that his/her feelings
are valid and that the doctor would have felt the same if s/he was
in the patient’s place. It is important here to refrain from expressing
sympathy instead, which would imply that the doctor feels sorry for
the patient’s plight.
can imagine how difficult it must be for you to live with your pain
for such a long time” is an empathetic statement, which is highly
desirable; a statement such as “Poor you, really feel bad hearing
your story” is an expression of sympathy which may not have the
desired therapeutic effect and also undermine the effectiveness of
communication.

Checking for understanding: From time to time during the session


the doctor needs to summarise patients statements or ask the
patient to comment on the summary. to ensure if s/he has
understood the problem and its associated feelings correctly.

An effective communication based on the above principles is


bound to form a bond and a relationship between the patient and
the doctor in which both feel understood and connected. It is this
feeting of mutual understanding that is traditionally described by
patients as Hatf my ittness was retieved after tatking to my doctor.

WhiLe the principles of effective communication should be part of


all clinical interactions between a doctor and his patient, the best
use of these principles is in counselling individuals, couples, family
members or groups.

2. Counsetting
Counselling is a technique that aims to hetp peopte help themselves by
the development of a therapeutic relationship between the counsetlor
and the patient or family member, a colleague or anybody who seeks
counsel. The process aims at helping a person achieve a greater depth of
understanding, and clarification of’ the problem mobilises personal coping
abilities. It is not an ordinary every day conversation, in which one person
• asks the other for advice and gets the other person’s opinion on what to do.
Counselling is a limited supportive activity aimed at developing a person’s
ability to decide upon and initiate a constructive change. A doctor or a
medical student may come across a variety of situations in clinical settings
and professional interactions in which they may require counselling skills.
Some of the common scenarios where this skill can become a useful
intervention include: breaking bad news to patients or their families, or
resolving professional conflicts. These may include announcing that a
patient’s biopsy report has revealed a malignancy, or that cardiopulmonary
resuscitation has failed to revive the patient. It may be required as part of
sharing the news of a baby with congenital malformations or a stillborn
baby with the expectant parents, resolving a conflict between a colleague
and a nurse in the ward, or handling a relative who feels that his patient is
being ignored and denied a particular investigation or intervention. A coun
selling session aims to:

a) Establish a relationship of mutual trust and care in which patients


and/or their families feel secure and able to express themselves in
any way or form necessary.

b) Give patients or their families a chance to seek clarification and


expLanation of terms, issues and misgivings.

c) Provide an opportunity to patients or whoever is being counselled


to freely express his or her feelings and emotions.

d) Provide reassurance.
e) Achieve a deeper and a clearer understanding of a heatth related
issue based on scientific and evidence based data.
f) Identify the various choices and options alongside their pros and
cons through a process of discussion and dialogue between the
counsetlor and the patient.
g) Help the person make a decision or reach a solution that is most
suitable for him/her.
h) Seek support of the counsellor
i) Mobilise resources required to implement the solution.
j) Learn the necessary skills to cope or deal with the issue.
Under no circumstances is the counsellor expected to make decisions
on behalf of the patient or the one counselled. The responsibility of the
consequences of the proposed solution thus always rests on the shoulders
of the patient seeking counsel and never on the counsellor. If a medical
student or a doctor opts to take up the role of a counsellor s/he needs to
develop and exhibit certain attributes, discussed below.

What traits must a counsettor have?


Unconditionat positive regard
This involves a deep and positive feeling for the patient, being
non-judgmental and trusting.

Empathic understanding
This is the ability to accurately perceive others’ feelings, validating
them and communicating this understanding to them effectively.
As highlighted above, it is different from sympathy which implies
feeling sorry for the person.
Warmth and consideration
This can be achieved by remaining open-minded and non
judgmental. Avoiding over emphasis of your professional role and
being consistent in behavior helps convey that you are genuinely
there to help. Also by remaining respectful and tactful, the counsel
tor would be able to show warmth and consideration to his patient.
Clarity
The counselling relationship should remain clear and without
mystery to the patient. As a counsellor you are required to be clear
and explicit. Encourage the person being counselled to be similarly
explicit in his requirements. Use of the techniques of paraphrasing
and checking for understanding described above can ensure
successful communication.
Here and now thinhing
The distressed patients would like to talk excessively about their
past in order to avoid the reality of the present. As counsellor you
need to help identify present thoughts and feelings to enhance
problem solving attitude on the basis of here and now’, and focus
on the present day issue(s).
caun4Ipon

Do not ask why” questions, These imply interogatiorv


7 MIsconcptfons about Counselling.

Does not involve giving direct advice to patients

Do not say should ought or icarna chahiye tha. Does not solve people’s problems for them
These imply moralisation.

Do not blame the patient Does not challenge a patient’s feelings and perceptions

t)o not compare the patient’s experiences witi, Does not impose the counsellor’s own views
your own, or gite examples from your life onto the patient
The patient is a different petson from you
and has different life experiences. Does not make people less emotional

Do not invalidate the patient’s feelings, Does not work to fulfil the counsellor’s need
to make people feel better

3. Informational Care (IC)


Memoirs of a patient’s son
I took my etderly mother to a targe hospitat in our city when she became
sick She was very embarrassed to go to the doctor because she said that he
would examine her and cause bepardagi but! convinced her that they have
welt trained doctors who are trustworthy and wilt take care ofher without
causing her any embarrassment We went to the outdoor department where
we were told that she had a breast lump which coutd be °a tumOur.” This
was like a bolt of lightning for the whole family as we had heard that nobody
survives from cancer I borrowed 5000 rupees from a friend and admttted
her in the sUrgery ward in the big hospitaL We were hoping that through
these doctors, A(tah would help us through this trial, Ajunior doctor took her
medical history and started some medicines. I asked him whether my mother
woutd be okay, but he said he didn’t know yet and we needed some tests. He
then went away and a nurse gave us a slip to do some tests but nobody told
us how much the tests would cost woutd it be painful for my mother how
long would the results take, how tong would we need to stay in the hospitat,
any precautions we need to take for her recovery? When the test results came
thejunior doctor looked at the results and told us that the senior doctorsahib
wilt decide during “the round I thought maybe they wilt tell us when they
decide after the round The senior doctor sahib came for the round but he
discussed something in English with the other doctors and moved on from our
bed without telling us anything. Later on, a group ofstudents came to our bed
and said that they needed to examine my mother’s chest My mother was very
ashamed but they sqid that it was necessary for her treatment, and so We
had to agree Seven of them examined my mothers chest turn by turn and we
were constantly worried about how many peopte might be watching her like
this. Later on, I asked for the senior doctor sahib to find out about the treat
ment ofmy mother and the questions I had in my mind, but the peon said that
he was in a meeting. I asked for thejunior doctor who had taken our history
but he had left after his duty and would be coming back the next day I asked
the nurses too but they did not know anything about my mothers treatment
plan. A newjunior doctor came that evening on Ucity and told us that we had
to prepare for my mothers surgery two days later, and that we also needed
to arrange for3 units ofblood and about 20,000 rupees for the items required
in the surgery. We were very confused, as no one had discussed anything with
us about this surgery. When I asked thejunior doctor about how much money
we needed in 4otat how many days we would need to stay in the hospital
after thaL and if there was any other option besides the surgery he got angry
and said that Don tyou trust the doctors advice2 and you care about mon
eymore than your mothers health 57 was very hurt and embarrassed bythese
comments On the otherhand my mother and sisters were very hopetess as
they had heard that nobody sunuves from 5cancer even afterthe surgery
Veiywomed and confused we were totd bya neighbtur that a local pirsa
R2LJ
LI
We didnot know what to do All we really wanted was someone to listen and
answer some ofour quenes in this confusion and desperation, a consultation
with the pirsahib seemed like our onty ray ofhope So the next morning we
left the hospital fora meeting with the pirsahTh’_..

Recommended exercise
Read this case scenario once before studying this sectlonr and then a sec
H
ond time after completing the section Discuss whythis chath of events ted
to this tonsequence and what actions could have been taken differently
by the health care team to avoid such an unfortunate outcome

Informational care is defined as provision of information to patients using


principles of communication regarding the disease, the drugs and the
doctor (the 3 Ds). This helps to fill the gap in the patient’s knowledge and
understanding in these areas. In order for the patient to fully achieve this
understanding, informational care must be provided using Language that
the patient understands. During ill health, the patient and his caregivers
feel a desperate need to know what exactly is wrong, how it is being or will
be managed, who will deliver the care and how.
The amount of information provided, timing, Language and setting in which
informational care is imparted has to be tailored according to the individual
needs of the patient, This includes considerations such as what stage the
illness or recovery is at and what questions bother the patient the most.
Seven ESSePtIIS Ifl iflformatlonal Care:

The physician must set aside time within a consultation to


give a reasonable level of information to the patient and his family about
the disease and treatment.

The IC session must take place in the language that the patient can understand.

it must start with patient’s knowledge, understanding and expectations.


Aap apni bemari kal baray ma) kya Jantay haln The doctor must than remove any
myths and misconceptions that the patient mentions in his description. These
misconceptions must be clarified and replaced with evidence-based information,
The task of giving intormation should be professional, evidence
based facts are provided without fear of causing a negative reaction in
patient and/or family. It must however be done with compassion, empathy and
sensitivity. Vague statements and building false hope should be avoided.

Both aspects of the disease and treatment, negative and posItive should
be communicated to th, patient, but information overload is to b, avoided,

Use of simple figures, diagrams and sketches are often helpful to enhance
the patient’s understanding. Most patients or relatives may like to keep the
sketches at the end of the session, which consolidates their
interest and the titility of the IC etetcise in the therapeutic process.
The IC session ends with th. patient briefly summerising his new understanding
of the 3 Os. This helps to evaluate how much of the InformatIon has been retained,
The doctor finally reassures that any future concerns and clarifications
that ar. needed will also be addressed.
Seven Questions a Patient NeedsAnSwered man CSessian

What is wrong with me (diagnosis)?

Why have developed this disease (aetiology)?

Is there an effective treatment to my problem? Is the treatment safe? Are there any serious or
danoerous side effects (management)?

How long iIl I take to recover (prognosis)?

Is therea ‘Perhez’ (restrictions)?

Is there a risk of illness being spread to those APOUND me or passing It


onto my offspring (transmission)?

How will the illness and the treatment effect or influence my functioning?
(Can I continue to work or rest? What will happen to my
sex life, sleep, appetite etc.?)

4. Handling Difficutt Patients and their Families


Health professionals find certain types of patients and their families
exceedingly difficult to deal with. These include individuals who

• have long, meaningless and repetitive discussions with the doctor


• waste precious time.
• become too dependent and clingy
ask for undue favours
• make unprofessional demands.
• try to manipulate the doctor
• become angry when things do not go their way
• become rude or behave aggressively.
• refuse diagnostic tests and treatment.

Other patients who are seen as difficult are those with medically
unexplained symptoms (MUS) such as vgue physical complaints, aches
and pains, mentat health problems and patients who may be drug users,
are obese or mute.
Management:
It is important to be aware of factors operating in a health professional that
can give a false feeling that the patient is behaving in a difficult way. These
commonly include having a heavy work load and what time of the day the
interaction with the patient occurs, as health professionals tend to become
irritable towards the end of the day. Inadequate knowledge and skills to
deal with a demanding clinical situations may also cause the health pro
fessional to become panicked or overly sensitive. Lack of training in com
munication and counselling skills may worsen this situation. Some health
professionals trained in a biomedical model feel that addressing patient’s.
psychosocial and spiritual issues is not their job. They may, therefore,
become irritable when a patient brings up these aspects for discussion.
Whatever ones views may be, as a heatth professional you are likely to
come across at Least one if not all of the aforementioned situations.
The following steps may help in dealing with a difficult patient or family
effectively:

a) Have an understanding of the biopsychosocial model and


integrated health care model and believe in the effectiveness of
these well researched models.

b) Train yourself well in principles of effective communication and


counselling. Seek specialised training in handling of difficult patients
by trying to form a relationship or bond with difficult patients in the
ward. Looking at videos of how seniors ideally handle such patients
and discussions with health team members will help educate you.

c) Learn relaxation techniques to manage your own anger and feelings


of frustration.

d) Approach difficult patients with tolerance, patience and use of


principles of active listening and unconditional positive regard,
keeping your cool. Concentrate on breathing deeply and easily while
listening to the angry patient or a family member.

e) Do not take remarks being passed as personal insult’ or challenge to


your integrity or authority. Consider them a different viewpoint of an
individual who is hurt or is uninformed and unguided.

f) Allow the patient or family member to express anger and validate it


by statements such as “your anger is understandable”, “I can
understand your feelings”, “this must be frustrating for you’.
“mujhe andaza hal kaiye aap k tiye kitna mushkft waqt hal”

g) Offer a chair and a calmer setting to discuss the issue at hand in


more detail. Offer an apology or an explanation for any unintended
offense but do not appear defensive. Stay calm, maintaining an open
body posture, a safe distance and always keep an eye at the
emergency exit. Always ask for assistance from colleagues or staff at
the earliest signs of aggression or threatening postures by a patient
or famity members.

h) For difficult pai. its in particular, define the objectives and duration
of consultatio ri advance.

i) Offer referral to a colleague or a senior consultant, particularly if you


are not making any headway.

j) Use humour while collecting further data, reassure, undertake


detailed physical examination, and a more extensive diagnostic work
up. or seek opinion from a mental health professional.

k) Involve family members, friends or significant others in the life of the


patient for support as well as help in understanding of the patient’s
issues.
5. Breaking Bad News
Any news that adverseLy and seriously affects an individual.’s view of his or
her own future is considered bad news. There are many clinical situations
where bad news has to be communicated to patients and/or their
relatives, e.g. disctosing the diagnosis or relapse of cancer, birth of mal
formed baby or death of a loved one. Breaking bad news is an unpLeasant
task and can be learned from the senior physicians or through own profes
sional experience. Most patients and families expect full disclosure
delivered with empathy, kindness and clarity.
There are five different schools of thought regarding the provision of
information to patients. The biopsychosocial model has the least number
of limitations and is therefore strongly recommended for use in health
settings.
a) Blo-Psycho-Sociat Modet:
This model provides clear, crisp, evidence based information on the
patient’s condition but tailors the flow and amount of information accord
ing to the needs of the patient. A vertical flow of all data on the disease
(particularly the parts that the patient or his family have not asked for), is
avoided. The bad news is broken using principles of effective communi
cation, counselling and informational care discussed earlier. The patient
is encouraged to involve his family members, particularly the ones who
can provide psychosocial support, during the session as well as in the
long run. This model suggests the following steps for a session that aims
at breaking bad news:

Step 1: Seating and Setting (Environment):


Exclusivity
The environment where bad news is being broken can have serious
repercussions on the outcome of the interview. A patient’s mistrust
and antagonism may simply result from a poorly chosen location.
It is, therefore, worth trying to find a private room where the doctor
and patient can focus on the subject attentively.

invoLvement of significant others


Some patients like to have family members or friends around them
when they receive bad news, while others prefer to hear bad news
alone. Ask the patient who they would like to accompany them. If
there are more than a few people supporting the patient, ask one
person to act as representative. This gives the patient support and
alleviates some stress from the doctor in the face of an emotionally
charged interview.

Seating arrangements
It is advisable for the interview to take place with both octor and
patient comfortabty and respectfully seated next to each
other, preferably at a distance of an arm’s length. The arrangement
should never impart an intimidating image of the doctor. It should
provide an appropriate setting for discussions and any emotional
outbursts or ventilation of feelings that may arise.
Be attentive and calm
Most doctors feet anxious when breaking bad news and it is worth
spending some time to eliminate any signats that may suggest our
own anxieties. Maintain eye contact and show your attention. If the
patient starts to cry, try shifting your gaze because nobody Likes
to be watched while crying. This should however be done with
sensitivity and must never send a signal that you do not realty care
about the patients feelings.

Listening mode
SiLence and repetition of last few words that the patient has said.
are two communication skills that wiLt send across the message
that you are Listening weLl.

Avaitabitity
If you have appointments to keep, give your patient a cLear
indication of your time constraints but make yourself available to
the patient for all his queries and doubts for the duration that you
are with him or her.

Step 2: Patient’s Perception:


Ask: What do you know?
“Aap apni bemoan kai baray mai kyajantay ham?”

The principle involved in this step is “before you tell, ask.” Before
you break the bad news to the patient, try to ascertain as
accurately as possible the patient’s perception of his or her
MEDICAL condition. Obtaining this information depends on your
own communication style. As your patient responds to your
questions take note of the language and vocabulary that s/he is
using and be sure to use the same vocabulary in your sentences.
This alignment is very important as it hetps you assess the gap
between patient’s expectations and actual medical condition. If
the patient is in denial, try not to confront him in the first interview,
as denial is an unconscious defense mechanism that facilitates
coping.

Step : Invitation:
Ask: What would you tike to know?
“Aap bemari k baray mai kyajanna chahain ge?”

Although most patients want to know all about their illness but
assumption towards that should be avoided. Obtaining overt
permission respects the patient’s right to know or not to know.
Some examples to address this are: “Are you the kind of person
who likes to know alt the details about what’s going on?”, “How
much information would you like me to give you about your
diagnosis and treatment?”, “Would you like me to give you details
about what is going on or would you prefer I tell you about the
treatments I am prescribing to you?.”
Step : Knowledge:

Before you break bad news, give your patient a warning of some
sort to help him prepare e.g. “Unfortunately I have some bad news
for you Mr. X” or “I am sorry to have to tell you...” When giving your
patient bad news, use Language similar to his. Avoid scientific and
technical language. Even the most well informed patients find
technical terms difficult to comprehend in that state of emotional
turmoil. Give information in small bits and clarify whether s/he un
derstands what you have said so far, e.g. “Do you see what I mean?”
or “Is this making sense so far?” As emotions and reactions arise
during the interview, acknowledge them and respond to them.

Ask: What have you understood?


“Kya mal aap ko baat theek se samjha saka/saki hoon?”

Step 5: Empathy:

For most doctors responding to our patients’ emotions is one of


the most difficult parts of ourjobs. In our effort to alleviate our own
discomfort it is tempting to withhold certain information or give a
more hopeful picture than actually exists. These tactics may appear
to help in the short term but seriously undermine aft your efforts in
the long run. It is much more useful and therapeutic to acknowl
edge the patient’s emotions as they arise and address them. The
technique that is most useful is termed the empathic response. An
empathic response involves listening and identifying the emotion
or mix of emotions that the patient is experiencing and offer an
acknowledgement for them. Identify the source of that particular
emotion and then respond by showing that you understand the
emotional expression of the patient. Statements such as “mai bhi
agar aap ki jagah hon toh aisa he mehsus karoon” reassure the pa
tient that you understand the human side of the medical issue and
that you have a respect for his feelings.

Step 6: Summarise:

Before the discussion ends, recapitulate the information in a short


summary of all that has been discussed and give your patient an
opportunity to voice any major concerns or questions.

Step : Ptan of Action:

You and your patient should go away from the interview with a
clear plan for the next steps that need to be taken and the role
you both would play, in the management of the issues. Also allow
the patient to have a way of contacting you, through the hospital
exchange or after rounds the next morning, in case they have any
questions.
b) Individuatised Disclosure Model:
In this model the amount of information disclosed and the rate of its
discLosure are tailored to the desires of the individual patient by
doctor-patient negotiation. First the doctor and patient work together to
clarify what information the patient wants. The doctor then imparts that
information in a way that the patient understands. This is an on-going and
developing process. It implies a tevel of mutual trust and communication
that takes time and effort to develop. The distinguishing features of this
model are that it takes time and skills and its assumptions are supported
by evidence. It has the capacity to maximise quality of life for the patient.
The underlying assumptions in this model are that it takes each individual
a different amount of time to absorb and adjust to bad news. A
partnership between the doctor and the patient for decision making is.
therefore, in the patient’s best interest. Its disadvantages are that it is a
time consuming process that might be difficult for a busy physician to
undertake. It also tends to drain a health care providers’ emotional
resources. The advantages are that the amount of information given and
rate of disclosure is taiLored to needs of the individual and a supportive
relationship with the doctor is established.
c) FuLL Disclosure Model:
This model involves giving full information to every patient as soon as it is
known. It argues that this promotes doctor-patient trust and
communication and facilitates mutual support within the family unit. The
underlying assumptions in this model are that the patient has a right to
full information about himself and the doctor has an obligation to give it. It
assumes that all patients want to know bad news about themselves and
that patients themselves should decide what treatment is best for them.
The disadvantage of this model is that discussion of options in detail may
frighten and confuse some patients. The doctor insisting on providing
information may undermine defenses such as deniaL which are otherwise
important for the survival of the patient. The provision of full information
may, also, have negative emotional consequences for some. The mod
el holds some advantages as well, such as promotion of doctor-patient
trust, family support and allowing patients time to put affairs in order in
case of a poor prognosis. It also helps those patients who cope better
with their diagnosis by having the maximum amount of information about
their illness.
c) PaternaListic Disctosure ModeL:
This model implies that information about the patient’s disease is the
right of the doctor. The doctor delivers the information to the patient as
and when s/he deems appropriate, in a ‘sugar coating’ to minimise the
pain and distress of the patient. It also involves the expression of sympa
thy and a sharing of emotions on the part of the doctor. This model is no
longer recommended for use.
d) Non-Disctosure Modet:
This model is based on the view that under no circumstance should
patients be informed that they have acquired a lethal disease. It states
that deception should be used if necessary, on the basis that the patient
needs protection from the terrible reality of terminal illness. This model
has been traditionally adopted as part of a paternalistic and nurturing
attitude of doctors towards their patients. The underLying assumptions in
this modet are that it is appropriate for a doctor to decide what is best for
the patient; patients do not want to hear bad news and they need to be
protected from it. This model has obvious disadvantages such as:
denial of the opportunity to adjust to illness, which the patient is ob
• viousty experiencing

trust in doctor is undermined
opportunities for helpful interventions are lost
• patient compliance is less tikely
• patients may acquire wrong information that can lead to avoidance,
isolation and a perception of rejection
• the patient may experience a sense of loss of control in what is hap
pening to his own body
Advantages of following this model are that it is easier and less time
consuming for the doctor and suits those people who prefer not to know
their condition. This model s fast fatling out of favour and is now widely
rejected by modern day doctors as welt as patients and their families.
What expectations do the patient and [amity have when receiving bad
news?
According to research, the most important factor to the patient and family
receiving bad news, is the attitude of the health professional. The heatth
professional should, thus, be knowledgeable, empathetic and give hon
est and clear answers in simple language. The second most important
factor is the setting in which the news is broken. A quiet, private place
• where the news is broken in an uninterrupted way is preferred.

What are the common reactions that a patient experiences upon receiv
ing bad news?
The reactions that a person goes through when they hear bad news, can
be summarised as the stages of denial, anger, bargaining, depression
and acceptance. These stages are rarely clearly delineated, and often
patients go through one or more stages at the same time and for each
individual the length of time each stage lasts may vary. It is important
that the health professional empathise with and provide support for the
patient during each stage.

What are the common reactions in a heatth professionaL breaking bad


news?
Delivering bad news can be equally taxing and demanding for the health
professional. S/he may experience strong emotions of being a failure, or
of not having done enough for the patient. Feelings of helplessness, sad
ness and fear that they may harm the patient emotionally by telling them
the truth may be experienced. Some may feel shame and disiltusionment
with their profession, and others may experience fear of their own death
and disability.
These feelings are essentially normal reactions to a challenging and a
difficult situation. A young health professional is advised to share these
feelings with a senior colleague. S/he may even assist in a few situations
before undertaking this specialised communication in clinical settings.
ChaLlenges In Non-pharmacological Interventions
As health professionals the biggest hurdle we face in the administration
of any non-pharmacological intervention is the Lack of time. In busy cLin
ics and overcrowded wards where patients go from being humans with
names to beds with numbers, it seems impossible to find the time to give
someone all they need. It seems to suffice that we are there at alt, that
we are doing the bare minimum to keep afloat in the never-ending sea
of patients that threatens to drown us. In such a situation we must keep
in mind two things: Research shows that by not spending the required
amount of time the first time we see a patient, we tend to misunderstand.
misdiagnose and mistreat. The inevitable result of this is that not only
does a patient not improve, s/he may return to a different doctor in a
worsened condition. This means that for the majority of our time, we are
redoing work that a colleague has done improperly (due to lack of time)
and vice versa. There are millions of patients stuck in this loop, who keep
reappearing for consultations, thereby increasing the workload of heatth
professionals as a whole. Secondly. it is important to understand that our
job is not to mistreat the most number of people in a day, but to actuaLly
treat the minimum number of people we can to the best of our abilities.
Using non-pharmacological interventions, such as providing informational
care and breaking bad news saves us time in the Long term. For example,
if we are able to take the time to explain to a patient that the true mea
sure of whether their blood glucose levels is normal is fasting btood glu
cose, or an HbAic, they will not waste our time (and that of the path lab)
by getting random blood glucose Levels done and showing them to us.
Breaking bad news is another time-consuming procedure, how
ever, one of utmost importance. To inform an individual that s/he
may have cancer or AIDS is to inflict a major psychological trauma.
People will remember, for the rest of their lives, the details of the
occasions when important news was broken. No surgeon would
think of operating without booking an operating theatre and setting
aside sufficient time to do the job properly. S/he would not ‘skip
the anaesthesia” just because it takes time. The procedure for
breaking bad news must have a similar importance. A health pro
fessional must think for a moment how they would feel if they were
to receive such news. There is a world of difference between the
doctor who breaks this news in relaxed atmosphere with a support
ive attitude and the caLlous consultant who flings bad news at the
patient in a public ward.
Before teLling people what we think they need to know, we should
find out what they already know, or think they know, about the
situation and what their priorities are. If they use words like cancer’
or death’, we should check out that these words mean the same to
them as they do to us. ‘There are many kinds of cancer, what does
the word mean to you?’. ‘Have you seen anyone die? How do you
view death?’ will often reveal considerable ignorance and open the
door to positive reassurance and explanation. Too often, doctors
fail to invite questions and miss the opportunity to help people with
the issues that are concerning them most. The patient has a right to
know the truth about an illness, but we must respect their right to
monitor the amount of new and painful information that s/he can
cope with at any given time. It is just as wrong to tell people too
much, too soon, as it is to tell them too little, too late.
• Life-threatening illness can undermine our confidence and trust
and members of the caring professions can do a great deal to help
peopte through these psychosocial transitions. Accurate informa
tion is essential to planning. Many patients may react with relief
when they are told they have cancer, as without any information
they have already imagined the worst. It is easier to cope with a Le
gitimate diagnosis than to live in an unplanned state of uncertainty.
• Many of the different ways people cope with threats reflect the
coping strategies that have been found to minimise stress earty
in life. At times of threat, those who tack confidence in their own
resources may seek help of others, express clear signals of distress
and cling inappropriately. Those who lack trust in others may keep
their problems to themselves, bottle up their feelings and blame
hea[thcare providers or therapies for their symptoms. Their tack
of trust makes it necessary for them to control us rather than be
controlled by us. A few, who lack trust in themselves and others.
may keep a low profile, turn in on themselves and become anxious
and depressed. To those who lack self-esteem the most important
thing we have to offer is our esteem for their true worth and poten
tial. To those who lack trust in others we can show that we under
stand their suspicion and their need to be in control of us. Doctors
must act as advisors rather than instructors and show that they
accept that trust must be earned: and that ‘it is not our right to be
trusted’.

SUMMARY
The breaking of bad news is a difficult situation for both the health
professional and patient and the family members. The task should be
undertaken in an exclusive and an uninterrupted setting. The information
provided should be based on what the patient and famity wants to know.
The information should build on what is already known to them. Opening
statement should be on the lines of “I have to share information that may
be unpleasant” or “I know it may be tough for you to know.” The contents
may be broken into short sentences making sure that the patient gets
adequate chance to process the unpleasant data. Accept and respect
the emotional reactions that follow the sharing of the information. In the
end leave enough time for clarifications and questions. Always schedule
a follow up meeting and mobilization of any immediate support that the
patient or the family may need after learning the bad news. The session
should not end without assessing the risk of the individual harming them
selves, and putting in place clear preventive interventions in this regard.
Reassurance that you as a health professional stand committed to pro
vide support and be with the patient during these trying moments is a
source of comfort for the patient and the family. This is a safe note on
which to leave. A calm, compassionate. empathetic health profession
alwho has adequate knowledge of the patient’s condition can leave a
calming effect on the patient and the family.
Young health professionals need to be aware of the strong emotional
reactions that they themselves may experience before, during or just
after breaking the bad news. These feelings are normal and their impact
can be reduced significantly by sharing them with a more experienced
colleague.
6. Crisis Intervention and Disaster Management
The word crisis is derived from a Greek word meaning decision makkig.
Chinese language has an expression for it in two words; danger and
opportunity. A crisis is, therefore, a situation which holds potential for great
individual growth provided that the appropriate decisions are taken. People
in individual crises or natural disasters find themselves in situations that
require deep and insightful decision-making and lead to a permanent
change in their lives. Crises are periods of disorganization, characterised
by trial and error, disequilibrium, and attempts to reduce feelings of dis
comfort. Resotution of a crisis can result in either an increase or decrease in
person’s level of functioning or a return to the previous baseline of
functioning. Individuals and communities who undergo major disasters
may. however, never be the same again. At a psychological tevel, they may
become more vulnerable to future crises. They are at a higher risk to
become victims of a variety of post traumatic conditions such as
post-traumatic stress disorder (PTSD). depression, anxiety and/or
dissociative states. They may become resilient and battle-hardened’, and
thus, better equipped to deal with challenges of life. This change that
foltows major trauma may be the basis of the positive shifts in human be
haviour called post-traumatic growth occur in response to stressful periods
of human maturation and transition. These inctude childbirth, early child
hood, schooling, adolescence, marriage, parenting, divorce, hospitalization,
death of a loved one etc. A situational crisis is where a person is faced with
a stressful or traumatic event which could be a natural or a manmade
disaster e.g. ftoods, earthquakes, rape, terrorist attacks, war, murder etc.

Ak.
GENERATE AND EXPLORE
ALTERNATIVE RESOURCES AND
COPING SKILLS

EXPLORE FEELINGS AND EMOTIONS


(USING ACTIVE LISTENING AND VAUDATION SKILLS)

IDENTIFY DIMENSION OF PRESENTING


PROBLEMS. INCLUDING CRISIS PRECJPITANTS

ESTABLISH RAPPORT AND COLLABORATIVE RELATIONSHIP

PLAN AND CONDUCT CRISIS AND BIOPSYCHOSOCIAL ASSESSMENT


(INCLUDING LETHALI1’! MEASURES)

Roberts’ seven stage modet of crisis intervention


Communication Strategies In Crisis intervention

Using silence gives the person time to reflect and become


more aware of feelings. Silence can prompt elaboration. Simply
being with the person can make them feel supported.

Using non-verbal communication- maintaining eye contact, head nodding,


caring facial expressions, and occasional “uh-huhs lets the person know
that you are in tune with them

Paraphrasing, expressing understanding, empathy and interest are conveyed by


repeating portions of what the person said. Paraphrasing also checks for accuracy
,clarifies misunderstandings, and lets people know that they have been heard.
You could say, “So you are saying that..”, or”Aap k kehne ka matlab ye hai kaL.

1efiecting feelings helps the person identify and


articulate emotions. You could say. “You sound angry...” or “You look scared...”

Allowing the expression of emotions is an important part of healing.


Venting often helps the person work through feelings
and helps in problem solving

Disaster Management:
A crisis involves three main phases: emergency phase, rehabilitation phase,
and recovery phase. Each of these phases has its unique characteristics.
The common factors for a medical student to remember regarding
disasters include:

a) The consequent trauma is never surgical and medical atone: nearly


all those affected suffer psychosocial changes.
b) Most psychosocial consequences of trauma are essentially normal
reactions to the overwhelming nature of the disaster. More people
dont get PTSD than do get PTSD.
c) Groups most vulnerable to deyeloping post traumatic conditions
include women, children and the elderly, but young adults and
males are not immune to developing psychological reactions.
d) Provision of early psychosocial support by trained professionals
prevents long term psychiatric morbidity. It also enhances the
impact of surgical and medical interventions and promotes early
recovery from the physical, psychological and socioeconomic
effects of trauma.
e) All medical and psychosocial care should be part and parcel of the
larger disaster relief in form of food, shelter, clothing and security.
Psychosocial and mentat health care should be made an integral
part of medical and surgical care. This will help to prevent stigma
of mental and psychosocial issues. It also helps to ensure a holistic.
biopsychosocial model of health care delivery
fl Traumatised individuals and communities best recover through pro
viding psychological, social and economic support to each other
(rather than relying on outside help alone). An early return to their
homes or shelters close to home and active participation in social,
educational, economic, and reconstructive activities ensure a
quicker rehabilitation.
H

g) The use of psychotropics, particularly. the benzodiazepines should


be avoided and simple. evidence-based, culturally rooted, non-phar
macological interventions are preterabte. Only short-term use of up
to two weeks for morbid anxiety and insomnia may be advised.
h) Rescue workers and health professionals involved in provision of
relief work require psychosocial support. They need adequate rest
and recreation as well as constant appreciation and patronage. They
should work using the buddy system, which involves individuals
teaming up and caring for and monitoring each others psycholog
ical and physical wellbeing. Preferably they should remain in touch
with their families and friends back home. This prevents early
fatigue. burn out and long term psychosocial complications.
i) Up to one third of the affected population may develop long term
post traumatic conditions characterised by disabling flashbacks of
the trauma, autonomic hyper-arousal. avoidance (of cues, settings
and circumstances that are [inked in anyway with the traumatic/di
saster event), anxiety, depression and dissociative states.
j) A second disaster wave hits soon after the first. This is largely in the
form of epidemics, wound infections, malnutrition, death and dis
ease due to exposure to extremes of temperature.

7. Conftict Resolution
Conflict is a state where two forces oppose each other. Conflicts arise in
situations where individuals and groups are not getting what they want
or need. This includes marital conflict, conflict amongst colleagues, the
attendant of a patient and the nursing staff, medical students on a clini
cal rotation in conftict with hospital staff, or the college administration etc.
Conflicts are inevitable situations and are usually seen where there is poor
communication, power seeking. dissatisfaction with management style.
weak leadership. lack of openness and change in leadership. Conflict has
the quality to divert attention from the main activity, undermine morale,
polarise people and groups, reduce cooperation, sharpen differences and
thus Lead to irresponsible or harmful behaviour. It is, therefore, important to
understand that at times the individuals involved may be unaware of their
needs or wants. Conflicts have the potential to be constructive when they
are raised in the spirit to clarify and solve problems. In these circumstances
conflict and timely resolution may help relieve tension and pent up
emotion as well as help build cooperation through learning more about
each other.
a) Common Causes of Conflict in Heatthcare Settings:
i) Assumptions are being made e.g. the doctor assumes that the
patient knows that his absence from the ward is on account of an
unavoidable academic commitment like attending an international
conference. The patient instead may not be aware of the activity or
may not attach the same importance to it as the doctor.

ii) Knowledge is minimal e.g. the family has inadequate information on


the indication of biopsy in a patient and may see it as a suspicion of
‘St Las/a Zombie
malignancy in the patient.
___________
__

CAUSES OF CONFLiCT IN
iii) Assumptions are being made e.g. the doctor assumes that the NEALTHCARE SETTINGS
patient knows that his absence from the ward is on account of an
HEALTH OUTCOMES
unavoidable academic commitment like attending an international
conference. The patient instead may not be aware of the activity or PERFORMANCE
may not attach the same importance to it as the doctor.

iv) Knowledge is minimal e.g. the family has inadequate information on


the indication of biopsy in a patient and may see it as a suspicion of
malignancy in the patient P.ERK5ANOPPWEXGE

v) Expectations are too high: e.g. the patient believes that a course of qUA#voPuFg V V

chemotherapy should have completely cured him of his lympho


ma, when instead s/he develops a complication of the treatment
and deteriorates.

vi) Personality, race, gender or social class differences exist e.g. a


trainee nurse may have a tow frustration tolerance and take offence
of an innocent remark by a patient; a visitor or a medical student or
the janitorial staff goes on a strike for being poorly paid.

vii) Needs and wants are not being met e.g. a patient dissatisfied with
food, bedding or facilities in the ward.

viii) Values are being tested e.g. a welt-clad female patient reluctant to
allow a male student to examine her.

ix) Perceptions are being questioned e.g. a confident medical student


distressed about not being given a chance by the surgical resident
to undertake an incision and drainage procedure independently.

b) Methods of Conflict Resotution:


The underlying emotion in all conflicts is bottled up anger, frustration
and/or an impression of being ignored, or of being ‘taker for granted’.
The most common underlying cause is often not a clash of interests but a
faulty communication or unfounded concerns. The worst ways of dealing
with conflicts is to brush them under the carpet, or to ignore or postpone
their resolution.
A formalised strategy to resolve conflicts is called Organised Conflict
Management (0CM). The following steps of 0CM help resolve most
conflicts readily:
/
i) Meet conflicts head on.

ii) Show mutual respect by separating the person(s) from the problem.
Do not try to corner, attack or undermine the individual(s) involved
in the conflict.

iii) Set goals that lead to a win-win situation for both the parties in
conflict rather than a victory of one party at the expense of the
other.

iv) Resolve the conflict through free communication.

VV._ VV1
v) Be honest about concerns and reservations and verbalise them as
early as possible.

vi) Agree to disagree. as healthy disagreements lead to better


decisions.

vii) Leave individual egos out of negotiations, and avoid serving or


pleasing one individual. Aim at satisfaction of the group, rather
than the leader alone. Exploitative and deceptive methods of
resolving confticts can succeed temporarily but are Likely to
generate bigger issues in future.

viii) If you are the one coordinating the dialogue, tet the negotiating
team create solutions rather than handing over the solutions -

people support what they create.

ix) Discuss differences in values openly.

x) Undertake a deeper anaLysis of the situation that generated the


conflict. Conftict resolution based on a superficial analysis is likely
to result in a bigger conflict in future. A conflict arising out of hurt
feelings, and emotional reasons is likely to settle on its own with
passage of time and an improved communication between the
parties. A conflict that arises out of morality issues, religious
differences, or cultural differences is unlikely to resolve. Here
the two parties can agree for a peaceful co-existence in spite of
the conflict. Realistic, fact based conflicts can be best resolved
through creative solutions put forth by the parties themselves.

The best method of dealing with conflicts is, however, by preventing


them. A sound management system in medical colleges. departments,
wards and hospitals helps to ensure this. It involves mechanisms of quali
ty control, free horizontal interactions and sharing of information. Leader
ship should be based on principles of following a middle path rather than
extreme measures. Ensuring equity and justice and imparting feelings
of security and predictability can prevent the rise of conflicts as welL as
ensure their early resolution.

Do’s and Dont’s In Crisis Intervention


Do Say: Dont Say:
These are normal reactions to ft could have been worse
an abnormal situation

You can always get another


It Is understandable that you i this W8
car/house or have another child

It was not your fault; you did the best you could It is best if you just stay busy

lam sorry that this happened I know just how you feel

Things will get batter, and you will I You need to get on with your life
feel better, although things may
never be the same again I
Empathy
The single thread that tinks alt the above non-pharmacological interven
tions is the demand on the doctor to empathise with the patient and the
family. The most important step in building a therapeutic bond is the doc
tors ability to experience the feelings of his patients and to gain a deeper
understanding of their distress, disease or disability.

Most medical students start their career in medical college with a huge
capacity to empathise. Alt that they have to learn is to communicate it
effectively. The biomedical modeL with its emphasis on the disease, rather
than the person experiencing it provides few opportunities to develop
and use this skill. Medical students, eager to perform well, are rewarded
for their abiLity to memorise anatomical and biochemical facts, causes of
diseases and classification systems. Their ability to empathise or relate
with patients at a human level is not marked, rewarded or appreciated.
As a resu[t of this, slowly but surely they start to focus more on acquiring
knowledge, with their skill at treating patients as humans and empathis
ing with them fading into the background.

The best time to learn how to empathise is in your relationships with each
other as medical students. The first step in this direction is to opt to study
in a group rather than alone. Once you are part of a group, try and under
stand the reactions of a fellow student who is struggling with language.
or a concept; who fails in a class test, a sub-stage or a viva. Sitting next to
someone who has failed, or is in pain, and thinking of how s/he is feeling
is an important exercise by which you can eventually learn to empathise
with patients. Let the person you are trying to empathise with, express
their feetings. The best technique in this pursuit is to share their silence.
Sit quiet. Listen actively: let the person know, that you care and it is ok
for them to share feelings with you. This effort on your part to empathise
with your colleagues in the first couple of years in medical college will
make you comfortable with your own world of emotions. It is this import
ant ability in a human to stay in touch and be aware of one’s own feelings
that helps them to relate with feelings of others and thus enhances their
ability to empathise. During clinical years, try and sit with patients, even
after you have taken the history and have completed the clinical exam-.
nation. Encourage them to talk about how they feel in reaction to their
illness, hospitalisation, and treatments being offered to them. Share their
fears, disappointments and sorrow without trying to take sides of the
health professionals and hospitaL authorities. Ask questions about the
influence of the disease and the treatment on their life at home, at work
and in general. These apparently irrelevant” steps will take you ctoser to
your patients and thus increase the chances of empathising with them. It
is this ability to bond, and eventually feel the way your patients feet, that
wilt help you have an insight into how patients think.

i
SAMPLE MCQ FOR SECTION A

1. A patient from a vilLage in ChoListan desert presents to a health


professionat in Lahore. He appears to be suffering from diabetes
mettitus, according to his HbAic and fasting bLood glucose reports.
The heaLth professional does not speak or understand his tanguage.
The best way to provide informational care would be:

a) Drawing pictures
b) Using sign language to communicate effectively
c) Seeking help from a colleague who partially knows the patient’s
language
d) Exclude medical jargon from communication and state essentiat
facts through an interpreter
e) Make an attempt to learn patient’s Language and then communicate
effectively

2. During a counseLling session, the most important aspect is:

a) Charismatic personality
b) Asking why the patient feels the way they do
c) Speaking to the patient in their language
U) Unconditional positive regard.
e) Empathising with the patient’s situation

3. Active Listening is best accomptished by the doctor understanding


and using:

a) Body language.
b) Paralinguistic aspects
c) Active prompting
d) Adequate eye contact.
e) Responses to open ended questions.

4. White deaLing with a patient who is fearfuL about not waking up from
anaesthesia for her hysterectomy, an empathic response is:

a) I assure you that your concerns are not scientific, everybody wakes
up from anaesthesia
b) I do understand your concern, in your situation I, too may have felt
the way you are feeling
c) I know that you are scared but you are a brave person who can face
this
d) We are experts in the field; we will make sure that nothing happens
to you.
e) Please relax, everything will be fine
5. Effective communication skills are considered essentially important
for a doctor. The most important reason for a doctor to develop
effective communication with his patients is:

a) It is a vital tooL in clinical settings.


b) It forms the basis of the interaction between the doctor and patient.
C) It is used in informationat care sessions with the patient and their
famiLies.
d) Doctors can convey their message on various health topics better.
e) Doctors will gain respect and popularity with the public.

Sampte Short Essay Question For Section A

Qi. Briefly describe the seven questions that need to be answered in an


informational care session.

Q2. What are the steps invoLved in breaking bad news to a patient?

Answers

l.a
2.d
3.b
4.b
5.b

For answers to short essay questions see Chapter 3, Non-pharmalog


ical interventions.
b
ECTION B
edical Ethics, Professionalism and Doctor-Patient
elationship

Retevance of Ethics in the Life of a Doctor


Medical. Ethics
Ethical. Omissions in Medical. Practice
Ethical. Dil.emmas in a Doctor’s Life
Doctor-Patient Rel.ationship
Ptofessiona[ism in Heal.th Care

I sb,ear [‘p scuLapus, Lpgeta. anb anacra. anb 3 take to b%tite%%


all tfje gobs, all d gobbeaea, to keep arcorbing to mp aWftp anti nip
ubgmeiu, tt (otLobng atfj. to con%iber bear to me aS mp parentS
tdm to(io taug me tl.ü art: to Cibe m cmnmmt it im anti if nws%arp
to sbare nip goats tnittj m: to took upon his cljilbrcn aS nip aton
.—c beatberL to teach tm this art it tfrp so bcsirr k%tfjmd fee or britten
omt%e; to Impart to nip sans anb tbe SonS of ttje master [‘ibjo taughi
me anti tk bisnptes bfjo babe enrotkb ttjemwetbes anti babe agreeb to
tlje rute% of the profeSSion, but to these atont the prtce5 anti tljt
instruction. I UnIt prescribe rtgimtns tar the goat of nip patients
berboarmloanpcnc
,, acarbingtompabititpanbmpiumeiUanbne
r to please no one tofU I prescribe a beatitp brug nor gibe abbece Wijicfj
map cause $S tieatlj. lor bsI( I gibe a boman a pessarp to prorure
abortion. ut I bitt preSerbe the puritp of nip (itt anti nip art. I bill
nof cut tar Slant, tUtu for patients in bhom the tüea%t is manitt%t
I bill (taUt tts operation to be performeb [‘p practitioners, sptclat&s
arLInebtrpbousebfrrticome)bstt.tronlptrxdgxb
at mp patients. keeping mp%elt far tram alt intentional Ilt-boing anti
Th altsebwtionanbesptdaltptromtttp(easureso((bstfjbionietior
beth men, be the? fret or stabes. Alt that map come to mp knobeebgey
tn the exercise of mp profession or In baitp commerce bath mni Wjlcfj
ongljnoftobesprtabaheoab,3bittkeepftcrtianbbellntbertebeaL -

It I keep (tO oath taithtuttp. map 3 tnwp nip tite anti practice nip ..

art. etsptcteb bp alt men anti In alt times; but it I s’wtrbe (rain U
I map the reberft be mp tnt.”

- -. --— .-

7
Medical Ethics and Professionalism
After ordering a hot cup of Doodh Patti, the tocat preferred version of English
tea Ahmed, Fazat, Javed and Safdar started their evening chat. The topic
today was not potitics but the attitude of doctors.
Ahmed remarked ‘The new Doctor Saheb who has opened his ctinic is very
different from Dr Raheem who died tast year after serving the community as
a generat practitioner for thirty years. Dr. Khatid has a neon sign ofhis name
and qualifications outside the clinic. He runs his clinic more like a ‘health
shop He asks his patients to deposit a fixed amount with him prior to the con
sultation, irrespective of their financial status. The other day Dr Khatid insisted
that I shoutd get the Hepatitis vaccine whether I like it or not without giving
me a choice to do so.”

Safdar quickly added: “Dr. Saheb removed my appendix in surgery, a few


weeks ago, after telling me that the appendix would burst if I did not get it
operated there and then.”

Javed recalled the differences between the two doctors: “Dr Raheem was a
very kind man. He never charged the poor He always discussed matters with
his patients.” He cited Dr Raheem’s gesture of explaining atl the advantages
and disadvantages of contraceptives to his wife and then asking her to make
a choice, before he put heron the contraception pilL Javed then came to Dr
Khalld’s rescue and remarked, “Yaar, everything and everybody has changed,
how can doctors be the same?! Dr. Khatid needs a much larger amount of
money than Dr Raheem to run his home and family His family sold a large
piece of land to afford his medical education and is still under debt, while Dr
Raheem went to a medical college with hardly any expenses involved. He is a
fine surgeon and knows much more than us about illness; why should he ask
us about medical matters as long as he means well?”

Fazal had another story to telL “Dr Khatid gladly accepted a new mobile
phone from a female patient so that she could call him for telephonic advice.
He also went on a holiday to Bhurban with his family and a friend, with all
expenses paid by the pharmaceutical company where lam emptoyed.”
Javed again came to Dr Saheb’s rescue, “But then he never charges any fee
from the medical students and his colleagues and recently appeared on the
television channel and gave free advice on important health matters.”

Ahmed concluded the discussion by saying “Dr Khatid is like all of us; he has
his positives and has some negatives too. Hebannot be compared with Dr
Raheem as the ethics of the medicat profession may have changed over the
last three decades.” Safdar remarked on his way out of the tea bar, “Let us
wait and watch Dr Khalid’s progress.”

F— -
ReLevance of Ethics in the Life of a Doctor
The discussion at the tea bar shows how doctors are regularly viewed and
critiqued in terms of their vaLue systems and behaviour by the community.
It also shows that doctors vary in their practice of ethical and moral issues.
This hightights the need for clear guidelines on how doctors are expected
to behave to be considered ethicat professionaLs’. The community puts
doctors on very high pedestals. They are expected to be kind, caring, and
hetpfu[. They are expected to be committed to heaLth provision and keep
the interests of their patients above their own. They must never harm
anybody, be just and equitable and show character and resilience. They
must also be able to communicate effectively, compassionately and
fearlessly. They are supposed to respect the laws of confidentiality when
it comes to their patients’ data. Alongside these expectations are the set
of laws of the state governing medical profession, and the regulations of
the Pakistan Medical and Dental Council, that a doctor must adhere to.
The behaviour of doctors is called upon to reflect the traditions and values
associated with them over centuries of the history of their profession.

Doctors face various ditemmas and difficult choices in their daily practice
such as taking consent from patients and families with low literacy and
inadequate understanding of health issues. They come across controver
sies such as abortion, euthanasia, human rights, and gender issues. They
atso deal with powerful sections of society while compiling medico-legal
reports. Their relationship with their patients, their families, the pharmaceu
tical industry, media and the challenges posed by the internet and modern
technologies and treatment options all raise ethical concerns.

ALL in atl, ethical principles are required for good medical practice and
come into play in almost all ctinical decisions that a doctor makes. They
remain under scrutiny even in their personat and private life. A good doc
tor is one who adheres to ethicat principles, regulations and customs of
his profession under all circumstances. S/he must, therefore, have a clear
understanding of what medical ethics are and what their scope is.

1. Scope and Meaning of Medicat Ethics


MedicaL ethics is the study of moral aspects of a doctor’s professional life
The two branches of ethics relevant to a medical professional are:

ETHICS

I I
DES IVE NORMATIVE
Whatwas,isorwllb , What ought to or shoId happen
What Is at What Is ideal
Normative Ethics: What heatth professional should do?
Normative ethics refers to what actions are right and wrong in principle, i.e
what the norms are. It serves to create moral standards that people should
foLlow. These provide the theoretical and ideaL framework that can guide
a doctor dealing with a practical problem e.g. Should a doctor be required
to take consent for surgery from an iLLiterate man, with the fear that the pa-
tient may make the wrong choice? (As was the case in the tea bar dialogue,
when Dr. Khalid choe to remove Ahmeds appendix without his consent)
Should public money be used to treat patients of drug abuse and AIDS?
Should the Population Control Division pay the bills of an employee
seeking a test-tube baby or in-vitro fertilization (IVF)?

Descriptive Ethics: What health professionals actually do?


Descriptive ethics are concerned with researching the morals, be[iefs and
behaviours people actually have. It is a study of the behaviours of health
professionals and what sort of moral values they follow. This involves var
ious medical issues and ethical dilemmas e.g. Should Dr. Khalid charge a
large consultation fee irrespective of the patients paying capacity?

2. Guiding PrincipLes of MedicaL Ethics


The guiding principles of medical ethics in the practice of a health profes
sion, also known as the ‘four pillars” are autonomy, beneficence,
non-maleficence, and justice.

a. Autonomy: This implies that it is the patients right to choose whether or


not to undergo a certain treatment. The doctor must give full
information such as therapeutic effects, risks, side effects, positives and
negatives to the patient. The patient should then be allowed to make a
meaningful decision without any external influence or compulsion. In the
tea-bar example, Dr. Khalid should have explained the risks and benefits
of undergoing an appendectomy to Ahmed. He should have then let him
make a choice about undergoing the surgical intervention. The underlying
principle of this approach is called informed consent (explained in more
detail later in this chapter).

b. Beneficence: This calls for all medical professionals to do good for all
patients under all circumstances, the same way as ordinary citizens are
required to do good for their parents and children alone. Doctors, therefore,
have a special relationship with their patients as they demand care from
them as a duty and an obligation. (Javed’s objection to Dr. Khalid’s choice
of charging the poor was based on the norm of beneficence).

c. Non- maleficence: This refers to the obligation a doctor has to do no


harm (or minimise it as much as possible) to his patients. It is an exten
sion of the principle of beneficence. This requires the doctor to protect
his patient against all forms of harm and always act in his best interest. Dr.
Khalid’s decision to protect Ahmed from the risks of an inflamed appendix
by operating on him even without his consent was an expression of this
principle.
The three principles listed above are accepted in legal terms as prima fcicie
(assumed to be correct until proven otherwise). They form the basis for all
other ethical guidelines, but none of those guidelines can be in contradic
tion to them.
3. Common EthicaL Issues in MedicaL Practice
a. Informed Consent and Confidentiatity
The two commonest ethical concerns for a physician are consent
and confldentiaUty. These are also the two paramount norms that
are most often ignored by physicians. Consent is omitted by most
doctors on account of two reasons. Firstly. the paternalistic attitude
of doctors thinking they know what is best for the patient. Secondly,
the common folly of thinking that an illiterate or an ignorant patient
may miss out on a cure if ‘crucial” time is wasted” in obtaining their
approvaL There is also a fear that the patient may say no’ when data
on the risks/hazards of intervention is communicated.
Confidentiality of patient is also regutarly compromised when we
share the details of the patient’s condition and clinical details with
his relatives, friends and ‘well-wishers’, without the patient’s prior
permission. The practice of undertaking interventions and heatth
decisions without informed consent, as well as sharing a patient’s
clinical data with anybody without his or her prior permission, are
unethicat.
What is Informed Consent?
Consent is the agreement by the patient to undergo an examination,
procedure, or treatment. It can be given orally, by signing a form or
a written statement or simply agreeing by a gesture e.g. offering to
pull up the shirt for letting the doctor examine the abdomen. It must
always be obtained in advance. In order for the consent to be valid,
the patient should be competent, that is. should have the capacity to
make a decision (free of a defect of mind or judgment and not be a
minor). The patient must give consent freely and voluntarily (without
coercion or threat). S/he must also be given options to choose from
and not fear rejection or neglect by the doctor in case he or she re
fuses to agree. The patient must be offered all the information that a
reasonable professional should have (prudent professional standard)
and as much as a patient would like to know (prudent patient stan-
dard). The minimum information required is, the risks involved, likely
success rate, the side effects, and a comparison with other options. In
Ahmed’s case, Dr. Khalid should have informed him of the risks of an
aesthesia, and abdominal surgery. The doctor must also ensure that
the information has been clearly understood. The whote process of
seeking informed consent must be made after building a trusting re
lationship with a patient. The patient should be able to see the doctor
as caring, competent and reliable.
Exceptions to the rule of informed consent are patients:
brought into emergency and accident departments
unconscious and alone
requiring life-saving measures,
children under 16 years of age (the parents have the right
to provide informed consent)
with an impaired capacity to give consent.
Consent is a reflection of the norm of autonomy highlighted above as
the foremost pillar of medical ethics.
Questions for Informed Consent
by Peachy Keen Birth Services
taceboak.com/Peachyteen6irth

Use your B.R.A.I.N.S.


When trying to decide if a precedure is right for you and your baby, here ore the questions
to ask to make sure you hove cii of the information necessary to make an informed choke.

D Benefits What are the benefits of this procedure? What are we hoping

U to achieve?
Risks—What are the risks or side effects? What other interventions will
R go along with this?

Alternatives What are our other options?


I intuition— Doesthe informationyou’rereceiving make sense. Doyou


I need additional info, or a second opinion?

[%J Nothing What if we do nothing, or wait a while before deciding?


S ‘Scuse Me — Can we please have some time alone to discuss and decide?

Informed consent shoutd answer these questions for patients

Confidentiatity: How much? When can it be breached?


It is the common law duty of a doctor to respect the confidence that
a patient has in him, Doctors are expected to preserve absoLute con
fidentiality on all that they know about their patient. This applies not
only during the treatment but also after it and even after the patients
death. This is a means of expressing respect for the patients right to
privacy and hetps the patient speak freely to the doctor. Confiden
tiality is the foremost part in the traditional Hippocratic Oath that alt
doctors are committed to. A doctor’s registration with PMDC can be
cancelled and misconduct charges in a court of Law can be pressed if
found responsible for an unauthorised breach in confidentiaLity.
There are instances, however, in which breaches of confidentiality
can be made:
What you
• When a patient authorises it in situations such as while say in here
seeking medical fitness to gain employment or getting a
stays in here.
medical report prepared for a second opinion. Patient has Unions:
the right to ask for a copy of the medical report. They must • Someone In honing yons
• You want to hurt samson.
always be explained the contents of the report and should • You ssant to hmtyosrsatt
• You glue nsa pornsi.sionto
sCans with a trusted adult
always understand what they have consented to.
• When information is to be shared within the health care
team
• When the disclosure is in the best interest of the patient, as
in the case of physical or sexual abuse, where
confidentiality would mean a continuation of a perpetual
hazard. In case of a terminally ill patient who does not wish
to know the prognosis fully, the doctor may make the family
or next of kin aware of the outcome, with the patients
consent.
• When the disclosure is in public interest and the doctor’s
attempt at holding back information may harm members
of the society. This includes instances such as reporting
the matter to ticensing authorities when a patient is unfit to
drive, or has a sexually transmitted disease e.g. is HIV
positive and the wife is at risk. In all such circumstances.
the patient should first be persuaded and counseLed to
report the matter themselves.
• In case of a Legislative requirement. This refers to matters
of Laws on Public Health and control of diseases such as
chotera, small pox, plague or venereal diseases.
• For research purposes and case reporting as anonymous
data, or if the identity of the patient may be surmised from
it, after the patient’s consent.

Medicat Students and the Issues of Consent and Confidentiality:


Medical students may only be called to observe a clinical interac
tion after the permission of the patient has been sought in advance
and not after a patient walks into a setting where the students are
already seated. A medical student may onty undertake a procedure
after an informed consent has been taken. The patient must be
briefed about the training level and status of the student. Adequate
safeguards should always be put into place in all such settings
and the patient duly informed about them. As regards confidenti
ality, medical students have the same duty as the treating doctor.
They must adhere to the same principles as those outlined for the
members of the health team. Medical students may themselves be
required to give informed consent if they opt to serve as healthy
volunteers in research projects. The Dean/Principal of their medical
cotlege must be kept informed in all such matters.

b. Decision-Making Capacity
Capacity in health ethics refers to the ability of the individual to
understand the nature of their illness, the treatment options and the
consequences of the decision. All adults are assumed to have the
capacity to make decisions about their health and treatment options.
This capacity may be impaired in certain conditions. A psychiatric
consultation is not necessary to estabUish capacity of every adult.

It is incorrect to assume that alt patients with psychiatric disorders


lack this capacity at all times. Patients with psychiatric disorders or
head injury must be clearly shown to have impaired judgement and
inability to weigh options at the time of decision making. Patients
with neurological illnesses like dementia do not automatically lack
this capacity till late in the progression of their disease. They can
make informed decisions in early stages of their illness and should
be encouraged to do so by the family and the treating doctor.
Next of kin may make decisions about a patient in a coma or veg
etative state. The decision of continuation of life support in a brain
dead patient, however, needs to be made by the medical authorities
in collaboration with (but not necessarily with the consent of) the
family of the patient.
With regards to minors, parents have the right to make decisions on
behalf of their children. It may be inappropriate to have the same
rute for teenagers. The physician should, however, encourage teen
agers to participate in the decision making process and have a say
along with their parents.

Religious beliefs of an individual cannot be trespassed in informed


decision making. All due respect should be shown to a decision
of refusal of treatment on the basis of a religious belief. An organ
transplant or transfusion may be refused by a patient on the basis of
his beliefs. Parents do not, however, automatically have the right to
use their belief system in decisions about their children’s health and
treatment options. A court can authorise a decision in their place in
situations where a conflict arises on this basis.

Can an individual make advance directives regarding health-related


decisions? The answer is yes. This can be done by making a Living
witt. In this the individual makes their own decisions regarding treat
ment choices they would or would not want in case their capacity
becomes impaired. This option is particularly useful in decisions
related to resuscitation, organ transplant, dialysis or organ donation.
Patients may Leave specific instructions regarding cardiopulmonary
resuscitation as Do Not Resuscitate (DNR), Comfort Care (a general
withdrawal of life prolonging care in favour of the patients care), and
Palliative Care. This can also be done by proxy, where the individual
nominates a person who will decide on his or her behalf once he or
she loses capacity to do so. Legally this is referred to asa power of
attorney).

The ethical dimension to be kept in mind is to give maximum control


to the patients during the times when they can make sane, rational,
and objective decisions about their life and death.

c. Euthanasia
Euthanasia is when a physician administers a Lethal drug to a patient,
with the patient’s futl consent and voluntary cooperation. Euthanasia
must be distinguished from Physician Assisted Suicide. Physician
assisted suicide refers to where the physician dispenses (but does
not administer) a lethal drug to a patient with intact capacity for
the purpose of they themselves bringing an end to their life. Both
are considered illegal and unethical in our setting. Certain Western
societies have sanctioned voluntary euthanasia with strict controls in
ptace.

U. Malpractice
In order to state that a health professional has indulged in mal
practice, it must be established through adequate and sustainable
evidence that the physician has wronged a patient and/or harmed
them. It must be shown, however, that the physician had known
better. It implies that the health professional was negligent and did
not meet the required standards of practice. This includes failure to
undertake informed consent.
e. Inclusion of Patients in CtinicalTriaLs
ClinicaL tria[s may only be started after approval of institutional eth
ica[ committees. Patients can give informed and written consent to
participate in clinical trials after being provided due explanation of
details of the triaL Patients must have the option to opt out of the tri
al at any stage. It is obtigatory for the principal investigator to predict.
be aware of, and inform the patient of any dangerous consequences
of the triaL

4. Common Ethical Ditemmas in a Heatth Professional’s Life


On account of their unique position in society, doctors have access to the
most intimate areas of peopLe’s lives. They are, therefore, likely to come
across unusual social situations. The doctor is not expected to respond to
these situations like other members of the society and a minor stumble on
their part may compromise their position. Some of the common situations
where he needs to show prudence and not take the obvious course are
discussed below.

a. Euthanasia
Euthanasia or physician assisted suicide is considered one of the
most prevalent problems when dealing with the ethics of patient
management. A worldwide debate continues to rage on the subject
of the ‘right to die.” Should people have the right to end their own
tives when prolonging it will only cause them more pain? Should
families who love someone so much that they don’t want to lose
them continue to cause them more pain by keeping them alive?
From the Greek term for “good death”, euthanasia means com
passionately allowing, hastening or causing the death of another.
Generally someone resorts to euthanasia to relieve suffering, main
tain dignity and shorten the process of dying when death appears
inevitable. Euthanasia can be voluntary if the patient has requested
it or involuntary if the decision is made without the patient’s consent.
Euthanasia can be passive simply withholding heroic life saving

measures or active deliberately taking a person’s life. Euthanasia


assumes that the intent of the physician is to aid and abet the pa
tient’s wish to die.
Most of the medical, religious and legal groups in both the United
States and UK are against euthanasia. The World Medical Associa
tion issued the following declaration on euthanasia in October 1987:
‘Euthanasia, that is the act of deliberately ending the life of a patient,
even at his own request or at the request of his close relatives, is un
ethical. This does not prevent the physician from respecting the wilL
of a patient to allow the natural process of death to follow its course
in the terminal phase of sickness.”
It should be noted that the Pakistan Medical and Dental Council
also holds the same view on euthanasia. Practice of euthanasia by a
doctor is considered a criminal act.
b. Accepting gifts from patients
Sharing of gifts as an expression of gratitude is a common norm in
nearly all societies, especially ours. In certain subcultures, in fact, the
gift giver may feel insulted if his offerings are not accepted. A clear
set of guidelines should therefore be fottowed by health
• professionals which may then become a well-known custom of the
medical community in the society. Citizens would then also
gradually start to follow these customs.
A safe recommendation in this regard is to accept a parting gift at
the end of a successful treatment, as long as it is in form of a bou
quet of flowers, a box of sweets or chocolates. You may accept this
graciously. Patients who bring gifts during the treatment may cause
problems. It may be an expression of the patients need for “more
than usual” attention. They may be interested in developing a per
sonal friendship, or being part of your non-professional life. Extrav
agant and expensive gifts must never be accepted. This is because
they signify that the patient is putting you under a heavy obligation
or has elevated you to an extraordinary pedestal. Both scenarios can
land the doctor into serious trouble in the long run. If a patient does
so it is safe to return the gifts saying “I will not be able to accept this
gift, as it is against my professional ethics. I assure you that my care
and concern for your health wilt continue to remain the same.” In
the scenario discussed previously, the patient who brought a mo
bile phone for Dr. Khalid may have an agenda beyond the obvious
meaning of taking medical advice readily. Dr. Khalid should have
politely refused the gift and reassured the patient of his availability
as and when required to provide professional advice, preferably in
person.
c. SexuaL boundaries violation: sexuat retationships in medicaL setting
Doctors operate in odd hours, in close and sometimes intimate
settings for long hours and without clearly defined boundaries of
age, gender and social class. They work with fellow, senior and
junior colleagues, nurses, paramedics, patients and their families,
and visitors. They may also become associated with professionals
from departments of sociology, social work, psychology, NGO5, the
pharmaceutical industry, and other related organizations. Alt forms
of liaisons and relationships involving personal intimacy of a sexual
nature in hospital settings are considered unethical and illegaL This
is to protect the sanctity of the medical profession and the hospital.
A sexual liaison between a patient and his or her doctor is prohibited
by law and the regulations governing the profession, the world over.
At a psychotogical level such a relationship is considered at par with
incest. The same rule applies to a medical student or any health pro
fessional working with a patient. Patients are vulnerable to develop
ing an erotic attachment with their doctor, a medical student or any
health professional involved in their care and may even declare their
passion. This can be handled by explaining in no uncertain terms
that it is impossible for you to continue as their care provider in such
a situation. Medical students themselves run the risk of being ex
ploited by senior professionals and even teachers in the hospital and
college settings. They must always report the matter to the Dean or
Principal and to do so with immediate effect,
without fear and prejudice. It is useful to remember that a predator
or exptoiter who threatens dire consequences if you inform a
concerned authority is essentially a coward. Never feel fearful or
overwhelmed by such an individual or a group.

d. Charges and Fee: Patients, CoLleagues, Teachers, Medical Students


The medical profession has traditionally been a service with no pri
mary commercial interests. With the privatization of health services
and involvement of health insurance in some capitaList countries,
the delivery of service is nearly always associated with financial
transactions, atbeit not always directly at the point of delivery. It is
not unethical to charge a fee for a consultation, procedure or an
intervention: it is against the customs and norms to base the doc
tor-patient relationship on their capacity to pay. The charges should
never be extravagant, or vary from one set for the poor and another
for the rich. A simpLe ethical rule is to determine a fee structure that
does not render a doctor, a procedure or an intervention, beyond the
reach of an average citizen. Traditionally, doctors are not expected to
charge their coLleagues, teachers, medical students or the extreme-
ly poor.

e. Retationship with the Pharmaceuticat Industry


Doctors and the pharmaceutical industry need to have a congenial
but professional relationship. This must be aimed at mutual pooling
of resources to promote welfare of health institutions and patients
and investment in research. The use of this relationship for personal
gains or profiteering is unethicaL. It is unethical for doctors to seek
financial assistance from the pharmaceutical industry, for travels
abroad, material benefits for themselves or their families. They may,
however, seek support in scientifically valid research pursuits provid
ed the research is not aimed at promoting a particular product of the
sponsor. They may also receive travel grants from the sponsor if they
are traveling to present findings of this research on an academic
forum. Grants from the pharmaceutical industry towards setting up
or improving a health facility, or a service exclusively for the welfare
of patients are a[so acceptable.
Prudent use of pharmaceutical human and material resources to
improve health literacy amongst patients and the community could
be an ethical and useful pursuit. The pharmaceutical industry human
resource and customised software may be useful in improving treat
ment adherence.
A doctor in particular and all health professionals in general must
always guard against becoming biased by the promotional literature
distributed by the pharmaceuticaL representatives. The preferred
resource for information regarding a particular drug should always
be peer reviewed medical literature. Health professionals must aLso
refrain from luncheons, dinners and meetings held at holiday resorts
or hotets under the cover of academic activities such as panel
discussions’ and lectures etc. Dinner and Lunches with sponsored
educational talks, or for the inclusion of patients in clinical triaLs, are
not unethical: however a professional needs to use his or her own
prudence as regards the reaL agenda of such a sponsorship.
Medication samples can be accepted only for the exclusive use of
deserving patients on the basic ethics principle ofjustice. These
samples are not for the use of physician’s family and friends.
Health professionals walk a tight rope in their relationship with the
pharmaceutical industry. If used prudently this relationship can en
hance the image of the medical profession and bring great advan
tages for research and patient welfare. A minor slip on the doctor’s
part or an overenthusiastic nonprofessional interaction can, however,
cause him great harm. The single driving force for doctors when
they prescribe drugs has to be the benefit of their patient. The doc
tor under all circumstances must prescribe those drugs and inter
ventions that are the most efficacious, cost effective and supported
by most evidence-base.

f. Media and Medicine


The last two decades have witnessed a rapid growth of print and
electronic media. This has resulted in our excessive dependence
on the media for all kinds of information. Whether it is the latest
development in politics or the day’s weather, the addictive nature of Artist Laura Zomt

media technology keeps most people glued to their TV or computer


screens for considerable lengths of time each day. The news media
are also an important source of information on health and medical
therapies. There is, however, widespread concern that some media
coverage of scientific issues may be inaccurate and over-enthusias
tic. Journalists and media managers have been criticised by scien
tists and physicians for misleading the public over important medi
cal issues. A 1997 survey of scientists found that the majority of them
believed that reporters do not understand statistics wetl enough
to explain new scientific findings, do not understand the nature of
science and technology and are more interested in sensationalism
than in scientific truth. For instance, sensationa[ised reports on the
hazards of calcium channel blockers may have led some patients to
stop taking their prescribed antihypertensive medications, while op
timistic coverage of stem cell therapy resulted in patients requesting
this unproven treatment. Similarly the internet hosts thousands of
web pages offering all kinds of medical advice to the vulnerable
user. \5Vhere e-medicine and e-consultations have effectively by
passed the hurdLe of distance, they still remain vutnerabte to ex
ploitation by quacks and impersonators. Despite the controversy,
an effective and judicious use of electronic and print media along
with internet can be made in regard to prevention of illnesses and
promotion of health. The concept of e-Health offers many opportu
nities for prevention, choice, home based care, and chronic disease
management, and it has the potential to widen access to health
care for most patients. Continuous Medical Education fCME) is being
effectively imparted to a wide section of medical professionals
through authentic medical websites such as ‘Medscape’ and video
teleconferencing is an effective tool for sharing information through
long distances today.
The use of electronic and print media to improve health literacy is a
noble public health pursuit. Such opportunities do, however, run the
risk of misinformation. They also have the potential to be used
?‘

for personal projection by hea[th professionals. Patients and their


families may expLoit such opportunities for seeking free advice with
adverse and dangerous consequences. Health Literacy programmes
cannot be used for advertising a particular drug or intervention or
for provision of specific advice to viewers or callers on the show. The
use of internet and eHealth can be brought to ethical use through
organizing health promotion seminars and video conferencing to run
CME (contipuous medical education) and CPD (continuous profes
sional development) activities.
g. E-Consuttations and Tetemedicine
E -consultations and telemedicine has become a reality. Telemed
icine is currently linking far flung areas and cities like Gilgit and
Skardu with Islamabad. The phenomena of telemedicine is being
practiced without boundaries around the world, and with video call
ing features available in every social media app, it is likely to spread
even more. Surgical interventions can now be monitored by experts
in the field from a distance through video links, especially in war torn
areas.
The appearance of doctors on television channels is an important
way to promote health Uteracy. It is, however, unethical to attempt
to diagnose and treat patients during these interactions. Medicat
advice should not be given out to peopte cat[ing in” to television
shows. It is also not ethically correct to attempt to diagnose and
treat patients on video calls.
h. TechnoLogy-assisted Medicat services
E -consultations and telemedicine has become a reality. Telemed
icine is currently linking far ftung areas and cities like Gilgit and
Skardu with Islamabad. The phenomena of telemedicine is being
practiced without boundaries around the world and with video call
ing features available in every social media app, it is likely to spread
even more. Surgical interventions can now be monitored by experts
in the field from a distance through video links, especially in war torn
areas.
The appearance of doctors on television channels is an important
way to promote health literacy. It is, however, unethical to attempt
to diagnose and treat patients during these interactions. Medical
advice should not be given out to peopLe “calling in’ to television
shows. This is ethically incorrect.
i. Declaration by a Medicat Student or a Trainee Heatth Professional
It is mandatory for all medical students and trainee health profes
sionals to declare their exact role and identity. It is unethical for
medical students to pose as junior doctor’ or doctor on duty’ in front
of an ignorant patient or a family member. Trainee psychologists,
clinical psychologists and other health professionals working in
hospital settings can be mistaken for doctors. They often do not
object when addressed by the patients or their families as ‘doctor.
This tendency must be curbed, in favour of revealing your exact
identity and role in the health team and taking pride in it. Most
patients enjoy talking to medical students and trainees from other
health disciplines as they have more time for them and are more
interested in their condition
5. Doctor-Patient Relationship
If you have come to hetp me
You are wasting your time
But ifyou hove come because
Your tiberation is bound up with mine
Then let us work together

- An Australian Aborigine’s statement to a Doctor

The basis of the unique relationship between doctor and patient is the
capacity of the doctor to appreciate the complexity of human behaviour. A
doctor must be sensitive to the effects of history, culture, and environment
on his patients. At the center of this therapeutic retationship is the trust that
a patient has in the doctor, This trust is built on the unconditionaL positive
regard that the doctor holds for the patient, irrespective of their gender,
social class, caste, colour or creed. The bond that forms in the relationship
can take three forms:

The vertical model, where the doctor completely takes over the
process of care with the patient having virtually no role e.g. when a
patient is unconscious, immobilised or in an altered state of con
scious, or is incapacitated.
The teacher-student model, where the doctor plays a roLe similar to
that of an authority figure (such as a teacher or a parent), who dom
inates, controls and guides the patient e.g. in the case of a patient -

recovering from a surgical intervention.


The mutual participation, horizontal modeL where the doctor and
the participation behave as partners in the process of healing and
care. Each augments and supports the other’s effort. The patient in
this is fully aware and informed and plays an active role in the treat
ment process e.g. a patient of diabetes metlitus who understands
the nature of his/her illness and undertakes lifestyle changes to
manage it with the doctor’s cooperation. is an active partner in the
management process.
F,b

‘4(1’-”

ZL:Z
/
/

- j Eysenbach G, ]adadAR
; 2 Evidence-based Patient Choice
and Consumer heatth
informat/cs in the Internet age
] Med Internet Res 2001,3t2):e19

URL: http://wwwjmit
org/2001/2/e19
DOl: lo.2196/jmir.3.2.e19
PMID: 11720961
PMCID: PMC1761898

48
It is important to note that the relationship between doctor and patient
should be based on empathy, not on friendship or affection and love.
Such a model is not always unethicaL but may turn the relationship into an
unprofessional one with obvious repercussions and dangers. The major
dangers in this relationship include:

The doctor assuming the role of a savior and fantasizing that only
they can reEscue the patient from all the troubles of the world

The doctors inability to switch off and leave behind the patients
problems when away from the clinical setting.

A need to control everything in the patients life and to try and pre
vent death, which may not be possible in all cases.

A doctor visualising his or her own complexes and difficulties in the


patients issues e.g. a doctor having a disturbed relationship with his
father, assuming that the patient must also be going through such
difficulties or a doctor avoiding discussion of such an issue with
their patient even when it is of clinical significance.

The doctor becoming judgmental in the relationship with the pa


tient and starting to determine what he or she ought and ought not
to do.

An enthusiastic medical student handing over financial or material


help to a patient or seeking them from a patient. Both transactions
are against the norms of professionalism.

Rights and Responsibilities of Patients and


Doctors

a. Rights of the Patient


It is useful to have a list of rights that medical ethics grant to a patient. Doc
tors have a duty to communicate and assist patients in protecting these
rights. Patients have the right to:
• have informed consent taken from them. This is after they have been
educated about the illness and its treatment, alternative treatment
options and side-effects involved. Costs of the proposed treatment
and any further costs associated with rehabilitation and details of
support services must alo be communicated to the patient.

withdraw consent at any time.

refuse experimental or research treatment.

• obtain a second opinion.

• confidentiality regarding details of a condition and treatment being


maintained by medical and hospital staff.
• leave a hospital at any time (except in the cases of infectious disease
or certain psychiatric conditions). If the patient leaves against medical
advice, however, s/he is liable for any injury or illness caused, or ag
gravated by. the action.
‘! L’1’
Z/—

be treated with care, consideration and dignity.

• request medical files from the doctor.


Whon 14t WSflI .‘c— cJon,
• obtain legal advice regarding any matter arising from the treatment. wtflQ,nfl ID rDu.

• contact friends, relatives, soLicitors, members of the religious/faith


group or his or her wards if he or she is the parent or guardian.

• ask to stay with a child at all times except where separation is


necessary for medical reasons.

• inform nursing staff if he or she wants to or does not want to see or


speak to a visitor.

b. ResponsibiLities of the Patients


Besides their rights, patients have certain responsibilities, such as to:
• Know their own medical history including medications taken

• Keep appointments or advise / inform those concerned if they are


unable to do so.

• Comply with the treatment advised / supplied.

• Inform the doctor if they are receiving treatment from another health
professional

• Know how their charges of treatment are best covered.

• Conduct themselves in a manner which will not interfere with the


welt-being or rights of other patients or staff.

c. Rights of the Doctor


A doctor has the right to refuse to undertake an action, a procedure or an
intervention which is against his/her personal ethics or beliefs. S/he also
has the right to refuse to treat a certain individual (provided the individual
is not in any immediate Life threatening danger). In such an eventuality the
physician is under obligation to refer the patient to another professional.

d. ResponsibiLities of the Doctor


It is a doctor’s responsibility to provide scientific information on disease,
diagnostics and treatment options available to the patient. S/he is also re
sponsible for addressing a patient’s concerns and taking informed consent
for all therapeutic actions. Doctors must respect the patient’s decisions
even when they are in disagreement with them. They must uphold the
interest of their patient above their own (fiduciary). They must also never
use their authority in any cause other than the best interest of the patient.
Miss X, a twenty year old student from the local intermediate cottege, was
admitted in a long stay medical ward undergoing treatment for pulmonary
tubercutosis. Mr K a third year medical student was assigned to take a de
tailed medicat history and examination. During the course ofhistory taking,
Mr Ynoticed that only after a few minutes ofinteraction, Miss Xsat up in the
bed and became cheerfuL Even while describing the details ofher cough,
toss of weight and night sweats she woutd smile. At the end of forty minutes
assignect for taking the aacount, Mls.XinsistecTthdt Mr. Yshould tal?e a bite
from the fruit basket or else take a cup of tea with her She mentioned that
some of the important details of the history have not been yet covered and
asked the medical student to come in the evening and she wilt be glad to
furnish further details. Mr Ywas impressed by the ‘unusuat’friendly attitude
of the patient He turned up in the evening, to find that Miss X was waiting
for him. She not onty gave a detailed account of her iltness but also reported
‘feeling much better”since her morning interaction with Mr Y. She told him
how impressed she was with the ‘kind and pleasant personatity” of Mr Yand
that she had never seen such a wonderful ‘doctor.” She asked for Mr Y’s mo
bite number so that she could talk to him when she felt distressed or unwelL
Mr Ygtadly agreed.
On his way back from the ward, Mr Ywas thinking about the “positive impact”
that he had on his patient He decided that he should visit her regularly during
his clinical rotation to “assist her feel better” Next day he bought fruits,juices
and chocolates for the patient He did so secretively, so that the ward staff
and his batch mates should not ‘make stories’ or ‘get ideas The gesture went
down very well with Miss X In a few days, Ywas a regular visitor to Miss X
even after his batch moved on to the surgical rotation. Miss X would regularly
call on his cell phone to seek advice. She started to share her unhappiness
and her arguments with her mother and how much she missed her kind and
affectionate father whom she had lost as a child. Yin the meanwhile start
ed to advise her on social and domestic issues, how to deal with her “cruel
mother” He started to feel like the only ‘saviour’ and ‘friend’ in X’s “miserable”
life. When the ward staff began to object to his visits, Mr Ystarted to get into
arguments with them and insisted that he was doing so to keep the patient
happy and healthy. In a few days, Miss X was discharged from the hospitaL
One evening Mr Yresponded to a knock at his hostel room’s door to find
Miss Xstaning there with a bag in her hand. She told him that she had left
the home and her callous mother who wanted to marry her off to a cousin
against her wilL She stated that Mr Ywas the only one whom she thought
“really understood and cared”for her and could save her from the wrath of
her mother Mr Ywas flabbergasted and did not know what to do. He hurried
ly made her sit in the WRoom and started to insist that she return home. He
categorically stated that he could not really take responsibility of Miss X Miss
Xbroke down into tears initially and then started to shout and cry, accusing Y
ofmisguiding her, and saying he was “no different from the rest of the unkind
world.” Word started to spread in the hostel and the warden arrived. Miss X’s
famity was contacted and she was returned home with great difficulty. Mr Y
was severely reprimanded and for many weeks became the laughingstock of
his class. Miss X was admitted in the psychiatry ward with an attempt at de
liberate self-harm as she took an overdose ofher anti TB drugs and slashed
her wrists. Mr Ywas called by the treating psychiatrist, where he learned of
the psychological reactions that had led to his and Miss X’s ordeaL It took
many weeks ofintense psychotherapeutic work for the mental health team to
work through the issues. In the course of Miss X’s treatment and Mr Y’s coun
selling sessions, the phenomena of transference, counter-transference and
resistance became clear to Mr K
Psychological Reactions in Doctor-Patient
Relationship

The primary expectation of patients from their doctor is that they show
empathy, that is, understand their feelings, show kindness, interest, and
a non-judgmental approach. They also expect to be considered active
partners in care. In Pakistan the doctor is given the status of someone who
always makes the better decision for you in matters of health. This leads to
either feelings of sympathy (feeling and experiencing the emotions of the
patient) and over-identification with the patient, or distancing and isolation
from the patient. Both reactions on the part of the physician can make the
relationship complicated or take a turn that undermines professionalism.

There are a variety of social and psychological reactions in a typical sus


tained doctor-patient relationship. These phenomena are most intense in
psychotherapeutic interactions but can occur in any helping relationship in
a milder or a somewhat modified form. They are:
• social bonding
• dependence
• transference
• counter-transference
• resistance
• physician burn-out
These reactions occur in the mind of the patient as well as the doctor
without one’s normat awareness. They can help a doctor understand why
a certain patient is reacting in a particular way or why they themselves are
behaving differently with a certain patient.

a. Social bonding
Pakistan is a unique mix of urban, rural, semi urban, modern, pagan, east
ern and western cultures. Its Islamic heritage and connection with the Arab,
Central Asian and Persian tradition further defines the nature and form of
its relationships. The modern doctor who practices allopathic medicine is
linked with the British Raj. Fotlowing the independence in 1947, the tertiary
care hospitals were run by doctors trained in Britain. The traditional rela
tionship that the common man has with the doctor is similar to his bond
with the ruling elite in the 19th and 20th century. which was heavily under
the Western influence. The common man, therefore, has an urge to form
a closer social bond with the doctor, who is seen as part of the elite. The
doctor in his own need to be part of the elite makes constant efforts to
socia[ise with high ranking government officials, miLitary, politicians, and
others in power. This arrangement grossly undermines the professional
nature of the bond that should ideally exist in an ethical health setting. It
results in the so-called VIP culture in hospitals, and grossly undermines the
• founding principle ofjustice in medical ethics.

The quality and nature of social bonds between doctors and their patients
is expected to take a new shape with the advent of social media. All at
tempts at forming social bonds that can challenge the professional
nature of doctor patient reLationship must be guarded against. This in
c[udes befriending patients on sociaL networks or making them privy to
doctors personaL Lives. This is because this shifts the focus of the doctor
patient reLationship to the doctor, instead of remaining on the patient and
their treatment. It also transforms the doctor patient relationship from a
therapeutic to a social one. This may also lead to serious issues of trans
ference and counter transference.

b. Dependence
The vertical nature of the existing relationship between doctors and their
patients puts health professionals on a higher pedestal, where they are
asked to make crucial health decisions concerning the Life of their pa
tients. Traditional family physicians even have a say in personal and family
decisions of the community that they serve. This unique status gener
ates strong psychological dependence of patients on their doctors. If the
patient has dependent personality traits, this dependence can become
counter therapeutic and lead to negative heaLth outcomes. A dependent
patient can start to tax health resources, a doctor’s time and energies.
They may, then, translate their dependence into hostility and anger
towards the health profession. An ethical doctor ensures earty detection
and management of this psychological reaction. If there is a failure to
manage this state it is safe to refer the patient to a colleague for further
management after briefing them on the issue.

c. Transference
Transference is when feelings, attitudes and desires originatly linked with
a significant figure in a patient’s life (usually childhood) are projected or
transferred onto the doctor. Transference may be positive or negative.
The significant figure may be a patient’s parents, sibling or someone that
the patient was close to. Depending on the nature of the relationship of
the patient with that person in childhood, the feelings for the doctor can
be positive or negative. In the case mentioned above, Mr. Y, was seen as a
kind and compassionate repLacement of the Miss X’s [ate father.

CIii1c

114e D€TO Tht TO

-::.--J1
If an adult patient in a medical ward wants to be examined by one partic
ular doctor, wishes the doctor comes to their bed first and spend longer
time in their company, it may be on account of a paternal transference. The
doctor, on account of his physical appearance, mannerism, or personal
ity, may remind the patient of their father. The feelings for the father that
the patient felt as a child and were tong forgotten, may come to the sur
face during their admission. This often happens as patients in a ward feel
dependent and cared for, the same as children. This behaviour amongst
grown-up patients of reverting to child-like behaviour is catted regression.
In this state they start to feel a strong bond for the doctor, similar to one
they once had with a parent figure as a child.

When a patient is seen showing aggression towards a doctor, with no ob


vious cause in the present, this may be an expression of negative transfer
ence. A doctor unaware of the phenomenon of transference, may find this
behaviour threatening and offensive.
Similarly, positive transference feelings amongst patients commonly occur
on long stay wards for the female nurses attending them. The femate doc
tor or nurse may remind them of a mother or a caring elder sister. Patients
in such a state may openly express their desire to bond with the nurse or a
doctor as a sister, brother, mother, father, uncle or aunt. Due to the patient
experiencing this transference, s/he may give the doctor an elevated sta
tus, and start to compliment them unduly. It is important for the doctor or
other health professional in this situation to reaLise that the patient knows
next to nothing about them personally and is merely identifying them with
• someone they knew intimately. This may become difficult, as the show of
this emotion may be quite flattering, especially if coming from the opposite
sex.
$ Unresolved and unaddressed transference can tead to prolongation of
the patients stay in the ward. Patients may begin to ask for personal phone
numbers and home addresses of the carer. They may also begin to invite
the doctor or the nurse to meet outside clinical settings (as in the case of
Mr. Y and Miss X). The ethical dilemmas that may follow this behaviour call
for a constant awareness and understanding of the phenomenon of trans
ference.
d. Counter-transference
In countertransference the emotional responses of the doctor are directed
towards the patient. Similar to transference, countertransference may also
be negative or positive. The patient in such situations reminds the doctor
of a welt-loved or hated individual from the past. and fulfils an unfulfilled
psychological need. In the case of Mr. Y, (mentioned before) his deep

Poctor, I had a
dfficuIt childhood.
seated desire to feel like a rescuer was fuLfilled by Miss Xs reaction. In
another setting, a young doctor during his house job began spending long
hours in the care of a 60 year old patient with hemiplegia. He would miss
his ward rounds, emergency duties and even his rest hours to be on the
bedside of the patient when there was no cLinical need to do so. A deeper
took into the situation revealed that the patient’s looks greatly resembled
the doctor’s deceased father, who died of stroke many years ago, and who
the young doctor had failed to took after. In this case, the doctor experi
enced countertransference towards the patient. Unaddressed counter-
transference can greatly jeopardise the professional life of a doctor anci
compromise the quality of the doctor-patient relationship.

How can transference and countertransference issues be dealt with?


The most important measure in all health care settings is to have a con
stant awareness of transference and countertransference. A conscious
understanding of the feelings, positive or negative, that a doctor or their
patient is having can make behaviour clinicat and prudent. Transference
or countertransference may become overpowering and adversely affect
the doctor’s ctinicatjudgment or progress of the patient. In such a case,
care of the patient should be shifted to a more experienced colleague. It
may become necessary to report the matter to the consultant or head of
department to make alternative arrangements. This is an action that Mr. Y
should have undertaken at the very onset of his discovery of the phe
nomena of transference and countertransference; Mr. Y instead kept it a
secret as he was unaware of the psychological reactions setting in.
In case there are repeated episodes of transference and counter-trans
ference with a particular doctor, it is appropriate for them to seek psycho
therapeutic advice from a mental health professional.

It is also important to work towards ensuring the therapeutic relation


ship is strictly professionaL Making statements to a patient such as “you
remind me of my mother”, or referring to patients as Uncle, Amma or Cha
cha is not helpfuL The use of titles such as Mr., Ma’am, or Bibi or Sahab to
refer to patients is respectful and culturally appropriate.
e. Resistance
Some patients may be seen constantly defying a doctor’s instructions in
spite of repeated warnings of the serious consequences. An example of
this is a patient of coronary artery disease who refuses to give up smok
ing, eats red meat regularly, finds evening walks “boring” and makes no
attempt to shed weight. This patient is exhibiting what is referred to as
resistance in the doctor patient relationship. Resistance is a result of use
of unhealthy defense mechanisms such as denial, avoidance, rationaliza
tion and suppression (discussed in detail in section E). The patient is often
unaware of this and it may be useful to discuss these in a therapeutic
interaction. This will help to make the patient conscious of the basis of
their faulty behavior and work towards improving it.
Resistance can seriously disrupt the doctor patient relationship. A doc
tor who is unaware of it may gradually start to withdraw from the care of
such a patient or refuse to treat him at alL The patient may become even
more resistant to treatment following or develop hostile feelings towards
the doctor. Resistance may be a transitory stage in the treatment process.

I
gradually receding on its own. It may be resolved with a couple of sessions
addressing the issue in a meaningful discussion with the patient. Clinical
ty, resistance may present as or non-adherence to treatment on part of
the patient. When a patient is repeatedly seen to do so, it is important to
consider it a psychological reaction that needs a deeper insight and un
derstanding.

f. UnweLL Physician / Burn-out


With increasing personal and professional demands. there is a proportion- ‘ I .

ate rise in stress experienced by health professionals. This increased stress - ‘-

may lead to maladaptive methods of dealing with the situation, causing the ,. . ‘
heaLth professional to burn out.” Burnout refers to a form of psychological
stress caused by mental and physical exhaustion. It leads to an increase ..

in the number of health professionals who develop depression, become


suicidal, indulge in drug abuse, alcoholism and other risk taking behav- Arfist Laura Zombie
ior patterns. This adversely affects their professional and ethical standing.
These health professionals are a risk to themselves, their patients and their
own family and community.

Early signs of burnout in physicians include the following:

• Long working hours without any time for exercise, healthy family life,
and interaction with friends
• Loss of temper and anger outbursts at work and at home
• Chaotic family life
• Impaired clinical decision-making and deteriorating performance
• Frequent job changes
• Un-prescribed use and misuse of painkillers, tranquillisers, smoking,
alcohol abuse

It is ethical tooffer and provide help to a colleague who is showing early


signs of impairment in performance due to burnout. In case they refuse to
seek help and continue to see patients, the matter may need to be report
ed to PM&DC or the employers.
Professionalism in Health Care

A healthy doctor-patient relationship is dependent on the professional


excellence of a doctor. Professional excellence is based on the knowledge,
skills and attitude of the physician, who is expected to treat both the psy
che’ (mind) and the ‘soma’ (body). The following attributes of knowledge,
skills and attitude are essential for any doctor to be considered a
professional.

a. KnowLedge
i. Distinguish normality from abnormality from a medical. social and
psychological perspective
ii. Relate biological factors with psychosociaL factors in health and
disease
iii. Learn the use principles of behavioural sciences in clinical interviews,
assessments and management plans
iv. Request and justify not only laboratory, radiological, and
electrophysiological investigations but also make social and
psychological inquiries
v. Use pharmacological as well as non-pharmacological interventions.
vi. Apply evidence-based research findings to clinical situations

b. SkiLLs
Written Communication ShiLLs:
i. Demonstrate competence in medical writing
ii. Write a comprehensive history of the patient
iii. Update medical records clearly and accurately
iv. Write management plans, discharge/transfer summaries and referral
notes

VerbaL Communication ShiLls:


i. Establish professional relationships with patients and their caregivers
to obtain a history, a physical examination and make an appropriate
management plan
ii. Demonstrate usage of appropriate Language in bedsiçfe sessions,
outpatients, E-communication, seminars,
iii. Demonstrate the ability to communicate clearly and sensitively with
patients, relatives, other health professionals and the public
iv. Demonstrate competence in presentation skills
v. Provide informational care and counsel patients
f. Use principles of effective communication (section A) in all his/her
clinical interactions.
Patient Management Skitts:
i. Interpret the history and examination findings and arrive at an
appropriate differential diagnosis and final diagnosis
ii. Demonstrate competence in cLinical problem identification, analysis
and management of the probtem using appropriate resources
üi. Prioritise cLinical probtems for interventions
iv. Use evidence-based pharmacological and psychosocial
interventions
v. Independently undertake counselling and informational care
sessions

Skitts in Research:
i. Undertake relevant literature searches and collect evidence based
guidelines for use in clinical practice
ii. Interpret and use resuLts of peer reviewed and standard articles to
improve clinical practice (and learn to not rely on data published by
groups with a vested interest)
iii. Organise and actively participate in educational, training and
research activities

c. Attitudes
Towards Patients:
i. Establish a therapeutic and ethical relationship with all patients
ii. Demonstrate commitment to the biopsycho-’social model in the
assessment and management of patients
iii. Demonstrate sensitivity, empathy and understanding while
performing physical and mental state examinations
iv. Consistently show consideration towards the interests of the patient
and the community and place them above personal interest
v. Adhere to principles of medical ethics under all circumstances
vi. Exhibit highest standards of professionalism through the practice of
integrity, compassion. honour. humanism and respect for patients.
colleagues, seniors and juniors,
vii. Demonstrate ability to work as a team member as well as a leader

Towards Setf Development:


i. Demonstrate consistent respect for every human being irrespective
of ethnic background. culture. soclo-economic status and religion.
ii. Dealwith patients in a non-discriminatory and prejudice free manner.
iU. Deal with patients with honesty, equity and compassion.
iv. Demonstrate flexibility to adjust appropriately to changing
circumstances.
v. Foster principles of self-education and reflection in order to
constantly update and refresh knowledge and skills
vi. Recognise stress in self and others.
vii. Deal with stress and support colleagues and allied health workers.
viii. Handte criticism by colleagues or patients constructively.
ix. Obtain and value a second opinion on clinical matters.
x. Demonstrate effectiveness as a member as well as a leader of the
health team

Towards Society:
i. Exhibit sensitivity towards the social, ethical and legal aspects of
health care provision
ii. Offer cost effective professional services

In addition to the above attitudes s/he must demonstrate a commitment


towards following measures of professionaL character deveLopment.
These measures can be used by the trainers of medical students and
post-graduate trainees to measure professionalism for assessment and
feedback.

ProfessionaL Attire:
A medical student or a doctor is expected to dress in serious, non-
provoking and non-offending attire. The bearing of the health professional
should help patients become comfortable. It should not in any way give an
image of self-neglect or non-concern.

Respect for time and punctuatity:


A doctor is expected to be punctual and prepared for ward rounds, lec
tures, and procedures for patients. This attitude affects the morale of
patients, colleagues and students as it reflects commitment towards the
profession. A lack of this attitude marks lack of discipline, structuring and
organizing capabilities of the doctor.

Grasp and knowtedge about patients under care:


A doctor is expected to have a grasp and knowledge of the patients s/he
interviews and should be equipped with basic literature about the patients’
illness. S/he is also expected to present the patient’s history to the consul
tant the next day of hospitalisation. An expression of this pattern is viewed
as a positive professional attitude. Lack of this indicates poor work ethic
and insufficient intellectualstamina.

Conscientiousness:
Taking responsibility in carrying out clinical assignments reflects interest in
learning and efficient patient care. A doctor is expected to have a responsi
ble attitude about his/her patients, which profiles them as a conscientious
professional. Inconsistency of this attitude indicates health problems, am
bivalence towards career, and inability to become a real professional.
Integrity in reporting patients’ findings:
A doctor or a medical student is expected to adhere to the basic human
value of understanding detaits of a patient’s clinical findings and reporting
them with accuracy, integrity and confidentiality. A failure to acknowledge
one’s mistakes and omissions in reference to patient’s clinical information
qualifies for serious professional dishonesty and merits dismissal from
training.

Avaitabitity to the patients:


Doctors are expected to be available to their patients on a daily basis
through appointments, and/or telephone. When on leave or out of station
on duty, they must be available on telephone to the patients they consider
need attention. They must inform the patient of this in advance and intro
duce them to another doctor who has been briefed about their condition
and will care for them in their absence.

Relationships:
Relationships with patients, hospital staff, fellow students, colleagues
and faculty member are expected to be of mutual support. respect, and
professional honesty. Difficulties in dealing with or failure to cooperate with
any one or more of these people may reflect health problems or serious
personality issues.

Assessment of Attitudes (Professionalism) Score


in Doctors
Professional attire!demeanour 7

Iactf9rumeand -.
puncwky::
-
•1I

Grasp and knowledge of awn patients 3

Integrity in reporting patient findings

Relatbnsblps with colleagues, hospital staff and patients *

Score 7-70 -.
SAMPLE MCQ FOR SECTION B

1. A young man undergoes a three month tong treatment by a femate


doctor. Upon his recovery, to show his gratitude he brings her an
expensive gift. Themost ethicaL way to respond wouLd be:

a) Report the matter to the head of department.


b) Ask the patient to leave immediatety, or you will call security.
C) Tell the patient you will be right back and send in a male colleague
to manage the situation
d) Take the gift but tell the patient to never bring you anything ever
again
e) Politely refuse to take the gift and explain to the patient that it is
against medical ethics to do so.

2. A 25 yr old male presents to you in emergency, after a road traffic


accident. On examination he is seen to have had profuse blood toss,
has low BR and cotd clammy skin and he is drowsy. What is the most
appropriate action:

a) Try to wake him up so he can give consent to proceed further.


b) Wait for the patient’s brother to arrive so he can give consent
c) Start Life saving measures immediately as the situation is an
emergency and does not require informed consent.
d) To refuse treatment without available consent.
e) Ask your senior to decide.

3. A patient recently diagnosed with Type II Diabetes mellitus refuses to


cut back on the use of sugary drinks and sweets, despite being given
informational care by his doctor, and warned of the serious
consequences. He is displaying the phenomenon of:

a) Transference
b) Resistance
c) Counter-transference
d) Non-compliance
e) Emotional instability.

4. You have been asked to taLk to a patient who has refused diaLysis for
renal failure. The most appropriate strategy would be to:

a) Provide detailed informational care in an exclusive setting


b) Listen and empathise with unconditional positive regard
c) Explain the pros and cons of her decision and then leave the choice
of dialysis to her
d) Explain to the patient’s family the importance of dialysis and ask
them to convince her
e) Take the patient for dialysis without telling her

I
5. A young man reports to the medicat OPD with swotten tymph gtands,
genital uLcers, and chronic fatigue. He is admitted and upon testing is
found to be H IV-Positive. According to the principtes of medical ethics,
the most suitable action is to:

a) Not give any one the information to protect confidentiaLity


b) Tell the patients family to tell everyone he has been in contact with
C) Inform the patients wife immediately.
U) Convince the patient to keep the information quiet, to protect him
from the stigma
e) Counsel the patient regarding the transmission of HIV, and persuade
the patient to tetl his wife himself. Inform him of your responsibility to
teLl her yourself, in case he refuses to do so.

Sample Short Essay Question For Section B

01. What are the essentiaL principles of medical ethics? HighLight the
two most commonly ignored ethical norms in medical practice.

02. Differentiate between transference and counter-transference.

Answers

i.e
2. c
3. b
4. c
5. e
ECTION C
sychology in Medical Practice

OUTLINE •
Role of Psychology in Medical Practice
Principles of Psychology
• Learning •Metacognition Memoty
• Perception Thinking
• .Emotions
• Motivation Intettigence Personatity

Neurobiological Basis of Behaviour

Chapter 1
Psychology in Medical Practice
Human thought, behaviour and interactions follow a set of psychological
processes and principles. The role of these principles and factors in the main
tenance of health and illness is of crucial importance. Some of the hea[th and
disease situations influenced by psychological factors are as follows:

a. Rote of psychoLogicaL factors in the aetioLogy of


heaLth probLems
There are many habituaL patterns of behaviour that can adversely affect
one’s health and increase ones susceptibility to illness. There is ample ev
idence that chances of developing a heart disease are increased amongst
those who smoke, consume a fatty diet and adopt sedentary lifestyles.
Obesity, misuse and abuse of tranquillisers, alcohol, street drugs like
cannabis, heroin and stimulants, risk-taking, and thrill seeking behaviour,
are all known to lead to diseases and consequences that include diabetes
meltitus, cirrhosis of liver, psychiatric disorders, and road traffic accidents
with obvious morbidity and mortality. Public health interventions to address
these unhealthy lifestyLe factors and promote healthy behaviours in the
community can decrease the subsequent risk of the illnesses mentioned
above.

b. Rote of psychotogicat factors in the precipitation


(triggering) of iLLnesses
Physiological processes in humans are directly affected by psychological
stress, e.g. the immune system. the endocrine system, and the sympathet
ic and parasympathetic systems in the body etc. There is scientific
evidence that traumatic life events (called critical incident stress) and
persistent high stress situations (called cumutative stress) can trigger either
the first episode or a repeat episode (called recurrence) of a variety of dis
eases. Some common examples incLude ischemic heart disease,
asthma, allergies, acid peptic disease, migraine etc.

c. RoLe of psychotogicat factors in the management


of iLLnesses:
Even after a disease process has started, psychological and social fac
tors ptay an important role in the course of iLlness, both in aggravating it
or recovering from the illness. For example, positive behavioural changes
are an integral part of the management of diseases like diabetes meWtus,
hypertension, and ischemic heart disease. Similarly social support ptays
a vital role in the management of serious illnesses like drug dependence
and schizophrenia, among many others.

d. Rote of psychoLogicaL and sociaL factors in diseases


causing disabiLity, handicap and stigma
ALL illnesses which resuLt in temporary or permanent disability or handicap
give rise to serious psychological reactions in patients and their carers e.g.
despair, hopelessness, guilt, anger, frustration, and Loss of motivation etc.
Similarly, illnesses associated with social stigma. e.g. HIV/AIDS, are also as
sociated with various psychological and social challenges, and addressing
these issues is an integral part of the management of such illnesses.

e. Rote of psychotogicaL factors in patients’ reactions


to iLLness
A persons attitude towards his/her illness can play a vital role in determin
ing a person’s future health. Poor compliance with the doctor’s prescrip
tion continues to beone of the major hurdles towards the success of any
treatment. This non-adherence to treatment is often on account of exces
sive and inappropriate use of psychological defence mechanisms. Some
people develop a liking towards some aspects of their sickness, such as
the excessive attention they get from everybody while they are ill and thus
develop sick-roles, which can detay their recovery, and return to work. A
positive psychological reaction to illness on the other hand can hasten the
recovery and the healing process.

f. Medicatty UnexpLained Physicat Symptoms (MUPS)


MUPS include a wide variety of apparently physical illnesses that have
been shown to have a psychological causation. These are seen in most
medical and surgical specialties and include disorders Like irritable bowel
syndrome, non-utcer dyspepsia, temporo-mandibular joint dysfunction,
writer’s cramp, atypical facial pain, chronic fatigue syndrome etc. Besides
the psychological causation, their management also involves various psy
chologicl interventions.
A list of psychological phenomena that shape human behaviour in health
and disease, therefore, needs to be analysed. A study of these processes
of normal human psyche can help a medical student to enhance under
standing of their own behaviour as well as assist in developing a deeper
insight into t[e behaviour of cotleague, other professionals and most
importantly, patients and the individuals around them. These include:
1. Learning
2. Metacognition
3. Memory
4. Perception
5. Thinking
6. Emotions
7. Motivation
8. Intelligence
9. Personality

Chapter 2
Principles of Psychology
1. Learning
Master A, an 8 year old boy, used to wet his bed almost every night He was
ashamed of this and was ridiculed by his cousins. His parents became
worried and took him to a doctor who referred him to a psychiatrist. A’s
therapist decided to teach him to remain dry during the night by using a
number ofbasic learning principles. He was given a buzzer which would
sound the moment he passed urine in bed at night, thus waking him up. The
idea was to associate the stimuli from a full bladder and the urge to urinate
with waking up. If Master A woke up in time, he could go to the bathroom
before he could wet his bed. In addition to this device, A and his mother were
explained a behavioural technique by which A had to change the sheets
himself with no help when the bed was wet On the other hand when the bed
was dry, A was given a chocolate. After 21 consecutive dry nights the buzzer
would be removed, but the behavioural method of rewarding a ‘dry night’ with
a chocolate would remain in place. A review after two months of the start of
A’s treatment, his buzzer was removed, since he had remained dry for 21
consecutive nights. He wet his bed twice after this but that was all. In the next
18 months, MasterA stopped wetting his bed. How in your view did he
overcome his bed wetting problem?

The psychological principle, that formed the basis of Master A’s treatment,
is called the Learning theory. Learning is the process by which new be
haviour patterns are acquired, This is a key process in human behaviour. It
plays a central role in our language, customs, personality traits and even
our perceptions. Humans have instinctual patterns similar to those of an
imals but their complex behaviour patterns are a result of their advanced
learning capacity. Learning is a relatively permanent change in behaviour,
for better or worse.
a. Operant Conditioning
Learning theory implies that learning new behaviours or changes
in behaviour occur on the basis of the environmental conditions or
responses to it. This is known as operant conditioning or
instrumentaL conditioning.
Operant conditioning was established by the work of BE Skinner.
Operant conditioning occurs when a behaviour that is not a part of
persons naturaL responses is learned (or unlearned) by consequenc
es in the form of reward and punishment. Operant conditioning
explains the learning of voluntary behaviour, such as motor actions.
The famous Skinner Box demonstrated operant conditioning by
placing a rat in a box in which the pressing of a small lever produc
es food. Skinner showed that the rat eventually learns to press the
bar regu[arty to obtain food (reward). If, however, the rat pressed the
[ever and received an electric shock or nothing at aLl, (punishment or
[ack of reinforcement) it stopped pressing the lever.
Skinners work showed that operant conditioning works on the princi
ples of reinforcement. Reinforcement refers to any event that increas
es the chance that a response will occur again or a behaviour will be
repeated. There are three types of reinforcement: Positive reinforce
ment (reward), negative reinforcement and punishment.
Positive reinforcement is when one receives a reward for a behaviour,
which results in the behaviour being reinforced, and thus, repeated.
This reward may be in the form of attention, praise, success, a mate-
nat gain. An example is a child who consistently gets a praise or hug
for picking up his toys. after playing with them will learn to be neat
and orderly.
Operant Conditioning

Reinforcement Punishment
Increase Behavior Decrease Behavior

I I I
Positive Negative Positive Negative
A.d appetatrve stimulus nordous stimuli gggg appetative stimulus
following correct behavior following behavior following behavior
Gn,,,g hrdwus rxioog

Escape Active Avoidance


Bcmv noxious stimuli Behavior avoids noxious
following correct behavior stimulus
.,., ,*,,,, dorr xywn,.., vwwco6,d,*
,frw .,,no.6Avn

Postive presence of a stimulus

Negative absense of a stimulus

Reinforcement increases behavior

Punixhment decreases behavior

Escape removes a stimulus

Avoidance prevents a stimulus


Punishment is when a behaviour is followed by a painfuL stimulus,
resutting in the behaviour being stopped, and not repeated. If a child
puts her finger into an electric socket and gets electrocuted (punish
ment), she is not likely to do this again.
Negative reinforcement is not the same as punishment. Negative re
inforcement is when a negative stimulus is stopped only when a de
sired behaviour occurs. This is experienced commonly as a person in
‘iSt Hijab Zainab
the car behind you constantly pressing the horn until you move your
,nish,nent or Negative Hi if ‘i car out of the way. In this case the noise of the loud horn is the nega
tive stimulus. The desired behaviour is moving the car.
The lever bbx experiment by Skinner mentioned above proved that a
behaviour will occur more frequently if given positive reinforcements
and witl decrease in frequency by punishment. Master A, described
in the beginning of the section was instrumentally conditioned to re
main dry by the use of the buzzer. The reward that he got in the form
of a chocolate was a positive reinforcement whereas the act of get
ting up and washing his clothes was a negative reinforcement. The
various types of reinforcement and punishments are summarised in
the table below.
An important consideration in changing and manipuLating behaviour
is the relative effectiveness of the various types of reinforcement.
Positive reinforcement is the most effective method in changing be
haviour, while punishment is the worst. This is why you may notice
that a person has to be punished many times before his/her unde
sired behaviour is stopped.
Another factor to keep in mind with children is their need for attention
which acts as -positive reinforcement, even if the attention involves
being yelled at or punished. A child who is repeatedly scolded for
nail biting, or thumb-sucking may persist with these habits beyond
chitdhood.

Feature Effect on behaviour Example Comments

Positive Behaviour is increased Child increases his Reward or reinforcement


Reinforcement by reward kind behaviour increases desired behaviour.
towards his younger Master A got chocolate
brother to get praise whenever he succeeded in
keeping dry at night
- VV VV - -. - . -
V.

,Chlnc,ees*hte t1veavoIdanceofan
P.inem.ntt.t or escape kind beheviour stimulus increases
- towsids hbyouj áed beha6ourThect
brtoavd%w.ehinghIsclothes

E
Punishment Behaviourisdecreased Childdecreaseahis D.ilve,yofanaversive
by suppression hitting behaviour stimulus decreases
after his mother unwanted behaviour rapidly
scolds him but not permanently. The
buzzer was th. aversive
stimulus in A’s case
VV_VV_
V - VV•V
V_VVVV

V•_V VV_VV_
-

liminated C Iopehlg kxtInctlefl I. meN


Iviau’ ,h5iIlIhm.nt
V

cement -

-
V

Vii rnothe# V

V mwanted b.hevloUt. V

S.V;VV!V VV
V

Conditioning Principles in reot tile


Shaping and Mode[ting
Shaping and modelling are also theories of learning derived from the
aforementioned principtes.
Shaping involves rewarding closer and closer approximations of the
wanted behaviour until the correct behaviour is achieved. An example
of this is when a child learning to write, starts with writing on a paper
with straight lines and is rewarded with a star’ for doing so. He then
learns to write in straight lines on a plain paper, or a medical student
who starts to learn stitching of wounds, makes mistakes and then at
tains perfection by shaping’.
Modelling is a type of observational [earning. This is what occurs
when a student starts to talk, dress and behave in a manner similar to
that of an inspiring teacher; ivhen someone gets a haircut to took the
same as their favourite film star or a medical student decides to be
a surgeon, after doing a rotation with an excellent surgical specialist.

b. Ctassical Conditioning
Classicat conditioning is another principle of the learning theory. It
involves three factors: A neutral stimulus, a stimulus that causes a V
biological response, and the biological response itself. It is a form of
learning that occurs when these two stimuli are paired. The neu
tral stimulus is known as the conditioned stimulus. This is usually a
neutral object such as a bell, a picture or a smell. The other stim ne..
ulus is known as the unconditioned stimulus. The unconditioned
stimulus is one that is inherently capable of producing a biological
response such as food or increased room temperature. The biolog
ical response is known as the unconditioned response. Salivation, Classical conditionin
Unconditioned stimulus: light butt
piloerection, sweating and autonomic arousal are types of biological Conditioned stimutus: cheese
Biologicot response: seen in the mous
responses.
When a biological response is elicited with a stimulus that causes it,
such as food causing salivation, this is known as an unconditioned
response.
When a biological response is coupled with a stimulus that would
not normalty cause it, such as a belt causing salivation, this is known
as a conditioned response.
Classical conditioning Operant Conditioning

Stimulus is provided Stimulus is provided


before reflex after reflex

Deve’oped in luuia by Pavlov Developed lntheUsbysldnner

Relies on association between


stimulus and response Relies on reinforcement

lnvolufltaiy, automtlc Voluntaty,opemtes on


environment
A

Passive learning Active learning


-,-t-.
— -,. -/.-.—....

Ivan Pavlov, a Russian physiologist in the 1890’s established many of


the basic principles of classical conditioning. He designed an appa
ratus that coutd measure the amount of saliva being produced in a
dog’s mouth in response to food (unconditioned response). Pavlov
noted dogs salivated when presented with food. He also saw that
dogs did not do this when he rang a bell. He then trained the dog by
sounding the bell and shortly afterwards presenting food. After the
sound of bell had been paired” with food a few times, he tested the
effects of training by measuring the amounts of saliva when he rang
the bell but did not present food. He discovered that some saliva
was produced in response to the sound of the bell (conditioned
stimulus) alone. This is a conditioned response. Similarly, people in
many parts of the world have learnt to associate the golden letter
M of McDonald’s with tasty fast food and have developed a con
ditioned response (of salivation or the smell of the ‘Big Mac’, to the
mere sight of the large golden arches).
Extinction is the process by which conditioned responses decreases
if the conditioned stimulus is never again paired with the uncondi
tioned stimulus. For example if only the bell is rung but no food is
presented, the salivation in response to the ringing of the bell will
eventually stop, i.e. become extinct.
r. Learning principLes in Clinical Settings
Use of Operant Conditioning against bad habits
How can we use learning principles to discontinue bad habits?
The following techniques derived from the above mentioned
principles of the learning theory could offer some hetpful
possibitities:
1.Try to discover what is reinforcing a bad habit, and remove,
avoid or delay the reinforcement
Exampte: Asif is a medical student who developed the habit of
taking longer and longer breaks when studying. He realised that
the breaks were usually lengthened by watching TV. The
pleasure of watching TV was also reinforcing more frequent
breaks.
Solution: To improve his study habits, Asif should either resolve
to stay out of the TV room untilwork is done (avoid reinforce
ment), or else complete 2 hours of study for half an hour of TV
watching (delay reinforcement).
Example: Farah, another medical student, has a different prob
lem. When she reads in the evening, her periods of concentra
tion, last only about 15 minutes. They are usually followed by a
trip to the kitchen for a snack. In addition to falling behind in her
studies, she is gaining weight.
Solution: Taking snacks are rewarding her impulse to avoid read
ing. Firstly she should do her reading at school or at a library, so
that there is a delay between the impulse to eat and the reward
of snacking. At home she should only keep foods that require
preparation, so that a separate trip to the market is required for
the snack. Requiring a walk to the market will also help her in
weight control.
2. Avoid or narrow down cues that elicit the bad habit.
Exampte: Mr Javed is not ready to give up smoking but would
like to reduce it. He has taken many smoking cues out of his
daily routine by removing ashtrays, matches and extra cigarettes
from his house, car and office. He has also been making an effort
to stay away from smokers, take a walk after meals (leaving his
cigarettes at home) and put chewing gum in his mouth whenever
he feels nervous.
Solution: To further improve his controt of smoking MrJaved
should try narrowing cues. He could begin by smoking only
inside the building, never outside or in his car. He could then limit
his smoking to his home and then to only one room at home and
then to one chair in that room. If he succeeds in getting this far,
he may want to limit his smoking to one uninteresting place such
as a bathroom, basement or garage.

3. Make an incompatible response in the presence of stimuti that


usuatty precede the bad habit.
ExampLe: Maryam bites her nails so much, that they are painful
and unsightly to look at. She has identified several situations in
which she is most likely to bite her naiLs and would Like to break
the connection between these and her habit.
Solution: Maryam should make a list of incompatible behaviours
she can engage in, when she has the urge to bite her nails. These
can include putting her hands in her pockets, taking notes in her
class, sketching pictures, crossing her arms, chewing gum or
combing her hair.

4. Use negative practice to associate a bad habit with discomfort.


Exampte: Hassan has a facial tic that appears when he is nervous
or tired. The tic, which looks like a wink with his right eye often
leads to social embarrassment.
SoLution: In negative practice a response is repeated until it
becomes boring, painful or produces fatigue. This increases the
awareness of the habit and tends to discourage its recurrence.
Hassan could stand in front of a mirror and repeat the tic until the
muscles used become quite uncomfortable. He could also wear
a rubber band on his wrist and stretch it to rebound on to the skin
causing a mild pain each time he winks. Similarly if you have a
habit of saying “you know” or”muttab hai” “uh” or “like” too often
when speaking, set aside 15 minutes a day and repeat the error
over and over while thinking “I hate the way this sounds.

5. Utitise feedback to change bad habits.


Atmost any habit wiLl benefit from simply keeping a score. Keep
track of the number of times daily that you arrive late to class,
smoke a cigarette, waste time during studying, bite your finger
nails. swear or whatever other response you are interested in
changing. This will serve as a ‘feedback’. You will soon find the
frequency of these bad habits, going down.
Uses of Classical Conditioning in CLinical Settings
Since much of medical practice involves behaviour and changing
behaviour, the principles outlined above can be seen in clinical
settings. A few examples will be discussed here.

i. Acquisition of fear and anxiety about hospitats


Children, as well as many adults have a great dislike of doc
tors and hospitals. Children, in particular, are known to cry and
scream at the sight of the doctor’s white coat and at the smell
of the antiseptic, after they have received injections or intrave
nous drips in hospital settings. It is quite likely that they associ
ate (pair”) neutral stimuli such as white coats with nasty, painful
events (injections, drips), It is for this reason that many paediatri
clans these days do not wear the traditional ‘uniform’ associated
with the medical profession. so that the “paired” response may
not take place. After a few visits to such a doctor the classical
conditioning that occurred between the white coat and injections
witl undergo ‘extinction’. The child will discover that the uncon
ditioned stimulus and the conditioned stimulus are not “paired
anymore.

ii. Chemotherapy for treating cancer


Chemotherapy for treating cancer is a highly unpleasant expe
rience for patients. It usually involves a series of weekly injec
tions of powerful cytotoxic drugs that have marked side effects.
The patient’s hair falls out, they feel nauseous and sick, and
are unable to eat. It is a common observation that conditioned
anticipatory nausea and vomiting occurs in these patients simply
at the sight of the medical staff and smell of the hospital setting.
Recently, attempts to use the principles of classical condition
ing have been tried to help children overcome specific dislikes
of food which have been induced by cancer chemotherapy.
Frequently children have been found to associate feelings of
being sick with the last food they ate before their treatment and
so were refusing it afterwards. The children were given a strong
tasting sweet after their last meal but before the chemotherapy.
The investigators found that children given the sweet ate more of
proper nutritionally good food afterwards than children given no
such intervention.

iü. Treatment of Phobias


A phobia is defined as an intense, irrational fear of an on object,
animal, or a situation, leading to its avoidance in fLiture. It is per
fectly reasonable to feel intense fear at the sight of a poisonous
snake. It is unreasonable, however, to feel intense fear of ele
vators, lizards, heights, thunder etc. A form of therapy known as
Systematic Desensitization based on classical conditioning can
be beneficial in treating patients whose lives have
become dysfunctionat and miserable because of their phobia.
The method involves firstly getting the patient to relax fully,
through the use of specific relaxation techniques.
After that, the patient is asked to imagine a picture only remote
ty associated with the feared object, or situation, In the case of
someone frightened of dogs, for example. the patient wilt imag
ine a simple outLine drawing of the dog. Whenever the patient
feeLs any sign of anxiety, he is asked to signal it by raising a little
finger. When that happens. s/he is instructed again to fully relax
using relaxation methods. Eventually, the images are brought
closer and closer to the real phobic stimulus and continue into
real life so that finally s/he is able to maintain a relaxed state of
mind firstly in imagination and lafer in the presence of a dog in
real tife. The principle invotved is to associate a calm, relaxed
state of mind with something which once instilled terror, and
irrational fear.

2. Metacognition
Metacognition is defined as thinking about how we think. It refers to knowl
edge people have about their own thought processes. It is not uncommon
for many medical students to take notes of only those aspects of a lecture
that they consider difficult. Sirnitarty, often while studying we notice that we
are having trouble learning a certain chapter but are able to learn another
chapter by mistake. This understanding of our own learning processes is V

metacognition. The examples illustrate a student’s awareness of their cog-


V

nition and hints at a strategy for managing learning based on this aware- :“
V

ness.
VVV2

Metacognitive awareness is not seen in children as it develops later in life. V

It may compensate for tower levels of ability. It enables students to coordi


nate the use of extensive knowledge and develop many separate strate- Metocnition-HitingA
gies to accomplish complex learning goals. Medical students are expected
to learn complex concepts and a rote learning of these is neither possible,
nor required Metacognitive techniques can help achieve this task. Efficient
learners are highly aware of their own thinking and memory and use this
V
information to regulate their learning. Their knowLedge includes the how,
why and when of learning. The degree of metacognitive ability varies from
person to person and some basic level of automaticity comes only through
regular practice. One way to practice is to use monitoring checklists in
which students can check off component steps in monitoring their learning.
Metacognitive strategies for medical students

tive reading Involves you In a process of actively questioning the matedaIu read. Befor.you n address
the questions of is It any good? and “what does It meanr you must understand whatycu are reading. Here
are acme helpful tipw
1. Quickly read the title page prefuce or abstract to get an Idea of the topic of the article or book and cetego
rize It In your mind. Is It really a report of research findings or Is It an anecdotal account of somebody’s Isolat
edexperlencei
Z. Study the table of contents or the headings In the.ardcle to get a sense of Its stmcture.Thls alerts you In
advance about what to expect
3. Read any boldface excerpts or boxed summaries (like this one) to ascertain the maIn points or Ideas.

4. Leaf through the whole article dipping Inhere and there to follow the logic.
5. FInd the Important and unfamiliar words and use racoure like a glossary or dictionary to determine their
meaning.
6. HIghlight key points or conclusions by underlining or puWng notes In the margins.
7. Be able to say with certainty that you understand what you have read before you crltidze It
8. Compare what you have read in one study with whatyotW have read cwnulatlveiyon a topic.
There ate three main phases in Learning:
1. PLanning
2. Monitoring
3. EvaLuation

1. PLanning phase
This is the phase in which the Learner is preparing to tackle what is
to be Learnt. In the planning phase, Learners may ask themseLves the
folLowing questions:
• Why do I need to know this? Write down Learning outcomes of the
chapter to be read or the task assigned.
• What do I know aLready about this topic? Before reading the
chapter, write down a few facts or concepts that you aLready know
about it.
• How wiLL I Learn it? Use Learning strategies such as mnemonics.
watch a video reLated to the subject or reLate the disorder being
described in the chapter to a patient you have recently seen.

2. Monitoring phase
WhiLe going through the materiaL to be Learnt, Learners can monitor
their Learning by asking the foLlowing questions:
• How am I doing in grasping these concepts? Do I understand what
am I studying? Think or repeat the important points in the mind
after reading a few paragraphs.

3. Evatuating phase
During this phase, after completing what was to be Learnt, [earners
can ask themseLves the foLlowing:

• How weLL did I do Learning this?


• Is there anything I do not understand?
• How shouLd correct my errors?
• Were there any distractions or behaviours that were Limiting my
concentration while studying?
• Have I accomplished the goals I set for myself? Assess yourself on
the basis of the objectives that you listed at the start.
• Is there a practicaL appLication for what I have learnt?
3. Memory
Zahra was considered one of the brightest students in her ctass. Her peers
were envious of the ease with which she managed to remember every sin-
gte detait by quickLy skimming through her textbooks. Now, as a doctor her
performance is phenomenat as she knows the name of every patient and
their family member and is able to recall their medical histories with ease. Her
friends often commented that her memory was “photographic” and that “her
mind works like a computer.”

Memory is one of the extraordinary phenomena of the natural world. Our


sensory experiences, perceptions and actions change us continuously and
determine what we are later abte to perceive, remember, understand, and
become. Human memory is unique in its characteristic ability to store visual
recordings. aLong with associated emotions and feelings. A subsequent re
call of various memories includes reliving the associated emotions. It is this
quality that makes our memories pleasant and unpleasant. Higher mental
functions such as speech, thinking, perceptions, moods, psychomotor
skills and consciousness of surroundings are based on memory. Without
memory there can be no mind. (For details on the neurobiological basis of
memory, refer to Chapter 7)

a. Stages in Memory
Human memory resembLes a computer, consists of an information pro
cessing system in three separate stages:
i. Encoding: Sensory information is received and coded or trans
formed into neural impulses that can be processed further or
stored for later use. Just as a computer changes keyboard entries
into binary digits that can be stored on a disk, sensory information
is transduced, so that it can be used and stored by the brain. Apart
from transduction a great deat of encoding process appears to be
devoted to rehearsing or repeating the input.
ii. Storage: Like a computer program, the encoded information must
be stored in the memory system. Atthough some bits of information
are stored briefly or used only once, and then discarded, others, like
certain telephone numbers, are used frequently, and are therefore
stored on a more permanent basis.
iii. Retrieval: Once a file has been named and stored on a computer,
we can call it up by its name and use it again. Human memory works
in much the same way. When we recall or bring a memory into
consciousness, we have retrieved it. This recall process is known as
memory retrieval.
Human Memory

Sensory Memory Short-term Long-term


(1sec) Memory Memory
tVrkIng Memory) IL ft
t1 miii)

Explicit Memory implicit Memory


(.i)flsCiOU( (unconsdousl

Declarative Memory Procedural Memory


(facts, even(s) (skIlls. tasis)
I
Episodic Memory Semantic Memory
(events. experiences) (fads. concepts)

b. Types of Memory

I. Sensory Memory:
Sensory memory is a memory or storage of sensory events such as
sights, sounds and tastes with no further processing or interpreta
tion. Sensory memory provides us with a very brief image of all the
stimuti that were present at a particular moment and therefore has
the potential to be quiet large. Sensory memory appears to Last only
briefly, about one half to one second, depending on which senso
ry system is involved. For instance if you see an object, an image
persists for about one-half second afterwards. Similarly information
you hear is held as a brief echo in sensory memory for up to two
seconds.

ii. Short-term Memory fSTM):


Not everything seen or heard is kept in memory. Lets say a TV com
mercial is running in the background as your friend reads you his
notes on pharmacology. Do you remember the words of the TV ad?
Probably not, because selective attention determines what informa
tion moves on to short- term memory. Short term memories are also
brief, but longer than sensory memories. Attending to your friend’s
narration will place his technicaljargon in your short-term memory
(while allowing you to ignore the voice on TV saying ‘talk shawk”).
Short-term memories can be stored as images, but more often they
are stored as sounds, especiatly in recalling words and letters. Short-
term memory acts as a temporary storehouse for small amounts
of information. Unless the information is important, it is quickly
dumped from STM and is lost forever. Short-term memory prevents
our minds from collecting useless names, dates, telephone num
bers and other trivia. At the same time it provides a working memory
where we do much of our thinking, dialling a phone number, doing
mental arithmetic and remembering a shopping list.

iii. Long-term Memory (LTM):


nformation that is meaningful and important is transferred to the
third memory system called long- term memory. In contrast to SIM,
long-term memory ([TM) acts as a permanent storehouse for infor
mation. [TM contains everything you know about the world and yet
there appears to be no danger of running out of room in LTM,
Type Description Examples

Implicit! Memories used In making responses Remembering how to dissect


Procedural and skilled actions. pass a nasogastrlc tube, play
tennis or drive a car.
Semantic Our store of general and specific Vt.r balls at 700 C, Paklstanl
knowledge fri Asia, and heart leon the left,
motorco.kInthemedlel
frontal lobe
. ‘

Episodic Memories of personal events. Your first day of college,


birthday, your patient’s clinIcal
state

which is considered to have a Limitless storage capacity. Information


in the LTM is stored on the basis of meaning and importance and not
by sound or image. When new information enters STM, it is related
to knowledge stored in LTM. This gives the new information mean
ing and makes it easier to store it in LTM. It is therefore important
to buiLd new information on what you atready know (stored in your
LTM). The hippocampus is of particular importance in LTM. Recent
research in human memory has revealed that there is more than one
type of LTM. The type of information being processed influences the
nature of the stored memory. Three major categories of LTM have
been proposed, as iLlustrated in the above table.

iv. Rehearsal.:
The process of rehearsal consists of keeping items of information
in the centre of attention, by repeating them silently or aLoud. The
amount of rehearsal given to items is important in the transfer of
information from short term memory to tong term memory. Ex
periments have indicated that the sheer amount of rehearsal may
be tess important than the ways in which information is rehearsed.
Just going over and over what is to be remembered (maintenance
rehearsal) does not necessarily succeed in transferring it to tong
term memory. What reatly works is eLaborative rehearsaL which is an
active process involving giving the material organization and mean
ing as it is being rehearsed.

hiomIng SensoTy
Information Memoly

+
Forgotten Forgotten

t4r’ti niy is thougnt to invotve at least three steps. Incoming intormation is first hetd far a second or
t&o by sensory memory. Information selected by attention is then transferred to short term memory.
If new information is not rapidly encoded, or rehearsed, it is forgotten. If it is transferred to tang term
memory, it becomes retativety permanent, althaugh retrieving it may be a problem.
Forgetting:
Forgetting is the inability to recall information. This means that much
of what we think as forgotten is not reatly forgotten because it was
never encoded and stored in the first ptace. The information, due to
tack of attention, may not have reached short term memory from
the sensory register. Alternatively due to inadequate encoding and
rehearsaL the information may not have been transferred from the
short term memory to the tong term memory. How much of the
information is forgotten depends on the following factors:
• Interference: Experimental evidence as well as everyday experi
ence indicates that learning new things interferes with our mem
ories of what we learned earlier and prior learning interferes with
our memory of things learned later.

• Retrieval problems: Finding information in the organised long


term memory store is aided by retrieval cues, or reminders which
direct the memory search to the appropriate part of the tong term
memory Ubrary. Without the retrievat cues, the sought for item
stored in LIM may not be found and seem forgotten. While we
often cannot recall something while actively searching for it, we
may later recall the sought for information, when we have given
up the search and are doing something else. The new activity in
which we are engaged gives us another set of appropriate re
minders or retrieval cues. Perhaps the new situation leads us to
search through portions of our LTM store not examined before.
It is therefore a good idea to ask the examiner to go to the next
question in viva voce, rather than continuing to retrieve an answer.
It will come to you in a while, when you are answering another
question.

Motivated forgetting: The theory of motivated forgetting was in


troduced by Sigmund Freud when he described a key concept of
psychoanalysis viz, repression. Repression refers to the tenden
cy of people to have difficulty in retrieving anxiety- provoking or
threatening information. This helps to explain why people gener
ally remember pleasant events more often than they do unpleas
ant ones: the unpleasant memories have been repressed. This
aspect should encourage to make ones learning a lot of fun, en
joying your studies and clinical work rather than making it painful
and a burden. This will happen if you will leave your exam prepa
ration to the few days before the exam.

c. The Anatomy and Biochemistry of Memory


The study of organic memory disorders has ted researchers to identify
brain structures that appear to mediate short term and long term memory
processes. Two of the key parts of the limbic system, the hippocampus
and the amygdata embedded under the temporal lobes are essential in
receiving new information and storing it. The diencephalon also contains
a number of structures, the most important for memory being the mam
mitlary bodies, thalamus and hypothatamus. Lesions to these structures
generally result in problems with encoding of new information. Although
CyIic-Anp UodiflUonof
lIly
DNAnter.tn
(mk,pl pu’ —..

Undtory Ue.,on, Nw pt.n


fo.mtion

F,,,tjo,of
NewSynpti IrVb
Connoton

FormaUon of Long Term Memory

no-one has yet found the exact physical changes in a cell that accounts for
a memory, many new discoveries have been made about the physiologicat
basis of learning and memory. One of these findings is the role that Long
term potentiation fLTP) may play in memory formation. Studies reveal that
repeated electrical stimulation of nerve cells in the brain can lead to a sig
nificant increase in the likelihood that a celt will respond strongly to a future
stimulation. This effect can tast for a long time and may be a key mech
anism in the formation of memories. Studies also indicate that a specific
type of receptor viz. NMDA receptor is extremely active in Long term poten
tiation as is the role of catcium. An important part of memory formation is
an increase in the sensitivity of certain nerve cells to acetytchotine.

U. Methods to Improve Memory

• The example of Zahra given in the beginning of the section clearLy indi
cates that some individuals have specific techniques through which they
facilitate there memory and learning of new information. Some of these are
as follows:
• Know[edge of resuLts: Learning occurs most effectively when feed
back or knowledge of resuttsãLtows you to check and see if you are
teaming. Feedback also helps you identify material that needs extra
practice, and it can be rewarding to know that you have answered or
remembered correctly.
• Attention: A setting that enhances your focus while studying, and
ensures minimal distraction will turn your attention to the memory
job at hand.
• Recitation and rehearsat: This means repeating to yourself what you
have Learned. If you are going to remember something, eventually
you will have to retrieve it. Recitation forces you to practice retrieving
information as you are learning. When you are reading a textbook,
you should stop frequently and try to remember what you have just
read by summarizing it aloud. -

Organise: Assume that you must memorise the following tist of


words; north, man, red, spring, woman, east, autumn, yellow, sum
mer, boy, blue, west, winter, girl. green, south. This rather difficult list
could be organised into chunks as follows: north-east-south- west,
spring-summer-autumn-winter, red- yellow-green-blue, man-worn
an-boy-girl. Similarly by making up stories using tong list of words
to be memorised makes remembering the list much easier.
• Selection: The Dutch scholar Erasmus said that a good memory is
like a fisherman’s net. It should keep all the big fish and let the little
ones escape. If you boil down the paragraphs in your textbooks to
one or two important terms or ideas, you will find memorizing more
manageable. Practice careful and setective marking in your text
books and marginal notes to further summarise ideas. Most students
mark their texts too much instead of too little. If everything is under
lined, you haven’t been selective.
Seriat position: Whenever you must learn something, be aware of
the serial position effect. This is the tendency to make most errors
in remembering the middle of a list. If you are introduced to a long
line of people, the names you are likely to forget will be those in the
middle, so you should make an extra effort to attend to them. The
middle of a list, poem or speech should therefore be given special
attention and extra practice.

• Mnemonics: A mnemonic is any kind of memory system or aid.


People demonstrating extraordinary feats with their memories are
actually using mnemonics to perform this. Mnemonic techniques
rely on the linking or association of to-be-remembered material with
a systematic and organised set of images or words that are already
firmly established in long term memory and can therefore serve as
retrieval cues. Similarly medical students often invent mnemonics
for memorizing the names of cranial nerves; amino acids etc. and
can remember these lists for lengthy periods. The basic principles in
the formation of mnemonics are:
- Use mental pictures
- Make things meaningful
- Make information familiar
- Form bizarre, unusual or exaggerated mental associations.
Attach emotions, feelings or link up with an event. We hardly ever
forget what is emotionally significant in our lives. Senior citizens
continue to recall events of emotional significance even when their
short-term memory is grossly impaired.
• Using mental pictures or visuaL imagery to enhance retention. One
way is to simply imagine an unusual scene that includes the various
elements you want to remember, For instance on your way home,
you have to shop for a newspaper, notebook, pen and shaving
cream. To remember this list of items, you have to make up a men
tal image of something funny and bizarre such as you, opening up
the front page of a newspaper to find a picture of a person reading
a notebook with one hand and using a pen to wipe off the shaving
cream on his face with the other hand.
• Overtearning: Many studies indicate that memory is greatly im
proved when study is continued beyond bare mastery. In other
words after you have learned material well enough to remember it
without error, you should continue to study the material. Overlearn
ing is your best insurance against going blank on a test because of
anxiety.
Spaced practice: Spaced practice generally is superior to massed
practice. Three 20 minutes study sessions can produce more learn
ing than one hour of continuous study.
• Whote versus part learning: Generally it is better to practice learn
ing whole packages of information rather than smaller parts. Try to
study the largest meaningful amount of information possible at one
time. This means, reading a textbook is better than reading con
densed notes. The only notes that will benefit you will be the ones
that you make yourself based on steps listed above (selection).
• Steep: Sleeping after study is helpful and reduces the interference.
Similarty breaks and free times in a schedule are as important as the
study periods. Staying awake the whole night before your exams is
not the smart thing to do
• Review: Reviewing shortly before an examination is helpfuL though
one should avoid the tendency to memorise new information at that
point. This review should take about an hour not the whole night
prior to the exam

e. Pathological Changes in Memory

Normal Brain Brain with Alzheimer’s

Codu

‘I-
Before considering the pathological basis of memory toss, it is important to
run simple tests of a person’s cognitive functions such as attention, con
centration. registration and motivation (see table of MMSE on next page). In
case of an impairment of one or more of these higher mental functions, the
function of memory does not come into play. People with tow mood, poor
motivation and consequent Lack of attention and concentration my there
fore complain of Loss of memory.” This state is catted pseudodementia.
Another situation, in which people sometimes experience a significant al
teration in their memory or identity, occurs on account of loss of integrative
function of the brain due to an emotional challenge or a stress that could
be sociaL psychological or structurat. Such stress can interrupt learning
new information, recalling old information, or change the ability to think
and process information. This results in disruption of memory and identity.
These alterations in memory (and or of identity. or consciousness) some
times lack a clear physical cause and are called dissociative disorders.
The principle symptom in such situations is an inabitity to recall important
personal events and information of personal significance.
Disturbances in memory and identity that have clear physical causes
inctude amnestic disorders and dementias. Amnestic disorders affect a
person’s memory exclusively, either for events before an amnesia inducing
trauma or for information learned after it or both. They are caused by med
ical conditions, such as thiamine deficiency, hypothyroidism, hypogtycae
mia, chronic alcohot or substance abuse, head injury or other problems
that can adversely affect the physical functioning of the brain. Dementias
involve mote than just memory ate characterised by deficits in other areas
of cognitive functioning. such as reasoning and problem solving. These
kinds of dementia are caused primarily by degenerative diseases that af
fect specific areas of the brain. The most common amongst them is Alzhei
mer’s disease.

4. Perception
I never woutd hove dreamed that ye/tow is.. soyettow. I don’t have the words,
I’m amazed byyettow. But red is my favourite colour Ijust can’t believe red. I
can’t wait to get up each day to see what I can see. And at night I took at the
stars in the sky and the flashing tights. You coutd never know how wonderfut
everything is. Isaw some bees the other day, and they were magnificent. Isaw
a truck drive in the rain and throw a spray in the air It was marvellous. And
did I mention. i saw a fatting teafjust drifting through the air”
Bob Eden, who had his sight restored after being btind for four decades.

Perception is the process of making sense of the stimuli in our environ


ment. To activate a particular receptor. a specific type of energy must be
present: light waves for vision, movement of air molecules for hearing.
molecules in a liquid solution for taste, and so forth. You cannot shine a
flashlight in your ear and expect to have a visual response since there are
no light-sensitive receptors there.
Neurons operate on the basis of changes in their electrical charge and the
release of neurotransmitters. Physical energies of light. sound, odour and
taste molecules must be changed into etectrochemical forms that the
nervous system can process. This conversion of stimuli from the recep
tors into etectrochemical energy that can be used by the nervous system,
is catted transduction. Continued presentation of the same stimutus can
cause receptors to become tess sensitive to that particular stimulus. This
process, known as adaptation, occurs very rapidLy when odours and tastes
are involved. To understand perceptual processes, we focus on visual per
ception. Many of the processes we wilt discuss also apply to other senses.
a. Motivation and Perception:
We do not perceive everything in our environment. Our motives,
needs, drives, and even prejudices may distort or determine what
we perceive. A thirsty traveller lost in a desert is bound to experi
ence a mirage, a false perception of water, influenced by his motiva
tion to search for a water body. A patient in a ward waiting for visitors
in the evening, would from a distance misperceive a stranger as a
relative. A medical student may hear his name called (erroneously)
while waiting for his turn for the viva voce. Similarly. certain stimuli
are more likely to attract our attention than others.ing is your best
insurance against going blank on a test because of anxiety.
b. Attention and Perception:
We cannot possibly attend to and process all the stimuli that our
sensory systems receive at any one moment; some of them must
be filtered out. A good example of the need to filter information is a
typical healthcare situation in our settings. You might be running a
busy medical OPD with many patients surrounding you, white you
desperately try to focus on the information being given by the pa
tient you are examining. If you try to listen to the shouting and crying
of the patient waiting for the turn and one in front, you will probably
find yourself switching your attention back and forth between them
and becoming quite confused. This is why it is important to not have
more than one patient by your desk or examination table. Having pa
tients wait outside the room will help to limit the amount of sensory
input and focus your attention better.

c. Basic Perpetual Abitities:


i. Patterns and Constancies: We perceive objects in our environ
ment as having features such as pattern, constancy, depth. and
movement. Our perception of these objects and their features is
so automatic that we often take them for granted. They are, how
ever, crucial components of perception.
ii. Pattern Perception: Among the most basic perceptual abilities is
the ability to perceive patterns. To survive in modern society, we
must be able to perceive a staggering numbor of shapes and fig
ures. A few of the patterns we perceive everyday are the letters of
the alphabet, traffic signs, friends facial features, food items in the
grocery store. the buildings in an apartment complex, and cars in
a parking lot. The ability to discriminate among different shapes
and figures is known as pattern perception.
iii. Perceptuat Constancies: Once you have identified an object, you
continue to recognise it even if it changes its location and distance
r

and, therefore, casts a different image on your retina. A change in


the retinal image does not signal a change in the object. This ten
dency to perceive objects as relatively stable is called perceptual
constancy. The importance of perceptual constancies should be
obvious; they allow us to deal with our environment as relatively
stable and unchanging. We do not have to treat every perceptual
change as if our environment had changed completely different
positions.
iv. Shape Constancy: Shape constancy means that your perception
of the shape of an object does not change even though the image
projected on your retina does. In other words, the shape of an ob
ject is perceived independently of the image it casts on the retina.
This phenomenon is easy to demonstrate. Look at this book from
a number of angles. You see nothing but a book being held in dif
ferent positions. The same coutd be said for the opening and clos
ing of a door or the image of a car making a left turn in front of you.
The image on your retina changes drastically, yet the object you
perceive does not. Almost any moving object displays the prin
ciple of shape constancy. For the perception of shape constancy
to occur, however, the object must be famitiar and must be seen
in an identifiable context. If there is no context or background to
whfr-h the object can be related, it appears to float in space. and
you cannot judge its correct orientation. Shape constancy. thus,
disappears.
v. Size Constancy: Size constancy also helps us maintain consis
tency in our perceptual environment. As objects move toward us,
their retinal images enlarge; as they move farther away, their reti
nal images diminish. We do not perceive those objects as chang
ing their size, we perceive them as moving toward or away from
us. Size constancy depends on the familiarity of the object and
the ability to judge distance, When we are dealing with familiar
objects and can easilyjudge distances, we are more likely to per
ceive the objects as having a constant size. When we are dealing
with unfamiliar objects and our ability to estimate distance is poor.
the objects may appear to change size. Size and shape constan
cy may seem rather simple because they are automatic process
es, but these constancies involve a great deal of processing. We
use familiar background objects for purposes of comparison (size
constancy) and to anchor our perceptions (shape constancy). Au
ditory constancies are another important aspect of perception.
We perceive words as the same when they are spoken by various
individuals with different voices. Similarly a melody is recogniz
able even when it is played on different instruments and in differ
ent keys.
vi. Depth Perception: In addition to a world of constancies, we ex
perience a third dimension: depth perception. The surface of the
retina is two-dimensional (top to bottom, side to side). yet we are
able to judge distances and locate objects in space (three-di
mensionalty) quite well. Two main types of cues, binocular and
monocular, are used to create our perception of depth. Binocular
cues involve the use of both eyes, whereas monocular cues are
processed by only one eye.
L
vii. Binocular Cues: Among the most important binocular cues are
adjustments of the eye muscles and binocular disparity. The mus
cles that move the eyeball to get the best possible view provide
feedback for judging distance. When objects are near, the eyes
rotate toward a centre point. You can feel the muscle tension
when you look at objects that are very close. To experience this
sensation, focus on this sentence and move the book closer to
your eyes. The closer the book gets, the more eye muscle strain
you feel; the farther away the object is, the less eye muscle strain
you experience. It is important for a medical student who spends
long hours reading to place the book at a distance which causes
minimal or no strain on the eyes.
viii. Monocular Cues: Monocular cues, which can be perceived by ei
ther eye, also help determine depth. For example, when the cii
lary musctes change the shape of the lens in accommodation,
the muscle adjustments are sensed and used to help determine
distance. Artists use numerous monocular cues, including super
position (the fact that neat objects partialty obscure mote distant
objects), texture gradient (that the texture of a surface becomes
smoother with increasing distance), linear perspective (that paral
lel lines appear to converge as they recede into the distance), and
relative brightness (that brighter objects appear closer) to create
the illusion of depth. These cues also operate in our day-to-day D
environment.
* ix. Perception of Movement: We often come across messages on
electronic signs that has letters and words that appear to move
across it. Our perception of separate words is created because of
the proximity of the letters that make up each group (word) and .)
the spaces between successive groups of letters. Unlike the let-
ters you are reading in this text, the letters on the electronic sign
are made up of separate, unconnected points we complete them
using closure. This sign adds another dimension to our consider
ation of perception: although the words do not really move across
the sign, they appear to do so. Apparent motion is the illusion of
movement created by turning the lights on and off in a particular
sequence. The same phenomenon is seen in movies, television,
and videocassettes. All of these forms of entertainment rely on
the brain’s ability to create the perception of motion from a series r
of still pictures. F
d. AbnormaLities of Perception
i. Illusions:
An illusion is a misperception of a real external stimulus. Percep i i
tion involves the brain’s attempt to interpret and make sense of
the stimuli we receive from our environment. Most of the time our
perceptual hypotheses are quite accurate, but sometimes they
can be wrong. For example, how often could you have sworn that
your professor had said an exam was next Thursday, not next
I ‘4::

Tuesday? Have you ever been absolutely sure that a traffic light
was green. not red? It is easy to trick our senses into developing Artist: Hj/ab ZaTha

an incorrect perceptual hypothesis. Such incorrect perceptual hy The influence of mThd-.cet on perceptio
Who do you see in these pictures? Th
potheses form the basis for perceptual illusions. These illusions Ilustrates how a pat vnt a doctor and
are misperceptions or interpretations of stimuli that do not follow family member may peicewe the sam
problem dit’erentt
the sensations received by the eye. Illusions are often seen in pa
tients in Intensive Care Units, such as those in a toxic confusional
state or delirium, as well as people under the effect of alcohol and
drugs such as LSD.
ii. Hattucinations:
Hallucinations are perceptions without any stimulus. They can be
in any of the five senses but the most common hallucinations are
of auditory and visual type. They are seen most commonly in pa
tients with serious mentat illnesses such as schizophrenia. These
patients may hear voices talking about them or doing a running
atiucinations
tist Lam Zombie commentary on their actions. These abnormal perceptions are
real to the patient, which is why s/he can be seen to be in con
versation with the imaginary voices. The most common cause of
visual hallucinations is delirium tremens.

e. Extrasensory Perception (ESP):


This refers to the occurrence of experiences or behaviours in the
absence of an adequate stimulus. Such occurrences are considered
to be paranormal or beyond our normal sensory abilities and are
studied in parapsychology. ESP includes the phenomena of:
• Clairvoyance: The ability to perceive events or gain information in
ways that appear unaffected by distance or normal physical
barriers.
• Tetepathy: Extrasensory perception of another person’s thoughts
or in other words the abitity to read someone’s mind.
• Precognition: The ability to perceive or accurately predict future
events. Thus prophecies and prophetic dreams about the future
are included in precognitions.
• Telekinesis: This phenomenon studied by parapsychologists is
basically the ability to exert influence over inanimate objects. In
other words “mind over matter.”
To date there is little scientific evidence to prove the existence of
extrasensory perceptions.

5. Thinking
Thinking: The During most of our waking hours, and even when we are asleep and
talking of the soul dreaming, we think. Thinking is a mental process invotving the manipula
with itself tion of both information from the environment and the symbols stored in
tong term memory. Thinking is evident when we solve a problem or make a
decision. Thinking can take the form of images or concepts. Visuat imagery.
the experience of seeing even though the event or object is not actually
viewed, can activate brain areas responsible for visual perception, such as
the occipital lobe. Imagery does not have to be visual, however; it can be
auditory or olfactory. Visual images allow us to scan information stored in
memory, answer questions and help us plan a course of action. Suppose
we need to describe the size of an acre. How might we convey this infor
mation? We could say that there are 43,560 square feet in an acre. Would
that hetp you understand how large an acre is? Perhaps not. If we used a
visual image, however, and said that an acre is about the size of a
football field, this woutd probably make it much more meaningful.
Similarly, a consultant listening to a house officer describe a patient on the
telephone would rely on her capacity to build visual images of the clinical
picture, before she can give some urgent instructions.
a. Concepts
What would life be like if we had to deal separately with each indi
vidual animal, event, object, and person in our environment? How
could we learn the names of all of them? We avoid such problems
by using concepts or mental representations of a class (students,
nurses, chairs, politicians). A concept is a symbolic construction rep
resenting some common and general feature of objects or events.
Concepts lighten the load on memory and enhance our ability to
communicate. They also allow us to make predictions about ani
mals, events, objects, and people. Much of what we learn in school,
especially primary school, involves concepts such as colours,
letters, species of living organisms, whole numbers and fractions,
time, and distance. The use of such concepts makes communicating
a great deal of information possible with relative ease. Concepts are,
therefore, an important class of language symbols used in thinking.

b. Problem Sotving
Every day we encounter a variety of minor problems; occasionally
we face major ones. You may find that your computer has fallen
prey to a virus, your shirt button has fallen off, or your motorbike
won’t budge. Some problems are easy to solve, others require great
effort, and some may be unsolvable. The problems we must solve
can differ along several dimensions. One way problems differ is that
some of them are well defined and others are ill defined. Well-de
fined problems have all their components specified. as in algebraic
equations; the goal of the problem is quite clear. Ill-defined prob
lems have a degree of uncertainty about the starting point, needed
operations and final product. A well-defined problem. for example.
might take the form of How should I use the word processor to fit a
500-word essay on two pages%’ An ill-defined question might take
the form of How can I write the type of paper that will get me a
higher percentage of marks?”

Problem-Solving Methods: When you recognise that a problem ex


• ists, you can remember whether you faced a similar problem in the
past. If so, you can retrieve the solution from memory and appLy it to
the current problem. If the problem is new and there is no solution
in long-term memory tLTM), you can use several strategies to attack
the problem. Computers have provided scientists with a model that
can be used to understand human thinking. However, to use the
computer as a model of human thought, researchers need to know
what human beings do when they solve problems. We use two gen
eral approaches to solving problems and these can be programmed
into a computer: algorithms and heuristics.
i. Algorithms: An algorithm is a recipe or rule for solving a problem
that guarantees a solution if there is one. A simple example of
an algorithm is the mathematical formula used to determine the
area enclosed by a rectangle: length multiplied by width gives the
answer, Algorithms cannot, however, provide answers when the
probleni is not clearly specified. There are no procedures that can
be set up in advance to guarantee a solution for sLich problems.
Moreover, some problems are so vast that algorithms are simpty
out of the question. For example, chess players could not rely on
algorithms because it would take centuries to examine all possi
ble arrangements of the chess pieces, even if the players could
evaluate them at a rate of several million per second. Computers
playing chess are, however, capable of doing so.
ii. Heuristics: Heuristics are educated guesses or rules of thumb
that are used to solve a problem. The use of heuristics does not
guarantee a solution but it makes more efficient use of time than
algorithms. For instance, a doctor looking to obtain informed con
sent prior to a surgicat intervention may heuristically solve this
problem. There exists no international standard for information
disclosure for a medical or psychiatric situation but as a rule of
thumb, five areas of information are generally provided: diagnosis,
treatment, consequences, alternatives and prognosis.

Obstacles and Aids to Probtem Sotving


Researchers comparing the problem solving techniques of experts
and novices and found that experts have more information that they
can use to solve problems. Experts know how to collect and orga
nise information and are better at recognizing patterns in the infor
mation they gather. As health professionals we often come across
situations that challenge us and expose the limits of our clinical
experience. Such situations merit a consultation from our senior
colleagues, and are considered a norm in medical practice. At times
the strategies used to solve problems may become obstacles in
problem solving. Some examples of this are:
• Functional fixedness or rigidity
• Mental set
• Assumptions
• Misleading information
• Transfer

Mentat set
According to Sternberg and Sternberg (2012), a mental set consists
of a frame of mind involving an existing model for representing a
problem, a problem context, or a procedure for problem solving.”
In most cases, it is helpful in problem solving to use a solution that
has worked before. It may, however, become a problem if you insist
on using a particular strategy to solve a problem (even if it does not
work) and cannot think of any other way to do it because this was a
strategy that had worked in the past. Mental sets can make it difficult
for a doctor to determine the cause of an illness.
Functionat fixedness
Functionat fixedness is a particular kind of mental set that only
allows objects to have a fixed functionality. This refers to the ten
dency to try and solve problems only in one particular way. It occurs
when we are unable to recognise that an object or thinking tool may
be used for something other than its intended use. This prevents us
from creatively solving a problem. A clothes hanger, for example, is
an object intended to hang clothes in a cupboard. Functional fixed-
ness is the mental block that dictates this is its only use. This would
prevent one, for example, from using the hanger to unlock a car
door when the keys have been left inside.

Misteading information
In a complicated problem there may be large amounts of data avail
able, which may distract from the information required to arrive real
solution. This may occur, for example when we are unabLe to arrive
at a diagnosis for a patient with pyrexia of unknown origin, C PUO) be
cause s/he has been tested for everything under the sun, with some
reports having positive findings but none seemingly indicating the
cause of the infection. It is important, therefore, to be able to sepa
rate information with respect to relevance when solving a problem.
This is especially the case with admitted patients who have under
gone many investigations.

Assumptions
We may at times be unable to solve a problem because we may he
assuming that certain obstacles to the solution exist when this is not
the case.

Transfer
Transfer refers to “the extent to which knowledge and skills acquired
in one situation affect a person’s learning or performance in a subse
quent situation. (Ormrod, 2014) This is obviously an important prob
lem solving technique, but can cause obstacles as welL Negative
transfer occurs when one attempts to solve a second problem with
the same strategy as the first one. An example of negative transfer
is when one switches from driving a manual transmission car to an
automatic one and keeps trying to switch gears.
Problems Involving Transfer, a method of problem solving

The Radiation Problem


imagine you are a doctor treating a patient with a malignant stomach
tumor. You cannot operate on the patient because of the severity of the
cancer. But unless you destroy the tumour somehow, the patient will die.
You could use high-intensity X-rays to destroy the tumour. Unfortunately.
the intensity of X-rays needed to destroy the tumour will also destroy
healthy tissue through which the rays must pass. X-rays of lesser intensity
will spare the healthy tissue but they will be insufficiently powerful to
destroy the tumour.
What kind of procedure could you employ that will destroy the
tumour without also destroying the healthy tissue surrounding
the tumour?

The Military Problem


A general wishes to capture a fortress located at the centre of a country.
All of them are minefleldi Although small groups of men can pass over
the road sally. any large armyofsoldiers will detonate the mines.
A futl-scale direct attack is, therefore, Impossible.
What should the general do?

Think about this: What are the commonaifties between the two problems.
and what is an elemental strategy that can be derived by comparing the
two problems?

Correspondence between the Radiation and the Military Problems


What are the commonalities between the two problems, and what is an elemental
strategy that can be derived by compared the two problems? (After Gick & Holyoak 1983)
Military Problem
Initial State Coal: Use army to capture fortress
Resources: Sufficiently large army
Constraint: Unable to send entire army along one road
Solution Plan: Send small groups doing multiple toads simultaneously
Outcome: Fortress captured by army

Radiation Problem
Initial State Goal: Use X-raysto destroytumour
Resources: Sufficiently powerful rays
Constraint: Unable to admInister high-intensity rays from one direction only
Solution Plan mister) densfty rays from pie directions simultaneousty
Outcome: Tumour destrcted by rays

Convergence Schema
initial State Goal: Use force to overcome a central target
Resources: Sufficiently great force
Constraint: Unable to apply full force along one path alone
Solution: Apply weak forces along multiple paths simultaneously
Outcome: Central target overcome by force

M.L Cick and K. 3 Holyoak fl983), schema induction and Analogical


Trans fer’ Cognitive Psychology. Vol l5pp 7-38.

I
A c.ction/d.clsion COflsqLeficei

a chofrr

A
I

The Seven Step Decision Making Process

c. Decision making
Each day we make dozens, perhaps hundreds, of decisions on every
aspect of Life: what to wear, eat, say, do. Some decision making
involves choosing among proposed solutions to a problem. Some
of these decisions are easy: others are not. How do we make such
decisions? The human brain’s prefrontaL cortex, through its vast
connectivity with the rest of the brain, enables us to process vast
amounts of information quickty and accurately. Heuristics is one
method of doing so. Decision making is a seven step process, as
seen in the diagram above.

d. Creative Thinking
As a medical student, you may have noticed that a few of your peers
or teachers deal with probLems in a unique and extraordinary way.
They are able to visuaLise and understand difficult and boring sub
jects in a fresh and new way. Perhaps you are one of those peopLe
who think and act creatively. The creative thinker whether artist, stu
dent or scientist is trying to create something new. Creative thinking
involves a new and unique way of conceptuaLising the worLd around
us.
Creativity ProfiLe
Creative people possess the following qualities:
• Unusual awareness of people. events and problems.
• High degree of verbal fluency.
• Ftexibility with numbers and concepts
• Flexibility in social situations.
• OriginaLity of ideas and expression.
• Sense of humour.
• Ability to abstract, organise and synthesise.
• High energy and activity Level.
• Persistence in tasks of interest.
• Lmpatience with routine or repetitive tasks.
Willingness to take risks,
• Vivid and spontaneous imagination in childhood.
, Verification
A -

Illumination
-
A Incubation
Preparation
Orientation

Five step procs of crcitit’e thinking

Stages of creative thinking


Creative thinking is a five stage process:
i. Orientation: The problem must be defined and important
dimensions identified.
ii. Preparation: In this stage creative thinkers saturate themselves
with as much information related to the problem as possible.
iii. Incubation: Most major probLems produce a period when all
attempted solutions appear futile. At this point problem solving
may proceed on a sub-conscious tevel for some time.
iv. Illumination: The stage of incubation is often followed by a
wave of insight and produces the Aha!’ or eureka” experience.
v. Verification: The final step is to test and critically evaluate the
solution obtained during the stage of illumination.
Common barriers to creative thinking
There are three types of barriers that impede creative thinking and
problem solving: Emotional, cultural and perceptual barriers.

Emotional barriers:
O Inhibitions
‘Fear of failure.
Inability to tolerate,
‘Ambiguity
• Excessive self-criticism.
Cultural barriers:
• Value systems that consider fantasy and imagination a waste of
time.
• Being taught that playfulness is an exclusive domain of
children.
• Categorisation tat home and as a society) of reason and logic as
good but feelings, intuition, pleasure and humour as bad

Perceptual barriers include habits leading to a failure to identify


important etements of a problem.
6. Emotions
Being the first in his pharmacotogy class to detiver a presentation, Shahid
was feeling terrified on the podium with the microphone in his hand. Though
the lecture hail was nicely air conditioned, he felt hot As he began to speak,
he could feel a strange sense ofhaving a lump in his throat and a feeling
of discomfort in his stomach. He tried to clear his throat but his dry mouth
and trembling hands were making the task too difficult “Is this the same
tecture hall where I have been attending classes every day and had fun with
my friends: he thought The growing restlessness and sarcastic expressions
on the audiences faces was not a real threat but Shahid felt as ifit was. His
heart was now beating at the rate of 140 bpm and his ctothes were drenched
with sweat The rationat part of his mind totd him that his imagination was
running wild but he remained unable to shake the fear and dread that had
taken over the control ofhis body.

Life would be dull and colourless without emotions. Feelings and emo
tions add pleasure and excitement to our lives. Have you ever waited untit
someone was in a good mood to ask for a favour? If so, you are aware
that emotions have a powerful influence on everyday behaviour. It is a
common observation that it is easier to make decisions when you are in a
good mood, People who are happy are more likely to help others in need.
Similarly we are all aware of the importance of love, optimism, acceptance
and joy in human relationships. However emotions also have their negative
effects as we saw in Shahid’s case.
Definition
The word emotion is derived from the Latin word which means “to move”
and emotions do indeed move us. An emotion is a feeling with its dis
tinctive thoughts, psychobiological states, and range of propensities to
behave. Human emotions can be disruptive (as in Shahid’s case who was
experiencing “stage fright) but often they aid survival. This seems to be
why emotional reactions were retained in evoLution. For details on the
neurobiology of emotions, refer to Chapter 3, The Neurobiological Basis of
Behaviour.
a. Types of Emotions
There are two types of emotions: innate (primary) emotions and
mixed (secondary) emotions.
Innate or primary emotions: The concept of primary emotions was
first given by Robert Plutchik (ig8o) who believed that there were
eight primary or innate emotions: fear, surprise, sadness, disgust, an
ger, anticipation,joy and acceptance. A baby by the end of first year
can express most of the primary emotions.
Secondary or mixed emotions: Primary emotions can be mixed to
give rise to the secondary or mixed emotions. For instance the emo
tion of love results from a combination ofjoy and acceptance. The
mixture of disgust and sadness gives rise to the secondary emotion
of remorse. Jealousy can be considered as a mixture of love, anger
and feat. Most secondary emotions are acquired. Greed, lust, preju
dice, paranoia, hatred, and shame are some examples of acquired
emotions.
b. Expression of Emotions
Facial expressions appear to be central to the expression of emo
tions and have been retained through the evolutionary process.
Body gesture and movements (body Language) also express feel
ings, mainly by communicating emotionaL tone rather than specific
messages. While the language we use to express ourseLves verbaLly
may vary with race and geography, body language and faciaL ex
pressions are Largely universaL
C. PhysioLogicat Differences amongst Emotions
It is usually difficult to differentiate one emotion from the other on
the basis of physiological changes but some differences provide
important cues. For example. people cry when they are sad, some
times even when they are happy, but almost never when they are
angry or disgusted. Establishing the physiological specificity of
emotions does not require that every emotion have a unique phys
iological signature, only that some emotions differ from others in
consistent ways. Finding such evidence has not been easy because
emotions are generally short lived, tasting for only a short white.
Over the years, research suggests that there are several reliable
differences amongst various emotions. One of the most consistent
findings is the tendency for anger to be associated with cardiovascu
lar changes. The heart rate increases with anger, fear, and sadness,
it decreases with disgust. Compared to anger. fear is associated with
lower blood pressure. cooler surface temperature and less blood
ftow to the periphery of the body. Our language reflects some of
these physiological differences. We use phrases such as “blood boil
ing” when we talk about anger but not when talking about disgust.
happiness or sadness. The description “white with fear” reflects the
cooler skin temperature associated with the emotions
U. EmotionaL Intettigence
Most people may find it difficult to identify what emotion they are
experiencing and when. For instance, if a student has failed a sub
stage. he may be feeling a myriad of different emotions: shame, em
barrassment, hopelessness, fear of the future. Now if his best friend
topped this exam, this may complicate his feelings even further. In
this entire scenario, however, if asked, how he is feeling, he may only
respond with: “bad.” Recognizing what particular emotions we are
feeling is a key element of the concept of ‘emotional inteLLigence’.
The abitity of an individuat to recognise their own and other’s
emotions, reason through them and use them to their advantage is
referred to as emotionat intelligence. This is measured as
emotional intettigence quotient or EQ. This will be further discussed
in the section on ‘Inteltigence.

e. Lack of Emotions
Not all of us possess the capacity to express our emotions to the
same degree. Some individuals feet a great difficulty in expressing
their emotions and understanding those of others. This may, inevi
tably. lead to a difficulty in maintaining relationships. This emotional
difficulty is called alexithymia. People with alexithymia are not aware
of their own feelings and may even lack the words needed to
.4,:.

communicate their emotions. A common example in clinical practice


is that of patients who are unable to express their distress, depres
sion or anxiety. They describe these feetings in terms of physical
symptoms such as headache, gas or ‘gala’. These patients more
often than not are silent sufferers of atexithymia. presenting with a
somatoform disorder.

7. Motivation
We alt seek different goals, some more vigorously than others, The same
goat may be pursued for different reasons. While alt of us may want to
be good doctors, our motivation for this may differ. Some may want this
because they would like to treat their patients welL Others may be inter
ested in getting famous and stilt others are concerned with becoming rich.
Our behaviour is driven and pulled towards goats. The driving force which
results in persistent behaviour directed towards particular goats is called
‘motivation’.
Motives cannot be observed directly and are in fact inferred by us after
we have observed people working towards certain goals. n other words,
motives are inferences from behaviour. They are powerful tools for the
explanation of behaviour and allow us to make predictions about future
behaviour.
BiologicaL motivation: The biological motives are rooted in the physiologi
cal state of the body. There are many such motives including hunger, thirst,
sexuat desire, temperature regutation, steep, pain-avoidance and a need for
oxygen. As regards the biological motives, the body tends to maintain a
state of equilibrium called homeostasis.
Sexual motivation: This depends to a large degree on sex hormones.
These hormones organise the brain and body of developing humans so
that they have male or female characteristics. The activation of sexual
motivation in humans, however, seems to be controlled more by external
stimuli and learning than by sex hormones.
SociaL motivation: Social motives are learned motives that involve other
people. Examples of social motives include the need for achievement,
need for approval and need to attain power. Power motivation is when the
goal is to influence, control, persuade, lead and charm others and enhance
one’s own reputation in the eyes of other people. A special form of power
motivation with negative objectives is termed Machiavettianism. It is char
acteristic of people who express their motivation to become powerful by
exploiting others in a deceptive and unscrupulous fashion.
Self-actuaLization motivation: This was first described by Abraham
Maslow, who spent most of his life studying healthy people. Self-actualiza
tion refers to an individual’s need to do what he or she is capable of doing.
Setf-actuatisers’ are people who make the most use of their capabilities,
and are able to maximise their potentiaL The goals may vary from per
son to person. According to Maslow, self-actualization is considered the
topmost in a hierarchy of needs or motives in life. Esteem needs include
the need for prestige, success and self-respect. Betongingness and love
includes the need for affection, affiliation and identification. Safety needs
include the need for security, stability and order. The basic physiological
needs include hunger, thirst and sex. The order in which these are listed
are important since the physiotogical needs must be
satisfied before any of the others can be met A starving man is preoccu
pied with the search for food. He is not bothered about what happens to
morrow as only today’s meat counts. Once he is assured of eating today, he
can begin to worry about his safety needs and thus climb up the hierarchy
by one step. Most of us do not make it to the top of the pyramid partly be
cause of the state of the society in which we live. It is only when our basic
needs are met, that energy is available to strive for greater understanding
of ourselves and our surroundings.
Mastows Hierarchy of Needs in HospitaL
The patient in a hospital environment in the grip of an illness, experiences a
fall to the baseline step of Maslow’s hierarchy of needs. S/he is concerned
solely with fulfilling their basic physiological needs and restoring physical
or mental health. A comprehensive health care plan should, therefore, not
only focus on the management of the disease but also make an attempt to
facilitate the patient’s upward ascent on the hierarchy of needs.
White the provision of food, drugs, bedding and a comfortable ward setting
by the paramedics in the hospital caters for the basic physiotogicat needs,
hospitat administrators provide for the second tier of needs i.e. security
The need for betongingness and tove isjeopardised once the patient is so
lated from home, famity and significant others. The nursing staff is uniquely
placed to fulfil this need. They can provide a surrogate environment that
gives unconditional regard and respect. so the patient does not feet aban
doned with respect to this third tier of needs. Addressing the patient by
their preferred name, rather than bed number 12 or 13 (which undermines
the sense of belongingness and need for bonding). greeting patients with
a smite, staying by their side when they are in distress or pain, and com
forting them are aLl gestures that a sensitive nursing staff ensures in their
interaction with patients.
A patient’s illness, its correlates and consequent disabilities, undermines
their self-respect and setf-esteem. The steps of making him/her wear a
patient’s uniform, taking away alt tiberties of movement, food, and choices,
confinement to a bed or a room and frequent examinations of body parts
(by medicaL students, trainees, residents and consultants) conducted in
groups can be embarrassing for the patient. This take away the fourth tier
of needs of esteem and recognition, as described by Maslow. As leader of
the health team, the doctor should take charge of this most vital need. A
healthy doctor-patient relationship based on mutual participation and in
formed consent can enhance the self-esteem of the patient. A doctor who
greets his/her patients on arrivat; addresses the patient with his/her pre
ferred name and adds sahib’ or ‘soft/ba’ (or culturally appropriate phrase of
respect); seats them respectfully; takes permission before undertaking an
examination or a procedure and looks after them using the bio-psychoso
ciat model of health care, not only adequately provides for the self-esteem
needs of the patient but also enhances adherence to treatment, and the
chances of recovery from itlness.
The diseased state, amongst other influences, also limits a person’s social
and occupational functioning. The need to setf-actuatise and realise one’s
full potential is, thus, also obstructed by illness and hospitalisation. The
ideal health system caters for social and occupational rehabilitation of pa
tients during hospitalisation as well as following discharge. The concept of
tertiary care and rehabilitation following injury and disease augments the
process of self-actualisation. Adding psychological and spiritual
../
(

Maslow’s hierarchy of needs

dimensions to the care plan can enhance it further. This goat is only possi
ble through the implementation of the bio-psycho-social model of health
care. It is the responsibility of health policy makers and pubtic heatth pro
fessionals to develop and implement a health system that ensures fulfiL
ment of unique potentials of the citizens and helps them become
self-actualisers.

Profite of a setf-actualiser
• A doctor has the unique opportunity to achieve his/her own self-actual
ization as well as make a contribution towards the achievement of self-ac
tualization in others. Maslow found that self-actualisers shared a number
of characteristics irrespective of whether they were rich or poor. famous or
unknown, academically distinguished or uneducated. These are:
• Efficient perceptions of reality: Most of Maslows subjects could judge
situations correctly and honestly.
• Comfortable acceptance of self, others and nature: SeIf-actuatisers
were able to accept their own shortcomings as well as those of
others.
• Spontaneity: Maslow’s subjects extended their creativity into everyday
activities. They tended to be unusually energetic. engaged and
spontaneous.
• Task-centring: Most subjects had a mission in life that they vigorously
pursued.
• Autonomy: Subjects were free from dependence on external authority
and tended to be resourceful and independent.
• Fellowship with humanity: SeIf-actuatisers felt a deep identification
with others and the human situation in general.
• Profound interpersonal relationships. They were able to form
meaningful and sustained mutually beneficial interpersonal
relationships.
• Non-hostile sense of humour: Self actuatisers have a wonderful
capacity to laugh at oneself.
• Peak experiences: AtI of Mastow’s subjects reported the frequent
occurrence of peak experiences. These are experiences that are
marked by feelings of euphoria, harmony and deep meaning.

8. Intettigence
Intelligence is hard to define. It can, generally, be described as the glob
al capacity of the individual to act purposefully, think rationatly, and deal
effectively with the environment. In essence, intelligence is the prob
tem-sotving ability of an individuaL
The Wechs[er Adult InteLligence Scale (WAISR) and the Wechster Intelli
gence Scale for Children-Ill IWISC-llI) are the two most common methods
of assessing the IQ of adults and chitdren, respectively. Assessment of IC
includes the assessment of mathematical, verbal, spatial, and mechanical
proficiency.
The measure of intelligence is the Intelligence Quotient CIQ) which is ob
tained by dividing a child’s mental age with his/her chronological age and
multiplying it with 100.

IQ mental age/chronological age x 100.

Extremes of Intelligence
Individuals who enter the medical profession have been found to harbour
an IQ score higher than the average 10 of the population which ranges
from go to 110. Scores below 70 or above 130 occur in only about 5 per
cent of the popuLation. Individuals with such statistically rare scores are
considered exceptionaL Those with scores below 70 may be diagnosed
as mentally handicapped if they also exhibit significant deficits in adaptive
behaviour such as self-care, sociaL skills, or communication. The diagnosis
of a mental handicap also requires that the condition begin before age 18.
In many cases, deficits that occur after age 18 are the result of brain dam
age from traumatic injury to the head and brain. Individuals with 10 scores
about 140 and above may be identified as gifted.

Savant Syndrome: In 1887, J. Langdon Down described a group of mental


ly handicapped children who exhibited speciaL abiLities. Down eventually
became known for his description of Down syndrome but in a book enti
tled Qn Some of the MentatAffections of Childhood and Youth, he offered
a description of Savant syndrome. Savant syndrome occurs in individuaLs
who are severely handicapped in overaLt intelligence yet demonstrate
exceptional ability in a specific area such as aft, memory, or music. These
individuals are often atso gifted with extraordinary memories. For example.
despite very Low scores on tests of inteLligence, one of Down’s patients had
memorised a large number of historicaL facts.
Emotionat IntelLigence
Emotional intelligence is defined as the capacity to identify, understand
and control the expression of emotions. Emotional intelligence abilities can
be divided into four areas:
i. The capacity to accurately identify emotions.
ii. The capacity to use emotions to facilitate thinking.
iii. The capacity to understand emotional meanings.
iv. The capacity to manage emotions.
To be emotionally intelligent is to have interpersonal skills that characterise
a rich and balanced personality. Emotional intelligence includes, as Aris
totle put it. the rare ability “to be angry with the right person, to the right
degree, at the right time, for the right purpose, and in the right way.” The
measure of emotional intelligence is known as Emotionat quotient (EQ.)

Components of Emotional Intelligence


One commonly used version of Peter Salovey and John Mayer’s 1990 defi
nition of emotional intelligence includes abilities in five main areas:
i. Self-awareness: Recognizing one’s own feelings as they occur is the
crux of emotional intelligence. The ability to monitor feelings from
moment to moment is key to insight and understanding one’s own
self. Being aware of one’s emotions makes one more confident when
making important life decisions such as what career path to follow, or
whom to marry.
ii. Managing emotions: Having appropriate emotional reactions is a
capacity that builds on self-awareness. The ability to modulate
negative expressions of emotions such as anxiety, anger, and
depression is a crucial emotional skill. Emotional resilience helps
one to prevail over life’s inevitable setbacks. Those who lack
emotionat self-regulation are continualLy overcome by feelings of
distress.
iii. Motivating oneself: Being able to focus on a goal is essential for a
range of accomplishments. Emotional self-control such as delaying
gratification or controlling impulsivity is crucial in working towards
such life goats. Individuals who can harness their emotions, and
maintain hope and optimism despite frustrations, are generally more
productive and effective.
iv. Recognizing emotions in others: Empathy, another skill based in
emotionat self-awareness, is fundamental to interpersonal
effectiveness. Those who are well attuned to subtle social cues that
indicate what others feel are more successful in personal and
professional settings.
v. Handling relationships: The art of relationships requires skill in
managing others’ emotions. Social competence underlies popularity.
leadership, and interpersonal effectiveness.
EQ
EQ
Measures conttlv1ntece; Measures emoIion Intetigence:
ability to problem soIre1ea,Mm abtrry to manage emotions
10gb
Permanent, fixed at birth Flexible skiT can be learned and
Improved
Neocortex — Umbic stem
Street smort
Confronts threats head on Manages, resolves conflict
Effeciiie dung times of stress

EmotionaL InteLligence and I.Q.


In a sense the human brain contains two minds and two different kinds of
intelligence: rationaL and emotionaL. These two fundamentally different
modes of consciousness interact to constitute our mental life. The emo
tional and rational minds are semi-independent faculties, each reflecting
the operation of distinct, but interconnected, circuitry in the brain. The
complementary relationship and working of limbic system and neocortex,
particutarly of the amygdala and prefrontal lobes, means each is a full
partner in mental life. The emotionaL and rational minds operate in tandem
for the most part. Emotions contribute to and inform the operations of the
rational mind, and the rational mind refines and sometimes vetoes the
input of the emotions. When these partners interact welt, both emotional
intelligence and intellectual ability are enhanced.
There is, at best, a slight correlation between 1.0. and certain facets of
emotional intelligence, small enough to make it ctear that these are large
ty independent entities. When people with high 1.0. struggle in life, and
those with modest I.Q. do surprisingly wett, the difference often may be
attributable to emotional intelligence. Those with an extremely high 1.0. but
low emotional intelligence-or low 1.0. and extremely high emotionat Intel
Ugence are relatively rare. Unlike the familiar tests for 1.0. there is as yet no
corresponding test that measures emotional intelligence, although there
is ample research on each of its components. Some aspects of emotional
intelligence are best tested by studying an individual’s ability at the task.
Empathy can be evaluated, for example, by testing an individual’s accuracy
at interpreting another’s feelings from their facial expressions.
How to Improve Emotional InteLligence
While I.Q is targely under genetic inftuence, emotional intelligence is a set
of skills that can be learnt and enhanced. just like any other skills. Below
are a few guidelines to help a person get started with improving their
emotional intelligence:
Observe how you react to people. Do you rush to judgment before
you know all of the facts? Do you stereotype? Look honestly at how
you think and interact with other people. Try to put yourself in their
place, and be more open and accepting of their perspectives and
needs.
Look at your work environment. Do you seek attention for your
accomplishments? Humility can be a wonderful quality, and it doesn’t
mean that you’re shy or lack self-confidence. When you practice
humility, you say that you know what you did, and you can be quietly
confident about it. Give others a chance to shine put the focus on
-

them, and don’t worry too much about getting praise for yourself.
Do a setf-evatuation. What are your weaknesses? Are you willing to
accept that you’re not perfect and that you could work on some areas
to make yourself a better person? Have the courage to look at your
self honestly it could change your Life.
-

• Examine how you react to stressful situations. Do you become upset


every time there’s a delay or something doesn’t happen the way you
want? Do you blame others or become angry at them, even when it’s
not their fault? The ability to stay calm and in control in difficult
situations is highly valued in alL professional and non-professional
settings. Keep your emotions under control when things go wrong.
• Take responsibility for your actions. If you hurt someone’s feelings.
apologise directly don’t ignore what you .did or avoid the person.

People are usually more willing to forgive and forget if you make an
honest attempt to make things right.
• Examine how your actions will affect others before you take those
-

actions. If your decision will impact others, put yourself in their place.
How will they feel if you do this? Would you want that experience? If
you must take the action, how can you help others deal with the
effects?
9. Personatity DeveLopment
Personality
The word personality originates from the Latin persona”, which means
mask. Personatity can be defined as the deeply ingrained and relativety
enduring patterns of characteristics, behaviour, motives, beliefs, attitudes
and cognitions that an individual possesses. Awareness of our personality
and our ability to recognise ourselves separately from the environment is
the only thing that separates us from artificial intelligence.
While the exact nature of personality is a topic of intense academic de
bate, general consensus states that aspects of personality start to devetop
during childhood and are then strengthened and moulded tilt adulthood.
This chapter sheds light on multiple theories of personality development.
I NW@,
As alt chiLdren develop differentty and each has their own complex cog
nitions and characteristics, no single theory can account for it. Thus, it is
helpful to have a broader appreciation of personality development than
any one theory.

a. Piaget’s Theory of Cognitivó Development


The Swiss psychologist, Jean Piaget believed that alt children pass
through a series of distinct stages in intellectual development. He
believed children construct knowledge for themselves as
‘little scientists’. His theory hypothesised that children advance through
four stages of cognitive development with each stage building on the
previous one.

i. Sensorimotor stage (0-2 years): The child’s inteLtectua[ development


is targety nonverbal in this stage and is mainly concerned with [earn
ing to coordinate purposeful movements (such as neck holding.
crawling and walking) with sensory input, After the age of 18 months
the concept of object permanence begins to emerge. This refers
to the child’s ability to recognise the permanence of objects. For
instance ifs/he is playing with a ball and it rolls under the sofa, they
would not Look for it as they think it no longer exists. For the very
young child out of sight can literally mean out of mind,” This is why
playing peekaboo (“taa) amuses infants, as they think that if you are
hidden, you do not exist and when you reappear. they ate surprised.
The development of object permanence, therefore, is the under
standing that the ball stilt exists and now the child will search for it.

ü. Preoperational stage (2-7 years): During this stage the child devel
ops the ability to think symbolically and use language. The use of
language is not sophisticated, however, and they tend to confuse
words with the object they represent. The child is also egocentric
(they feel everything is about “Me!”) and unable to take the point of
view of other people. During this stage chiLdren also make conserva
tion errors. where they believe that simply changing the appearance
of objects can change their quantity.

iii. Concrete Operational stage (7-11 years): An important development


during this stage is mastery of the concept of conservation as well
the ability to reason logicalLy. Children have learned that pouring
liquid from a talL narrow glass into a shallow dish does not reduce
the amount of fluid. During this stage, a child’s thought starts to
grasp the concept of time, space and numbers. The child starts
using categories and principles.

iv. Format Operationat stage (12 years onwards): After the age of ii.
the child begins to break away from concrete objects and specific
examples. Thinking is based more on abstract ideas and the child
becomes tess egocentric. This stage represents the attainment
of full adult intellectual ability. The older adoLescent is capable of
reasoning and can conceptualise mathematics, physics, and phi
losophy. Piaget argued that attainment of this stage is not universal
and may depend on quality of education, the environment and the
society that the adolescent is living in.

b. Freud’s Psychoanatyticat Theory of Personality Devetopment


Sigmund Freud proposed that an individual’s personatity develops through
a series of five stages stretching from infancy to adulthood. These stages
are called psychosexual stages because each is characterised by efforts to
obtain pleasure centred on one of several parts of the body catted
erogenous zones. According to Freud, the five stages of psychosexual
development are the oral, anal, phallic, latency, and genital stages.
I Oral Stage: Pleasure-seeking behaviour in the oral stage focuses on
the baby’s mouth. Young children can often be seen biting, sucking.
or placing objects in their mouths. Freud hypothesised that if oral
needs such as the need for food are delayed, the child’s personality
may become arrested or fixated. An individual whose development
is arrested at this stage will disptay behaviours as an adult that are
associated with the time of life during which the fixation occurred.
For example, fixation at the oral stage may manifest in behaviours
such as chewing on pencils, smoking or overeating and in person
ality characteristics such as excessive dependency, optimism, and
gullibility.

ii. Anal Stage: From about 18 months until about years of age, the
3
child is in the anal stage. As the child gains muscular control, the
erogenous zone shifts to the anus, and the child derives pleasure
from the retention and expulsion of faeces. The key to this stage
is toilet training. The way parents approach toilet training can have
lasting effects on their children. If the parents are strict and demand
ing, the chitd may rebel, and the result will be fixation at this stage.
Individuals who are fixated at this stage may be overly rigid, obses
sional and orderly as adults and are referred to as anat-retentive.
People who have obsessional traits in personality tend to be per
fectionists. These people can become easily distressed and anxious
when their orderliness or time tables are disturbed.

iii. Phattic Stage: The phallic stage, which begins at about age to
4 5.
is ushered in by another shift in the erogenous zone and the child’s
pleasure seeking behaviour, During this stage, children derive
-

pleasure from fondling their own genitals. Children also begin to


differentiate between males and females. It is during this stage that
children begin to identify with the same sex parent and compete
with them for the other parent’s attention. This is also the stage
where boys devetop a feeling of wanting to possess the mother and
the desire to replace the father what Freud called the Oedipus

complex. Conversely girls develop a simitar set of feelings but for


the father in what is referred to as the Etectra complex. A disturbed
phallic stage is seen in adults in the form of disturbed relationships
with either the opposite gender or both.

iv. Latency and Genitat Stages: At about age 6, children enter a peri
od when their sexual interests are suppressed. This period, which
lasts until the beginning of adotescence, is called the latency stage.
Sexual interests are reawakened at puberty and become stronger
during the genital stage. In this stage, sexual pleasure is derived
from heterosexual relationships. At the beginning of the genital
stage, most adolescents have difflculty developing true affection
and caring for others: they still experience the narcissistic qualities of
earlier stages of development. As they mature, they develop greater
ability to establish such relationships, thus setting the foundation for
adult relationships.

Although Freud’s psychosexual stage theory of personality development


was fascinating and audacious given the conservative era of its concep
tion, it was based upon case studies and hence lacked repeatability and
reliability. The theory was based predominantly on male development and
there was little mention of the psychosexual development of females. It
can also not be verified through empirical research, as constructs utilised in
the theory cannot be measured.

c. Erikson’s Stages of Psychosociat Development


Erik Erikson added to Freud’s theory by concentrating on human develop
ment beyond puberty. He concluded that human personality is determined
not only by childhood experiences, but also those of adulthood.

denlity nerfi’iky

PR[- cUIOUI..- AllcttJ YOUNG MI[[ WR


INFANT IDD([R A11T Afl
(WOOt[A A[R A[

GvIt tnf€iatil

https//sites.googte.com/site/hool?appsychotogy;a/
He described 8 stages of the life cycle fsee table below)
I. Infancy. The first of Erikson’s stages is “trust versus mistrust” and
occurs from birth to 1 year. The child devises either a trusting or mis
trusting relationship with the world around it based on whether its
immediate needs are met. These needs are generally concerned with
physical cravings (food, sleep, and comfort) and feelings of
attachment.
ii. Early Childhood: The second stage of development Erikson is
“autonomy versus shame and doubt” which occurs between 1 to
3
years of age. During this stage children learn to be independent but
only if they are sufficiently encouraged to explore their world and
given freedom to do so. Children with overly restrictive or anxious
parents who restrict their children’s creativity and independent
exploration of their environment, start to self-doubt and become tess
confident.
iii. Late Childhood: From the age of three to six, children pass through
the stage Erikson refers to as “initiative versus guitt.” During this
period of development, children take the initiative to further explore
their environment and gain new experiences. The aspect of guilt
comes about when there are unforeseen consequences involved in
these explorations.
iv. Shool Age. The final stage of childhood development is called
“industry versus inferiority,” and it lasts from age six to 12. In this stage
children learn to read and write and learn specific skills. Children seek
to win approval by demonstrating skills that are valued by the
society and develop a sense of accomplishment. Mastery in these
skills, with adequate support from parents, teachers and peers, brings
about a sense of overall competence. Failure brings about a
sense of inferiority in the child.
v. Adolescence. During the years of 12 to 18, the stage of identity versus
rote confusion occurs as the vital transition from childhood to
adulthood takes place. This is the time when the child evaluates
his/her identity and decides the role he wiLl occupy as an adult.
Erikson claimed that some uneasiness would be felt as the adult tries
to feel comfortable in their changing body and success during this
stage wilt lead to fidelity. This is because the individual will only be
able to accept others who may be ideologically different once s/he
becomes comfortable with whom they are themselves.
vi. Occurring in young adulthood (ages i8 to 40 yrs.), in this stage of
intimacy versus isolation” individuals become more intimate with
each other and explore retationships that lead to long term
commitments with people who are not family members. Success
during this stage leads to a sense of commitment and healthy
relationships whereas resistance to intimacy may lead to isolation.
vii. During mid adult years (40-85) individuals reach the stage of
“generativity versus stagnation.” They become established in society
and their careers as well s give back to their society by educating
their children and working. Failure in this stage by not playing an
accommodating role in society can tead to stagnation and feelings of
worthlessness.
viii. The last and final stage of integrity versus despair” occurs after 65
years of age where productivity deteriorates and individuals reflect on
their roles in life and their accomplishments. Erikson believed that
people who do not feel satisfied with what they have achieved
through their role in society have a tendency to become depressed
and feet helpless.
Although one of the most influential theories of development. Eriksons
work was rather vague in the description of factors that may affect each
stage and of the behaviours that would lead to successfuL completion of
each stage.

AGE PEflIOD ERIKSONS EASIC DEVELOPMENTAL DEVELOPMENTAL FREUD’S


HAZARDS psycHosExuAl.
COMPONENTS TASKS STAGES

infanqv - - Trust Trtrst In others and feelings Abuse. neglect Oral stage
Biahtol8 months vs of security based on how in Infancy. Pleasure
- Mistrust th infant is trgated by the premature or focused on
harsh weaning mouth chewng.
parents/caretakers.
-

sucking and
biting

Early Childhood Autonomy Child begins to view his/her Overprotective Anal stage
Pleasure is
l8monthstothree - vs self as an individual In their parenting can lead
focused on
years Shame own right apart from their to selFdoubt in
bowel and
parents but still dependant the chIld. bladder
on them, Toilet training and Conditions that emptying
controlling temper lead to the child

I tantrums begIns feeling inadequate


or learning skills
like walking or
talking are S
hazard In this
stage

Late Childhood initiative Child feels g sense initiative Oaedy strict Phallic stage
3-Sysars -. vs toundarsl.andtheworld. disdplinv bythO 3-6 years
and their main word goes parents can PISxJrB


-
- Guilt
- -
from 7’JoI to whyT suppress the become
pleasure
ftation of adult behaviOur child’s

Ir
-.

zones. Gender
occurs. The child’s identity forms.
- imagination starts to Oedipus.
Electra
- - . - Reality testing begins complex
occurs

Industry Child feels Failure due to Latency


School Age 6 years
vs accomplishment personal
6-lZyeere
inferiority in learning and limitations or
puberty
mastering extreme
Playing with
now skills. competition leads the same
to sense of gender.
inferiority and dormant
poor work habits sexual feelings

Identity Identity development of Society fails to Genital Stage


Adolescence Puberty
12-20 years vs who one is pfoeide well
onwards
Role Confusion defined roles or
Seauci
roles are interests
undesirable mature
leading to
contusion

Capacity for love, mutual isolation or lack of


Young Adulthood Intimacy personal relations
vs dafdoliort commitment to
18-25 years -

latIon workand’telatlonships:
impdrsohal reiationshlps.

Adulthood Generetivigr Creativity, productivity. FaIlure to perfect


25’65 years vs concern for others, self- and master
Stagnation indulgence. developmental
impoverishment tasks, leading to
Of self stagnation
--

Acceptance Of the worth Ineblityto find


Old age integrity
and uniqueness of oneg llfe Meaning in life
6syearstOdeeth vs
Despair sense orioss. contempt tor
Qthers
-

--
Determinants of Personality
The study of personality cannot only be based on how it is manifested and
changed overtime but rather what factors determine, or affect personality.
Charles Darwin in his ‘Origin of Species’ proposed that a person’s person
ality originates in the mother’s womb with genetic and hereditary factors
contributing to our primal personality traits. Scholars like John Locke
argued that the human psyche and psychological traits are formed pri
marity due to the environment and surroundings that the person is raised
in. Angyat (1941) believed that the determinants of personality are neither
exclusively organismic nor environmental but rather a combination of both.
When studying the development of personality, therefore, one must take a
holistic approach and consider biologicaL psychological and social factors.
Biotogical factors
Biological determinants of personality include inherited traits and
characteristics as well the workings of the nervous system. glands
and blood chemistry (Kumar, 2015). Factors such as heredity are de
termined at birth and can be referred to as physical structure, mus
cle density, facial structure, attractiveness that one inherits from the
biological makeup of their parents. Basically. the way we look plays
an important role in how others treat us and how we in turn, treat
others. Krueger and Johnson (2008) found that genes contributed to
individual differences in alt of five of the Big Five personality fac
tors. As we grow older we acquire different aspects of our parents’
personalities. Siblings may differ in their appraisals of situations as
well as in their overall outlook on life. Proponents of lhe nature over
nurture argument would suggest that these personality differenc
es are due to the differences in the children’s’ observation of their
parents’ behaviour.

Sociat/Environmetita[ factors
Factors such as social structure and the environment play a vital
role in the formation of a child’s personality. Numerous studies have
shown that a constructive learning environment, social support and
a nurturing family, all positively affect the development of personal-
ity. Environmental factors may include the culture that children are
raised in and the societal norms that guide behaviour. Environmental
factors can be used to manipulate/suppress the genetic predisposi
tion to express undesirable emotional and behavioural responses of
individuals. In 2000, Collins et at found that factors such as parent
ing and other early life experiences affect the expression of genetic
personality traits.

Personality Types: Relevance to Clinicat Conditions


Many readers would have observed that it is a common tendency for peo
ple to categorise others on the basis of their personal characteristics. Even
in your class there are bound to be ‘shy’ students and the ‘sociable out
going’ type. Classifying people into types is one device by which we try to
make sense out of others’ behaviour and anticipate how they wilt act in the
future. The notion that people can be classified into certain types is one of
the oldest ideas about personality Personatity classifications date back to
400 BC. Hippocrates, a Greek physician grouped people into four types
on
the basis of their temperaments. These were sanguine (cheerful, vigorous,
optimistic), metancholic (depressed). choteric (hot- tempered) and
phlegmatic (slow moving. unexcitable).
According to the Type theory of personality, people can be divided into
extroverts and introverts. Extroverts share a tendency to be outgoing,
friendly and talkative whereas introverts can be described as people who
share characteristics such as shyness. sociaL withdrawal and a preference
to spend time alone,

Personality types and heart disease


Heart disease has traditionally been linked to smoking, obesity, diabetes
and inactivity. In recent years, however, a link between heart disease and
personality types has emerged. Two specific behaviour pattern types are
now known to be associated with increase and decrease chance of coro
nary artery disease; Type A and Type B.
Type A persons are driven and competitive. They tive under
constant pressure, which is usually setf-created. They also take on
multiple activities with rigid deadLines to meet. These people may
function weU. most of the time and may be competent and efficient.
Under stress, however, they are likely to become hostile, anxious
and disorganised. They may feeljittery and irritable prior to
examinations and are constantLy in a hurry’. The underlying
cause of an increased incidence of heart disease in Type A
personality is the constant outflow from the sympathetic nervous
system in response to stressors.
Type B persons are easy-going, non-competitive, placid and cooL
They remain calm and composed under stressful states. Given
a task to do, Type-A’s usually perform near their maximum capacity
no matter what the situation is. Type B persons only work hard
when given a deadline. Interestingly, when placed in Long lasting
stressfuL situations over which they have little controL Type A
personalities tend to give up. They exhibit he[p[essness and be
come less responsive than Type B personalities.

PersonaLity types that can influence heatth personnel:


There are distinct personatity types that doctors and medical students may
come across in hospitals and in their personaL lives. KnowLedge and under
standing of personality profiles can help them dealwith these individuals
more effectively. The foremost amongst personality types that may be
difficult to handte are those with histrionic or sociopathic personatity traits
or types. These individuals have the following characteristics;
• manipulative behaviour
superficiaL charm
• loud in their expression (both in language as well as dressing and
demeanour),
• tell fantastic stories (mostly false),
• exaggerate their symptoms,
• take advantage of naive medicaL students (asking for uncalled for
investigations, petty cash, small time favours, drugs. cigarettes).
• prefer to be treated by junior doctors (for fear of being discovered or
identified).
The common ploys used by such patients include stories of being mal Cluster A CMad”)
treated by their own family, work setting, or the society in generaL These
Paranoid: Distrustful, preoccupi
individuals readily share sob stories of being ‘cheated” or ‘abandoned” in with conspIracy theones
life. They may say they are in dire need of money to pay for a lifesaving
procedure or drug. A dangerous strategy they use is appearing to develop
‘strong feelings of love” for the doctor or the medical student. The story of
Schizotypal: socially odd, magica
Miss X in section on psychological reactions in doctor patient relationship thinking
in Section 2, revealed that she had histrionic traits. The case report goes
on to reveal the complications that can arise if a medical student/doctor Cluster B (“Bad”)
is caught unaware. While it is crucial to be wary of such personalities and
Histrionic: provocative, dramatic
to be constantly on the lookout, it is equally important not to judge, pun shallow emotionally
ish or prosecute them. These individuals have deep rooted psychological
conflicts and complexes and require professional psychiatric help. This fri’—I [ I

help should be offered to them at the earliest. It is always better to hand


Nardssisstic: Grandiose, self loving
over the care of patients with such attributes to the senior members of lack empathy, sense of entitlement
the health team. Clinicat descriptions of their behaviour must be entered Borderline (emotionally unstable): Impu
in their case histories, for future reference. The table briefty describes the swe. unstable intense relationships. proni
to deliberate self-harm, drug abuse
important features of the various types of personality disorders.

Obsessive compulsive: perfectionist


inflexible morals, preoccupied with
rules/orderliness
Chapter 3
Neurobiological Basis of Behaviour Dependant: submissive, helpless,
clingy, excessively needy

The evolution of our understanding of the mechanisms of human be Personatity disorder


haviour has undergone many different stages (see table). The most recent
and perhaps the most dependable expLanation is the neurobiological basis
of behaviour. While we study exhaustive texts on iow the brain works in
terms of our most basic functions, we as students tend to gloss over the
anatomy. biochemistry and neurology of what makes us who we are.
The purpose of this chapter is to shed light on some of these areas. It is,
however beyond the scope of this book to go intodetails of neurobiolo
gy and cognftive neuroscience for which the reader must consult a more
exhaustive text and ongoing research articles on the subject. It must be
brought to the reader’s attention that the human brain is not like a house
with rooms, in which certain functions take place only in certain areas. It
is, in fact, the connectivity between various areas of the brain that leads to
this. The following text works to highlights some of these connections, and
should work to create awareness that such connections exist.”
We do not h’now everything, and not alt of what we betieve we know is
correct,” Anon
-

Religion

ch

Neurosdence

- -,,---_j
Ge

Progression of our understanding of human behaviour


Emotion
Emotion may be defined as a feeling with its set of unique thoughts, elec
trical and biochemical changes, and the range of behaviours it can Lead to
(See Section C, Chapter 2). There are two different kinds of emotions: inher
ent and learnt. Inherently found emotions include those that all human be
ings are born with the capacity to experience. These can range from anger,
sadness, fear, love and amazement. These emotions are derived primarily
from our basic human drives such as hunger, thirst, sex and the need for
social interaction.
The Learnt emotions include: envy, pride, guilt, pity. resentment and af
fection, alt of which are housed in the frontal cortex. The more complex
emotions like jealousy, greed, paranoia, and lust are a mix of the inborn
and learnt emotions. There is Less known about the neurobiology of these
highly complex emotional states but there is a fair amount of research on
the inborn and learnt emotions.
Neuroanatomy of Emotions
The neuroanatomical site for the manufacturing’ of emotion is the limbic
system. The range of behaviour patterns that may result from a given emo
tion is decided by the frontal cortex.
The frontal cortex is the smart” part of the brain where planning for the
future, decision making and executive functions are carried out. It is the
chief executive of the brain, and what makes us “ashraf-ut-mahhtooqaat”
(The Superior Species). The prefrontal cortex is what allows us to behave in
a civilised fashion even when our emotions urge us to do otherwise. This is
essentially what it means to be human, as animals have a limited capacity
to perform these functions. These functions are performed specifically in
the prefrontal cortex which can be divided into 3 parts: Dorsotateral,
Ventromedial and OrbitofrontaL
The dorsolateral prefrontal cortex is primarily involved in managing
cognitive processes. moral decision making. inhibition of emotion, altruism
and telling the truth (or tying!). It allows the individual to have the ability to
entertain multiple ideas at the same time and plan for the future. It is also
involved in motivation, attention and the drive to complete tasks. It is par
ticularly active in tasks that require deductive and syllogistic reasoning.[3;1
It also plays a role in the behavioural response an individual has to anxiety
and works to prevent in behaviour likely to be harmful to the individual. [61
The ventromedial prefrontal cortex has a role in emotional regulation and
decision making involving morality. [31 This was famously seen in the case
of Phineas Gage who suffered damage to his ventromedial prefrontal
cortex. [41 It is the part of the prefrontal cortex that allows one to [earn from
their mistakes. It also allows people to detect irony, sarcasm and detect
tying in other people. The ventromedial prefrontal cortex allows people
to use emotions to make moraljudgements. This can be explained using
the Trolley dilemma (Thomson,1g86), in which a person is asked to save
five people from certain death by a trolley crashing into them by pulling a
switch redirecting the trolley to kill only one person. The majority of people
felt that it was moral to let one person die and save five people. If, howev
er, people were asked to physically push one person in front of the trolley
to save the five people, they refused, despite the end goal of both being
the same. The ventromediat prefrontal cortex used the emotions telt in the
situation to come to a decision, as seen by Green et al. (2011). When the
amygdala becomes active in situations causing anger the ventromedial
prefrontal cortex is able to control the urge to act impulsively. [51
The orbitofrontal cortex (so called because it represents the area above
the orbits) plays an important role in emotional regulation, specifical
ly anger management. It functions to associate possible outcomes of a
certain behaviour by weighing pros and cons based on reward and pun
ishment and makes a decision. Iii In one study people with more outgoing
personalities were found to have higher volume orbitofrontal cortex, while
more introverted people were found to have a higher volume ventromediat
prefrontal cortex. [2]
A part of the cingulate cortex, the anterior cingulate cortex (ACC) is the area
where the integration of emotional input with attention occurs. It, therefore,
controls emotional arousal and emotional self-controL It is involved in gen
erating empathy and social awareness and becomes active when individu
als indulge in acts of bravery. [41
To summarise, the prefrontal cortex (the smart’ part of the brain), takes into
account the emotions elicited and decides the most logical and rational
action to take in a situation. 171
The limbic system, specifically the amygdala, works to integrate inputs
from the thalamus and frontal cortex with outputs involving the endocrine
and autonomic nervous systems. This results in the expression of emotion.
• The amygdala is the main mediator of the fight, flight or freeze response.
It is invo[ved in the mediation of aggression, fear, sexuaL orientation and
social interaction, including the number and kind of people one is friends
with and interacts with in a social context. It also regulates personal space,
in that it is stimulated when another human being gets “uncomfortably
close.” [81
The right lobe of the amygdala houses negative emotions such as fear and
anxiety. The left lobe, on the other hand, mediates and stores both positive
emotions such as pleasure and happiness and negative emotions such as
fear. In one study, poLitical conservatism was linked to increased volume of
the right amygdala. [9]
Distinct gender differences have been seen in the male and female amyg
data. The mate amygdata has a larger right lobe, while the female amygda
Ia has a larger left lobe. As the right lobe is associated with negative emo
tions and action, men are more likely to react physically in situations that
are emotionally stressful. The left lobe is important in the recall of emotion
ally charged memories and details, leading to more intense thought, which
may explain why women are more likely to not react in physical ways to
emotional stress.
The amygdaLa also mediates the formation of memories that have an emo
tional content. When sensory input is received, it is relayed by the thalamus
to both the frontal cortex and the amygdata. The amygdala then checks
with the hippocampus to see if there is an emotional memory of a similar
experience. If one is found, we tend to react in a similar fashion, if not; the
prefrontal cortex kicks in to make something of the novel experience and
forms a new response. Activity of the amygdala has been linked to clinical
depression, anxiety disorders, and posttraumatic stress disorder. liD]
Despite the tact that the prefrontal cortex is the smart part of the brain, the
amygdala exerts far more control over it than the prefrontal cortex does
over the amygdata This can be understood it the amygdata is equated
with the moon which despite its size may eclipse the sun. This “limbic
eclipse” is th reason that “love is blind” “ghussa akal ko khaajaata hai”
(anger makes you irrational) and durr se behosh hona” (fainting with fright)
occur. As seen in each of these expressions, an emotion that is essentially
less ‘smart” completely clouds the judgement of the smarter higher centre,
the frontaL cortex.

Neurochemistry of Emotions
Biochemically, certain neurotransmitters have been [inked to feeling spe
cific emotions. Amongst the most highty researched of these are serotonin,
dopamine, oxytocin and norepinephrine.
Serotonin has been hailed as the feet good” neurotransmitter in the brain.
It, among other things, leads to a feeling of well-being and satisfaction liii.
Lower than normal levels have been linked to clinical depression. This is
evidenced by the improvement of symptoms of depression with the use of
serotonin reuptake inhibitors tSSRIs & SNRIS) h21 [131.
Dopamine is the main neurotransmitter released in anger, aggression and
excitement. Dopamine levels also increase when an individual experiences
complex emotions [Ike paranoia and jealousy. Dopamine is also the
primary neurotransmitter in the reward pathway. The reward pathway is
activated by drugs that lead to increased dopamine, gambling (where
winning leads to dopamine release), and playing video games. This is also
why all of these activities are addictive. Dopamine is responsible for the
kick” that normal people experience in moments of excitement and
anticipation when they are looking forward to something. Higher than nor
mal levels of dopamine can lead to difficulties in impulse control,
aggression and eventually, psychosis [‘41 [151.
Oxytocin is the neurotransmitter of love and bonding. It exists in the body
both as a hormone and a neurotransmitter. It was previously studied as
being released in targe amounts during childbirth, and immediately after,
for breastfeeding. In newer research it has been hailed by many as ihe
propagator of the human species.” This is not only due to its role in child
birth and reproduction (large amounts are released during sexual inter
course) but also due to its ability as a neurotransmitter, to cause ‘pro-
social” behaviour 1161. Higher levels of oxytocin have been found t make
humans more likely to make decisions that promote the well-being of a
group rather than the individuaL The bond, kinship and love that any two
human beings share is mediated by the presence of oxytocin, which is why
it has been called the love hormone. It has also been known to decrease
stress and anxiety and lead to a greater degree of trust, altruism and feel
ings of safety between people. In a study published in the 2013 in the
Proceedings of the National Academy of Sciences of the USA Journal
(PNAS), administration of oxytocin was revealed to cause greater fidelity
between couples, making men more likely to be monogamous and more
sensitive to other people’s emotions t171.
It is thus helpful to see dopamine as the neurotransmitter released when
one first becomes intrigued by a mysterious stranger, oxytocin as released
when one falls in love and serotonin as causing the sense of satisfaction
and well-being when we live “happily ever after.”
I I
a’n
Lepreon ftorFit

—-—
5ota*
-

-Oytocm

Progression otour understanding of human behaviour

The Limbic System


Corpus callosum

COMPONENTS IN
THE CEREBRUM
COMPONENTS IN Cingulate gyrus
THE DIENCEPHALON
Anterior group of Parahippocampal
thalamic nuclei gyms

Hypothalamus Hippocampus

Mamillary body
References
i.Kringelbach, M. L. (2005) “The orbitofrontal cortex: Unking reward to he
donic experience. Nature Reviews Neuroscience 6: 691-702.
2. Schoenbaum G, Takahashi Y, Liu T, & McDannatd M (2011). “Does the orbi
tofrontat cortex signal vaLue?” Annals of the New York Academy of Sciences
1239: 87-99

3. Hu, C; Jiang, X (2014). “An emotion regulation role of ventromedial pre


frontal cortex in moratjudgment.” Front. Hum. Neurosci 8: 873. doi:10.3389/
fnhum.2o14.oo873.
4. Nicolle, A. & Goet, V. (2013). What is the role of ventromedial prefrontal
cortex in emotional influences on reason? In I. Btanchette (Ed.), Emotion
and Reasoning. Psychology Press.
5. Schwartz, Carl E., et aL “Structural differences in adult orbital and ventro
medial prefrontal cortex predicted by infant temperament at 4 months of
age.”Archives of general psychiatry 67.1 (2010): 78-84.
6. Carlson, N. (2012). Physiology of Behaviour filth Ed.). Hartow: Prentice
Hall.
.
Chan, R. C. K., Shum. D., Tou[opoutou, T. & Chen, E. Y. H., R: Shum, D: Tou
lopoulou, I; Chen, E (2008). “Assessment of executive functions: Review of
instruments and identification of critical issues.” Archives of Clinical Neuro
psychology. 2 23 (2): 201-216.

8. Kennedy, Daniel P., et al. “Personal space regulation by the human amyg
data.”Nature neuroscience 12.10 (2009): 1226-1227.
9. Kanai, Ryota, et at “Political orientations are correlated with brain struc
ture in young adults.” Current biology 21.8 (201;): 677-680.
10. Sheune, Yvette I., et at “Increased amygdata response to masked emo
tional faces in depressed subjects resolves with antidepressant treatment:
an fMRI study.” Biological psychiatry 50.9 (2001): 651-658,
ii. Peirson AR, Heuchert JW. Correlations for serotonin levels and measures
of mood in a nonctinica[ sample. Psychot Rep 2000:87:707-16,
12. Flory JD, Manuck SB, Matthews KA. et at. Serotonergic function in the
central nervous system is associated with daily ratings of positive mood.
Psychiatry Res 2004:129:11-9.
13. Muldoon MF, Mackey RH, Williams KV, et at. Low central nervous sys
tem serotonergic responsivity is associated with the metabolic syndrome
and physical inactivity. ] Clin Endocrinol Metab 2004: 89:266-71.
14. Schultz W (2007). “Multiple dopamine functions at different time cours
es.” Annual Review of Neuroscience 30: 259-88.
15. BjOrktund A, Dunnett SB (May 2007). “Dopamine neuron systems in the
brain: an update.” Trends in Neurosciences 30 (5): 194—202.
i6. Lee HJ, Macbeth AH, Pagani JH, Young WS (Jun 2009). “Oxytocin: the
great facilitator of tite.” Progress in Neurobiology 88 (2): 127-51.
17. 01ff, M.. FrUling. J. L., Kubzansky, L. D., Bradley, B., Ettenbogen. M. A., Car
doso, C & van Zuiden, M. (2013). The role of oxytocin in social bonding, stress
regulation and mental heatth: an update on the moderating effects of context and
interindividuat differences. Psychoneuroendocrinotogy, 38(9), 1883-1894.
Motor and Sensory Regions of the Cerebral Cortex
Primary motor cortex Primary sensory cortex
(precentr& gyms) (postcentml gyms)

Somatic motor association area


(premotor cortex)\\ / Somatic sensory association area

Prefrontal cortex
Visual association
//_ area

Sri

Broca’s area—
(production of speech)

Visual cortex
Auditory association area
I
/ -

Wemicke’s area
--

Auditory cortex (understand speech)

Language
Language forms a quintessential part of what it means to be human.
Human language is unique compared to the communication techniques
used by other animals. Other animats communicate using a finite num
ber of ideas that can be expressed. Human language. on the other hand,
is open-ended and productive meaning. Humans, can, thus produce an
infinite range of expressions from a finite set of elements to create new
words and sentences. [ii
Speaking is the default mode for language in all human cultures. Humans
produce language using control of the lips, tongue and the rest of the vo
cal apparatus. They are able to differentiate spoken sounds white attaching
contextual meaning to the sounds. In other words we can understand what
is being said, and what it means in a certain situation. These abilities are
[inked to a neurological apparatus to acquire and produce language. 12]
While, language is processed in various areas in the human brain, the two
areas that are mainly involved in language processing are Wernicke’s area,
located in the posterior section of the superior temporal gyrus, (temporal
lobe) and the Broca’s area, located in the posterior inferior frontal gyrus
(frontal lobe) of the dominant hemisphere. Wernicke’s area is used for lan
guage comprehension and Broca’s area is responsible for language artic
ulation. Language is the only human behavior that has two controlcentres.
thus the famous idiom pehte tolah phir boto’ (think before you speak).
Language comprehension is known to consist of three distinct steps. The
first step known as phonological processing, takes place in the frontat
lobes, where individual sounds, such as vowels are recognised. The sec
ond step is known as texicat processing and is localised in the left temporal
lobe. Lexical processing matches heard sounds with words or sounds that
already exist in the individuat’s memory. In the third step, known as seman
tic processing. words recognised during the lexical processing step are
connected to their meaning. Semantic processing activates the middle and
superior gyri of the left temporal lobe. Brain areas required for the under
standing of the conceptual content ot words are distributed all over the
cortex, as been in brain imaging.
Language production occurs in the opposite direction from language
comprehension. This basically means that it proceeds from the cortical
semantic processing to the teft temporat lexical processing, finally going to
the phonologicat processing area (for speech) or the graphomotor system
(for writing).
Prosody, the emotional and ffective component of language. or ‘body
language.” appears to be [ocalised in the right hemisphere. Linguistic
functions such as intonation, tone, stress, and rhythm form part of pros
ody. Prosody provides information on the various non-semantic aspects
of communication, such as the emotional state of the speaker, the nature
of what is being said as welt as the presence of irony or sarcasm. These
nonverbat elements are created using motor operations of the face, mouth,
tongue, and throat and are associated with Broca’s area in the left frontal
lobe. The processes needed to understand these nonverbal elements oc
cur in the right-hemisphere perisylvian area, particularly Brodmann area 22.
Language exhibits a high degree of hemispheric lateralisation. Damage
to the right inferior frontal gyrus diminishes an individual’s ability to use
the nonverbal aspects of communication such as conveying emotion or
emphasis by voice or gesture. Damage to the right superior temporal gyrus
diminishes an individual’s ability to understand the nonverbal
meaning of the voices and gestures of others.
In summary, as is largely known, the left hemisphere is the part of the
brain that is mathematical, calculating and analytical. To put it in terms
of an unfair stereotype the left hemisphere is the ‘accountant”, while the
right hemisphere is the “artist’ where lies the appreciation of art, music and
literature. In other words. while the Wernicke’s area that processes lan
guage is in the left hemisphere (in most right handed individuals), the right
hemisphere is where the processing of the context, sarcasm, irony, body
tanguage, facial expression and intonations occurs. In any given spoken
sentence, therefore, the left hemisphere hears what is said while the right
understands how it is said.

References
1.Hockett, Charles F. (1960). ‘Logical considerations in the study of animal
communication.” In W.E. Lanyon: W.N. Tavolga. Animal sounds and animal
communication. pp. 392-430.

Trnsk, Robert Lawrence (1999). Language: The Basics (2nd Ed.).


2..
Psychology Press.
Memory
Memory is defined as the mental capacity to encode, store, and retrieve
information. [ii
Clinically we are concerned with three, testable periods of memory which
have distinct anatomical correlates: Immediate memory functioning over
a period of seconds: recent memory applicable over a scale of minutes to
days: and remote memory ranging over time periods spanning months to
years.
Immediate memory can be understood as the ability to follow a train of
thought. It is divided into phonological and visuospatial components. The
phonological component is localised in the left hemishphere and the
visuospatial in the right.. Immediate memory is often related with recent
memory using the concept of working memory. Working memory is defined
as the ability to store information for several seconds while other cognitive
operations take place using this information. The dorsolateral prefrontal
cortex is required for working memory along with the certainty with which
the information is known. [2] Some researchers locaUse working memory
predominantly to the left frontal cortex.
Three brain structures are critical to the formation of memories: the medial
temporal lobe, certain diencephalic nuclei, and the basal forebrain.
The hippocompus is part of the medial temporat tobe. Adjacent to the ante
rior end of the hippocampus is the amygdata. The amygdala and the hip
pocampus work in tandem to store emotional experiences. The amygdala
rates the emotional importance of an experience and activates the level of
hippocampal activity. This phenomenon is called the memory enhance
ment effect and is the reason why emotionally intense experiences are
etched in memory. This is why most of us would find it hard to recall what
we had for lunch last Tuesday, but remember in great detail what the menu
for a close family wedding three years ago was or the exact sequence of
events when vie were in a car accident.
The amygdata also plays a role in the formation of long-term memory by
moduLating synaptic plasticity, which helps to retain a memory. It helps to
visuaLise memory as a grassy path, that when used repeatedly becomes a
place of common fare.
Long-term memory for learning events is not formed immediately. These
memories are slowly made over time, through a process called tong term
potentiation These are enhanced and made permanent when they are
potentiated by experiences. This is also why you don’t remember last
minute cramming after a few weeks. It is, therefore, imperative for a medi
cal student to ensure that all forms of knowledge are enriched by seeing a
patient or dealing with a relevant clinical experience in real Life settings of a
hospital. The lesser the gap between studying and clinical experience, the
higher the chance that the knowledge will remain in memory for the long
term.
The association areas are required for the formation of memory for motor
acts. The performance of a new action requires feedback from the sensory
cortex and association areas. Neuroimaging has shown activation of the
large parts of the cortex, during performance of unskilled acts. When per
forming repeated activities initially the medial temporal lobe is activated,
however with time the performance of the act results only in the activation
of the premotor and left parietal cortex. This phenomenon is known as the
corticalisation of motor commands, The repeated acts of [earning how to
pass a catheter or intravenous injection ensure perfection and thus the
idiom practice mal?es perfect!
Within the diencephaton. the dorsal medial nucleus of the thatamus and
the mamillary bodies appear necessary for memory formation.

References
1. Gerrig, Richard J. & Philip G. Zimbardo. Psychology And Lite, ;6/e. Pub
tished by Allyn and Bacon. Boston, MA. Copyright (c) 2002 by Pearson
Education. Reprinted by permission of the publisher.

2. Sadock, Benjamin J., and Virginia A. Sadock. Kaplan and Sadock’s synop
sis of psychiatry: Behavioral sciences/clinical psychiatry. Lippincott Wit
hams & Wilkins, 2011.

ArousaL
Arousal is defined as the physiotogicat and psychological state of being
reactive to stimuli. While there are many different neural systems involved
in the establishment and maintenance of this state, research has shown
the involvement of mainly five systems. each originating in the brainstem.
These systems are based on five neurotransmitters: norepinephrine, ace
tylchoUne, dopamine, histamine, and serotonin.
The noradrenergic system. originating in the locus coeruleus causes wake
fulness by the release of norepinephrine. The cholinergic system based in
the pons and basal forebrain causes cortical activity and alertness. Both
the dopaminergic system and the serotonergic system’s neurons project
into the limbic and prefrontal cortex and are important for mood control
and regulating motor movements. The histaminergic system neurons proj
ect into the cerebral cortex, thalamus, and the basat forebrain, stimulating
the release of acetytchohine into the cerebral cortex. All these systems are
related to the development of a feedback mechanism to establish and
maintain arousal.

(,rW\

I
\

Image is taken from Christof Koch ((20 p4) “Figure 5.1 The Chotinergic
Enabling System” in The Quest for Consciousness; A Neurobiological
Approach, Roberts & Co., p. 91 ISBN.’ 0974707708. with permission
.

from the author under license


There have been attempts to explain different temperaments in humans by
examining variations in a person’s brain stem, Umbic system, and thatamo
cortical arousal system using Electroencephalogram (EEG). [1] Limbic sys
tem activation has been [inked to neuroticism, with high activation showing
high neuroticism.[2] High cortical arousal has been shown to be associated
with introversion. People with high extraversion and low neuroticism have
been shown to have the towest overa[[ levels of internal arousat. Converse
ly people with high extraversion and high neuroticism have been shown to
have the lowest intrinsic thatamocortical excitation.

References
1.Robinson, David (6 November 2000). UH0w brain arousal systems de
termine different temperament types and the major dimensions of per
sonality.” Elsevier. Personality and Individual Differences i: 1233—1259.doi:
10.1016/s0191-8869(oo)0o2;1-7.
2. Robinson, David; Gabriet, Katchan (22 February 1993). Personality and
Second Language Learning(PDF). Personality Individual Differences i6 (1):
143—157.dOi:10,1016/0191-8869(94)90118-x. Retrievedl2 November 2012.

Steep
Steep is a recurring state of altered consciousness, imperative to nor
mal brain and body function. Approximately one third of our lives is spent
asleep. Steep is characterised by decreased awareness and interaction
with surroundings, lowered sensory activity and inhibition of voluntary
muscles.
The awake state is characterised by beta and alpha waves on the electro
encephalogram. Beta waves are commonly seen during active mental con-
centration whereas alpha waves are seen when a person c[oses their eyes
and relaxes.
Each stage of the sleep cycle is characterised by a specific wave form on
EEG. Mapping the transition of sleep from one stage to another is known as
sleep architecture and this changes with age.
The normat sleep cycle is divided into rapid eye movement (REM) steep
and non-rapid eye movement (NREM steep). REM sleep is a period of high
levels of activity in the brain and a level of physiological activity similar to
when the person is awake. During NREM sleep, physiological activity is less
than when an individual is awake. NREM has four stages (1-4).
Hypnograrn
,,•.Jlri4’1 .dnhiig

w \:

St1ge 2

MIiiighi 11311

By!, RazerM, CC BY-SA 3.0, https://commons.wikimedia,org/w/index.


php?curid=;7745252

Stages of Steep
When we ñrst fall asleep, we enter into a NREM cycle which lasts forgo
minutes. These go minutes are composed of the following four stages:

Stage 1 is the lightest stage of sleep with theta waves on EEC and is char
acterised by a sense of calmness, slow pulse, respiration and a decrease in
blood pressure. This constitutes 5% of the sleep cycLe.

Stage 2 shows sleep spindles and K complexes on an EEG. This stage


constitutes about 45% of the sleep cycle making it the largest portion of
sleep time.

Stage 3 and 4 are characterised by the delta waves or slow wave steep and
is the deepest and most relaxed stage of sleep. Sleep disorders such as
nightmares, night terrors, sleep walking and bed wetting occur during this
stage.

After about 90 minutes, the NREM cycle is followed by REM sleep. Rapid
Eye Movement (REM) steep is characterised by a saw-tooth EEG showing
beta, alpha and theta wave patterns. Dreaming occurs during this part of
sleep along with an increase in pulse, respiration and blood pressure. REM
periods of 10-40 minutes, occur about every 90 minutes throughout the
night.

Sleep disorders
The quantity and quality of steep changes with age. The elderly show more
frequent awakenings during the night white teenagers tend to remain
awake alt night and steep during the day. Changes in steep are believed
to be due to changes in internal body rhythm, (catled Circadian Rhythm),
emotional stress, physical illness and drugs. The chronic use of sedatives
and hypnotic is not known to improve steep. On the contrary they are
implicated in many of the dyssomnias. Due to stow metabolism the elderly
tend to accumulate more of the sedatives in their bodies which may lead
to delirium, daytime drowsiness and loss of equilibrium.
IF

Primary Sleep Disorders


Dyssomnias

Primary Insomnia Difficulty falling asleep, staying asleep, or


sleeping but feeling as if one has not rested during sleep

sle.p1n.ss lp)ng q .

Narcolepsy Sleep attacks during daytime.


Daytime naps relieve sleepiness

“••-•

i RNathlng ,alated . Abnormal breathing during si.ep


sleepdIiorder r 1.ads tesi pdIsuptlonvid diytlme sleepiness ,

- -______
Circadian rhythm Disturbance of sleep due to a mismatch between a persons
sleep disorder intrinsic circadian rhythm and external sleep wake demands

Parasomnias

Nightmare disorder Repeated episodes ofscary dreams that


wake one from sleep usually during REM sleep

d.pb’ rdungslee en bslMdusl may


sit up or scream end appear extremely frightened.
Occurs during delta sleep

Sleep walking disorder Recurrent sleep walking often


coupled to other complex motor activity

There are two types of sleep disorders: primary and secondary steep
disorders. Primary disorders occur as a direct result of disturbances in the
cycle. Secondary steep disorders occur as a consequence of
steep-wake
other disorders such as depression or due to a general medical condition
(e.g. pain) or substance abuse.

Sleep hygiene
The following factors contribute to improvement of sleep:
• Sleeping and waking up at around the same time daily
(even on weekends!)
• Increased physical activity in the afternoon and early evening hours
• Cooler room temperatures are more conducive to sleep than warm
temperatures
• Light bedtime snacks that have calcium and small amounts of sugar
• Evening relaxation routines such as progressive muscular relaxation
and evening prayers.
• Avoidance of long naps, especially during the later part of the day.
Making and cleaning your bed every day.
• Getting into bed only when ready for sleep.
• Eating at reguLar times daily and avoiding large meals near
bedtime.
• Avoidance of sensory stimulation at night by substituting TV and
cellphone usage with light reading.
• Avoidance of caffeine and fizzy drinks in the evenings.
Avoidance of excessive smoking in the evenings (as nicotine is a
stimulant).
• Avoidance of stimulant drugs such as amphetamines, cocaine and
MDMA.

Steep induction
Sleep control is a mental technique that you can use to enter normal physi
ologicaL sleep any time without the use of drugs, using the following steps
• Lie down in bed. ctose your eyes and graduatty guide your mind to
visualise a chatkboard. You witt mentatty have chatk in one hand and
an eraser in the other. Mentatty draw a targe circte on the chatkboard
Then draw a big X within the circle, You witt then proceed to
erase the Xfrom within the circte starting at the centre of the X and
erasing towards the inner edges of the circte. Be carefut not to erase the
circle in the teast.
• Once you erase the Xfrom within the circle, to the right and outside of
the circle write the word “deeper.” Every time you write the ward “deeper”
you will enter a deeper level in the direction of healthy sleep. Write a big
number 100 within the circle.
Proceed to erase the number 100 being careful not to erase the circle in
the least. Once the number 100 is erased to the right and outside of the
circle you wilt go over the word “Deeper”
• Every time you go over the word “Deeperyou wilt enter a deeper
healthier level of mind going in the direction of normal natural healthy
steep. You will continue using numbers within the circle on a descending
scate until you enter a normal natural healthy physiological sleep.
Whenever you enter sleep with the use of steep control you wilt awaken
at your customary time or you can remain asleep for as long as you desire.
When you wake up, you will feet well-rested and refreshed.

Attention to detail
It is important that you mentally draw a large circle not a small one, You
shoutd make the numbers in the circle large enough to reach the edges.
Then erase them carefully paying attention to details. If you are paying
close attention to details like this you will not be thinking of the day’s prob
lems and letting them keep you awake. In fact what you will be doing wilt
be boring, so boring that you will go to sleep rather then keep doing it. If
your attention wanders and you forget what number you are on, just start
again with any number. If you feel that the technique may not be working
properly for you, review the instructions and make sure you are doing it
exactly as instructed. Perhaps you are not erasing completely, for instance.
Once you are successful, you can start experimenting to see if you can find
a variation that works better for you. At first you may need to go through
quite a few numbers before you go to sleep but the next night you wilt go
to steep more quickly. Eventually you will be able to fall asteep in only a
few moments with this technique. It witt become very effective if you
persist until you succeed.

Coping with Insomnia


In primary insomnia it is useful to encourage regular habits and exercise
and discourage indulgence in tobacco, caffeine and alcohoL If insomnia is
secondary to another condition, this should be treated and general mea
sures to promote sleep (discussed above) should be advised. The use of
short term hypnotics, such as benzodiazepines, shoutd be avoided due to
their high addictive potential. Hypnotics may be prescribed for a few days,
in severe cases only. Withdrawal of hypnotics can lead to insomnia that is
as distressing as the originat sleep disturbance. Prolonged use can lead to
the development of dependence, tolerance (needing ever increasing dos
es to achieve the same effect) and impaired performance during the day.

Effects of
Sleep deprivation
- Irritability
-
Cognitive impairment
- Memory lapses or loss - - Increased heart rate variability
- Impaired moral -
Risk of heart disease
judgement
Severe yawning
Increased
-

I-
-

Hallucinations
-
reaction time
- Symptoms similar -Decreased accuracy
t0ADHD p.
-Tremors
- Impaired immune -Aches
system

- Growth suppression
- Risk of diabetes - Risk of obesity
Type 2 -
- Decreased
temperature

i-
I
- -.-—

SAMPLE MCQ FOR SECTION C

1. A medical student who has just failed his Anatomy viva returns to his
room in the hostet and sees that his roommate is watching a movie. He
immediately starts yelling at him for making too much noise. This
reaction is most likely due to use of the defense mechanism of:

a) Suppression.
b) Displacement.
C) Identification.
U) Reaction formation.
e) Repression.

2. A 14 month otd has to be taken to the hospitat for her vaccination. As


soon as she enters the hospitat, injection is even brought in, she sees
a doctor and smetts the antiseptics: This is an example of:

a) Systematic desensitisation
b) Classical conditioning.
c) Operant conditioning.
d) Shaping.
e) Modelling

. A first year medical student reports to a psychiatrist saying that she


faints with fear at the sight of the cadavers and therefore cannot
attend her anatomy lectures. She even has troubLe walking by the
anatomy lab atone. The psychiatrist determines that she has a phobia.
The best way to treat her phobia would be:

a) Anxiotytics prescribed for the first two years of medical coLlege


b) To ask her to consider giving up medicine as a career
c) Write her a report excusing her from attending the Anatomy lab, for
medical reasons
d) Systematic desensitization.
e) Reassurance

4. What kind of memory involves the recatt of the indications for


endoscopy?

a) Iniplicit memory.
b) Semantic memory.
c) Explicit memory.
U) Episodic memory.
e) Procedural memory.
I
5. Which of the following can be used to make a clinical assessment of
dementia?

a) History taking and physical examination


b) Mini mental state examination (MMSE)
C) Blood CP and Urine RE
d) MRI Brain
e) CT Scan Brain

SampLe Short Essay Question For Section C

Qi. Briefly describe the stages in the formation of memory. What type
of memory is required to remember how to suture?

Q2. What are the different stages of sleep? How do they appear on EEG?

Answers

i. b
2. b
3. U
4. b
5. b
ECTION D
ciology and Anthropology

— OUTLINE4
Sociology and Health
• Famity Social groups Social class Gender
• Child Rearing Practices Roles Sociot Support
• Religion •Stigma Sich Rote
• Death and Dying
Comptiance

Anthropology and Health


Culture Culture and Public Health

Chapter 1
Sociology and Anthropology
Introduction
Sociology and anthropology form an important knowledge base in the
understanding and practice of holistic medicine (Section A). It is necessary
to understand the social and cultural setting as welt as the person and the
physician to have a holistic view of health, or its absence resulting in dis
ease. This includes culture, values, norms and health belief models. These
factors are important for a health professional to study as they influence:

• rates of spread of diseases,


• preventive strategies,
• pathways to health care facilities,
• choices and options of treatment,
• chitd rearing practices
• social barriers to health care
• compliance or treatment adherence,
• cultural understanding of the disease
• illness behaviour
• utilisation of health services.

The study of sociology and anthropology can help a health professional in


making a sound psychosocial assessment (at par with biomedical mea
sures) to determine a comprehensive list of determinants of health and
disease.
Case ILlustration

Muhammad idrees was a security guard who was severety injured white
guarding a government office in Pakistan which was attacked by mititants
Both his Legs were fractured and his spinal cord was injured His treatment
costs were covered by his brothers pooting their monetary resources dona
tions by his emptoyers and Zakat from his neighbours His fractured Legs
were operated upon successfully but he became paratyzed due to his spinat
cord injury As he could no longer continue working as a security guard his
brothers and parents set up a smalL cornershop in his village His etdest son
stopped his education after Matricutation to assist his father in the shop.
He had to rely on his famiLy and friends to help with his tong term medical
care as there was minimal organisational and government support A local
NGO provided him with crutches and Later a wheel chair He went through
a period ofgriefand depression after the incident but finally recovered with
the support ofhis famity and friends When asked how he had coped with the
trauma and the permanent disability he responded that he had accepted
Allahs will and considered himselffortunate to have the opportunity to save
so many innocent tives that could have been tost in the attack

Greg Martin was a fire fighter from Belgium, who sustained injuries to his
spinal cord while rescuing people from a high nse building which was on
fire His wife and two children lived with him at the time As he was medically
insured, he received very good medical care and a motorised wheel chair: His
family and fnends initially prowded emotional cnd moral support but were
constrained by their own responsibilities and obtigations He retired from the
fire department with futt financiat benefits and received physical rehabilita
tion and counselling services to help him with adjustment after the accident
He alsojoined a community group ofretired fire service veterans to help him
cope with this life change His wife and chiLdren were very supportive and
caring He felt proud that he had been abte to hetp so many peopte during
his work as a fire fighter and this helped him cope with the disability His wife
had to take care ofthe home and the children mostly by herselfnow as wetl
as look after his increased medical needs after this accident She also re
ceived counselling to help her deal with this added stress Greg went through
a phase ofdepression andreceived psychotherapy for the problem At three
years follow up he was receiwng treatment for alcohol dependence that
he developed after the accident and his marital retationship had become
strained
1. SocioLogy and HeaLth
Sociology is the scientific study of patterns and systems of human inter
action. It is undertaken by focusing on the social structure of a society and
social interactions amongst its inhabitants.
The role of sociology in behavioural sciences is to study social determi
nants of health and disease. The following are some of the important sociat
determinants of health and disease:
Family
Family, as the basic unit oa society, is the building block upon which a
society is constructed. Family constitutes two or more people related by
blood, marriage or adoption living together. The traditional joint family
where grandparents, parents, siblings and cousins lived together is be
coming increasingly rare in Pakistan. A more common form now is the
extended family, where parents, grandparents (mostly paternal), and oc
casionally, unmarried aunts and uncles live in the same household, There
is a growing trend to tive as a nuctear famity. where the husband and wife
live with their children. The single parent family, where one parent, often
the mother, resides alone with her children, is as yet a rarity in Pakistan but
may not remain so in future. Famity structure and functioning is the way a
family is organised in terms of its boundaries, emotional bonding and inter
actions outside the family. These parameters greatly influence health and
disease patterns and reactions to both. A family that has cLosed boundaries
allowing minimal or no interaction with other units of the society. leads to
enmeshment or loss of boundaries within the family. This consequently
results in serious mental and physical health issues. A family with open
boundaries, with strong ties to each other forms the basis of healthy inter
actions as well as better physical and emotional health in its members.
The way we respond to symptoms. cope with stress, follow health ad
vice, or tet our illness affect our close relationships, points to the central
role family has in influencing health and disease. [[ness does not exist in
a socially neutral environment. An ill member of the family changes the
structure and functioning of this basic unit. This influence is maximally seen
in psychiatric disorders, chronic illnesses, head injuries, stroke, and cancer.
The influence of infectious diseases like hepatitis, tuberculosis, sexually
transmitted diseases, HIV-AIDS, skin diseases and neurological disorders
can be equally severe. In our social settings (with minimal or no govern
ment social services available for health care), the entire burden of care
fatls on one or all members of the family. A sensitive health professional is
familiar with the concept of ‘carer’s burden’ and its impact on the health of
the carer. Another influence that family exerts on a member’s disease is
the role it plays in decision making as regards pathways to care, choice of
treatment and even adherence to treatment. Health professionals should
regularly draw genograms (family trees) of the patients family to familiar
ise themselves with the structure and functioning of the family.
A health professional aware of this immediately wins over the key mem
ber in the family. They will then include him/her in the decision making
process to ensure their positive input. A typical example of this process
is the role that a pregnant lady’s mother plays during the pregnancy and
throughout the reproductive health process. A similar role is played by the
reactions of the in-laws towards the pregnant mother. Supportive handling
by the husband and in-laws can yield huge benefits in improving the
SOCIOECONOMIC FAMILY EXTENT OF
BACKGROUND COMPOSmON URBANISATION

FAMILY STRUC1UPE

ROLE POWER &


STATUS F.NMLVPELAONS

I I
DY AGE PYGENDEP

I
STAT U S OF
WOMAN. CILD 1I

IMPLICATiONS FOP

FAULT STASIUTYI
INDMDUAL

Conceptuat framework for understanding the rote of family as an


agency for attainment of health and disease

physical and mental health of the mother and foetus. Ihs is why the bio
psychosocial and integrated care models identify family as an important
partner in the assessment and the therapeutic process.
Social groups
A group is formed when individuals join together for a common pur
pose that cannot be achieved atone. Primary groups are small, intimate
social groups. These are commonly seen amongst family, friends and/
or like-minded people. An example of a primary group is the ‘committee
group’ bound by financial interest (money pooling) that also ensures social
interaction (‘kitty parties’) on a regular basis. Religious organisations also
form primary groups to spread their message. People who go to the same
mosque also behave as a primary group. These primary groups can play an
influential role in promotion of health and disease. They do this by serving
as a support system, influencing beliefs about health, referral pathways,
lifestyles changes and even preventing or causing spread of disease.
Secondary group is a larger groups where face- to-face interaction
amongst members of the group may not be possible, yet the common
ality of interest or purpose joins the members. Secondary groups can be
formed on the basis of communities, religious, cultural, politicaL or sec
tarian allegiance. The advent of social media has made the formation of
secondary groups far more common through social networks. Secondary
groups have a great deal of public heatth development potential. Health
parameters in Gitgit and Baltistan have improved on account of the positive
role by one such secondary group.
Seconday group

Smaller in size Usually large in size

High level of intimacy amongst members Less intimacy amongst members

Relationships are personal Relationships usually formal

frequent interactions between members Limited contact between members

Members are accepted regardless Members are accepted on the basis of


of who they are what they can do for the group

xample Family, group of friends Example Political party

Social Class
ALL societies have ways of pLacing people in social strata. These can be
based on weaLth, education, inheritance or other criteria. Social grading
may often have direct relevance for health care provision, interpersonal
communication, outlook on Life and knowledge. Providers of health care
must be aware of the prevailing division of the society into social cLasses.
The Western method of dividing a society into six classes is as fot[ows:
I: Professionats
II: Managers and Technicians
NI: Non-manuat-skitted
IV: Manual SkitLed
V: Partly Skilled
VI: Unskilled
Due to the remnants of the caste system stilt prevalent in the rural and
semi urban sections of the society, this division may not be retevant to us.
In our society individuals are classified on account of their land holding and
professions into
I. Feudal,
II. Businessman,
III. Technician.
IV Farmer.
V. Labourer,
Social class in cities in Pakistan is often determined by wealth. power, area
and size of housing. social connections or networking, and types ofjobs.
Health parameters may greatly vary amongst social classes. Social class
es may differ in types and patterns of diseases, prevalence of infectious
and lifestyle diseases, longevity, quatity of life and disabilities. The type
of health infrastructure utilised by each class is the single biggest health
variable, Lower socioeconomic classes rely largely on public sector health
facilities or are at the mercy of non-professional (yet culturally endorsed)
health workers such as quacks. omits, and charlatans. The higher social
classes largely rely on private health care infrastructure or treatment in
other countries.
From a behavioural sciences perspective it is useful to divide these sec
tions of the society into the advantaged’ and the disadvantaged’. This
division is based on access to education, scientift and modern health
services, clean water, sanitation and shelter.

Gender
Gender is reinforced at the family Level through socialisation to determine
• the male and female behavior in social life. Larkoy rotoy nohi hain (“boys
don’t cry’), and torkiyon oisoykaproynahipehnti”C’girls don’t dress that
way”) are statements commonly given to children to help them make
appropriate gender associations. Gender as a determinant of health and
disease is debatable but clear differences exist in terms of patterns, and
health parameters. On the whole, men are at a higher risk of dying earli
er (shorter life expectancy), and women are at a higher risk of ill-health.
Females report a higher number of symptoms in a clinical consultation and
are at a 2-3 times higher risk of developing anxiety and depression related
disorders. Females also seek medical and spiritual help far more com
monly than men. This could be one reason for their longevity as compared
to men. It is also interesting to note that marriage is a protective factor for
health in males and a risk factor for female health.
ChiLd Rearing Practices
Parents and families shape the behaviour and lifestyles of their children by
a combination of rewards and punishment (operant conditioning) and by
modelling behaviour which the child emulates. The growing middle class
means that more people are able to afford domestic help to care for their
children. With the advent of the nuclear family, children who were previ
ously cared for by aunts, uncles and grandparents are now being cared
• for by a domestic servant/maid. The implications of this family system are
many and manifest. The joint famity system as a melting pot of various in
dividuals and their experience was a place parents would have a chance to
learn from the mistakes and advice of other parents raising their children in
close proximity. In the absence of books about raising children in our part
of the world, training/workshops on parenting or experienced individuals,
new parents must learn the do’s and don’ts of the most important and diffi
cult job one can do, from trial and error. One recurring problem seen in this
generation is that of permissive (apologetic) parenting [see tablel. The last
generation of parents practised primarily authoritarian parenting, where the
parents’ word was law, and children were meant to be “seen and not heard.”
Punishments were handed out to those who disobeyed and the primary
emotion was fear. The result was well-behaved children, who were afraid to
voice their opinions. Partly as a reaction to this, when these children grew
up and became parents, they decided to approach parenting in a com
pletely different way to their own. Their children are the Little rulers of the
house: their wishes are always catered to and their word is law. The result is
children who are badly behaved and have a sense of entitlement.
One advantage of nuclear families is consistent parenting. This may not be
the case with joint families, where multiple authority figures such as the
grandparents may contradict each other.
A serious problem with families in which domestic servants are the primary
care-givers for all or part of the day, is that these children are at a high risk
for physical, emotional and sexual abuse. There is also little opportunity
7

Type of Parenting Description

----,,r- V

Parents impose rules and expect


i’.4-
.

Authoritarian
complete obedience.’

Permissive Parents submit to children’s demands

V
Parents demand obed’i
Autttatiw
also responsive totheirchlldren

for emotional and intelLectual development. n the absence of profession


al child care with both parents working, however, there may be no other
choice. Parents, especialty mothers who are doctors who may spend up
to 36 hours at a stretch on call and away from their children, may become
guilty for their absence and try to overcompensate in the time they do
spend with their children. They may do this by trying to stuff their children
with food, or even worse, spend all their time with their children “teach
ing” them coursework or completing homework. The problem with this is
that time previously spent with parents learning vatues. and attitudes and
having fun is now entirely monopolised by “studying.” This may lead to the
child resenting the learning process.
Effective child rearing depends on tailoring the demands to the intettectual
stage of development of the child and providing clear and consistent re
assurance and rewards. However some children seem to be “difficult” from
the very start. Such a child is characterised by a negative reaction to events
and objects (meal, stranger, new toys), magnified emotions, slow adapt
ability to new situations and irregular biological functioning (e.g. sleeping
and eating). Specialised family therapy techniques and behaviour therapy
can greatly reduce the degree of stress in a family with such an offspring.

Rotes
Roles consist of a set of expectations about how people should behave in
various circumstances. The doctor’s role, for example, is that of a carer who
is scientific, impartiaL knowledgeable and courteous. The patient’s role
(sick role) involves being excused from various obligations, a commitment
to wanting to get well and to following medical advice. Rote conflict refers
to when one individual has multiple roles which have conflicting demands
on the individuaL An example of this when a doctor has to run a busy clinic
(fulfil her rote as doctor) but also has to manage her household responsi
bilities (fulfil her role as homemaker) and take care of her children (fulfil her
role as a mother).

SociaL support
Research shows that having social support ameliorates the effects of
physical and psychological stress and hastens recovery from surgery and
illness. This social support may be in the form of family members, friends,
work colleagues or other agencies that provide emotional and/or practical
support. Support may also enhance adherence to health advice and reha
bilitation. it is, therefore, the duty of a doctor practicing holistic medicine to
mobilise social support around a patient as an essential therapeutic strate
gy. This can come from amongst family members, friends or volunteers
or formally enrolled social workers from the community. A social worker in
charge of mobilising and optimising social support for a patient must be
recognised as an important member of the health team.

Retigion
Retigion may be considered a collection of beliefs and practices that are
external expressions of spiritual experience. Spirituality may be considered
an orientation towards or experiences with the transcendental or sacred
• dimensions of life. It is possible for peopte to engage in religious activi
ties independent of having spiritual experiences. Similarly certain people
consider themselves intensely spiritual without being religious. Religious
practices such as regular prayers and fasting are an essential component
of almost all formal religions, especially the Islamic faith. Western co-re
lational studies have shown a positive association of religious/spiritual
involvement with improved health outcomes and longer life spans. Reli
gious/spiritual minded individuals suffer less from cardiovascular disease
and hypertension. They are more likely to be engaged in health-promot
ing behaviours, They also have a decreased risk of depression, anxiety,
substance abuse and suicide. They are better at coping with illness and
have better health-related quality of life. Despite this, doctors must never
make recommendations for patients to follow various religious practices
from their personal faith. This practice grossly undermines the respect that
every health professional must have for the patients’ religious beliefs and
what they might find comforting. It is, therefore, enough to highlight the
research based positive inftuence of faith, spirituality and religion in healthy
life styles and coping with stress and the challenges of chronic illness and
hospitalisation.

Stigma
Stigma is defined as a mark of disgrace or having a” shameful difference.”
Some stigmatised conditions such as infertility, delay in onset of menstru
ation, congenital malformations and physical deformity may be obvious.
Other conditions such as epilepsy or mental illness may have a stigma that
is hidden but just as severe. People witb stigmatised illnesses are socially
rejected, which can compound their difficulty. Often the families of such
individuals are also rejected by society. Health professionals, especially
doctors, should be aware not only of this additional stress on patients and
their families but also that they may be one of the few sources of support
and advice for these people. A common example of this is the public’s stig
ma against the mentatly ill due to the association of mental illness with vio
lence. The belief that people with mental illness are dangerous and unpre
dictable is a popular misconception which arises from sensational media
reporting. What the media fails to report is that only a very small minority of
mental patients commit violent or serious crimes, a finding which has been
established by several studies. Also it must be stated that the vast majority
of people with mental illness are not mentally ill.
Stigma can delay the detection of an illness, obstruct provision of scien
tific management and undermine the social, occupational and economic
standing of the patient. Some methods of dealing with stigma include
reaching out to stigmatised groups (such as the mentalLy ill) and integrat
ing their care in general health care and the use of media to launch destig
matising campaigns.
Sick RoLe
The sick rote invotves being excused from various obligations and duties,
and not being blamed for being ilL It occurs when a patient continues to
maintain the tote of an unwell person long after the initiat ittness has settted.
A patient adopting the sick rote, continues to have symptoms that are not
related to his/her previous illness and cannot be attributed to any other
disease or disorder. This exaggerated response is often an indication of
underlying anxiety. rote contusion, unresolved conflicts and personality
vulnerabilities. Social factors such as positive or negative reinforcement of
the sick rote can be disastrous and may tead to the patient becoming an
invalid.
The ideal intervention includes early detection, removal of reinforcing
factors, and withdrawal of the perks of the sick role. It is important that the
patient is neither confronted, nor bLamed. S/he needs to be reassured
of the benign nature of their symptoms and made to see the benefits of
health, The symptoms should be sensitively listened to, but not attended,
explained or rationalised. Uncatted for investigations and over enthusiastic
symptomatic treatment can further reinforce the sick role. The use of ‘gold
en injections’, ‘spirit ammonia’ or multivitamins (“taaqat ka sherbet’3 and
other dramatic measures shoutd always be avoided. It may help to invotve
psychiatric services in the assessment of the patient and rule out underLy
ing conflicts, stress or the possibility of a latent disease.
It is important to note that both over reporting of symptoms (as in a sick
role), and under reporting are influenced by socioeconomic factors. What
these symptoms mean to the individual at a psychotogica[ level is also of
significance.

Death and Dying


Death of a patient, in our cutture is seen as ‘will of God’ (Attah ki marzy). This
should never stop a doctor from undertaking a scientific enquiry/audit into
the causes of death in a patient. The attribution of death and disabitity to
nature is fast giving way to the influence of the west, where more and more
people may seek a scientific understanding of the cause of death. Such
a change is tikety to lead to the culture of suing health professionals in a
court of law for what may be perceived as negligence. SociaL institutions
of grieving such as ‘iddat’ ‘sog and ‘phuree’ provide psychosocial relief to
the aggrieved and hetp resolve grief. There are atso times when the friends
and family of the deceased are unwitting to accept the outcome and resort
to viotence against the health care providers. This is usuatty the conse
quence of a breakdown of communication between the treating doctor
and the family of a terminatty itt/a patient who dies suddenly. A constant
update on death being a possible outcome in such patients should be
shared with the famity from the very beginning and at regutar intervals of
the management process.

Impact of sociat factors on Treatment Adherence (CompLiance):


Failure to foltow heatth retated advice (sometimes termed “non-compli
ance”) is widespread. According to the WHO up to 50% patients do not take
medication as advised. This includes taking medications incorrectly or not
at all, forgetting or refusing to make essentiat behavioural changes and
persisting in behaviours such as smoking that may jeopardise their heatth.
FaiLure to adhere to treatment is determined by:
• patients age,
• socioeconomic status
• education status
type & chronicity of illness,
lack of social support
• patient-doctor trust
• miscommunication and ignorance of side-effects of treatment
• failure to understand the doctor’s advice
depression
• cost of drug
• dosing frequency
• multiple medication use
Non-compliance is counter-therapeutic and an economic drain. It should not
be underestimated. As a result of their failure to adhere to recommended
treatments, patients might become more seriously ill, and treatment-resis
tant pathogens may develop. Failing to recognise patients’ non-adherence
may prompt physicians to adjust medication dosages, and to be misled in
their diagnoses. Practitioners and patients tend to become frustrated by
non-adherence, and the time and money spent on medical visits is wasted.
Some of the clues which shoutd alert a physician that a patient might not be
adhering to the treatment plan are as follows:
• Indifference and lack of involvement in the treatment/healthcare pLan
• Appearance of unquestioning obedience
• Depressed patient

Lack of response or inconsistent response to treatment
• Confusing clinical picture
Treatment adherence, or the lack of it, is often on account of social reasons.
Advice from a neighbour, a wise man siyana’, or another patient is often the
basis of the path that a patient or the family will take.
Essential elements required to improve a patient’s compliance are sum
marised below:
• Accurate communication of information between patient and doctor.
• Emotional support for the patient
• Awareness of patient’s health belief models
• Choosing an acceptable course of action to which a commitment can
be made.
• Focus on the overall quality of life of the patient
• Development of a specific plan to implement the regimen
• Recognition of the patient’s depression or hopelessness
2. AnthropoLogy and HeaLth
Anthropology
Anthropology is the study of evolution of civilizations, their social charac
teristics. languages, cultures and ways of life. A health professional com
mitted to the biopsychosocial and integrated models of hea[th is a provider
of culturally sensitive heatthcare. This is why anthropology is relevant to
behavioural sciences.
A health professional must be aware of and accept cultural differences;
collect data about the patient’s culture, and adapt the scientific care plan
accordingly. S/he must also remain aware of and understands his/her
own biases and attitudes. For example, some doctors may find themselves
giving more attention and care to patients able to give histories in English,
rather than patients from a rural background. A culturally sensitive doctor
is interested in learning all the cultural variations in the patient population
and then tries to merge his/her professional knowledge with each patient’s
health beliefs. S/he needs to interact as directly as possible with individu
als from diverse cultural backgrounds and collect valuable data about their
norms, values and beliefs regarding health and diseases.
In a culturally sensitive health care approach social aspects are treated at
par with biological determinants of disease. The social dimension of health
is thus an integral part of history taking, assessment, diagnostics and ther
apeutics. The understanding of terms like culture, subculture, beliefs, val
ues, and norms, society, family, social class, social roles, and child rearing
practices is basic to this form of health care.
Culture
Untike att other creatures, human beings are unique in that they are abte to
transmit ideats, knowledge, betiefs, vatues, and patterns of behaviour from
one generation to the next. This sociat heritage is catted “culture. It is what
makes our species human and sets us apart from other primates. As one
anthropologist has put it: “Without the presence of culture, conserving past
gains and shaping each succeeding generation to its patterns. homo sapiens
woutd be nothing more than a terrestrial anthropoid ape, slightly divergent
in structure and slightty superior/n Thteltigenc& (Linton in ‘The Individual and
Society’ 1936).
CuLture is the set of values, mores, beliefs and perceptions of the world
that are passed on from one generation to the next. It is thus our social her
itage. We share it with other members of our society as our common world
view. It is our culture that makes us behave like those around us and thus
makes our behavior acceptable to others.
Culture is dynamic and, therefore, continues to evolve, although never
at the same pace as scientific, economic or technological progress. It is
revealed in people’s language, art, architecture, religion, norms, values
and health care beliefs and practices. ft is not inherited biologically, but is
learnt from the environment. Culture provides one with a sense of identity.
encourages group survival and gives its members a useful picture of the
universe. Pakistan is a country that brings together a variety of cultures in
its four provinces, Gilgit, Baltistan and Kashmir.
Subcultures may develop within a culture. These result in a distinctive set
of standards and behaviour patterns in which a small group within a larger
society operates. People living in and around shrines, with their distinctive
behaviour patterns and routines are an example of a subculture, The typ
cal example of subcultures relevant to health are the language, learned
acceptable behaviour patterns and a shared view of the world that prevails
in hospitals and medical colLeges. Cultural influences are seen in many
beliefs and practices of doctors as well, in addition to their patients. For
example, the doctor knows best; patients must comply with the doctor’s
orders if they want to get better. If they do not listen to the doctor, they
do not want to get better, and therefore, are not worthy of their time.” This
belief is part of the doctor/ hospitaL subculture.
Most people believe that their culture is superior to all others. Cultural and
societal biases, prejudices and ethno centrism must, therefore, never be
confronted in a doctor-patient relationship. They can best be addressed by
understanding the following:
The easiest response to transcultural conflict is to behave as if it does
not exist, and is not a barrier in the doctor patient relationship.
Every individual is expected to behave according to unwritten ethnic
customs. These traditions and the diversity of behaviour should be
accepted, and catered for, in outpatients. wards, waiting areas and
therapeutic interactions.
One
can make mistakes in transcultural interactions but one should
readily accept and acknowledge them and learn never to repeat
them.
Understanding Culture in Health Care
Beliefs, values, norms, mores, folk ways and laws form the fabric of society.
They are the basis for social control and the ‘do’s and donts’ of cultures
and subcultures. They tend to evolve and change but hardly ever com
pletely transform ih a single lifetime
Beliefs are tenets with a shared meaning in a culture that are held to be
true e.g. in most Islamic cultures, the benevolent role of God in the healing
process is an established belief. This is reflected in all doctor- patient inter
actions which nearly always end with a mutual prayer such as “Only Allah
granteth health” (Allah sehat denay wala hai).
Values are those aspects of a culture that are held in high regard, are
desirable and, therefore, worthy of emulation. A doctor’s deatings with
patients must never be commercial’, ‘elders always know better’, ‘the sick
must always be visited in the hospital’ are commonly held Eastern values.
Culturally sensitive doctors must keep beliefs and values in mind as they
may need to cater for them in the administrative running of wards. This
may save unnecessary worry over matters such as families not adhering to
‘visiting hours’, or ‘far too many visitors’, and relatives asking details about
the health and prognosis of the patient.
Norms refer to principles of right and wrong that govern acceptable and
unacceptable behaviour in a society. Abortions, euthanasia, intake of alco
hot, cannabis and heroin, nursing of femate and mate patients in the same
wards (except in emergency or intensive care settings) for example, are all
against local norms. A doctor needs to have a flexible approach regard
ing a patient’s beliefs, values and norms and try not to pass judgments
on them. It is equally important, however, to not break professional norms
(such as that of confidentiality) and not let the two clash.
Norms may be divided into three types on the basis of the kind of disap
proval that resutts when they are violated: fotkways, mores, and taws,
Fotkways are customary patterns of everyday life that specify what is
socially correct. These are social niceties such as referring to health profes
sionals as “Doctor Sahib”, offering a seat in the bus to a lady or an elder, and
not calling parents or elders by their names.
Mores are considered vital for the group’s welfare and survival, thus mak
ing them very important. They define what is morally right and wrong and
as a consequence, their violation results in strong disapproval and even
severe punishment. Respect for sexual boundaries by a health profession
al, for example, is a more. Persons who violate mores may be ostracised,
imprisoned, or killed. Conformance to mores is taken for granted and most
people in a society accept them without question. Unlike laws, mores are
not formally written but are respected with the same fervour and
commitment.
Laws are formally compiled written rules and regulations coded after
debate and deliberation. They are then enforced by organisations com
posed of persons authorised to use force if necessary. Laws are similar to
folkways and mores in many ways but are far more adaptable to changing
times.
Many mores are incorporated into law and serve to reinforce it. In fact,
laws are most effective when they are rooted in the mores. Abortion is
legally prohibited as well is considered against the mores in Pakistan. At
times, however, mores and folkways may conflict with the laws, The classic
example is that of the jurisdiction of Pakistan Medical and Dental Council.
This is a formal body that enforces regulations pertaining to health care
in the country. It is empowered to take punitive action against medical
practitioners when found guilty of ftouting a certain regulation. or in cases
of malpractice. It does not, however, currently have jurisdiction over prac
titioners of alternative medicine or even aamils, charlatans, and self-styled
hakims who openly practice their own methods of ‘health care’. This is on
account of the permissiveness to these practitioners by the mores of our
culture.

Inftuence of cu[ture on heatth care


Culture determines beliefs about health and disease. The way Pakistanis
view health, causes of disease, the meaning of various symptoms. ap
proach therapeutic interventions and determine their relationship with
health professionals is largely influenced by their culture.
For the predominantly Muslim population of the country. health and dis
ease, like life and death are predestined by the will of Allah. The process
of healing and cure similarly rest with the Almighty. Virtually every health
outlet, and the prescription letterheads of physicians have Arabic inscrip
tions from Holy Quran that endorse the same belief. This unfaltering belief
forms the basis of trust in the physician as a mere executioner on behatf of
God. Practitioners of health care for the same faith also share this cultural
and religious belief with their patients.
Our culture tends to classify diseases into those caused by supernatural
forces, stresses of life, unhealthy lifestyles and habits, environmental fac
tors and germs and chemicals. Social and family problems and feuds are
often seen as causes of the evil eye (“nazar”) or a curse leading to illness.
In our culture, most psychiatric disorders, epilepsy, fits and related chang
es in consciousness, and headache are attributed to supernatural causes
or ‘evil eye’. Heart conditions, diabetes, and hypertension are attributed
to stress, poor dietary habits, and unhealthy lifestyles. Liver and kidney
disease are attributed to eating the ‘wrong food’ ( “garam taseer”). Sexual
diseases are considered a curse of nature, caused by moral depravity.
Cultural views on treatments and interventions are largely diverse and vary
with social class, educational background. urban and rural setting and in-
fluence of prevailing subcultures. Altopathic medication are Largely viewed
as ‘goram’ and dangerous. and essentially lesser of the two evils when
confronted with disease. Pharmacological preparations are called ‘ongrezi
dawayi’ (a reference to the days of the British Pa]). This is to distinguish
it from ‘des! dawa’ (local or medicine of our own heritage). ‘Des! dawn’ is
considered ‘thand/’ and thus, more appropriate in a predominantly hot and
humid temperate setting. Local translations of the words ‘tablet’, ‘injection’
have dangerous connotations e.g. ‘tablet’ is translated as ‘got/which is the
same as ‘bullet’, injections are often just referred to as “su/”fneedle).
The most preferred and the first line of treatment in our culture are house
hold remedies and herbs based on past experience and conventional
wisdom. These are supplemented by prayers to The Almighty by the
individuaL his family, and sometimes the pit, the local faith healer, or the
prayer leader at the local mosque. Alms, sadqo, taweez, and holy water are
also used. It is only after the failure of these convenient, easily accessible,
inexpensive and trustworthy interventions that professional treatments and
interventions are considered an option. Alternative medicine, folk medicine,
healing through spiritualists, omits, sorcerers, shamans, and faith healers
are often reserved for patients with psychiatric disorders and epilepsy.
Often inhumane methods of ‘treatment’ are used by these agents in the
presence of and in connivance with the referring members of the family.
Cuftura[ty Sensitive Clinical Assessment
A culturally sensitive assessment may not he required in all patients and
not always at the start of a treatment plan. It would, however, be urgently
required when a patient is obviously from a different ethnic background
e.g. a patient from rural Sindh, central-Punjab, Balochistan or Northern
Areas being nursed in Lahore, Karachi or Islamabad. It is also important to
consider the concept of heritage consistency. This refers to how closely
an individual is influenced by or practices his/her particular ethnic back
ground. The lifestyle, health belief model, or practices of an individual may
not always be the same as their cultural heritage, In case a patient has a
high heritage consistency, the next step is to note how much of his/her
beliefs are influenced by the cultural background.
The following questions can be asked in a clinicat setting to ensure a
deeper cultural understanding of the patient’s explanatory model of illness
• What do you call your problem? What name does it have?
• What do you think has caused your problem?
• How do you think it started and what course do you think it will take?

A recommended approach for culturally sensitive health professional was


suggested by Fowkes and Berlin in lg8os. This approach can yield not only
invaluable information about the patient and family’s cultural understand
ing of the disease (their health belief model HBM) but also improves
treatment adherence and their ownership of the health care plan.
It can be remembered with the acronym LEARN:
1. Listen: Active listening and understanding of the patient and the family’s
cultural views is central to this approach, A conscious attempt to elicit the
health belief model of the patient and family has to be made by the health
professional.
2. Explain: At this stage the doctor explains the scientific basis of the dis
ease without chaltenging, rejecting or ridiculing the health belief model
of the patient and family. This view is presented in simple language using
symbols from the patient’s cultural context. Making simple drawings on a
piece of paper while doing so (as illustrated in the section on informational
care), can be helpful.
3. Acknowledge: The differences between the two explanatory models
are discussed to achieve consensus and common understanding. This witI
ensure treatment adherence and improve prognosis and follow up.
4. Recommend: A line of action regarding further assessment, diagnos
tic tests and short and long term management plan, with options, is then
recommended. A sensible physician will give the patient the final choice to
proceed with the plan or opt out, without feeling offended or hurt.
5. Negotiate: Offer a jointly conceived plan of action in which the doctor,
patient and family are all active partners. This adds to ownership of the
healthcare plan and chances of treatment adherence.

I Inftuence of sociocuLturat factors on therapeutics


Patients with different sociocultural backgrounds look at treatment from
different perspectives. The traditional concept of the “taaseer of the
medicine, whether it is ‘hot’ or ‘cold’ is respected by the culturally sensitive
doctor. S/he must adapt scientific knowledge to the patients’ language
to explain the different aspects of therapeutics. Many patients hold very
strong views about injections because in their culture and social setup,
injections are correlated with ‘dramatic results’. In other cultures, inject
ables are viewed with fear as they denote extreme illness or the last stage
of treatment’.
Interestingly, many Asian races are traditionally known to respond to low
doses of psychotropic medications as compared to their western coun
terparts, who need higher doses. A practitioner of holistic medicine must
understand these differences and belief models and address them tact
fully using language and context that the patient understands. The stigma
attached to many modes and methods of treatments needs to be ad
dressed on the same principles as stigma. Some patients need education
about their treatment options while others requirejust the word of doctor
to make them adhere to treatment. The doctor must understand the strong
influence these sociocultural factors play in the treatment of disorders.

ii. Cutture and PubLic Health


Clinics and hospitals are not the only places where a culturally sensitive
approach is beneficial. It also yields good results in public health settings.
Prevention of illness and promotion of health depends largely on an indi
vidual’s attitude towards help seeking and the value of health. The health
betief model has a number of variables, all of which contribute to some
extent to health behaviour, These include: concern with health matters, be
liefs about susceptibility to illness, ideas about illness and its severity, ben
efits and costs of various treatment and howwe[tthose measures will work.
The heatth belief modet also emphasises the importance of the opinion of
respected people in the individual’s life and the amount of personat control
that people perceive they have over events. The health betief model and
illness explanatory models of a community determine the pathway that a
patient will follow in pursuit of health. A comprehensive plan to promote
health literacy, develop health infrastructure and allocate resources must,
therefore, be based on this pathway. Such a culture sensitive approach im
proves community ownerships and participation in public health initiatives.
It also adds to acceptability, utilisation and sustainability of preventive and
promotive public health initiatives.
SAMPLE MCQ FOR SECTION D

1. A 35 yr old worker injured himself on the construction site and Lost a


Limb. The sociaL support, in our cuLture, that he can rely the most on,
will come from:

a) The owners of the construction site


b) Volunteers.
C) Health care system.
d) NGO’s
e) Family members

2. A 56 year otd tabourer diagnosed with hypertension is prescribed an


ACEI. He refuses to take it, insisting that “angrezi dawai garam hoti hai”
and is not appropriate to take in the hot weather. He says he wittjust
get his faith heater for dum (prayer) and this witl make him better. How
shouLd you respond?

a) Hand the patient the prescription and tell him to do as he pleases,


you’re too busy to argue.
b) Tell the patient that there is no such thing as “garam dawai” and he
will suffer and become very ill if he does not take it
c) ExpLain to the labourer’s educated son how important the
medication is for him and ask him to hide it in his food
d) Tell the patient you understand his concerns but this drug has been
tested on people in the region and they got much better, Faith
healers cannot cure hypertension
e) TelL the patient that the medication is safe, and he can take it as well
as see the faith healer and he wilt feet better, God witting

3. Stigma refers to:

a) Having a dangerous illness


b) Being shameful.
c) The inability to interact with normat people.
d) Suffering from mental illness.
e) Having a difference that is considered disgraceful or shameful by
society

4. Doctors can help reduce the stigma regarding mental iltness by:

a) Ignore the stigma and providing the best avaiLabLe treatment to the
mentally ill
b) Think of methods to rehabilitate the patient after s/he recovers
C) Start a sociaL media/public health campaign to change the mind of
the public about the mentatly ill
d) Reach out to the mentally ill; integrate their care in general heatth
care and use the media to run a destigmatising campaign.
e) Inform the family and everyone you know that stigma worsens the
effects of mental illness
. How should doctors behave in the face of cutturat differences between
the patient and themselves?

a) Try to read up on the patients cutture and behave with them


accordingly
b) Agree with patient’s notion that the culture he comes from is the
best one, in order to get him to do what is required
c) State categorically that we are all one human race and cultural
differences are meaningless
d) Behave as if the conflict does not exist and is not a barrier in
doctor-patient relationship.
e) State the doctor do not object to patient’s cuLtural practices.

SampLe Short Essay Question For Section D

01. Briefly discuss the concept of Beliefs, VaLues and Norms as it


appLies to BPS ModeL of heatth care.

02. How does culture affect treatment adherence?

Answers

i.e
2. e
3. e
4. d
5. d
OUTLINE
PsychosocialAspects of Health and Disease
Psychosocial Assessment
Psychosocial Issues in Special Hospital Settings
Psychosocial Peculiarities of Dentistry
Psychosocial Aspects of Alternative Medicine
Common Psychiatric Disorders in General Health Settings
Psychosocial Aspects of Gender and Sexuality
PsychosocialAspects of Pain
PsychosocialAspects of Aging, Death and Dying
Psychotrauma
Psychosocial Aspects of Terrorism
Stress and its Management

Chapter 1
Psychosocial Aspects of Health and Disease
The World Health Organization (WHO) defines health as a state of com
plete physical, mental and social well-being and not the mere absence of
disease. In order to understand the factors contributing towards a state of
psychological and sociaL well-being we must acknowledge that a human
being is a complex organism with a multi-dimensional existence.

HeaLth and Normality


The parameters that describe a state of psychosocial health and normality
include:
• Dynamism
• Optimisation
• Personal contentment
• Social responsibility-
Occupational efficacy
• Economic emancipation
• Relief from pain
Homeostasis
Dynamism
Dynamism refers to the various roles and functions a person has in life,
and their changing and evolving nature. In one tifetime s/he performs
many roles and contribute to their own growth, that of the society and the
world at large. One person may simultaneously be playing the roles of
family member, student, teacher, friend and breadwinner. The more roles
a person performs the more dynamic s/he is. Illness and disability limit the
dynamism of a human being.
Optimization
When a person performs most of his roles in an optimum state, s/he is
considered psychosocially healthy. While it may not be possible to stay
at your best alt the time, a healthy person views themselves as being in a
state of productivity in most roles a majority of the time.

Personat contentment
Personal contentment is when despite failures and difficulties and inability
to be the world’s best scientist or best parent, a person accepts them
selves as ‘good enough’. S/he is able to focus on the positive aspects and
achievements of their Life to attain a sustained state of satisfaction and
contentment.

Sociatresponsibitity
A healthy person takes responsibility of the roles and duties assigned by
the society. These begin from the immediate family to the neighbourhood,
town and country, to the world at large. S/he works towards making this
world a better place for their own self and subsequent generations.

OccupationaL efficacy
In order for an individual to be normal and healthy, they must be well-
versed in the knowledge, skills and attitudes required for their occupation,
i.e they must perform effectively. Such individuals are also helpful to so
ciety through their occupation, and attempt to pass their skill, knowledge
and wisdom to others.

Economic stabitity
A core component of health and normality is economic stability. This
means that whatever a person’ means of earning may be, s/he is free of
the pressure to acquire the basic necessities of life. This allows an individu
al to pursue their goals of self-actualisation.

Retief from pain and discomfort


In order for an individual to be healthy, s/he must be devoid of distress,
discomfort and pain at the physical as well as at a psychological level. S/
he must also be able to form and sustain relationships that are free of mis
trust, deceit, jealousy, prejudice, and ignorance.

Homeostasis
When a person respects the rights of others in their interactions with other
human beings and gives due importance to the laws of nature in his/her
interaction with the environment they are said to maintain homeostasis
with the world around them. S/he neither threatens the environment nor
do they feel. threatened by it, but instead make an earnest effort to improve
it. They are able to, thus, be in harmony with their internal and externat
environment.
Defence Mechanisms
In order to acquire and maintain a state of health and normality, a person
uses different psychological mechanisms which help to endure the chal
lenges of le. These are called defence mechanisms. Some of these are
basic defences of all human beings in a particular situation, whereas others
are more complex defences which come into play under certain circum
stances. Defence mechanisms function to help individuals cope with their
internal and external states of anxiety and distress.
Defence mechanisms have the following characteristics:
• They emerge in a developmental sequence from less mature to more
mature.
• They can be brought under conscious controL to ward off anxiety.
• They maintain a sense of wellbeing and safety.
• They may be episodic or become more habitual and pervasive.
• They may contribute towards formation of personality traits.
This is a list of many other defence mechanisms which people use
unconsciously. Some of these, in addition to those mentioned in the
example above, are:
repression.
• denial,
• displacement,
• projection,
introjection
• rationalization
• intellectualization
• identification
attruism
• sublimation

Repression
Unconscious exclusion of an unwanted or painful feeling, thought or mem
ory from the conscious mind is called repression. It is one of the basic de
fences of the mind that we almost regutarly use to push away unpleasant
happenings, thoughts and impulses from our active memory. One tends to
forget the painful details of an exam faiture, a setback, or an insult, through
the use of repression.
RationaLization
Unconsciouslyjustifying one’s feelings. impu[ses and thoughts that are un
reasonable and unacceptable in reality in order to seek retief from anxiety
or guilt is called rationalization. It is usually seen in people who have per
sonality traits of being obstinate and stubborn. People in stress and those
with limited capabilities, tend to use this defence mechanism as a routine.
Reaction formation
It is the devetopment of a conscious attitude, opposite to an attitude in the
unconscious, to avoid awareness of unacceptabe feeling, fear or impuLse
or a thought, It is usuaLly seen in peopte taking on sky diving to master their
fears of heights.

Defence Mechanism Definition Example

Displacement Discharging pent-up feelings A student who has just


on people less dangerous received a low grade on a
then those who initially ward test starts to shout on
aroused the emotion. the junior paramedical staff
over a ttIial
misunderstanding.

Symbolic satisfaction of A student struggling through


wishes through non-rational graduate school thinks about
thought a prestigious hIgh paying job
sure wants

Separating an emotion from A man learns from his doctor


an Idea or thought because that he has cancer, 14e studies
the emotional reaction is too the physiology and treatment
painful to be acknowledged. ofcancerwlthout
axperiendng any emotion

Rationalization Falsification of experience A patient misses out on


through the construction of hlslhex daily dose of
logical or socially approved medication and thInks that
explanations of behaviour. aJhe doesn’t need it any more

Repression UnconscIously keepIng Amen lsjealous of a good


unacceptable faellngs out fri.nd’t success but Is
of awareness, unaware of hIs feelings,
Grief: An exampLe of use of psychological defence mechanisms
A patient’s death is an unfortunate yetsometimes inevitable event at
hospitals. It bring into pLay a whoLe series of defence mechanisms. The
following cage scenario illustrates the sequence of defence mechanisms
that come into play in such a setting:

Case Scenario
A young doctor brings his 52 year otd father who has met a road-traffic
accident and suffered serious injuries, to the casuatty department of the
hospitaL Incidentatty the surgeon arrives tate to examine the patient. By the
time she examines the patient, he has already passed into a state of un
consciousness with falling breathing. Att resuscitative measures fqit and the
patient dies. The son is shocked but insisted that his father should be taken
to the intensive care and put on a ventilator, and that he was, in fact “not
really dead (shock and disbelief). He regained control after a while and with
a mask like face, started to take responsibitity for alt the affairs of the family
(denial; acting as if nothing has happened). He was seen handling his moth
er and other siblings and relatives, consoling them, arranging for the coffin,
transportation of the dead body, informing other relatives about the news,
making arrangements for the funeral, arranging for food and lodging of the
guests. Never during this whole period did he shed a tear. Eventually after the
burial the doctor returned to go to hisjob in a week’s time. During this peri
od, he found that he was unabte to steep property and felt exhausted every
morning. He tried to manage as best as he coutd, but one day saw his dead
father in a dream and woke up in a state ofpanic and felt a huge sense of
weight over his chest. Fearing he may be having a heart attack, he went to a
cardiologist When he was found fit and healthy by the physician, he suddenly
burst out making accusations and blaming doctors for being inept, inefficient
and irresponsible (anger). He blamed ‘all doctors and hospitals’for killing
his father and claimed that they are going to kill him as well (projection and
displacement; putting the blame and responsibility on somebody ets&. His
accompanying retatives took him home, but he continued with his outburst,
eventually exhausting himself and gave in to dying and weeping loudly tike
a child (regression). Since the death ofhis father this was the first time he had
cried in 20 days. The crying continued off and on for the next few days, until
one day he decided to visit Data Darbar. He prayed to Allah and promised to
give away food to one hundred people, ifgiven a chance to see his fatherjust
once (bargaining; “If God gives him back I’ll sacrifice so and so”). After dis
tributing the food to street dwellers, he waited for signs of his father’s return
but nothing changed. He started feeling responsible for his father’s death. He
would talk of the incidents when he had disobeyed his father; when he had
hurt peoples’ feetings, or had committed some other sins (depression). He
started to believe that Allah was punishing him (Introjection; taking the blame
on his own self). This tasted for few months until finatly he accepted that
nothing could bring his father back, no matter how hard he may try and a big
vacuum had been created in his life (acceptance; “There is a problem”). Grad
ualty he started to return to his life and his routines to resolve his bereavement
(resolution; “Life moves on”).

I
In order to bear the toss of his father the doctor in this scenario passed
through 7 different stages of grief, listed in the brackets as denial, anger,
bargaining, depression, acceptance and resolution. These stages invotve
use of unconscious defence mechanisms which hetp him resolve his
toss. These stages of grief are universal and present in all cultures. In fact
different cultures have varied rituals tohelp people move through the grief
e.g. the rituals of bain, Qul, Daswan and Chaleeswan’ or Chehtum’ all are
events that facilitate the resolution of the bereavement process.
As health care providers we need to be sensitive towards these stages of
grief and wherever possible must proactively work through with the griev
ing people to ensure a smooth and early return to routines of life.

PsychosociaL Assessment in HeaLth Care


Practitioners of a biopsychosocial model of health care are expected to
make psychosocial assessments of their patients as well as the biological
assessment. Psychosocial assessment is a study of the mental, familial and
cultural aspects that can inftuence health. These factors affect the etiolo
gy, presentation, diagnosis, interventions and management of illness and
prognosis in an individual. Certain questions asked by a health professional
may hetp develop a comprehensive understanding of the patient. The goal
of this is not only to determine treatment goals but also include social and
psychological treatments that the patient may need. Such an assessment
ensures the comprehensiveness of a treatment plan. While the psychoso
cial assessment is routinely undertaken for psychiatric patients, it is often
ignored in pediatric, medical, surgical and reproductivehealth settings.

CLinicaL Situations Demanding a Comprehensive


PsychosociaL Assessment
While psychosocial assessment should be routinely undertaken for all
patients, it becomes crucial in the following settings:
1. Patients affected by natural and human-made disasters and
earthquakes, tsunamis etc).
catastrophes (wars, violent crimes, floods,
2. History of psychological trauma
3. Cancer patients
4. Psychiatric patients
5. Sexually transmitted diseases like AIDS, and infectious diseases
6. Lifestyle disorders such as diabetes mellitus, hypertension, coronary
artery disease, cancer, cerebro-vascular accident fCVA), depression
7. Dementias
• 8. Intractable diseases such as spinal injuries, paraplegias.
How to undertake a psychosocial assessment?
A comprehensive psychosocial assessment must include the following
1. Identification of stresses in a patient’s life,
2. Emotional and psychological reactions to these stresses,
3. Symptoms,
4. Challenges to health,
5. MentaL state examination,
6. Assessment for risk of deliberate self-harm or suicide
7. Risk of violence or harm to others

F -—--

. . . . ..,

—. — 0<

It must result in a List of expectations or needs of the patient that are to be


fulfilled, particularly at the psychological and social levels and his personal
- L
goals of treatment.

Psychological reactions to ILlness and Hospitatization


As all of us who have ever fallen ill may realise, disease and hospitalization
are a source of major stress to a patient, The severity of stress and
individual’s response to it plays a major role in the suffering the patient
experiences and prognosis of the illness,
Major concerns about the illness include its infectious nature, the possibil
ity of passing it to the next generation through inheritance, how long the
isL Lora Zombie treatment will last and whether there are any consequent/residual disabil
ities. While these questions lurk in the patient’s mind, s/he often expects
the health professional to read their mind and many a time may never ask
them. In case the health professional is not sensitive to this need of the
patient, these questions can serve as a source of fear and anguish for the
patient. (How to address these questions and answer them appropriately is
discussed in Chapter 3)
The result is of this is stress that combines with a variety of other situations
to become an important perpetuating factor for the disease.

1. Stress due to ilLness


In addition to the effects of the illness the patient also experiences the
stress of the illness in the following ways:

Change of rote: The patient is usually relieved of alt his obligations and
consequently is assigned a passive role. S/he is not considered well
enough to manage anything or even think for their own self.
Financiat toss: The disease and the consequent disability may result in
a temporary or permanent loss of job. In the absence of insurance or
free health care the consultation with the health professional, inves
tigations. treatment and hospitalization result in a significant financial
burden on the patient and family.
Stigmatization: Cardiovascular diseases and iiabetes are considered
‘acceptable’ diseases. Psychiatric illnesses, tuberculosis, venereal
disease, sex-linked disorders, AIDS, epilepsy and skin diseases, on the
other hand, carry myths and stigma. This leads to concealment, secre
cy, delay in seeking help, somatization and ‘cover-up’ presentations.
Loss of seLf-esteem: The passive role, stigma. feeling of being a
source of financial stress and distress caused by the disease lead to
the patient feeling like a burden and having a poorer view of them
selves.
Fear of becoming handicapped: Failure of treatment, fitness resulting
in a disability, handicap or a cosmetic or functional loss all generate a
fear that haunts the patient all through the illness.
Uncertain prognosis: The short and long term prognosis as well as the
possibility of a relapse are a major source of stress for the patient and
his family.
Intervention:
Most of the factors Listed above require a mere explanation and reassur
ance based on facts and scientific data. The information sought needs to
be furnished in the Language best understood by the subject with minimal
technicaljargon. The stress can therefore be significantly relieved and the
resutt is a positive impact on the prognosis of the disease and a greater
patient satisfaction.
2.Stress of Hospitalization:
Hospital is a place associated with disease, disability and death that we
learn to fear from our childhood. The word hospital is synonymous with
bad news’ and thus a source of major stress. The prospect of being admit
ted in it heralds a variety of losses and [imitations on our being.
Anatomy and physiology of hospitals: The layout. arcnitecture and design
of hospitals, particularly in the public sector settings is, unfortunately. far
from pleasant. The most traumatic parts such as the trauma centre, the
emergency, and the intensive care units are set at the front of the hospitaL
A person accompanying the sick, visiting a patient or someone who merely
is there to get their blood pressure checked may be traumatised and fear
ful for the rest of their life at the prospects of going to a hospital.
The functioning of public sector hospitals is mostly bureaucratic, with a
series of long cues for a chaotic rush) at registration points, outpatients,
and pharmacies. Offices of consultants, as well are manned by often rude,
overworked assistants with little understanding of the stressful mind set of
a patient or an anxious family member accompanying them.
Private hospitals and clinics are often aesthetically pleasing and run by
staff that is not as stressed due to fewer number of patients but their func
tioning is driven by financial considerations, resulting in gross inequity.
Stresses relating to the hospitalization are thus over and above those of
illness. The common stressors include:
Loss of privacy: Once admitted in a hospitaL the individual finds him
self surrounded by patients and hospital staff. S/he is expected to eat,
steep and perform all routine tasks without any privacy, in the midst
of virtual strangers. This is usually not the case when one is well and
the patient may be helpless in changing this which may cause a great
deal of stress,.
Loss of autonomy: Once an individual is assigned the role of patient”,
his diet, dress, bedding, lodging, sleep and pattern of all routine tasks
will now be decided either by their care-givers or health personnel.
While some patients might enjoy this departure from decision mak
ing and feeling of control, others may find it disturbing. The patient
may find it impossible to adjust to this change and is, therefore, seen
seeking an early discharge from the hospitaL At times, the patient may
take a ‘ftight to health’ i.e. a sudden disappearance of all symptoms,
appearing to be ‘fit for discharge’.
Separation: Most patients are admitted without attendants and are ex
pected to meet no visitors or a minimal number, during visiting hours.
A sense of loneliness ensues and the day is spent in awaiting the
dear ones. The parents, children, spouse. friends, relatives and neigh
bours are all expected to visit and share the sorrow of the patient. Any
absentees are noted with concern, resulting in strained ties of friend
ship or family. Often these absentees may be cast out from the social
group, deemed callous and uncaring.
Need for doctor’s approval: Health professionals enjoy unparalleled
prestige and status. Their word is given a special consideration and
often considered irrefutable. The vulnerability of the patient further
enhances the aura of the doctor. S/he, therefore, tries to get the ap
proval of the ‘saviour’ by being the model patient who follows advice,
never argues or asks questions, and always nods ‘yes’ when asked
“Are you feeling better”? This model is gradually being replaced by
that of a more critical, argumentative patient who is informed of his/
her rights. S/he speaks as a client who is trading health services for
the large sum of money paid in cash, insurance or in taxes.
Handing over health matters: While the doctor is a fairly familiar figure
for the patient, the rest of the hospital staff are all new to the patient.
This may serve as a source of great stress when they take for granted
that the patient will hand over all matters relating to his physical and
mental health to them with little or no inhibition.
Threat to social or financial circumstances: A hospital admission can
mean loss of much-needed income, which may cause financial dis
tress to the entire family. Other causes of stress are the care of young
er children and etderly retatives.
Unsatisfactory information: Under most circumstances in the acute
situation leading to the patient’s admission the doctor is more focused
on management. Thegueries of the patient and the family are mostly
ignored or postponed. This adds to the patient’s and family’s anxiety
and causes stress.

3. Reaction of the Patient to Ittness and Hospitatisation


In the circumstances surrounding the illness and subsequent hospitalisa
tion the patient has a myriad of different reactions to what is going on in
and around him/her. These include:
• Denial
Anger
• Depression
• Dependance
Patients who are at a higher risk of reacting to the stress of illness and
hospitalisation include those suffering from psychiatric illness, those with a
past history of psychiatric illness, elderly patients, children and those with
poor pre-morbid psychosocial and psychosexual adjustment.
DeniaL: Hospitalisation may be an overwhelming challenge to some
individuals. Their response may initally be a state of disbelief and de
niaL The patient may refuse to believe that the admission is required.
or in some cases deny they are ill at alL Such a state may be shared by
the family and may last for few hours to a day.
Anger: The state of denial may give way to anger. The degree of anger
is influenced by the personality and emotional stability of the patient.
This anger generated may be directed towards self or the hospital
staff, When directed to the patient’s own self it may be in the form of
refusat to take medication or meals, disturbed sleep, and frequent
losses of temper with or without provocation. A team of health
professionals sensitive to this displacement of anger gives the patient
(who may be joined by the family / attendants) the chance to venti
late. It is important to avoid both an apologetic and a confrontationat
attitude.
Depression: A state of resignation, apathy and depression may ensue
by about the third day. The state is often transient as benefits of the
treatment start to become visible. A delay in recovery, worsening or
persistence of symptoms may however aggravate this stage. It may
lead to a depressive reaction or syndrome with adverse effects on
biological functions such as appetite and sleep.
Dependence: The combined effect of the disease, the stress of hospi
talization and the consequent stresses often result in a strong de
pendence-hunger’. The patient has a constant need on the part to be
pampered, sympathised and helped white eating, drinking or perform
ing routine tasks. The extent of this dependence varies with the kind of
experience the patient had when previously ilt, and the sociocuttural
context.

It is evident from the above description that the stresses of illness and hos
pitalization are responsible for ‘secondary suffering over and above those
directty caused by the illness. A vast majority of these can be prevented
by prediction, identification sensitivity and understanding at the appropri
ate hour. Nearly alt these states respond welt to the empathic attitude of a
considerate health professionaL
Chapter 2
Psychosocial Assessment

Modet Psychosocial Assessment And Management Protocol


You have been asked to undertake the psychosocial assessment of a 26
year old, single, male: MrX.

Presenting Probtem
Mr X is admitted in a medical ward with the diagnosis of Hepatitis B. He is
also being investigated for spikes of high grade fever, unexplained by the
diagnosis of hepatitis. In spite of the ward staff’s repeated requests, He
leaves the ward every now and then and returns after two hours or so. He
sometimes fights with fellow patients and ignores rules and regulations of
the ward. He occasionatly refuses treatment and states that he does not
want to live anymore.

Personal Status
Mr X is the youngest of six sibs. He has two sisters and three brothers. He
was born in Sheikhupura. and his father, a cobbler, shifted to Lahore when
X was a six year old, to put up a roadside stall. None of X’s sibs went to
schooL S/he collects waste from the waste disposal dumps on the banks
of River Ravi. S/he started to smoke cigarettes at the age often. He now
smokes chars (marijuana) and is addicted to heroin. S/he inhates the heroin
on a silver foil.
X is HIV positive. He gives a history suggestive of repeated urinary tract
infections and gonorrhea.

Famity History and Current Relationships


X has been in an extended family system for the first ten years of his life. He
lived in Sheikhupura in a three room pukka house with his paternal cousins
and uncles. The family practiced some basic Islamic rituals but was not
very religious. He was physically abused by his father and a paternal uncle.
S/he was bullied and sexually abused by a cousin regularly till his family
moved to Lahore. S/he never attended a regular school, but went to a
Local madressa to study religious text. S/he would get physical beatings
at the madressa as he was poor in studies. His father earned about five
hundred rupees on the days he worked. His mother worked as a household
servant to earn five thousand rupees a month. He often fought with his
siblings and had no confiding relationship except with his mother.

Positive Support Systems


Mr X feels comfortable in the company of his mother, and a friend from
whom he usually buys chars and heroin. He hates his father as he is ag
gressive and has beaten him repeatedly on account of his smoking and
drug use.
Lately Mr X has started to attend the local mosque. The prayer leader there
gives him sermons about leading a pious life and assures him that he will
be granted Allah’s mercy if he gives up his bad habits. He feels some com
fort in the company of the prayer leader. It is he who brought him to the
hospital for treatment.
Crime and Delinquency
Mr X has had repeated run-ins with the police on account of possessing
charas and heroin. He has also been arrested and sentenced for selling
drugs. He once beat up a fellow drug addict. He has been to prison twice.

Education
X has never attended a format schoot and was taught onLy to read Quran at
the madressa. He can do simple counting and count currency. He cannot
Write.

Emptoyment
X has been making three to four hundred rupees a day by selling used
syringes and ptastic bottles picked from the dumping grounds on Ravi.
He once worked for six months at a tea bar near Daata Darbar, Lately he
has been emptoyed by a syringe packing factory as a daily wager for two
months.

Readiness for treatment


X has opted for treatment of hepatitis but is still unsure about giving up
charas and heroin.

Available social, economic, occupationaL resources


X is admitted in a public sector hospital receiving free treatment, but has to
pay for interferon therapy. He has no job to return to and is a poorly skilled
labourer, working on waste disposal area which is a high risk setting. His
bond with brothers and sisters, and father is weak.

Personal Values and Attitudes


Mr X often prays at the mosque for the last six months but was raised in
a house with little or no religious values. He cheats and tells lies. He is
bisexual, and has had several sexual liaisons with mates and females since
the age of twelve. He has been involved in petty thefts from his home and
various shops.

Mental Status Exam


Appearance and generat behavior
Mr X is a young mate with a medium build. He appears older than his
age, is shabbily dressed, and poorly groomed. He is cooperative and
forms a fair emotional rapport but is restless and fidgety.

Speech:
He speaks in a low tone and volume, giving brief and relevant replies,
but starts to speak with pressure, and rapidly while discussing
religious beliefs.

Mood and affect:


His mood is anxious and defensive. His affect is appropriate to his
speech and mood state.
Thoughts.’
His thinking processes are normat. logical, and he has no delusions.
He does have some overvalued ideas about the relationship of his
medical and drug issues with his fate: he considers them Gods curse
on him because of his inability to serve his mother,

Perceptions.’
Mr X denies any visual or auditory illusions or hallucinations at presenc
He states that he does have hallucinations of police sirens getting
closer while under the effect of drugs. This state has never occurred
white he is sober and drug free. He has not noticed any change in his
perception of taste, olfaction or bodily sensations.
There are no obsessive-compulsive phenomenon. He does not
experience any phobias, dereaUsation or depersonalisation.

Cognitive Functions.’
He is fully conscious, attentive and is able to concentrate. He is
oriented in time, placeand person. and has normal short,
intermediate, and long term memory. His abstract thinking is intact,
and his arithmetical skills and general knowtedge are appropriate with
his educational and cultural setting. His judgment is not impaired.

Insight:
He is aware of the physical and drug abuse issues he is facing and
wants to seek appropriate treatment from the hospitaL

Suicide and homicide risk:


He has thought of killing himself by taking an overdose of heroin, but
has not attempted it so far, and has a moderate risk of self-harm. He
has no plans to harm anybody.

Diagnosis
AXIS I: (Psychiatric Diagnosis) Heroin and Cannabis dependence
AXIS II: (Personality): Sociopathy / Antisocial Personality Disorder
AXIS III: (Medical Condition): Hepatitis B, HIV positive, gonorrhea
AXIS IV: Problems related to family, police, and hospital administration
AXIS V: (Social and occupational functioning): Global Assessment of
Functioning (GAF) score: 71

PsychosociaL Management:
Major Issues:
Medical and Psychiatric illness
Antisocial personality
Poor socioeconomic status
Inadequate family support
Unemployment, and intermittent exposure to hazardous waste and
infected syringes due to occupation
No housing
Legal problems
Easy access to drugs of abuse

Strengths And Weaknesses:


Strengths: Cooperative, has insight, and adequate cognitive functions,
link with the local mosque and religious inclinations, strong
bonding with mother.
Weaknesses: Multiple psychosocial issues listed above, poor motivation to
give up drugs, antisocial personality traits and delinquency
history, mild to moderate suicide risk.

Management Goals:
1. Improvement of motivation to overcome drug dependence
2. Adherence to hospital norms and rules, as well as social values
3. Meaningful occupational skills and employment and giving up
current exposure to syringes and infected material
4. Mobilisation of family support particularly improved relationship with
father and brothers
5. Estrangement from drug dealers, and drug abusers
6. Risk reduction regarding sharing of needles, dissemination of
hepatitis B, suicidal ideation, and safe sexual practices
7. Informational care on medical and psychiatric diseases, drugs, follow
up, prognosis, cross infection, treatment adherence
8. Relapse prevention through active engagement with medical, and
psychiatric services --

9. Availability of medication .- -

Strategies and Interventions:


1. Motivational interview to deal with drug abuse issues
2. Informational Care session
3. Counseling and support to refuse peer pressure in drug abuse, avoid
risk taking, and high risk situations, mobilization and re-establishing
family bonds.
4. Occupational therapy
5. Improved treatment adherence and follow up
6. Conflict resolution at home and hospital settings
7. Medicolegal / forensic support
8. Mobilisation of economic support for treatment of medical and
psychiatric disorders
g. Cognitive behavior therapy to deal with issues related to suicidal
ideation, social bonds, drug abuse.
10. Anger management and impulse control methods
11. Group counseling / group therapy for drug abuse
Chapter 3
Psychosocial Issues in Special Hospital Settings

a. Coronary Care Unit


Coronary Artery Disease (CAD) is commonly seen in people who have a
characteristic combination ambition, time urgency, anxiety, competitive
ness and hostility (Type A personalities). It is more likely to occur in persons
with high environmental stress, life dissatisfaction, less social mobility, sta
tus incongruity and those with personal loss. These patients when hospi
talised to CCU nd themselves faced with a situation when their health and
environment is not in their control. The surroundings of CCU depict a very
grim picture. Patients are attached to different kinds of tubes, wires and
gadgets and neighbouring patients are being resuscitated or dying. For a
patient, therefore, the CCU may appear to be a ‘chamber of horrors’ and
further complicate his state. Certain patients, however, may feel calm and
“protected” in the same setting.
The common psychological reaction in patients in CCU settings is a state
of anxiety, fear, distress and/or a sense of loss following the cardiac event.
This is known to lead into clinical depression in up to one-third of the pa
tients, who may present with weeping spells, low mood, disturbed sleep.
loss of appetite and even non- cardiac chest pain. Some of these patients
may use the defence mechanism of ‘denial’. In this state they may appear
joviaL talkative and found cracking jokes. The co-morbid depression and
anxiety ma, delay recovery and adversely affect the short and long term
morbidity and mortality.
To manage this certain physicians may prescribe benzodiazepines in cardi
ac patients as a routine. Benzodiazepines are not currently recommended
in clinical guidelines for long term use due to their addictive potentiaL This
practice may result in benzodiazepine misuse, abuse and dependence.
Family members visiting the patient or those asked to wait outside the CCU
may be equally distressed and anxious. Their anxiety is often on account of
a lack of information and awareness about the events taking place inside
the CCU, hidden from their eyes, They are often found waiting on the edge
of their seats, walking anxiously in the corridors or feeling exhausted after
waiting for long hours in the open or the poorly furnished waiting rooms.

Interventions:
A medical student or a doctor on duty in CCU must actively look for
the above mentioned psychological reactions, make clinical notes and
promptly start treatment or make a referral for a psychiatric opinion. The
effectiveness of biological treatment and clinical outcome in CCU settings
can be greatly enhanced using nonpharmacological interventions (NPIs).
Use of informational care, counselling and ventilation sessions, progres
sive muscular relaxation, visual imagery, relaxation techniques, hypnosis,
meditation and biofeedback can help. Opportunities for stable patients
in CCU to interact in groups with each other and share their experiences
have tremendous therapeutic value. Group sessions conducted by doctors
and cardiologists to educate patients on behaviour modifying strategies
regarding smoking, anger, time and stress management can also enhance
the therapeutic outcome and prevention of future cardiac events.
Alt CCUs should ideaLLy ensure comfortable, dedicated waiting rooms and
rest rooms for famiLy members and attendants. A regular flow of informa
tion about the patients state progress based on scientific data should be
ensured through a medical officer or a senior nurse, trained in principles of
effective communication (Section A). This is preferable to the usual “tassalti”
with statements such as “atl is well ‘mareez theek hai or 5mareez ki halat
theek nahin.”

b. intensive Care Unit


Long periods of sensory deprivation and sensory overtoad will often give
rise to a state called CU- Psychosis or Intensive Care Syndrome. This
syndrome is characterised by increased wakefulness, disorientation and
visuat hallucinations, depression, social withdrawal, anxiety, delusions and
delirium. There is a combination of sensory overload and sensory depri
vation in ITC syndrome. Sensory overload occurs in the form of noise of
monitors, activity of the staff, constantly lit setting, and high level of activity.
The sensory deprivation as a result of immobility, restraints and bandages,
contribute towards the ICU syndrome. The lack of familiar orienting objects
such as clocks, calendars, windows, meals, and close proximity to other
patients further worsen this condition. An already distressed patient is ex
• posed to constant attendance by physicians, nurses and technicians. S/he
also suffers sleep deprivation, physical and emotional pain of disease, fear
of mutilation or death, unfamiliar medical procedures, (often conducted
without consent and information) and minimal control on one’s draping and
dress. All this leads to extreme stress and a feeling of powerlessness. The
• typical psychological reaction of hyperactivity, anger and anxiety in ICU
• patients should not, therefore, come as a surprise.
Interventions
A redesigning of CU’s architectural and administrative layout with the is
sues identified above in mind can greatly reduce the risk and frequency of
ICU syndrome. The steps that can be undertaken to improve psycho care
of patients in ICU can be divided into those focused on the inside of the
CU and those aimed at their family waiting outside.

Inside the ICU:


• Educate and prepare the patient.
Use orientation devices e.g. clocks and calendars.
• Minimise pain through pharmacological as well as non-
pharmacological interventions.
• Be careful while discussing the patient’s plight assuming the patient
to be unconscious, comatose or sedated. Many patients in these
states report having heard and painfully remember the negative and
sometimes insensitive remarks of the health care staff.
• Reduce environmentaL stressors.
Outside the ICU:
• Designate a space for the family and visitors.
• Provide support and educate the family using Informational. Care
principles.
• Set up a counselling or a ‘solace room’ where those in distress can be
attended to. The same space can be used to breakbad news.
Hospital staff shoutd be trained in counselling and non-
pharmacological interventions in ICU and CCU settings

c. The Emergency Department


The emergency department is the face of a hospital and often the most
frequently visited part. It is, therefore, crucial to incorporate psychosocial
care in deaLing with patients brought to this part of hospitaL With emotions
running high in a crisis situation, Emergency health professionals often
are forced to handle anger, frustration, guilt, depression, dependency and
infantilisation in patients and families. Patients who have gone through a
health issue requiring emergency care become reluctant to resume routine
tasks and adoption of sick role” is a common associated finding. The pa
tient’s family fears the unpredictability and outcome of the illness, change
of rotes and economic burden of treatment of the patient.
Interventions
• Educate the patient and the family.
• Keep the patient informed about the progress of his illness,
• Give the patient a chance to express him/herself and work through
the problem with them
• Manage ‘sick role’.
Aim for early rehabilitation and return to previous level of functioning.
A comprehensive psychosocial assessment and intervention in emergency
department can prevent uncalled for admissions and save the family, the
patients and the doctors a great degree of distress and resources.

d. PsychosociaL Aspects of Organ Transplantation


While the future holds promise for transplants of possibly all or many
organs, the most common are the kidney. bone marrow, liver and heart
transplants. Other than the ethical issues and the legal bindings now in
position in Pakistan, the psychosocial aspects of this intervention are often
ignored. While the recipient is of primary significance on account of his or
her severe illness, the donor must never be forgotten. Before, during and
well after donating the organ the donor has to be cared for with sensitivity
and concern. Other than the surgical and medical risks that s/he may carry,
S/he is looking for approval, and support from the medical staff. A profuse
and oft repeated thankful stance, and eulogising of his altruistic behaviour
is the least that we can do. This aspect is often left to the family of the
recipient or the recipient himself; which many a times is not enough. As a
medical student it is important that you ask for the donor, visit him, and use
principles of effective communication such as understanding for emotions,
active listening and above all, empathy, in interacting with him. A single in
teraction of this kind can lift his psychological state and may be enough to
comfort him. It is also important to give him or her a detailed informational
care session to remove his myths and misconceptions, respond to his con
cerns and reassure him that s/he is not a disabled individual after donating
an organ and can still lead a full and productive life. Particular emphasis
needs to be given to the impact on his diet, sleep, sexual life, return to
work, and address his concerns about Perhez.
e. The DiaLysis Unit
Memoirs of a young doctor;
The turning point in my medicat career came in final year MBBS when I was
on rotation in the medical ward of the hospital attached with our medical
cottege. I was to take histories ofsome patients and carry out physical ex
aminations. I was surprisingty drawn towards one 65 year oLd fernate patient
who was admitted in the ward with chronic renal failure. She needed a kidney
transplant but the family could not afford to have the surgery. She was under
going haernodialysis at regutar intervals in an effort to somehow maintain her
failing health. The frequency of dialysis required gradually kept increasing as
her kidney function deteriorated. As the family and the patient herselfslowly
seemed to be preparing for the inevitable outcome, I fett I was witnessing the
realistic limits of our health care system. As a medical student, I could not
offer her much technical assistance, but I would always take some time in the
evenings to come and listen to her I saw her progress from frustration and
anger to gradual acceptance. I was able to see how the whole family pulled
together in an effort to make herjourney easier, and how they found peace
and comfort in their cutturat and spirituat beliefs. She said, and I believed,
that the opportunity to talk to a caring and non-judgmental individual helped
her I realised that one of the gaps in medical care in Pakistan is that our
doctors and nurses are not trained in practical counselling skills that they can
use in their daily interaction with patients. That was when I decided to take up
mental health as my future specialization, becoming a psychiatrist and finally
a stress counsellor. The training also helped me answer another unresolved
question, ‘why was I drawn towards this particular patient?’! realised that this
patient unconsciously reminded me ofsomeone very important to me: my
mother

Psychosociat concerns and interventions in a diaLysis unit


Acute uremic encephalopathy, intra-cerebral thrombotic stroke or hemato
ma, dialysis dementia (characterised by dyspraxia. facial grimaces, myoc
Ionic seizures, schizophrenia like state, mania and depressive symptoms),
maladaptive coping behaviour, loss of independence, difficulty in perform
ing routine tasks, fulfilling family and financial obligations are a cause of
many psychosocial concerns in a patient admitted to a Dialysis Unit and
her/his family. These include:
1. The burden of treatment costs,
2. medication,
3. special diet
4. transportation,
5. loss and change in function,
6. roLe reversals, marital difficutties
7. Depression, with or without suicidal thoughts
8. Emotional reactions are seen with such patients such as guitt, hostility
and ambivalence.
Interventions:
• Emotional support
• Education and preparation of the patient, and the family through
regular Informational care and emotional support through
s and
counsetting sessions can greatly reduce the distress of patient
families in a Dialysis Unit setting.
ions
• It is also important to detect the co-morbid psychiatric condit
and treat them early.
f. Reproductive HeaLth
the defini
The WHO defines reproductive health, within the framework of
of compl ete physic al, mental and social well-being
tion of health, as a state
absenc e of disease of the reprod uctive system .
and not merely the
health in
Reproductive health is central to general health. It a reflection of
and sets the stage for health beyond the re
childhood and adolescence
women and childre n. It includ es women ’s health
productive years for both
safe mothe rhood, fertilit y regulat ion, preven tion of
in general as well as
manag e
infertility, prevention and management of cancer, prevention and
itted disease s.
ment of reproductive tract infections and sexually transm
sed here.
Only the key times in the reproductive life are however addres

Puberty
and
A fairly regular sequence of events happen between the ages of 9
first time and may lead
r6 years. Vaginal secretions are generated for the
or disease d’. Increas ed body fat
to misconceptions e.g. ‘the body is dirty
femini ne feature s mature . There are
is perceived as ugly” before final
se
concerns about breast size. Many negative reactions occur in respon
, particu tarly if it is precoc ious. All these factors genera te
to menstruation
and confid ing
anxiety. mood changes, anger and a desire for tonetiness
s and
relationships. Changing moods can cause family conflicts and friction
[ow self- esteem in them within the peer group.

Interventions
family
Close and healthy bond based on free communication within the
es and worrie s usually improv es the
(mother, elder sister) to share anxieti
should be sought also for delaye d / preco
situation. Professional advice
con
cious puberty, and a sensitive physician is expected to address the
ance. Myths
cerns of the adolescent and the family members with reassur
to be unclea n
such as perceiving normal vaginal secretion (“leukorrhea”)
ionals for treatm ent. It is
are often brought to reproductive health profess
this and other myths, instead of
up to the health professional to debunk
antibiotics,
“throwing drugs at the problem” as overuse of drugs. especiaLly
is already showing danger ous conseq uences worldw ide.

Pregnancy & childbirth


endowment,
During pregnancy both parent have concerns about genetic
intelligence, temper ament. physic al health and stature of the child. Cultural
l expec
practices and myths of the society greatly influence on parenta
pregna nt woman is
tations. A prevalent example of this is the diet that a
nt food groups out of
asked to consume. Various cultures, deem differe
research base.
bounds for the pregnant female, with next to no evidence or
experie nces and how prepar ed they feel to
The parents’ own childhood
greatly influen ces the parenta l expect ations. Con
become parents also
n, medica
genital anomalies due to non-inherent factors such as nutritio
the other con
tions, alcohol, caffeine, cigarette, toxin, infection are some of
contrib ute toward s quality of
cerns of parents at childbirth. All these factors
bonding of each parent with infant, The child bears high risk in this regard if
the mother develops puerperal psychosis or depression.

Pregnancies at risk for Devetoping Psychosocial Complications


• Primigravida, precious pregnancy, unwanted pregnancy
• Past history of psychiatric illness or family history of psychiatric illness
• Strained maritaL relationship,
• Poor social support
• Children under the age of 14 years
• History of drug abuse
Minor problems in pregnancy, like heartburn, nausea, cough, generalised
itching, varicose veins etc. may generate undue anxiety, particuLarly if
organic cause has not been effectively ruled out. Unlike the west, in our
culture adequate training for labour and psychological support during
labour is preferred from mother or elder sister rather than from husbands.
However with changing times more and more husbands are playing a
more supportive role during the pregnancy, labour and even in the care of
the offsprings.
Majority of deliveries in Pakistan are still done by Traditional Birth Atten
dants (TBAs) and Dais. This increases the risk of birth complications. On
the other hand a number of Caesarean sections are also done without
giving an adequate trial of labour. World literature provides evidence for
better results from natural childbirth.
Problems with breast feeding may occur, amongst mothers on account of
the misconception about disfigurement if they feed their children. While
mothers milk is considered to be, by tar the best feed for the baby.

Psychosocial issues during Puerperium


The risk of developing morbid psychological reactions during the puerpe
rium is higher amongst the primigravida, those with past history of psychi
atric illness, family history of psychiatric illness, past history of puerperal
illness, previous childbirth with congenital abnormality, poor social support
or poor marital relationship.
The common psychiatric conditions that can develop during puerperium
include:
• Postpartum btues: a self-limiting state of low mood, weeping spells
and disturbed sleep that lasts for 10-14 days.

• Puerperat depression: If the sad mood and disturbed biological


functions lasts for more than 2 weeks, puerperal depression should
be considered.

• Puerperat psychosis: A relatively rare state, characterised by


delusions and hallucinations and gross abnormalities of behaviour.
Both Puerperat depression and psychosis should be treated by a psychi
atrist. An early referrat of such patients to a mental health professional is
therefore mandatory.
Ctinicat situations with a high risk of psychosociaL morbidity in Reproduc
tive Health Settings
Couples with infertility, females with vaginismus and those who have
tong-standing premenstrual syndrome, dysmenorrhoea. chronic petvic
pain or those who undergo hysterectomy form a high risk group for devel
oping psychiatric morbidity. An early detection of psychosociat correlates
in these patients should be followed with informational care sessions and
the use of N P15. Long-term use of benzodiazepines, analgesics, placebos
and megavitamin therapies are often unproductive measures, and should
be avoided. In case the symptoms do not improve, a prompt referral to a
psychiatrist is the best option.

g. Paediatrics Ward
A poem
Alt the way to the hospital
The tights were green as peppermints
Trees of black iron broke into teaf ahead of me, as if
I were the lucky prince in an enchanted wood
Summoning summer with my whistle, Banishing winter with a nod.
Swung by the road from bend to bend, I was aware that btood was running
Down through the delta of my wrist
And under arches of bright bone
Centuries, continents it had crossed;
from an undisclosed beginning spiralling to an unmapped end.
Crossing tat sixty) Magdaten Bridge
Let it be a son, a son, said the man in the driving mirror,
Let it be a son. The tower
held up its hand: the college
bells shook their blessings on his head.

I parked in an atmond’s shadow blossom, for the tree was waving, waving at
me upstairs with a child’s hands At seven-thirty
the visitors’ bell
scissored the calm of the corridors. The doctor walked with
to the sticing doors.
His hand is upon my arm, his voice I have to tell you set another bell beat
- -

ing in my head:
Your son is a Mongol the doctor said.
How easily the word went in
-clean as a bullet leaving no mark on the skin, stopping the heart within it
This was my first death
Wrenched from the caul of my thirty years’ growing,
fathered by my son, unkindly in a kind season
by love shattered and set free.
You turn to the window for the first time. I am catted to the cot
to see your focus shift,
take tendril-hold on a shaft
of sun, explore its dusty surface, climb to an eye you cannot meet
You have a sickness they cannot heal the doctors say’ locked in
your body you wilt remain.
Welt, I have been locked in mine.
We will tunnel each other out You seal the covenant with a grin.
In the days we have known one another my little Mangol love,
I have learnt more from your tips than you will from mine perhaps:
I have learnt that to live is to suffer, To suffer is to live.
- Jon Stallworthy
PsychosociaL aspects of parenting a child with disabiLity

Having a child with a physical or mental disability is an emotionatty, phys


ically and often financially challenging experience. Many parents experi
ence periods of extreme stress as they adjust to the demands of parenting
a disabled child. These feelings are often similar to those commonly expe
rienced after a significant loss such as a divorce or death of a loved one.
1. Denial of the Childs Disability. One or both the parents may deny
their childs disabitity, especially if it is not an obvious one. A patent in
denial will avoid talking about the disability and will make up
excuses and alternate exptanations of the problem. They may behave
as if everything is fine, completely ignoring the child’s problems. A
child whose parents are in denial is at risk for being punished for
his poor performance. This is inappropriate and may cause
psychological damage to the child. This stage is especially difficult
when the parents disagree about the degree of the child’s disability
and how the problems should be handled.

2. Anger and guilt about the child’s disorder. Anger is a common


reaction in parents of children with disabilities. Parents struggling with
anger may become argumentative, demanding, and verbally
aggressive when dealing with a child’s problem. They may project
their anger toward a teacher, their spouse, and worst of atl, their child.
Some may also be angry with themselves over the child’s disability
and their inability to ‘fix” the problem.

3. Blaming others for the Disabitity. Some parents of children with


disabilities attempt to cope by blaming others for the disorder. The
parent in the blaming stage may believe or say that the other spouse.
doctors, school staff, or even the chitd themselves is at fault or
making excuses. Again, this stage is especially difficult and stressful
when both spouses disagree about who is responsible. Furthermore,
the blamer may become unable to get past blaming someone to
focus on coping with the child’s problems.

4. Grief: Parents of disabled children may go through a grieving process


that begins when they learn about the disability. Parents who grieve
over their children’s disabilities are usually concerned that their
children may struggle for the rest of their lives. They may worry that
the child will not be successful in life because of the disability.
Parents may feel new grief over the years if their child has difficulties
achieving various milestones when other children succeed. Passing
a driver’s test, college entrance exams, and similar events may trigger
this grief.

5. Worry: Worry and grief often go hand-in-hand in parents of disabled


children. Patents may worry about their child’s self-esteem,
achievement and ability to make it through school. They worry
whether the child will learn to read, write, or perform other important
life skills, whether the child will be able to attend college or a
vocational program, have a successful career; have a family and/or
lead a normal adult life.
ility
Psychosociat issues faced by a child with disab
handicapped leading
Various disorders can render a child partiatty or ftilty
e timitations over an
to restrictions on movement, ptay and/or diet. Thes
cial prob lems both at home
extended period of time can result in psychoso
low expe ctations of teach
and at school. Frequent absences from school,
self- estee m. Such children
ers and parents and poor grades lead to [ow
t is inad equa te or stow verbal and
have few or no friends at alt. The resul
mak e matt ers wors e. In the event of un
language proficiency, which may
and thinkin g may be affec ted and scars and
controlled seizures, memory
er decline the poor
in
disfigurement may occur. These issues lead to furth
gloom. Delayed sexual
self-concept and a state of chronic dysphoria and
causing poor peer inte
maturation may further complicate the situation by
gration and a state of alienation.
the proper guidance,
On the other hand, if a child with disabitity receives
come these disabilities.
psychosociat and practical support, s/he can over
highly motivated human
to become a highly successful professional and a
alt so calle d normal” chil
being. The basic principle is the same as with
g envi ronm ent in which they can
dren: to provide a safe, non-threatenin lop their
ilitie s and expl ore and deve
learn to manage their apparent disab
strengths.
Psychosocial interventions
ion with the family and
As mentioned earlier, timely psychosocial intervent
e to their prog nosis and integration
individual in such a situation is imperativ
s may provid e struc ture to such an
into society. The following guideline
intervention:
health team, in a
• Informational care by the senior member of the
to the patie nt and/ or pare nts in simple
frank and honest fashion
. Both parents must
language, as early as possible, is recommended
ltaneously.
be seen together and involved in the care simu
d so that issues can
• Adequate time for this interviewshoutd be spare
quer ies of the pare nts, patient and
be discussed in detail and alt the
out any rush. A hom e visit by the health
the siblings dealt with, with
er is regular practice in
professional or the mental health social work
a formal setting of
the West. This may not be possible locaLLy but
t be ensured for an
the hospital cad be used effectively. Privacy mus
an issue is being discussed.
open expression of emotions, when such
hologist or a social
• A multidisciptinary approach involving a psyc
or ideal. Electiv e surg ery, if indicated, should
worker, nurse and doct is
d of psychosexual
not be scheduled during the sensitive perio
avoid heightened
development between 4-7 years in order to
Ideally , the patient should
anxiety and possible psychological trauma.
latio n. S/he may be taught
not be segregated from the general popu
alon g to the vario us festivals
to be appropriatety dressed, and taken
long run this appr oach help s to reduce
and social gatherings. In the the
the patient and
the stigma and enhances the self-esteem of
play, telev ision
family. A fair amount of peer contact in games,
, telep hone conv ersat ions and read ing stories can work
watching
adeq uatel y guided.
wonders after the peer group has been
at the verb al and non-verbal level
Communication skills and training ication to
as poss ible. Verb al com mun
should be taken up as early
convey warmth, acceptance, empathy, care, while nonverbal
communication (tone of voice, facial expression) to convey humanistic
concern should be encouraged. Evaluation of the psychosocial
environment that influences the child has to be done on a regular
basis.
it is important to ensure emotional support for the family of a child with
a disability, as a whole. Recognition of the roles of all the members indi
vidually and due appreciation and valuation should never be forgotten,
Encouragement and facilitation of needs through parent-to-parent support
amongst the affected families is most helpful. it is also useful to teach and
train the family in the techniques of coping with stress (discussed later in
this section).

Dear Professionals: What Parents Want to Tett You


(Adapted from a piece by Muriel Hykes)

;. Please don’t predict the outcome for a specific case. You can give
general advice, but caution that nobody can predict how this child
wilt turn out
2. Do not write off a complaint as “Goes with the syndrome.” Just because
a child has one diagnosis, does not mean that all his other problems are
due to that
3. Ptease don’t get angry with our questions. We are worried and trying to
do our best for our child.
4. Be open. Realise that the parents are going to be looking for a cure and
will pursue alt avenues. Please respect that
5. When the parents are depressed and in shock, be avaiabte for them.
Find something good to say about the patient’s development
Appreciate their efforts
6. Promote a team approach to the child’s rehabilitation: practice
“family-centred” care. Do not be surprised if other doctors have told the
parents conflicting information or failed to tetl them something.
Coordinate care with alt other team members.
z Understand that most parents are new to this...atl the medical terms,
conditions and stuff So please go slow and be patient with us. On the
other hand, recognise that some of us are very wett versed on our kids
situation, and you need to address us as your peer in the treatment of
our child.
8. Remember that your patients are little human beings who feel and hear,
and do thingsjust like other people. Itjust takes them a bit tonger to
achieve those goats. Be careful how you talk about them in front of
them. Setf-esteem, confidence, and acceptance are major needs for
everyone.
g. Please be careful when using words like “handicap” and “disabled”
because they hurt us like knives. These are just children with special
needs, and nobody knows what our chitdren can achieve.
Psychosocial issues in chronic illnesses and disabilities
The reactions of a person who has been diagnosed with a chronic illness or
devetoped a disability, are similar to reactions after other major life loss
es. The individual experiences deniaL, anger. depression, and attempts to
figure out how this change will affect them. f these feelings are resolved
one can accept the new set of circumstances and [imitations. Long-term
painful thoughts and emotions can lead to alienation, Loss of friends,
more symptoms, feeling alone, and so on. The onset of a disability Lat
er in life creates dramatic changes in how to live and feet. Studies show
that people who are born with disabilities tend to be better adjusted than
people whose disabilities came later in life. A person who goes blind at 20
years old deals with issues around the loss of never seeing what’s around
them again, and the fear of, ‘Will I be okay? Can I still live without seeing?”
Someone who has done physical work his whole life may experience a
tremendous struggle. Finding out that s/he can no longer work or be inde
pendent can be especially debilitating.

Effects on care-givers
Many people who suddenly become disabled, or develop an illness
which creates a disability, suddenly have a greater need for support, both
physicalty and emotionally. At times, just when a family member thinks
everything is ‘under control”, more needs or emotions may arise. As many
disabilities and iltnesses take place later in life, many times these people
are in committed relationships. This creates a special set of conditions
affecting the spouse. It is a problem they have little control over. Many new
issues surface, not just in coping with the issues of the person who’s ill, but
also coping with what one’s commitment is, finances, change of lifestyle,
increased responsibility. It becomes an ongoing. lifetime struggle. Some
one diagnosed with multiple sclerosis, for example. sees their abilities
disappear. and can see their family trying to accommodate. They see their
state deteriorating and their need for assistance increasing and therefore,
feel Like more of a burden. It may become imperative to an individual’s
mental and physical health to get support from friends, family. or a coun
sellor/mental health professional.

h. OncoLogy
Patients admitted to Oncology Wards face a high morbidity and mottality
risk. They have grave concerns about the unpredictability of their illness
and about their loved ones. They are faced with the painful side-effects
of chemo and radiotherapy and have fears of disfigurement and disability.
This fear, and that of impending death and associated pain and distress be
come the major preoccupation for these patients. Some of these patients
would not even understand the meaning of their illness. Others may have
strange unexpected reactions upon hearing the news for the first time. This
may include manic reactions, dissociative reactions, denial, panic reactions
and depression. Some individuals may even begin to think about suicide.
The patient’s relatives may also have similar reactions to the news. Most of
them expect a miracle from the doctors and staff and it may take time for
the realities of the illness to set in.
In addition to the patients and relatives, the staff and doctors working with
patients of terminal illnesses also undergo slow and gradual changes in
their attitudes. Exhaustion and emotional burnouts are common amongst
oncologists. Working as a team, taking time out for catharsis, ventilation
and for relaxation and changing the monotony of this very demanding job
are some of the methods to prevent against such conditions.

Interventions
Studying and seeing the reactions that people have to hearing bad news
may give the naïve health professional the idea that it should be avoid
ed, As discussed in Section A. to decide what information is to be given
to the patient is not the prerogative or choice of the doctor. The patient
must be specificatly asked what information they would like to receive
regarding their illness. Breaking bad news to the patient and the relatives
is a common scenario in oncology wards. It requires some skitls in coun
selling techniques (See Section A, Non-Pharmacological Interventions).
Each patient and family is different and requires a unique and personatised
approach towards breaking the news. It is advisabte that the senior doc
tor or consultant should take the responsibility of delivering the news and
handling of the reactions, afterwards. With most patients this may be all
that is required. In some, however, the help of a mental health professional
may be needed to help the patient work through their sense of loss. The
patient’s role in the house may determine what the loss would mean to
the family when the patient is gone. Ifs/he is head of the family, they may
need time to delegate responsibiLities before starting the treatment. The
treating doctor should give due consideration to the patient’s needs. If
the patient goes into a state of dependency and loses all witl to fight his/
her illness or becomes depressed s/he may require treatment with anti-
depressants and professional handling. All available social and emotional
support must be mobilised for the patient. The queries of the relatives and
the patient about the illness and its treatment, likely outcome, side effects
of the drugs, prognosis and duration of the treatment and any alternatives
should be discussed. The doctor must arrange a separate session for
providing this informational care.

i. Operating Theatre
Operating theatres (01) are viewed as sinister places by the patients. They
feel that “nobody knows what goes on behind those closed doors, till they
themselves enter the room. Most of them are scared of what is going to
happen to them. The fears are of being vulnerable, being undressed and
exposed to the eyes of complete strangers, of going through pain, of being
under the knife and of dying on the table. When in fear people may react
with anxiety, restlessness, irritability, anger and frank
outbursts of aggression and panic. These reactions of the patient should
be viewed as natural response to a fearful situation. The existing envi
ronments in the theatres are such that they depict a coLd and indifferent
atmosphere to the patients that further increases their anxieties and fears.
Machines, surgical instruments, bLood spills here and there, shouts of panic
and heLp from staff members to each other adds to the patients’ anxieties
and Leave him/her aLone to handle them. The casual communication of the
doctors and nurses may further enhance the scary image of the OT. The
patient’s reLatives, waiting outside, are in a state of extreme distress, fear
and apprehension about the outcome of the surgery. The fears are not only
about the success or otherwise of the surgery and the Life of the patient.
interventions
ted by reassur
Most of the fears and anxieties of the patients are allevia
ure and its likely
ance and providing informational care. The surgical proced
expected all
results, how long the surgery will last, what kind of pain is
An introduction of
need to be discussed with the patient prior to surgery.
the surgeons help in removing apprehensions of patient s.
n or assistants
Receiving the patients in the theatre by the operating surgeo
many uncertain
in an empathic and professional manner helps removing
ties of the patients.
esia and tike[y
An informational care session about the procedure, anaesth
emotional
results to the relatives helps in mobilizing necessary social and
support to the patient.
in the surgery.
The patient has a right to be informed of any complications
discusses prob
This is less stressful for alt parties involved if the surgeon
.
lems that are tikely to occur with the patient before the surgery

I
Chapter 4
Psychosocial Peculiarities of Dentistry
,The oral cavity is one of the most tender and most vulnerable parts of our body.
We teed ourselves through it and kiss our loved ones with it The mouth is literali.y
a path to our innermost setf. The tongue is the only organ in our body which is fully
developed at birth and functions during the first 2 months of life. Infants are de
pendent upon it for nourishment, to communicate and express their feelings, and
to’explore the world (we all know how infants put everything they touch into their
mouthsD. During this early part of our lives, we are helpless. dependent vulnera
ble and unable to express oursetves fully.
A visit to the dentist is unlike any other medical experience. During dental interven
tions, we place our mouths in a vulnerable position. The feeling of hetptessness
that ilievitably arises from these infant experiences of dependency and vulnera
bility come from our unconscious minds. The resutt is anxiety. We place ourselves
in a physically vulnerable position and suspend our usual physical boundaries. We
render ourselves unable to communicate in the usual way (since our mouths are
what is being tended to) and anticipate pain, while remaining conscious and fully
alert. The close physical proximity of the dentist may also be perceived as threat-
ening. If we add to the mix the negative associations many of us have with doctors
• or other authority figures. it is easy to see how feelings otanxiety might arise in
typica dental settings.
DentaL anxiety and phobias
Dental anxiety or fear of the dentist is a major stumbling block for many people. It
usually prevents otherwise intelligent, rational peopLe from optimizing and main
taining their oral health.
Dental phobia is a fear and avoidance of going to the dentist or fear of any dental
treatment or care. It is believed that more than half the population fears dental
• treatment and because of this avoid seeking dental treatment It is a serious con
dition that leaves people panic-stricken and terrified. People with dental phobia
have an awareness that the fear is totally irrational but are unable to change t
They exhibit classic avoidance behaviour that is, they will do everything possible
to avoid going to the dentist People with dental phobia usually go to the dentist
only when forced to do so by extreme pain.
Other signs of dental phobia include:
• Trouble sleeping the night before the dental exam.
• Feelings of nervousness that escalates white in the dental office wailing
room.
• Crying or feeling physically ill at the very thought of visiting the dentist
• Intense uneasiness at the thought of, or when actually objects are placed in
the mouth during the dental appointment or suddenly feeling like it is
difficult to breathe.
Causes of Dentat Phobi,and Anxiety
There are maiy reasons why some people have dental phobia and anxiety. Some
of the common reasons include:
• Fear of pain is a common reason for avoiding the dentist,
This fear usually sterns from an early dental experience that was
unpleasant or painful or from dental ‘pain and horror” stories told by others.

170
Thanks to the many advances in dentistry made over the years, most of
today’s dentaL procedures are considerably less painfuL or even pain free.

• Fear of injections or fear that the injection won’t work.


Many peopte are terrified of needles, especially when they are
inserted into their mouth.

• Fear of anaesthesia and its side effects


Peopte may fear that the anaesthesia has not yet taken effect or was
not a large enough dose to knock out any pain before the dental
procedure begins. Some people fear the potential side effects of
anaesthesia such as nausea, dizziness or feeling faint. Others do not
like the numbness or “fat lip” associated with local anaesthetics.

• Feetings of hetplessness and toss of controt.


It is common for people to feel these emotions considering the
situation sitting in a dental chair with your mouth wide open,

unable to see what is going on.

• Embarrassment and toss of personat space.


Many peopte feel uncomfortable about the physical closeness of the
dentist or hygienist to their face. Others may feel self-conscious
about the appearance of their teeth or possible mouth odours. Most
individuats with dental phobias have had very negative experiences
with either unskilled or incompetent dentists.
The most important step for many patients to overcome dental anxiety is to
find an expert dentist. An expert dentist is one who:
• communicates effectively.
• is highty competent.
• endeavours to make each meeting pain free.
• genuinely cares about the patient.
• has the ability to nurture a patient through past traumas.
The most significant factor in reducing dental anxiety is the ability of the
dentist and staff to quickly develop rapport with each patient. With ade
quate rapport. the patient feels understood, safe and protected. Knowing
how to ask the patient what they are thinking and feeling is crucial to de
veloping a connection. By eliciting this information, the dentist can respond
to each patient appropriately and meaningfully. Additionally, a connection
with each patient is fostered when the dental office environment is relaxed,
welt organised, and the staff is friendly, warm and caring.
Psychological techniques to reduce anxiety
There are a number of psychological techniques that can be used to re
duce levels of anxiety.
i. Psychological support and counsetting
Counselling can help with a whole range of phobias. There are
several different approaches, but generally the aim is to discuss your
anxieties in a way that can hetp to overcome them. This may involve:
• exptoring the origins of a feat in chitdhood.
• looking at the thinking patterns that arouse a fear.
• facing fear gradually so that one may learn to cope with it.
I I • teaming ways to cope, such as relaxation techniques, breathing
exercises or meditation.

ii. Distraction techniques


It is sometimes heLpful to draw attention away from the treatment.
This can be done in a number of ways:
• Listening to music. For children cartoons may be played on a
tv that has a view from the chair
• Concentrating on relaxing each part of the body in turn.
• Thinking about something one is looking forward to.

iii. Hypnotherapy
Hypnotherapy creates an attered state of mind that feels [ike being
very relaxed. In this state, suggestions made by a therapist (or a
dentist trained in hypnotherapy) can help people to receive dentat
treatment. People can also be taught how to do this for themselves.
Hypnotherapy may not work for everybody, however, while some
people may be more susceptible than others.

Dental appearance and psychosociaL welt being


Aesthetic dentistry has become a prominent force in today’s popular
culture, with treatments ranging from bleaching to “invisible” braces being
commonplace. A healthy and attractive smile is undoubtedly valuabte,
given the current focus on aesthetics. It is understandable that any devia
tions from an “ideal” dental appearance could be detrimental to a person’s
• psychosocial well-being.
A number of investigations of craniofaciaL malformations (such as cleft lip
with or without cleft palate) have sought to eLucidate the psychosocial im
pact these conditions have on affected people. These people are generally
welt-adjusted psychosocially: however, they reportedly have decreased
social interactions, fewer close relationships and feelings of self-con
sciousness regarding their facial features. For these reasons, patients with
clefts should be treated early to prevent or interrupt negative psychosocial
outcomes.
Research has documented the effects of malocclusion and motivations
for orthodontic treatment. Malocclusion has been identified as a potential
threat to one’s body image and seLf-concept in both adolescence and
adulthood. Many of these patients have a negative self-image as
motivation for treatment. These individuals frequently report experiencing
a level of psychological distress concerning their appearance that warrants
intervention. It is essential that all health care providers understand how
dental health and appearance fits into the larger picture of overall health
and well-being.
Teeth are important to an individual’s self- perception during adolescence,
but by aduLthood other factors have greater significance. The importance
of having the perfect smile changes as people age. This is why although it
may make a minor contribution to an individual’s perception of self-worth,
orthodontics cannot be justified on psychological grounds alone.
Stress Reductive Anxiety Disorders and Oral Health
Objective disregard their oral health
• Patients with anxiety disorders may
for dental caries, periodontal
altogether and are at an increased risk
To condition muscle
relaxation response disease, and bruxism (grinding).
ety disorders can manifest
• Oral health problems associated with anxi
Lichen Planus, mouth ulcers.
in tle form of canker sores, dry mouth,
andibutarjoint dordes.
burning mouth syndrome, and ternporom
panic attacks. generalised
Symptoms primarily • Anxiety disorders, which include phobias.
(PTSD). are serious
related to anxiety anxiety and post- traumatic stress disorders
that can be treated with a
Symptonis secondarily conditions with oral health implications
aggravated by anxiety
variety of methods.
Bwxism or clenching activity and some anti-psychotics
Muscle pain Medications such as anti-depressants
uce saliva, which can increase
decrease the mouth’s ability to prod
periodontal disease. A liaison
Techniques
the risk of developing tooth decay and
dentists can benefit such
of the mental health professionals with
Relaxation techniques
Biofeedback training
patients.
Temporomandibutar and Facial pain
andibular and facial pain have mul
It is an established fact that temporom
g significant in aggravating them.
tiple causes with the role of stress bein
rome’ usually seen in elderly women
Condition such as Burning Mouth synd
n. Conversely, in cases where an
may sometimes have a psychogenic origi
ry of pain and dysfunction can pro
organic pathology exists, a chronic histo
nts may frequently display charac
duce psychologicat changes. These patie
is characterised by
teristics of ‘chronic pain syndrome’. This
orcement of illness behaviour. These
increased anxiety, depression and reinf
pain behaviour and an increase
features lead to an increase in the patient’s
ry of such a patient, it is prudent to
in pain perception. When taking the histo
pain or dysfunction keep you from
ask questions such as “What does this
nt alteration in occupational
doing?” If the patient complains of significa
a careful evaluation needs to be
or social activities as a result of the pain,
g may be therapeutically reward
done. This process though time consumin
ing in the long run.
Stress Reduction Techniques
in the body such as release of
Stress produces physiological responses
restriction of blood flow to the
adrenaline, contraction of sketetat muscle,
to the brain, increased heart rate
digestive system, increased blood flow
TM joint movement that results
and increased blood pressure. Pain in the
can benefit from a variety of stress
from stress induced muscle contraction
is the Jacobson’s technique
reduction techniques. Most famous of these
e of a particular muscle, usually
which teaches the patient to become awar
training is to condition the mus
by contracting it. The goal of the relaxation
for a prolonged period of time.
cle to achieve a relaxed state quickly and
audio tapes designed for the
This process is also facititated by relaxation
patient.
has been shown to be helpful in
Biofeedback is another technique that
Biofeedback teaches the patient
reducing stress-induced muscle activity.
lated functions of the body. It
voLuntary control over automatically regu
ng of the physioLogical parameters
provides the patieht with instant monitori
c (EMG) biofeedback monitors the
of stress. For example Electromyographi
monitor this through visual and
level of muscle activity and the patient can
ee of cont raction of selected muscles.
auditory displays indicating the degr
adequately relax these muscles
With good training the patient learns to

-—-- -j
Chapter 5
Psychosocial Aspects of Alternative Medicine
The term alternative medicine refers to atl medicine that has not been
tested using the scientific method, and has no evidence to prove its efficacy.
This includes homeopathy, acupuncture. hikmat and other forms of herb
al medicine. In our culture, alternative medicine is usually the first line of
treatment that a patient adopts when s/he first becomes ilL It is practiced
by anyone and everyone who has ever experienced illness, and alternative
medical advice can be sought from anyone, whether it is a fruit vendor, a
neighbour or your grandmother. The reasons for this are that traditionalLy
a[Lopathic medicine was considered a western invention, harsh’ on the
body and unsuitable for the subcontinentat climate. It was only to be turned
to if and when all else had failed, and the healing properUes of household
remedies had failed. A return to the use of alternative medicine is seen in
patients in situations where allopathic medicine suggests a painful cure, or
does not have one.
White anecdotal evidence states that these alternative methods are ef
fective there is no scientific evidence to back this claim. Homeopathy, in
particutar, according to the NHS UK has been cited as ‘performing no better
than placebos.” The National Institute of Health and Care Excellence (NICE)
guidelines do not recommend the use of homeopathy for the treatment of
any ailment. In a 2013 large scale study conducted by the Nationat Health
and Medical Research Council of Australia, 57 systematic reviews containing
176 individual studies, published between 1997 and 2013 were evaluated by
an independent contractor overseen by a body of homeopaths. The study
found that there is no evidence that homeopathy caused greater health im
provements than placebo. or caused health improvements equal to those
of another treatment. This finding has been repeatedly confirmed by find
ings in various peer reviewed journals. tl][21[3][41
As health professionals it is therefore, unethical to recommend homeo
pathic treatment to any of our patients, even if we have personally feel that
they are effective. Any patients who report that they are using homeopathic
medication must be informed that there is no evidence to support their effi
cacy. They must also be instructed to continue the use of altopathic medi
cine if they insist on using homeopathic or other remedies.

References
1. Fisher, P., & Ernst, E. (2015). Should doctors recommend homeopathy?.
www.bmj.com
2. Australian National Health and Medicat Research CounciL Statement on
homeopathy. 2015.
www.nhmrc.gov.au/_ffles_nhmrc/pubtications/attachments/camo2_nhm-
rc_statement_homeopathy.pdf.
3. Ernst E. A systematic review of systematic reviews of homeopathy. British
journal of clinical
pharmacology. 2002 Dec 1;54(6):577-82.
4. Campbell A. Homeopathy in perspective. Lulu, 2008.
5. Cucherat M, Haugh MC, Gooch M, BoisselJP. Evidence of clinical effica
cy of homeopathy. European Journal of Clinical Pharmacology. 2000 Apr
1:56t1):27-33.
6. Smith K. Against homeopathy—a utilitarian perspective. Bioethics. 2012
Oct 1;26(8):398-409.
Chapter 6
Common Psychiatric Disorders in General
Health Settings
The workings of the human mind have remained mysterious since the
beginning of time. Psychiatry. the medical specialty devoted to the study.
diagnosis, treatment, and prevention of mental illness, is just as Little
understood, despite being the oldest profession known to man. In prehis
toric times, people haunted by ‘evil spirits” were taken to medicine men
•xed!n
to have holes drilled into their skutts to let the spirits escape. The need to
istHyaKhan understand the way our minds work coupled with lack of scientific prowess
to investigate the mechanics of the brain led to the perpetuation of these
myths. The first psychiatric facilities were set up in the 8th century in the
Islamic world but methods more humane than exorcisms did not come into
use until the seventeenth and eighteenth centuries. Psychiatry has come
a long way since then but sadly the perceptions surrounding it are very
much prehistoric, especially in our part of the world.
The myths and misconceptions surrounding mental illness, treatments and
mental health professionals themselves are many and manifest. The war
against the stigma of mental illness is one that we alt fight. whether we are
protesting against being called “crazy” or fighting for a loved one to get
help. According to the WHO Mental Health Action Plan 2013-2020 Re
port, “mental, neurological and substance use disorders exact a high toll,
accounting for 13% of the total global burden of disease in the year 2004,
Depression atone accounts for 4.3% of the global burden of disease and is
among the largest single causes of disability worldwide Eli % of all years
lived with disabitity globally], particularty for women. The economic conse
quences of these health losses are equally large: a recent study estimated
that the cumutative global impact of mental disorders in terms of lost eco
nomic output will amount to US$ 16.3 million between 2011 and 2030.”
Some of the most common myths and misconceptions surrounding mentaL
health issues, their treatments and mentat health professionals are being
dealt with here hi:

Myth: Psychiatric Illnesses do not exist or are caused by magic and evit
spirits
Reatity: Psychiatric illnesses are caused by structural and chemical chang
es in various structures of the body, especially the brain, and are just as
much a “curse” as any other illness or adverse life event. The reasons for
this are genetic, biochemical, behavioural and environmentaL According
to a WHO Report, one in every four people in the world wilt be affected by
mental or neurological disorders at some point in their lives. Psychiatric
Illnesses constitute 15% of all diseases incurred in people throughout the
world. 33% of all hospital attendances are for psychiatric diseases.
Myth: Psychological factors do not cause any other diseases
Reality: Psychological factors are acause of the disease for at least 60% of
alt patients with any disease. In fact, mental illnesses such as depression
may predispose one to developing infections, heart disease, diabetes and
even cancer.
Myth: Psychiatric patients are dangerous: theycan harm tife and property:
Reality: gg% of aU.violence, crimes and homicides are committed by the
so-called “normaL” The large spectrum of patients with mental illnesses
is non-viotent and not dangerous to other people. In fact, according to a
study published in The Lancet, “people with psychiatric disabilities are far
more tikely to be victims than perpetrators of violent crime (Appteby, et al.,
2001). “People with severe mental illnesses, schizophrenia, bipolar disorder
or psychosis, are 2 Y2 times more likely to be attacked, raped or robbed
than the general population (Hiday. et aL,;ggg).

Myth: Psychiatric Patients Are ‘“Fraudulent, Matingerers, Hysterical”, Be


having Abnormalty To Fulfil Ulterior Motives
Reality: Extensive research has shown that those who were labetled
“fraud” ‘fake” “attention seekers” were in fact misdiagnosed and misun
derstood. Follow up studies conducted ten and fifteen years after people
were Labelled matingerers and frauds, revealed very high rates of death
and disease amongst the “frauds.” The nature of psychiatric disorders is
such that people are unable to cope with their daily routines, experience
a lack of motivation, and start to behave differently. They, therefore, seem
to be malingering or “seeking attention.” The real reason for this is the brain
experiencing neurobiologicaL chemical or structural changes. As all bodily
functions are controlled by the brain, patients with psychiatric illnesses
also experience changes in their appetite, sleep and sex lives for extended
periods of time.

Myth: Psychiatric treatments are lifelong, addictive, put you to sleep and
render you incapable of living your tife
Reality: Psychiatric treatment includes a myriad of different kinds of
treatments including medication and therapy. Medication is not addic
tive if used according to prescription, white therapy is conducted for a set
number of sessions. Only a certain class of prescribed medication induces
sleep. The main aim of treatment in psychiatry is, in fact, to ensure patients
are abLe to return to their daiLy routine and living their lives as fully as
possible.

Myth: Spiritual Interventions, homeopathy and “ghareLu totkas” can be


used in place of medication prescribed by the psychiatrist.
Reality: Medications and psychotherapy should be used in conjunction
with spiritual interventions and neither should replace the other. Both med
ication and psychotherapy use in the treatment of psychiatric issues have
been researched extensively and found to be effective. According to The
National Health Service of the UK “There is no good-quality evidence that
homeopathy is effective as a treatment for any health condition.” The larg
est to-date analysis of all data on homeopathic treatments, conducted by
the National Health and Research Council of Australia has concluded that
they are ineffective in the treatment of any clinical conditions in humans.

Myth: Psychiatrists give “electric treatment” to alt patients


ReaLity: Electroconvulsive therapy or electroptexy is a welt researched,
safe, painless and effective treatment used for serious issues in very specif
ic disorders.
Myth: “Once a psychiatric patient...Atways a psychiatric patient’
Reatity: Psychiatric itlnesses are not Like heart disease, diabetes or hyper
tension (all of which require lifelong treatment). Mote than sixty percent
of psychiatric illnesses are curable and adherence to medication as pre
scribed can lead to cure within 6 months to a year.

Following is a brief account of some of the most commonty encountered


Priority Psychiatric Disorders as far as the general heatth settings are
concerned. All health professionals must have an understanding of the
presentation and management of these disorders.

a. Mixed Anxiety and Depression


Mixed Anxiety and Depression may present initiatly with a history of two or
more weeks of physical symptoms such as fatigue and pain. Further inquiry
will reveal depressed mood and feeling of fear, apprehension, or a state
of gloom. When symptoms are not severe enough for the individual to be
diagnosed with either an anxiety disorder or a depressive disorder, a diag
nosis of mixed anxiety and depression can be made.

Diagnostic Features:
• Low or sad mood,
• Loss of interest in activities
• Loss of pleasure in things previously found enjoyable.
• Inability to cope with routines/duties, at work and/or at home
• Prominent anxiety or worry.
Disturbed sleep,
• Poor concentration expressed as forgetfulness,
• Change in appetite,
• Dry mouth,
• Tremors,
• Palpitations,
• Dizziness,
• Suicidal thoughts or acts.

Differential Diagnosis
• If either depression or anxiety is severe, consider depressive disorder,
or anxiety disorder as independent conditions.
• If hallucinations (hearing voices, seeing visions) or delusions (strange
or unusual beliefs) are present consider Acute Psychotic Disorder.
If excitement, elevated mood, rapid speech is present, consider
Bipolar Affective Disorder

Management Guidelines
If the diagnosis of anxiety and depression is made follow the steps as listed
below:
Provide Informational Care
The patient and family members must be provided informational care
regarding the aetiotogy, management and prognosis. The following
must also be mentioned, in tight of the stigmaandmythssurrounding
mental illness:
• Stress or anxiety has many physical and mental effects. The
symptoms are reat and understandable. They can be managed and
relief is possibte.
• These problems are not due to weakness or laziness; patients are
trying their hardest. Do not blame the patient for not trying hard
enough or consider him weak and timid. Do not repeatedly tell him!
her to use wilLpower, go on a holiday, change jobs, etc. to get better
Counsetting
• Teach the patient to practice relaxation methods such as progressive
muscular relaxation, deep breathing exercises or visuaL imagery to
reduce physical symptoms. Ask the family to remind and encourage
the patient to undertake the exercises.
• Ask about risk of suicide in explicit terms — can patient be sure of not
acting on suicidal ideas? (See box)
• Help the patient to plan short-term activities which are relaxing,
distracting or are known to build confidence in the patient. Resume
activities which have been helpful in the past.
• Identify exaggerated worries or pessimistic thoughts. Discuss ways to
chaltenge these negative thoughts.
• If physical symptoms are present, discuss the link between physical
symptoms and mental distress (Reattribution).
• If tension-related symptoms are prominent, advise relaxation
methods to relieve physical symptoms.

Medication
Consider antidepressant drugs if depressed mood is prominent. In
mixed anxiety & depression tower doses may be effectiv, e.g.
imipramine starting at 25 mg each night increasing to 150 mg by the
15th day, or fluoxetine 20 mg/ after breakfast. Explain to the patient
that:

• Medication must be taken every day


• Improvement wilt build over 2-3 weeks.
• Mild side effects may occur that usuatty fade in 7-10 days, or become
less disturbing. These may inctude dryness of mouth, constipatior’i.
excessive sweating.
• Check with the doctor before stopping medication as abruptly
stopping it may result in discomfort.
• Continue antidepressant for 6-8 months after the symptoms improve.

Specialist ConsuLtation/Referral
If suicide risk is severe, consider urgent referral tothe nearest
psychiatric facility after admission. Refer the patient to nearest
psychiatric OPD if the patient does not respond to the management
plan even after 06 weeks of treatment.
-..... ,, ,, .. . ..

L.

b. Panic Disorder
Presenting Complaints
These patients usually present to the Emergency Room with one or more
physical symptoms such as chest pain, dizziness, shortness of breath, feeL
ing of suffocation and fear of having a ‘heart attack.” Further inquiry shows
the full pattern described below.

Diagnostic Features
3-4 attacks of sudden onset of anxiety or fear in which there is a feeling
of dread, impending disaster, accident and the patient feels as ifs/he is
about to die. It often occurs with physical symptoms such as palpitations.
chest pain, a choking feeling, churning stomach, dizziness, feelings of
unreality, or fear of some disaster (losing control or going mad, heart attack
and sudden death).
A typical panic attack begins suddenly. builds rapidly. and may last only a
few minutes. Symptoms start white the patient is at rest. It often leads to
fear of another attack and avoidance of places where attacks have oc
curred. Patients may start to avoid exercise or other activities which pro
duce physical sensations like panic.

Differentiat Diagnosis
Many medical conditions may cause symptoms similar to panic attacks,
such as coronary artery disease (CAD) and asthma. These can be ruled out
with appropriate history such as onset of pain or breathlessness on effort.
changes in rhythm, rhonchi or crepitations on examination of the chest and
typical changes in ECG. This exercise should not be repeated with each
subsequent attack once the diagnosis of panic attack is established.

• If attacks occur only in specific feared situations, consider Phobic


Disorders.
• flow or sad mood is also present, consider Depression.

Management Guidelines
Provide InformationaL Care
Essential Information for Patient and Family

• Panic Disorder is common condition and effective treatments are


available.
• Anxiety often produces frightening physical symptoms. Chest pain.
dizziness, or shortness of breath are not necessarily signs of heart
disease, asthma or similar disorders. None of the symptoms being
experienced are dangerous or fatal. They are like a storm. that comes
and then passes. Alt you have to do is tie low. do not start to rush to
the hospital, or GP, and practice deep breathing and progressive
muscle relaxation. Keep repeating in the mind that the
symptoms of a panic attack are transient and always pass. They
cannot cause any harm to heart, brain or any of the vitaL organs. To
develop a belief that a panic attack is not a threat to life is an
important step in effective management of a panic attack.
• Mental and physical anxiety reinforce each other. Concentrating on
physica’. symptoms will increase fear.
• Do not withdraw from or avoid situations whereattacks have
occurred; this will strengthen anxiety.

CounseLling
the patient and family to:

Counsel
Concentrate on controtting anxiety. not on medical worries.
• Practice slow, relaxed breathing. Controlled breathing will reduce
physical symptoms.
• Identify exaggerated fears which occur during panic (e.g. patient
fears, he is having a heart attack). Discuss ways to challenge these
fears which occur during panic e.g. the patient could tet[ himself
am not having a heart attack. This is a panic attack and it will pass in a
few minutes.
• While the symptoms may appear scary, the patient must be left alone
when they are having a panic attack to practice their relaxation
exercise, and learn how to calm their ownsetves. The panic generated
by family members on seeing their condition will only worsen matters.

Medication

Many patients with panic disorder will not need medication. The use
of relaxation exercises and reassurance is sufficient.
• If attacks are frequent and severe or if significant depression is
present, antidepressants may be helpful e.g. imipramine 25 mg at
night increasing upto 75— 100 mg at night after 2 weeks.
• For patients with infrequent and limited attacks, occasional use of
anti-anxiety medication may be helpful: torazepam one mg up to
three times a day for two to three weeks but never more than five
weeks.

• Always taper the dose in allowances of 1/4th of the dose every fifth
day. Regular use of benzodiazepines may lead to dependence and
is likely to result in the return of the symptoms when discontinued.
Avoid unnecessary medical tests or therapies.

Speciatist Consuttation/Referrat
• Consider a referral for consultation with psychiatrist, if severe attacks
continue after the above treatment, for 4 weeks.

Avoid referral for medical consultation for exaggerated worrying
regarding medical symptoms.

_.1
t Comptainers
c. UnexpLained Somatic Complaints: Persisten
Presenting Comptaints
symptoms may vary ac
Any physical symptom may be present, but the
ngs of patients. Complaints
cording to the geographical and cultural setti
ical or unusuaL Some of the
may be singLe or multiple. but tend to be atyp
gas’ or ‘gota’, aches and pains
common complaints incLude a feeling of
che, back ache, shoulder or neck
from head to toe, headache, stomacha
ibed site that cann ot be explained on
ache in a peculiar and circumscr
nied by an extraordinary
medical or anatomical basis. These are accompa
disorder.
concern of harbouring a dangerous or dreadful
Diagnostic Features
on (proper history and
• Physical symptoms without medical explanati
this).
physical examination are necessary to determine
stigations that yield no
• Frequent medical visits and laboratory inve
unusual findings.
ical illness.
• Patient may be overty concerned about med
mon, but are not forth
• Symptoms of depression and anxiety are com
coming.
Differential Diagnosis
Panic Disorder, and
• If anxiety symptoms are prominent, consider
manage as mentioned above
ession. If it is severe or
• If low or sad mood is prominent, consider Depr
res such as
associated with suicidal ideas or psychiatric featu
nt, and seek psychiatric
delusions and hallucinations, admit the patie
consultation.
nt (e.g. belief that
• If strange beliefs about symptoms are prese
Disorder. Refer to the
organs are decaying) consider Acute Psychotic
nearest psychiatric facility.
plaints/somatization is
• If the diagnosis of unexplained somatic com
.
confirmed: move to implement following steps
Management Guidelines
Provide Informationat Care
Essentiat Information for patient and famity:
• Stress often produces physical symptoms.
vering their cause.
• Focus on managing the symptoms. not on disco
best life possible
• Cure may not be possible: the goal is to live the
even if symptoms continue.

Counselling
They are not lies or
• Acknowtedge that physical symptoms are real.
is not imag ining these symptoms.
inventions or “veham.” The patient
the symptoms, offer
• Ask about what the patient thinks is causing
mina l pain does not indicate
appropriate reassurance (e.g. abdo
focu s on med ical worries.
cancer). Advise patients not to
• Discuss emotional stresses that were present when symptoms arose.
• Reattribution: link physical compLaints ith emotional distress.
Discuss emotional stresses that were present when symptoms arose
such as feeling of anger. envy, jealousy, grief, loss, disgust. fear,
sadness or threat.
• Relaxation methods may help relieve symptoms related to tension
(headache, neck or back pain).
• Encourage exercise and enjoyable activities. Do not wait until all
symptoms are gone to return to normal routines.

Medication
• Avoid unnecessary diagnostic testing or prescription of new
medication for each new symptom. Offer reassurance, reattribution,
relaxation exercises, massages, and physical exercises in place of
symptomatic drugs.
• Antidepressant medication (e.g. fluoxetine 20 mg per day) may be
helpful in some cases e.g. those with headache, bowel symptoms,
atypical chest pain.
• Do not use benzodiazepines, antacids, analgesics. multivitamins or
placebos as symptomatic treatment in these patients.

Specialist ConsuLtation
Avoid referrals to medical specialists; these patients are best
managed in primary care. Keep in mind that patients may be offended
by a psychiatric referrat and seek additional medical consultation
elsewhere.

d. Dissociative and Possession States


These occur when a patient is brought by family or friends on account of
violent or unmanageable behavior due to being possessed by ajinn or evil
spirit. These states are often an involuntary attempt by the individual to
escape from a stressful or demanding situation.

Presenting CompLaints
• Reduced awareness of self and surroundings (I can hear sounds but
cannot respond to them) or a constricted level of consciousness.
• Disturbed memory. where simple information is forgotten for short
periods (patient is unable to answer queries Like what is your name,
who are you, where are you, how many Legs does a cow have?).
• Attention and concentration lapses.
• Wandering away from home, to be found at a place where others can
recognise him or her and return home, or sleep-walking.
• Change or loss of motor and sensory function of a part or whole body
or inability to talk, swallow, see, or hear without any neurological
signs to support the change in function.
a jinn’ or ‘bhoot’, or a supernatural
• Statements of being possessed by
language. voice tone, and
force or deity. causing a change in
assuming of another identity.

Diagnostic Features:
ility
s occur due to the individual’s inab
As mentioned earlier these symptom reali ty into a situa tion
of escape from
to cope with reatity. They are a form
where the individua l is more able to deat with their issues. Females are
es
ptoms. This is a result of their voic
more likely to suffer from these sym be an
ions not considered. It may
often being suppressed and their opin log
d stress, or physical or psycho
expression of a social, relationship-relate disas ters, and
iers, people affected by
ical stress. It may also be seen in sold stiga tions and
e and uncalled for inve
survivors of psychotrauma. Extensiv id
all pos ible physical causes should be avo
tests with a view to rule-out’ rders is a posi tive di
possession diso
ed. The diagnosis of dissociative and cho
dee per unde rstan ding of the patient and their psy
agnosis based on a tive com mun icati on skill s.
se and effec
social world using good clinical sen
gical functions, any suicidal
Questions about disturbed mood, biolo
t always be asked, Patient often re
wishes, and psychosocial stress mus or
lar presentation or has lived with
ports having a family member with simi
state.
seen another patient with possession
t of
‘evil-eye’ and a cuttura[ endorsemen
A belief about supernatural forces, and fami ly’s belie f syste m.
nt’s own
possession states often prevails in patie
DifferentiaL Diagnosis
ms can many a times be an earty
Dissociative and possession sympto olo
physicat. social or a psychiatric path
symptom of an underlying serious roa che d like PUO (pyre xia of
fore be app
gy. The symptoms should be there psy
orbid severe depression, anxiety,
unknown origin). Underlying or co-m and
ld be considered while managing
chosis, psychomotor epilepsy, shou iple scler osis
rders [ike cancers, mult
investigating the patient. Serious diso ve
may rarel y present initially with dissociati
and degenerative disorders
symptoms.
Management Guidetines ,
pted. Patient must never be ridiculed
No punitive measures should be ado are mali nger ing”, you are
ents like “you
challenged, or confronted. Statem
making it up”, “you have no disea se” ‘makar naa karo” ‘dramay karna band
karo”, should never be given.
ted in simple easy to understand
An informational care session conduc to
ment of this state. It is most important
terms is imperative to the manage e reass uring the patie nt
ptoms whit
highlight the involuntary nature of sym
are not life-t hrea tenin g or dangerous. It must be con
and family that they
the symptoms are an expression
veyed to the patient and the family that
ustion, fatigue and inability to cope
of an underlying conflict, nervous exha
like “Kya mareez per jin ya bhoot ka
with a situation. Answers to queries
statements tike “These phenomena
saya hai?” should be responded with
ever, try to find and treat the sci
are not my area of expertise, I can how
you and the patient understand the
entific basis of the problem and help itm
t of view. “Mujhe iss baray mel toh
condition better from a sLientific poin hal our mel
kehti hal k ye musta zehn ka
nahi hal mdgar sciencey tahqeeq ye n ga/g ifl
harne hi hoshish haru
iss ko science h nuqt-e-nazar se hut
Reassure that the symptoms may be toud and dramatic to look at but are
not dangerous and often result in recovery with specialised psychosociaL
care and medication if needed.
The family must be couneled against the use of force, undue attention
and gathering around the patient or trying to hold or physically handle the
patient during the attack. The family must leave the patient alone and
only approach the patient to prevent any serious bodiLy harm. The patient
must be encouraged to try and predict when an attack is about to occur
and use relaxation techniques to overcome the state preferably by staying
in a calm setting away from crowded places. A supportive, kind and re
[axed approach with the patient with repeated empathic reassurances can
help relieve the symptoms. Rest, adequate ftuids and food, adequate sleep
and tranquil settings help in early recovery.

Medication: There is no specific medication to relieve the symptoms of


dissociation. The use of antidepressants or small doses of tranquillisers
may help if they are associated with an underlying symptom of depression
and anxiety.

Specialist Consultation
Such patients should be referred for speciatised psychiatric help as soon
as possible, especially if they have associated depression or psychosis.
Earlyinvolvementofa mental health facility may help in making a quicker
diagnosis and they may be better equipped to manage such a patient.

e. Drug Abuse, ALcohoL & Tobacco Use


Smoking and drug abuse is common amongst adolescents, young adults
and sometimes even medical students and health persQnnel. Alcohol
abuse is also becoming common in the same sections of the society.
Depending on their specific personalities and various other environmental
factors, these people resort to drug abuse mainly for the following reasons:
• An escape from painful reality and responsibility of adulthood
• Peer pressures for identifying with their specific peer group
• To avoid physically painful conditions, e.g. narcotic injections for
relieving pain cause by a fracture.
• To seek novelty and thrill.
• Misconceptions about drugs such as ‘doing it once in a while does
not make you an addict’.
• To use drugs to copy or imitate an ideal or famous figure.

Management
The management of tobacco dependence and drug abuse is assessed on
the following parameters:
• Cutturat and social pressures that led to the start and are now leading
to the abuse of tobacco or a drug
• Patient’s motivation
nt that includes:
Create an individuaUsed pLan for the patie
• Detailed smoking history,
nicotine or the drug of abuse
• Assessment of degree of addiction to
patients smoking or abuse
• A bio-psycho-sociaL perspective of the
patterns,
Realistic expectations
• A specific date to quit.
withdrawal syndrome subside within
The physical symptoms of nicotine
1 -3 weeks but the psychotogical
addiction lasts much Longer. Nicotine
be used as alternatives during the
chewing gum and tow tar cigarettes can
stimulants, solvents and opiates
initial phase. The withdrawal state with
may last for one to two weeks.
cco, alcohol and drug abuse in the
It is important to view the use of toba
following states:
same bracket. All three can cause the
i. Addiction
ii. Physical and Psychological Harm
iii Ever-increasing Demand
iv, Withdrawal leading to:

- Tremors, chills
- Cramps

- Emotional problems
— Cognitive and attention deficits
— Hallucinations
— Convulsions and even death.
e need to be seen as disease states
The tobacco, alcohol, and drug abus
support. A medical or a dental stu
for which the sufferer requires help and
may initially seek counselling
dent who is challenged by these disorders
ces teacher and for severer forms
and support from the behavioural scien
ld be encouraged to learn and
with psychiatric services. Patients shou
situations such as negative mood
practice healthy coping skills for high risk
tions that cause anxiety. Many
states, sudden boredom, and social situa
alcohoL and drugs of abuse
medical students start the use of tobacco,
er hours”, improve memory”, and
to “learn quickly”, “stay awake for long
All of these states can be effec
sometimes to enhance “sexual pleasure.”
healthy means. Drugs of abuse.
tively managed through physiological and
in the short run but may cause
alcohol and cigarette smoking may work
serious long term damage.
to avoid cigarette smoking,
• Simple techniques that can be used
alcohol, and drug abuse include:
• distraction,
• detay,
• not giving in to the urge,
• deep breathing.
• physical exercise
• escape.
• positive statements,
• cognitive restructuring and recommitting to the benefits of quitting.

Physicat exercise, sports, and relaxation techniques have been found to


be particularly effective in dealing with nicotine withdrawaL It is important
not to abandon patients of tobacco, drug and alcohol abuse, to deal with
their challenges all.by themselves. They require support. guidance. and
constant patronage. They can be rescued far more effectively and quickly
through peer and social support accompanied with professional help. Con
stant bickering, criticism, long lectures on morality and hazards of smok
ing and drinking, and threats for dire consequences by parents. teachers,
elders, and friends may cause more damage than good.

The Healing Time Line


A realistic look at how long takes for your body to recover after your last puff

E • Fortysight One 10 nine 0we year. O the years: S Tenyasro: ) Fifteen


.Twenly Eight hours: years: Yaw rink
hours: Your months: The added risk Your steths Yotntiskotat
minutes alter The amount at heart disease
nerve endings Coughing. of heart disease risk nsaybe smoking-
quitblng, year of carbon non
03rtlnregnO dew csngen decleses to half reduced to that related cancers and orioking’
blood pe0550re snide hr yow retalod death is
bow, larigue, of that of a of someone wxhaskerg,
decreases. bleed drops mate, and
smoker. Mrs sever mouth, and nowskreiarto
bockto normal you can noelt arid diiorttmess
at breath nnohe4 moat decreases that of seone
oxygen and tunIC
decrease. by up to 50 one who never
increases to things bettor.
percent. nnoke
normal,
anawncavtnessocoooe
CNS Depressants CNS Stimulants Opiates Cannabinols

Barbiturates Axnphetarninas Morphine HaShish

Nicotine c—a

• chronic contdpa • mncNtJs


Long-term • Alcoholic hepati • Mak-,utrWon - Umg fectio
Effects tis, •Fatigu.
tion • Lung cancer
• Peptic ulcer dis • Impairad vision • LOSS Of ihofttwn
•Insomnla m.mcry concentration
ease -Mood swings and abitract tNnldng
• Sevrn anxiety
•cNs damage Instability •AnxI.ty. Personality
• Pana.atith • immunosuppr.s disturbances Daçx.sslrs
slon • As tolerance • Aniotivadon Syn
•incraased bid develops, the user dmmW set of parsonall
denc. otcaicer of • BizarneMoient can no longerget tychangeslh which
head nedcoe behatilour the pleesurabie users less active and
effects, but must ambitious Unconcerned
sophagus, stom • Amphetamine about th. future and
ad-i. hepatic,co psychosis continue taking
unwilling or unable to
ionk. end tI.aIg the drug to pie ma long-tacos plans.
cancer vent withdrawaL • Amotjvadoni vyrrdrotr
• Totersnc.. tolansory
end other peychologica
affects occtan es icon a
theseosrdorthkdd,
of auccaisjve LSDw.
-Olthcets.dbyplbnl
cal hlurywhan ISO use
Impalrsjudgnsanteg
about trflcoraper
wn’sebllltytofly

•Wthrae:ç•1Nmo

4.upIa
.H.adacb.
nood
f Suicide and Detiberate Self harm (DSH)
All patients of depression must be screened for suicide risk as go% of those
committing suicide or deliberate self harm are suffering from a psychiatric
illness. It is one of the top 10 causes of death in all age groups and one of
the top 3 causes in young adults and teenagers. Certain risk factors are
particutarty associated with completed suicide including:

• Gender (elderly white males being at highest risk),


• Psychosis,
• Alcoholism,
• Chronic physical illness, lack of social support and use of generally
lethal methods in the past (e.g. Gun rather than overdose of
medication). (For more, see box)
Medical students and young doctors may be reluctant to explore suicid
al ideation in the mistaken belief that asking about suicide may actually
increase a patient’s risk or give them ideas of suicide.” In fact, the opposite
may be true. Assessment of suicidal tendencies usually reassures patients,
reduces anxiety for both patient and doctor and facilitates partnership
in suicide prevention. The assessment should begin gradually by asking
questions such as Sometimes when people who are going through the
circumstances that you are, they may start to feel like life is not worth tiv
ingr and then asking more specifically about a history of suicidal attempts.
any specific current plans, hopelessness and any specific current inten
tions.

Deliberate self-harm is a term used to describe an act in which an individ


ual deliberately causes injury or harm to themselves, without the intent to
commit suicide. This may be in the form of cutting, or ingesting substances
in non-lethal doses.

Where doctors globally have a lower mortality rate from cancer and heart
disease relative to the general population, they sadly have a significantly
higher risk of dying from suicide [3]. In fact, according to a survey in 2015,
doctors are the most professionals to commit suicide, followed closely by
dentists. The leading cause of this is the prevalence of depression in med
ical students and postgraduate trainees. Completed suicide is also more
prevalent amongst medical professionals partialLy because of availability
and access to lethal means. Medical students and trainees are at particular
risk because they are unlikely to report a history of depression. Sadly. even
when it is reported, it may be neglected. This is due to myth that people
who ctaim to be having suicidal ideas are not likely to commit suicide. The
truth is 67% of individuals who committed suicide had confessed to some
one that they wanted to kill themselves. The famous Urdu saying o garajte
ham wo baraste nahi’ is untrue when it comes to a risk assessment of sui
cide. Any suicidal ideation being reported should, therefore, immediately
be considered as serious and requiring intervention. A detailed suicide risk
assessment must be carried out in all such cases. ‘The biggest risk of sui
cide is a direct statement of intent” [41 These statements should never be
.

ignored, or responded to with statements of how the person is just looking


for attention” or ‘doesn’t reatly mean it.”
Protective factors of suicide

Sane. of connectadneas

Having Chfl*.n

Good health

flestrictad Access to means

Individualised Family Related Social Environmental


Risk Factors Risk Factors Risk Factors Risk Factors

• U&. gender •Slngh • Socioeconomic .. Easy access to methods


disadvantage cuturaisuppcttersuicisie
• Early adufrage •

• Migrant population M.dio


• Psychological/emotional • Separated
problems UflernpbYrneflt Suicide among peers
• Family conflicts
• P,dcal health problems •Soc
• History oFchUdhood abuse
•Stranful I wants •Family history of suicide .

• Major Depression
•4polerAfbctlveDbuitler
• Subetanc.abus. plenrd.r
• Antioclal Personality
Dlsord.r
• HLstoryofpsychlattlccare
• Previous 5ulcldai
b.hauiow
r
g. DeLirium
Delirium, also referred to as acute confusional state, refers to an acute de
dine in consciousness, cognition and attention. There is an incidence of up
to 55% in medicat and surgical inpatients 12].
Presenting Complaints
Families rather than the patient may request help because the patient is
often in a state of confusion or agitation. Delirium may commonly occur in
patients who are hospitalised for medical conditions, particularly in inten
sive care units and acute surgical units.

Diagnostic Features
Delirium is characterised by its sudden onset of hours or days. Confusion
is often present and patient struggles to understand surroundings. The
following neuropsychiatric symptoms are common:
• Clouding of consciousness
• Poor memory,
• Agitation,
• Changi ng/ftuctuating emotions,
• Loss of orientation,
• Tremors
• Nystagmus
Asterixis
• Urinary incontinence
• Wandering attention,
• Auditory hallucinations (hearing voices without anyone speaking),
• Withdrawal from others,
• Visions or illusions,
• Suspiciousness
• Disturbed steep/reversal of steep pattern.
Symptoms often develop rapidly and may change from minute to minute
and hour to hour, but are worse at night. There may be periods of clear
consciousness or lucid intervals. DeLirium may occur in patients with previ
ously normal mental function or in those with dementia. Mild stresses such
as medications or mild infections may cause delirium in the elderly.

Dilferentiat Diagnosis
Attempt to identify and correct any physical causes of confusion. Common
causes include;
• Alcohol intoxication or withdrawal
• Drug intoxication or withdrawal Cncluding prescribed drugs)
Severe infections
• Metabolic changes (e.g. liver disease, uremia, dehydration
electrolyte imbalance, alkalosis/ acidosis)
If symptoms are persistent and delusions and disordered thinking predom
inates, consider Acute Psychotic Disorder.

Management Guidelines
Informational Care
Essential information for patient and famity:
Strange behaviour or speech is a symptom of a medical ilLness; as
primary cause is treated, patient will return to complete normality.

Counsetting

Take measures to prevent the patient from harming himself or others
(e.g., remove unsafe objects, restrain the patient if needed).
Supportive contact with familiar people can reduce confusion. Visits
by unfamiliar individuals should be stopped.
Provide frequent reminders of time and place with clocks and
calenders in full view of the patient to reduce confusion. Place the
patient’s bed facing a window, trying to keep tights as dim as
possible at night. Introduce yourself to the patient even if you are a
close friend or a relative of the patient.
Medication
ons such as
• Avoid use of sedative hypnotic medicati
t of alcohot or sedative
benzodiazepines (except for the treatmen
withdrawat).
otic medication e.g.
• To controt agitation, prescribe antipsych
tion up to three times per
hatoperidol 1-2 mg by mouth or by injec
day.

SpeciaList consuLtation
admitted, preferably in
• Patients in delirium should always be
intensive care settings. Nece ssary inve stigations should be
assis tance and advice from consultant
conducted promptly and
diagnosis and
physicians should be urgently sought, for
management of underlying cause.
residual psychiatric
• Referral to psychiatrist is only needed if
treatment of the primary
symptoms are seen even at the end of the
cause of delirium.

References
1 http://depts.washington.edu/mhr
eport/facts_violence.php

2. Kaplan & Saddock’s, Synopsis


of Psychiatry
nti
Suicide rates among physicians: a qua
3. Schernhammer ES, Co[ditz GA. s). American Jour nal of Psyc hi
tative and gender assessment (meta-analysi
atry. 2004 Dec ;;;61(;2):22g5-3o2.
s Tom. Chapter 16. Suicide and Deliber
4. Cowen Philip, Harrison Paul, Burn
book of Psychiatry 6th edn.
ate self harm. fl: The Shorter Oxford Text
Chapter 7
Psychosocial Aspects of Gender and Sexuality
The World Health Organisation defined sexuality as a central aspect of
being human that encompasses sex, gender identities and roles, sexual
orientation, eroticism, pteasure, intimacy and reproduction. Sexuality is ex
perienced and expressed in thoughts, fantasies, desires, betiefs, attitudes,
vaLues, behaviours, practices, roles and relationships. While sexuality can
include all of these dimensions, not all of them are always experienced or
expressed. Sexuality is influenced by the interaction of biological, psycho
logical, sociaL economic, political, cultural, legal, historicaL religious and
spiritual factors.” (WHO, 2006a).
In society at large and since the beginning of time, sexuality and the fac
tors associated with it have remained a source of taboo, and thus, curiosity.
This is not the case in clinical settings where health professiOnals are often
faced with issues related to gender, psychosexual issues, sexual dysfunc
tion and sexually transmitted disease. While the medical aspects of these
can be found in physiology and pathology textbooks, there is a need to
clarify terms in use and the provide insight into how to manage issues
of such a delicate nature. As health professionals we may feel not only
ill-equipped to handle these issues, but even uncomfortable in speaking
of them without educating ourselves on the matter. This chapter aims to
relieve some of that discomfort by educating health professionals in the
basic factors pertaining to human sexuality.
Human sexuality depends on the following factors:
Sexual identity
• Gender identity
• Sexual behaviour
Sexual orientation
These factors are also known as psychosexual factors as they determine
the interaction between personality and sexuality in a human being.

Sexuat Identity
This refers to the biological sex of the individuaL This is determined by
chromosomes, internal and external genitalia and hormones. Embryology
studies show that alt genitalia in both XX and XY embryos is initially female.
The presence of genes such as SRY and SOXg leads to the production of
androgens which then cause the genitalia to differentiate into male in the
6th week of life. In the absence of these genes the genitalia devetops and
differentiates as female.
In normat individuals all these factors may fall into one or the other classi
fication of either male or female. In individuals with ambiguous genitalia, or
chromosomal abnormalities such as Klinefelter’s (XXY) and Turner’s syn
drome (XO) this may not be the case. In hermaphroditism both ovaries and
testes may coexist in the same individual white having a genotype of 46XX
or 46XY, Virilising adrenal hyperplasia, the most common female intersex
disorder can be caused by an XX foetus becoming exposed to excessive
androgens.
“Z_z.

Gender Identity
s as belonging to a certain
This refers to how individuals identify themselve
sexual identit y of the individuaL
gender. This is most often retated to the
gend er identit y that an individual
but may not always be the case. The
factors : the temp eram ent of the child, and
takes on is determined by two
de towards the child.
his/hei interaction with the parents and their attitu
by the time they are three
The gender identity of a child is often solidified
category of boy or girl
years of age. Children are now placed in a certain
children start to refer to
and treated as such. This is often seen as when
give statements such as
themselves with gender specific pronouns. They
impo rtant to notice here that
“Mai nahi khetta” or “Mai nahijaoon gi. It is
uage are gend er neutrat, which helps
these statements in the English lang
culture , lang uage and sociat factors
to illustrate the influence the effects
Gend er identit y is deter mine d largely by
ptay in determining gender.
the child.
parental inftuence and that of other adutts surrounding
important that the child
In cultures such as ours, it is considered extremely
er categ ories as early as possible. This is be
be placed in one of two gend
minant of what behaviour is
cause this may be in many cases the only deter
cause a great deal of
expected and acceptable in a child. A boy child may
laughed off when a girt
concern if seen playing with dolls, while it may be
ghar. This is reversed
chitd prefers to ride her bike rather than play ‘ghar
is cons idered a time in which
when the individual reaches puberty as this
mine what is approp riate. Girls, in
the gender of the individual will deter
differe ntly, sit, walk and eat in an appro
particular, are expected to dress
es physically. Boys
priate manner and become more aware of their bodi
the house while girls
of that age may be allowed more freedom to leave
ly a measure adopted
may have a lesser degree of it. While this is certain
may underm ine or cause
for the security of the child, certain measures
individ ual. One such example is that
dissatisfaction with the gender of the
to leave the hous e only in the company of a
of teenage girls being allowed
In the same way. while
mate relative, even if the male is younger than her.
ns, boys are expect
girls may often be allowed to express all their emotio
ham”) and often the only
ed to remain more indifferent, (“Larkay rotay nahi
te aggres sion.
emotion that may be considered acceptab is
girls. are largely similar in
Research reveals that children, whether boys or
boys are more likely to
their behaviour. Some notable differences are that
than girls. Girls are also less
exhibit aggression (both physical and verbal)
mon ths of age.
likely than boys to throw tantrums after a8
due to concern that
Parents often bring their sons to health professionats
be worrie d that the boy
the boy “does not behave like a boy.” They may rather than
to play with dolls
spends mote time in the kitchen, or prefers le sibtings
male amo ngst many fema
cars. In situations where there is one
nce of a strong father
this becomes more common, especially in the abse
emulate and at times
figure. The male child has only female modets to
him as a girl and a lack
this confusion is worsened by the sisters dressing
be ridicuted for being
of activities considered “boyish.” These children may
e gend er identities. Often this
effeminate by other children with mote definit
with age and more male exposure to
may be a phase in a child’s tife and forcing the
s should be disco urag ed from
emulate may disappear. Parent as this may
riate”
child into only doing what they consider “gender approp s
rch yields no benefit
cause significant distress and according to resea
(Zucker 2006).
Sexuat Behaviour
Certain areas of the brain have been identified as ptaying pivotal totes in
human sexuality. The limbic system, as previousty mentioned, is involved in
generating emotion and sexual desire. This has been determined in studies
in which electrical or chemical stimulation of hippocampus, the preoptic
area, anterior thalamic nuclei resulted in penile erections. During orgasm,
female brain areas involved in anxiety and feat showed significantly [ow
activity.
The regulation and processing of sexual desire and stimuli is conducted by
the cerebral cortex. The prefrontal cortex is involved in the inhibition of
sexual impulses. white the orbitoftontat cortex processes emotional input
from the amygdala. Right caudate nucleus activity factors into the
determination of whether arousal will lead to sexual activity. The left
anterior cingutate cortex ptays a role in the sexual arousaL as well as
hormonal controt.
Afferent input from the pelvic, pudendal and hypogastric nerves conveys
sensory stimuli to the spinal cord, where arousal and orgasm are. Sexual
reftexes are mediated in the lumbosacral segments.
Neurotransmitters that play a role in sexual function include serotonin,
dopamine, oxytocin, norepinephrine and epinephrine. Oxytocin is released
following sexual activity and leads to bonding between sexual partners,
as wetl as feelings of contentment, calmness and security. Stimuli that
increase dopamine increase sexual desire. Inhibition of sexual function is
mediated by serotonin released by the pons and the midbrain.
Certain hormones have long since been known to mediate sexual be
haviour. Chief amongst these are testosterone and oestrogen. In both
mates and females, testosterone is known to increase sexual desire.
Oestrogen, in females teads to increased sensitivity to sexual stimulation
and teads to lubrication caused by arousaL Progesterone. cortisol and
excessive prolactin lead to decreased sexual desire.

Gender differences in SexuaL Behaviour


Contrary to myths regarding the matter, men and women both experience
sexual desire (Hyde 2005). Men, however, tend to experience a greater
frequency of sexual thoughts, and readiness to engage in sexual activity.
It must be stated here thatdue to human socialisation, sexual desire may
not be the only reason to engage in sexual activity. This is especially true
for women. Women are more tikely to engage in sexual contact (even in the
absence of desire to do so) for reasons such as pleasing and maintaining
the interest of their partner, as welt as establishing a closer relationship. It is
also important to note that in women physiological arousal may not always
coincide with a subjective sense of arousal or a desire to indulge in sexual
activity, That is to say, physiological sexuat arousal in women cn occur
independently of psychological arousal.
Men and women differ in the determination of their sex drive (Baumeis
ter, 2000). In men sex drive appears to be biologically determined, white
in women, factors such as cultural background, education, religion and
parenting all may play a part. Furthermore in culfures such as ours, wom
en may indulge to hold on to” their male counterparts, as they have their
sense of security (financial and otherwise) invested in the mate partner.
Masturbation
ntly discussed, more
No other form of sexual activity has been more freque
univer sally prac ticed than masturbation.
roundly condemned, and more
Kaplan and Saddock, Synopsis of Psychiatry
-

genitalia by oneself for


Masturbation refers to the physical stimulation of
xual devetopment. It is
sexuat pteasure. This is a normal part of psychose
desp ite the fact that nearly
an activity shrouded in shame and mystery
fourths of fema les indutge in it
all post-pubescent males and up to three
(Kinsey).
become aware of
Children become aware of their genitalia just as they
y. howev er, due to the in
their other body parts. Upon reaching pubert
ent of seco ndar y sexual characteris
crease in sex hormones and developm
tics, masturbation may become a regu lar activit y.
result in loss of
Myths and misconceptions such as “masturbation may
can caus e mental illness”
sexual prowess and potency” and “masturbation
of man hood and mate virility” abound,
or “masturbation may result in a loss
e no scienti fic evid ence to supp ort these
especially in our culture. Ther is ce (not a
sym ptom of an emo tiona l distu rban
claims. Masturbation is a
control and
sexual one) only if it results in the individual losing
compulsively indulg ing in it ho].

SexuaL orientation
sexual orientation
According to the American Psychological Association,
ly and romanticatl.y attract-
refers to the sex of those to whom one is sexuat
n (accord ing to cens us data worldwide)
ed. For the majority of the populatio
ual i.e. individ uals are attra cted to the oppo
sexuat orientation is heterosex
lation of the world
site sex. Statistics suggest that about 1-5% of the popu
be understated, how
has different sexual orientations. This number may
ing homosexuality in most
ever, given government and religious taws bann
i.e attra ction to the same sex,
parts of the world. This may be homosexual,
and asex ual, i.e no sexual attrac
bisexual, i.e attraction to both genders
uals exists, who feel that sexuality cannot
tion. Another subset of individ
s as “queer” or pansexual
be labelled. These individuals refer to themselve
e individuals, sexual
or polysexual. Some research indicates that for som
true for women (e.g.. Dia
orientation may be fluid. This may be especiatly
mond, 2007; Golden, 1987; Peptau & Garnets, 2000),
naljurisdictions have laws
As of July 2015, 72 countries and five sub-natio
tries, inctuding Paki
criminaUzing homosexuality. A majority of these coun
stan are in Asia and Africa.
any variations that
According to Darwin, natural selection dictates that
an organism’s struggle
occur have to be ‘useful to man’ i.e. they must aid in
s laws of natural selection
for survival and procreation. Homosexuality defie
-sex sexu al behavi our. Despite this,
by preventing procreation due to same
trait in hum ans and animal s, resulting in
it remains a stable population level
els propose that genes
a so-catted Darwinian “paradox.” Evolutionary mod
fit on heterosexual
influencing homosexuality have a reproductive bene
hom osexuality
carriers of the gene 121. This may be the reason that
continues to persist.
In 2006, a study found that human sexual preference has a significant ge
netic component. [41. It was also seen that biological and congenital factors
regulate human sexuatity 13]. These findings were corroborated in a study
2015, in which a Large scate genome-wide scan resulted in findings that
support the existence of genes on chromosome 8 and Xq28 influencing the
development of mate sexual orientation. ti] Another study found linkage
.

between male sexual orientation and markers on the X chromosome of the


mother in some famities. The study found that the X chromosome played
a role in regutating sexuat orientation in a subgroup of homosexual males
[41. Some studies indicate females with hyperadrenocorticalism are more
likely to be homosexual as welt as bisexual than the general population.
Research in this field has been, for the most part, inconclusive. This may he
due to the tack of definitive samples, the abstract concept of attraction or
the taboo associated with nonheterosexuality. Continued research efforts
are required to enhance understanding of the genetics factors affecting
human sexuality.

Psychiatric morbidity
Research shows that non-heterosexual individuals are more likely to suffer
from poor general health [71. They are also twice as Likely to suffer from
depression, panic disorder, generatised anxiety disorder and have a higher
risk of suicide [51181. This is especially true for younger adults [6]. They also
have significantly higher rates of alcohol dependency and drug abuse Ig].
One factor in this is of course, the societal pressures and religious taboo
associated with non-heterosexuality. Due to this taboo, such individuals
may often present to a health professional with medically unexplained
symptoms, and/or severe depression and anxiety.

Sexuat Disorders
According to the International Classification of Disease, Tenth Edition, sex
ual disorders encompass three categories: sexual dysfunction (not caused
by organic disorders), disorders of sexual preference (paraphilias) and
gender identity disorders (gender dysphoria).
These are of importance due to the higher number of psychiatric morbid
ities associated with them. These individuals are more likely to suffer from
anxiety, depression and somatoform disorders. They also have higher risks
of suicide and deliberate self-harm. Often the clinical presentation may he
that of persistent headache, backache, abdominal discomfort. and gener
alised aches and pains. Low mood and other depressive features,
especially guilt and severe anxiety may he present.

SexuaL Dysfunction
Sexual dysfunction occurs when there is inability, difficulty or pain involving
sexual intercourse. This includes disorders such as those described in the
table above, The presence of any sexual dysfunction is a difficult situation for
not just the individual but also the sexual partner. In cases where the dys
function is severe enough to not allow for consummation of the marriage or
lead to reproductive problems. the entire family may become involved. This
can be a source for great discomfort for the individual. This is especially true
for males as they are expected to be ‘more informed’ in the sexual act and
all its nuances), even if they have no prior experience. Also in males, sexual
prowess or lack thereof, is closely connected to their self-esteem.
Sexual Dncti

Sexual desire disorders Sahibrdonism

Sexual arousal dIsorders Fetishism

Or5asm disorders
Paedophilia

Premature Ejaculation Sadomasochism

L Sexual pain disorders


L FetishiStic transvestitIsm

for it.. n
This makes it much harder to confront a problem and seek help
the case. Due to the taboo and empha sis attached
females, this is tess often
own anatom y.
to virginity, however, they may be poorLy acquainted with their
variati ons,
as well as poorly educated about the sexual act and its stages.
uncomfort
and nuances. Most health professionals may feel embarrassed,
ing such patient s. especia lly f
able or nervous, while talking to and manag
ant here to remind onesel f of the
they are the opposite gender. It is import nor
gender in a clinica l situatio n. One is neither a ternate
neutrality of one’s
thus, best
a mate in the white coat, simply a heater. This situaton is,
ation:
approached as one would approach an invasive physical examin
like to discus s their sexual
begin by informing the patient that you would
that they might feel uncom fortable
problem in further detail. Acknowtedge
only asking to unders tand the proble m better
but that as a doctor you are
ce based solutio ns. Encou rage the individ ual to be as
and provide eviden
ber to reitera te
frank as possible. Provide frequent reassurances, Remem
share any of
that laws of confidentiality apply and, therefore, you cannot
t their consen t.
the information they give you with anyone thou
be solved
Often mild to moderate problems of sexual dysfunction may
in which expec
with an open, frank and honest informational care session
acknow tedged and discuss ed.
tations are managed and fears and anxieties
cases of sexual dysfun ctions are
In the absence of organic causes, severe
best managed by a psychiatrist, and a referral must be made

Disorders of SexuaL Preference/ Paraphitias


and what is ac
As previously discussed, a major determinant of sexuality
ceptable is society and culture. Paraphitias refers to sexual acts or interests
, Accord ing to Diagno stic and Sta
that are considered abnormal by society
ers. 5th ed.2o;5 , these disorde rs collectively
tistical Manual of Mental Disord
persist ent sexual interes t other than sexual interest
involve “any intense and
partner s.”
in phenotypicatly normal, physically mature, consenting human
able to achiev e
Individuals suffering from these disorders are onty
sexual pleasure from performing abnormal sexual acts. This does not
inctude people who may hve abnormal sexual thoughts; only those that
behave and iridulge in abnormal sexuat acts. Some of these disorders are
considered punishable by law, according to the Pakistan Penal Code. As
such, if a health professional is confronted with a situation in which the
patient is a perpetrator of such an act, confidentiality must be breached
and the individual reported, especially if a human victim is invoLved.
There are two main types of disorders of sexual preference:
i) Disorders reLating to the object of sexuat interest.
2) Disorders relating topreferences in the sexual act.

Disorders relating to the objectbf sexual interest include paedophilia,


objectophilia and fetishism. These involve sexual interest being directed
towards children, inanimate objects, animals or non-consenting humans.
Disorders relating to the abnorml preferences in the sexual act include
sadomasochism, exhibitionism and voyeurism. Such individuals are able to
achieve sexual pleasure only from either causing or receiving painful stim
uli, exposing themselves inappropriately or observing the sexual activities
of others. I.

These disorders are best managed by a mental hea[th professional, and an


urgent referral must be made.

4 I

Gender Dysphoria (DSM V) or Gender Identity Disorder (lCD io)


According to Robert Stotter gender is socia[ and sex biologicaL: “Most
often the two are relatively congruent, that is, males tend to be manly and
females womanly.” In some individuals, however, this may not be the case.
An individual who is biologically male, may feel like “a woman trapped in
a man’s body and vice versa. Transgender individuals experience what is
referred to as “gender dysphoria” in the Diagnostic and Statistics Manual,
Fifth Edition. Individuals who are transgender may biotogically be one sex
but identify as the other, i.e there is a discord between the gender they
have been assigned and the gender they experience or feel themselves to
be. The International Classification of Disease, ioth revision refers to this as
Gender Identity Disorder. The term transgender encompasses those who -

are:
a) Transsexual: individual who prefer to have the body of the sex
opposite to their own.
b) Gender-queer: Individuals who prefer to belong to neither gender
c) Cross-dressers (Dual Transvestitism): Individuals who identify with
their biotogicat sex, yet prefer to dress in clothes of the other gender.
They do not wish to change genders.
Management of Gender and Sexuatity Issues
Some say that sexuot orientation and gender identity are sensitive issues.
I understand. Like many of my generation, I did not grow up totking about
these issues. But! teamed to speat’ out because tives are at sta1’e, and be
cause it is our duty under the United Nations Charter and the
Universat Declaration of Human Rights to protect the rights of everyone,
everywhere.’
— UN Secretary-Generat Ban Ki-moon to the Human Rights Council,
7 March 2012

In the management of atl clinical problems, a health professional is ex


pected to react with empathy. Often in issues related to gender, however,
lt
socialisation, religious beliefs and stigma may make t extremely difficu
to do so. The doctor may find themselves at a loss as to how to put them
or
selves in the shoes of an individualwho is dealing with gender dysphoria
ity. In such a situatio n, the most import ant emotio n
issues with their sexual
that a health professional can express is compassion. A health profession
al’s presence may be the only place where an individual with these issues
can expect to not to be judged. In most clinical situations related to gender
and sexuality issues, non-judgemental compassion and giving the patient
room to express their fears and problems may be the only intervention re
quired. It is also important at this juncture to remind oneself of the role as a
.
health professional, rather than a member of a particular society or culture
who has presen ted to a health profess ional is not lookin g
The individual
for a personal opinion, a religious judgement or an argument. The health I.

professional is equipped only to treat any health related issues and should
ing to
limit their interaction to this alone. It must be added here that accord
ical Ctassification of Diseas es
the ioth revision of the International Statist
and Related Health Proble ms ((CD-b ): “Sexua l orienta tion alone is not to
er. Individ uals who are nonhet erosex uat (homo sex
be regarded as a disord
uals and bisexu als) are, howev er, as mentio ned earlier , at a higher risk for
developing psychiatric morbidities. If signs of a major depressive or anxiety
disorder, or risk of suicide are present, the individual must be referred to a
psychiatrist urgently. In all patients with such issue, on account of the high
risk of suicide, it must be directly enquired if the individual is having suicid
al thoughts or has made any attempts in the past (See Section D). Again,
this merits an urgent referral to a psychiatrist.

I
References
1.Sanders AR, Martin ER, Beecham GW, Guo 5, Dawood K, Rieger G, Bad
nerJA, Gershon ES, Krishnappa RS, Kolundza AS, Duan]. Genome-wide
scan demonstrates significant linkage for male sexual orientation.
Psychotogicat medicine. 2015 May 1:45(07):1379-88.
2. Burn A, SpectorT, Rahman Q. Common genetic factors among sexual
orientation, gender nonconformity, and number of sex partners in female
twins: ImpLications for the evolution of homosexuality. The journal of sexual
medicine. 2015 Apr 1;12(4):1004-11.

3. Jannini EA, Burn A, Jern P. and Nove[ti G. Genetics of human sexual be


havior: Where we are, where we are going. Sex Med Rev 2015;3:65-77.

4, Bocktandt S, Horvath 5, Vilain E, Hamer DH. Extreme skewing of X chro


mosome inactivation in mothers of homosexual men. Human Genetics.
2006 Feb 1:118(6):691-4.

5. Prajapati AC, Parikh S, Bala DV. A study of mental heatth status of men
who have sex with men in Ahmedabad city. Indian journal of psychiatry.
2014 Apr:56(2161.

6. Bränström R, Pachankis JE. Hatzenbuehler ML. Sexual orientation differ-


ences in mentaL health morbidity: A population-based longitudinal study.
The European Journal of Public Health. 2015 Oct 1:25(suppl 3):ckVl74-062,

7. Fredriksen-Gotdsen KI, Emlet CA, Kim HJ, Muraco A, Erosheva EA, Gold-
sen J, Hoy-Ellis CP. The physical and mental health of lesbian, gay male,
and bisexuat (LGB) o[der adults: The rote of key health indicators and risk
and protective factors. The Gerontologist. 2013 Aug 1:53(4):664-75.
8. Cochran SD, Sullivan JG, Mays VM. Prevalence of mental disorders,
psychological distress, and mental health services use among lesbian, gay,
and bisexual adults in the United States. Journal of consulting and clinical
psychology. 2003 Feb;71t1):53.

9. Cochran SD, Mays VM. Relation between psychiatric syndromes and


behavioraLly defined sexual orientation in a sample of the US, population.
American Journat of Epidemiology. 2000b;151:516-523. [PMC tree artictel
tPubMedl
io. Kaplan and Saddock Synopsis of Psychiatry, Eleventh Edition, page 570.

V
Chapter 8
Psychosocial Aspects of Pain

Was it onLy a tooth ache?


Mrs H was a Pathan tady married into a Punjabi family. In the third year other
house
marrJagMrsJ±devetaped pain in her upperjaw She initially used
hotd remedies for a few days withou t any retief She then saw a dentist who
decayi ng.
advised a tooth extraction of the third tett upper molar which was
She experienced temporary reliefbut the pain persisted and started to radiate
towards the teft ear and eye. The dentist gave her short courses ofparac
d
etamot, aspirin, and mefenamic acid, but nothing worked. She then referre
her to the eye and ENTspeciatists. No abnorm alities were tound and she
came home with escriptiOtis of more pain killers. Many months passed. yet
sed
Mrs H was not relieved of her pain. She then saw a physician who discus
her pain in greater detaiL Mrs H describ ed her pain as “dull, and disgus ting.”
The physician then asked about the symptoms that were associated with her
chronic pain in thejaw. Mrs H readily expressed that she has had disturbed
steep, a change in appetite, and has become irritabte and angry The sensi
tive physician asked her about her domestic circumstances and personal life.
He found out that Mrs H had a difficult mother-in-tow and a husband who did
ew
not find Mrs H attractive anymore. Mrs H broke into tears during the intervi
and totd the doctor that she was Ted up” with life and often wished for death.
A detailed assessment revealed a diagnosis of depressive disorder Mrs H
was counselled and placed on Amitnptytine, a type of antidepressant, for six
months. The doctQr involved Mr H and the family in the care plan to mobilise
emotional and sociat support for Mrs H. Within six weeks, Mrs H’s pain was
gone and she was symptom free at the end of the six month period. She then
n
managed to form a fairly loving relationship with hr husband and a retatio
r
ship based on mutual respect and healthy communication with her mothe
in-law.

c
It is fairly well established now that pain is not a straight forward specifi
sensation that is transm itted from the periph ery to the brain by a simple
transmission system. Instead it is a ‘complex perceptual and affective
(emotional) experience determined by the unique past history of the
individuaL by the meaning of the injurious agent or situation that s/he
ascribes, by the state of mind at the moment, and the sensory nerve pat
ch
terns that evoked physical stimulation” (Gregory. 1987). Such an approa
wouLd suggest that ‘pain’ has two aspects , a sensor y and an affecti ve
(emotional) one. It is primarily the affective aspect which imparts the un
cance. In
pleasant quality to pain and is, therefore, of greater ctinicat signifi
the case of Mrs H, mentioned above, for example, the pain did not respond
to analgesics alone and a combination of antidep ressant medica tion and
psychosocial support relieved the chroni c jaw pain. The cognitive dimen
sion greatly influences the patient’s reaction to pain as welt as attempts at
analgesia, both in the short and in the long term. The psychology of pain.
.
therefore. revolves around its affective, motivational and cognitive aspects
Factors that influence the content, and the quality of psychological
correLates of pain are:
• intensity of pain,
• meaning of the pain to the individuat, (which in itself is influenced by
the location and quality of pain),
• personality traits of the individual,
• psychosocial factors at home and at work,
• past psychiatric morbidity
• cultural settings of the patient.

Psychosocial aspects of pain are different in different kinds and types of


pain:

i. Acute pain
emo
The subjective experience of acute pain is distinctly unpleasant. The
tional content of such pain can be judged by the descriptions used by the
(‘dull and disgus t
sufferer. “Wretched”, “excruciating’. “boring”, ‘sickening”
ing” in the case of Mrs H) are few of the words used in the expres sion of
pain. White the sensory words communicate a somatic sensation that can
be at least vaguely tocalised to part of the body, the emotional words (e.g.,
pounding, throbbing, terrifying, killing) do not point to a specific region.
These describe an emotion instead. Anxiety is another correlate of this
kind of pain, characterised by a feeling of impending disaster and dread.
in
• Patients with repeated episodes of acute pain often have traits of anxiety
more analge sic medica tion
their premorbid personatity. They may require
after surgery than other patients. Relief of anxiety by employing reassur
ance, instilling a sense of control and the use of anxiotytics contribute to
analgesia in an acute episode of pain.

ii. Chronic pain


Various psychoLogical abnormalities have been described in patients with
chronic pain. The exact ‘cause and effect’ relationship between chronic
pain and associated psychological morbidity remains to be established.
s
Studies have shown presence of clinical depression in over 75% of patient
with chronic lower backac he. A change in the form of consta nt preocc u
g
pation with the pain, seeking repeated medical check-ups and radiolo
ical and laboratory .investigations, often just for reassurance is comm only
in
seen amongst such patients. The chronic pain attains a central position
ar, social, intelle ctual
the sufferer’s life. S/he loses interest in extracurricul
t
and spiritual pursuits. Patients of chronic pain often seem to lose interes
in family and friends, while their depend ence on them for physic at and
emotional support increases. Loss ofjob, financial worries and substantial
hospital bills worsen the already grim picture. Poor self-esteem, a state of
boredom and unhappiness, follow as a prelude to severe depression which
of
can at times result in deliberate self-harm or even suicide, Disturbances
are often seen in patient s
sleep, changes in appetite and undue irritability
of chronic pain (as in Mrs H).
PsychotogicaL symptoms in chronic pain
Chronic pain is often associated with psychiatric morbidity. Chronic pain
has been proposed as a symptom of various psychiatric conditions, es
pecially in cultures with a poor recognition and awareness of psychotog
ica[ symptoms. People without a known cause of chronic pain have been
shown to have a higher prevalence rate of depression, as compared to
those with painful disorders such as rheumatoid arthritis. Dyssomnias
tabnorrnatities of pattern of sleep). loss of libido and sociaLwithdrawalwere
more common and more se\Jere in this group. These patients also have
a higher percentage of positive family history for mental illness and often
have a physically abusive spouse. Emotional conflicts are also common
features. The foLlowing somatic expressions of depressive features are
often seen in patients with chronic pain.
• Insomnia
• Anorexia
• Constipation
• A feeling of indigestion
- Gases
• Lethargy
• Exhaustion
• Fatigue
Presence ot such a symptort complex is associated with poor pain toler
ance drid these patierts run a higher risk of developing chronic pain.
The commonest sites for chronic pain are the precordium. genitals, face
and the cranium, Females are twice as more prone. Pain is virtualLy contin
uous white the patient is awake, yet never wakes him/her from steep. The
site of pain cisualty has a symbolic significance for the patient. It responds
k to analgesics Backache, a common site for chronic pain. may be
aggia’ated by underlying negative emotions such as unexpressed anger.
h ust ation. jealousy or chronic psychiatric disorders such as depression
.nd anxiety. The characteristic of such pain is that it is usually manifested
in the extensor groups of muscles. Fibromyalgia is another common entity
which reveals itself in the form of chronic widespread pain, fatigue and
?‘( essive pain in response to tactile pessure and is precipitated by stress

md other psychotogicat factors.


iv. Psychogenic pain disorder
ihere is a proportion of patienls whose pain and associated symptoms do
not fit any of the known psychiatric morbidities. The term used for such
patiei its is ‘Psychogenic Pain Disorder’ and it has its own diagnostic criteria.
The pain in these cases does not originate in response to a noxious stim
ulus or a pain transmission anomaly. nor is it secondary to a demonstrable
underl.,’ing psychiatric / structLiral illness. It has a mechanism similar to
hallucnation, dreams or has ioots in the psychotogicat development of
the individual. Psychodynamic mudets in the Freudian tradition and learn
iiig theories have been proposed to explain this disorder but both remain
speculative. It is also suggested that psychogenic pain could be mereLy a
hieiqhtuned response to somatin pain seen in certain predisposed individ
cials and. therefore. may not need to he considered a separate entity. It is
rolunqecl and severe and inconsistent with anatomical distribution
of the nervous system. The onset of psychogenic pain disorder is related
to environmentaL stress.

v. Pain behaviour (sick rote)


If the patient has adopted the sick rote and pain has become a behaviour
rather than a symptom. the following features start to determine the inten
sity. type and location of pain:
• Functional disabilities,
• Overuse of medications.
• Sleep disturbances
• Decreased social activities
The typical behaviour pattern in such cases is a new unhealthy lifestyle
characterised by avoidance based on the feat of pain and reinforcement
of passive learning. This involves the individual being in too much pain to
work/study/indulge in physical activity but fine while watching TV, reading
or indulging in other passive pastimes.

Psychological management of pain


The aim of psychological management of pain is to make a thorough
and methodical evaluation based on the approaches discussed
above. A treatment strategy tailored towards the specific needs of the
patient can be developed by:
• Allowing the patient to express feelings of distress.
• Providing an opportunity to describe the experience and
• Taking the patients views about the cause
• Discussing the perpetuating and the precipitating factors
All this has long term positive effects on the treatment of pain and its
outcome.

Steps of Management
These inctude:
• Psychological assessment
• Psychotropic drug treatment
• Psychological Interventions

Psychological Assessment
• Establish the primary cause of the pain i.e. is the pain organic. a
symptom of a psychiatric morbidity or primarily psychogenic in
origin.
• Define the exact role of psychological factors (psychodynamic
or cognitive) in the clinical scenario: are they etiologicaL
precipitating or perpetuating in their influence?
• Make an assessment of the various environmental factors i.e.
interpersonal, social, occupationat. economic and cutturat as
determinants of pain somatization.
Psychotropic Drug Treatment
eroidal
Other than the long Ust of narcotic / non narcotic and non-st
-

sics, certain psycho tropic drugs


anti-inflammatory (NSAID) analge
es
have also been found useful in the treatment of pain. This includ
some antidepressants, e.g tricycl ic antidep ressant s (Imipr amine)
ing the
relieve pain by blocking biogenic amine uptake, thus enhanc
pair
inhibitory action of serotonin and nor-adienatine on the spinal
transmission pathways. Phenothiazines and drugs [ike
L.
Carbamezapine. Valproic acid and Gabapentin are aLso hetpfu
of associa ted anxiety .
Benzodiazepines are primarity used for relief
to deaL
The following psychologicat interventions can be empLoyed
with chronic pain:
• Relaxation methods
• Operant Techniques
• Cognitive Strategies.
• Social skills training
• Assertiveness training
• Coping Strategies Approach
Relaxation Methods
Using muscular biofeedback (EMG Biofeedback), progressive
ion the
relaxation, cognitive relaxation and cue controlled relaxat
thereby breakin g the pain
muscular tension is brought under control
cycle.
Operant Techniques
problems
These are based on learning theory and aim to identify the
and modify an individ ual’s respon ses to the proble m.
and
Techniques include activities training, medication reduction
ues resuLt in
activities focused on functional disability. The techniq
improv ed physic al fitness and
decreased disability levels and fear, and
body image. The steps include:
• Measuring the baseline level.
• Selecting target activities,
x ones
• Starting with simple activities and moving on to comple
• Gradually increasing the activity level,
• Reinforcing the activities by providing feedback and verbal
praise.
• Checking progress and modifying the target as necessary.
• Selecting a new activity once the first activity has been
accomplished.
training
Operant techniques are also used in medication- reduction
in chronic opioid use. The drugs are dispen sed using a “time
ue.
contingent’ rather than a “pain contingert’ treatment techniq
pain retief, decrea ses pain behavi our and has a
This improves
tion is
decreased addictive potentiat. In this technique the medica
pain intensi ty. The dose
provided at regular time intervals rather than
is then systematically reduced.
*
*Von KorffM, MerrittJO, RutterCM, Sullivan M, Campbell CI, Weisner C.
Time-scheduled vs. pain-contingent opioid dosing in chronic opioid
therapy. Pain. 2011 Jun 30;152f6):1256-62.

Cognitive Strategies
Cognitive strategies aimed at pain management focus on
Dealing with stress
• Modifying pain related cognitions
This involves the stress oriented techniques like relaxation therapies
and cognitive techniques such as
a) Imaginative Inattention: thinking about something incompatible
with the pain experience e.g. relaxing in a beautiful quiet place
alongside transformations of context, that is, imagining that pain
is actually occurring but under more appropriate circumstance
b) Attention diversion methods: attention is diverted toward another
engaging task e.g. counting or reading.

iv. Sociat Skilts and Assertiveness Training


Social skills training focuses on increasing social activities at work and at
home. Assertiveness training is similar but focuses more on communica
tion skills. It enhances quality of life and reduces pain-related stress.

v. Coping Strategies Approach


This focuses on the patients’ interpretation of the pain problem and various
coping methods. Patients with chronic pain syndromes usually use passive
methods of deating with pain such as hoping pain will go away). Coping
skills training identifies methods of dealing with pain, belief about pain and
resulting disability. Relaxation and activity programmes can later be added
to the plan.
Active coping:
• Engaging in physical exercise or physical therapy
• Clearing your mind of bothersome thoughts or worries
Passive coping:
• Restricting or cancelling your social activities
Taking medication for the purposes of immediate pain relief
Chapter 9
Psychosocial Aspects of Aging

Every man desires to tive tong, but no man wants to be old.


Jonathan Swift

Medical advancements, improvements in heaLthcare and Living conditions


mean people can now expect to live Longer lives than ever before. Today,
e
the proportion of elderLy people in the world population is on the increas
in the past. The rise in
mainly because people are living longer today than
in
the proportion of older people has been accompanied by an increase
research on the intelligence and person ality of the elderly and the psycho
social aspects of old age.
).
Gerontology (the study of aging) and geriatrics. (the care of aging people
have emerged as important new medicaL fields Late adulth ood or old age
usually refers to the portion of human life cycle that begins at age 65.
Wherever the elderly may live, they are recognised by characteristic
physical changes. Aging is generally characterised by the declining ability
ity
to respond to stress, increasing imbalance and increased risk of morbid
and mortality, Aging refers to aging of cells The aging proces s or senes
cence is characterised by a gradual decline in the functioning of all the
body systems. The biological changes of aging occur at a cellular level
,
and effects the immune, musculoskeletal, reproductive, cardiovascular
nervou s system s.
gastrointestinal, respiratory. endocrine and the central
A popular theory holds that each cell has a genetically determined life
dies
span during which it can replicate a limited number of times before it
Structural changes in cells occur with age In the centraL nervou s system
for instance, neurons show signs of degeneration ri certain age related
disorders such as dementia of Alzheimer’s type, signs of degeneration are
much more severe and characterised by severe memory toss and toss of
intellectual functioning.

Neurobiotogicat changes of aging


• The weight and volume of the brain decreases by 5% between ages
of 30 and 70, by 10% by the age of So, and by 20% by the age of go
The ventricles and the sub-arachnoid spaces also increase in size
proportionately.
• Cerebral blood flow in frontal and temporal lobes decreases with
age.
Nerve cell loss occurs in the cortex, hippocampus, substantia nigra
and cerebellum.
The links between neurofilaments and microtubutes called Tau
proteins can accumulate to form neurofibriltary tangles in some nerve
cells.
• Senile plaques are found in the normal aging brain in the neocortex.
amygdala and hippocampus.
Psychologicat changes of aging
IQ peaks at 25 years of age, plateaus until 60-70, and then declines.
Performance 10 drops faster then verbal 10. This may be due to
reduced processing speed or to the fact that verbal 10 depends
largely on familiar information while performance IQ involves novel
information.
• Problem solving deteriorates due to declining abstract thinking abiLity
and increasing difficulty in applying information from one situation to
another.
• Short term memory fSTM) does not alter with age. Working
memory, however, shows a gradual decrease in capacity and this is
worse with increasing complexity of task and increased memory load.
• Long term memory tLTM) declines, except for remote events of
personal and emotional significance which may be recalled with
great clarity.
• There is a typical pattern of psychomotor slowing and impairment in
the manipulation of new information.
• Well-rehearsed skills such as verbal comprehension show little or no
decline.
• Given the physical and cognitive declines seen in aging, a surprising
finding is that emotional experience improves with age. Older aduLts
are better at regulating their emotions and experience negative
emotions less frequently than younger adults.
To keep cognitive functions and attitudes flexible, the brain must be exer
cised. The elderly brain is as capable as the young brain of growing new
connections between cells with use. Researchers have reported evidence
of increased dendritic branching in healthy, aged human brains, which
suggests the brain remains plastic even in old age. Many scientists have
discovered that cognitive functions of the brain in the elderly can be main
tained and even improved with training and environmental enrichment.

Social problems of old age


With the breakdown of traditions and family bonds in many modern soci
eties; increasing numbers of the elderly population. especially in the West,
live alone or in homes for the aged. Social problems include loss of their
social status, independence, and/or the toss of a spouse/partner. Most
of the elderly popuLation. especially in our society. have limited incomes,
and are unemployed as well as dependent. Medical problems compound
deficiency and care needs. Unlike the West where the eLderly face variable
degrees of isoLation, marginaLisation and stigmatisation, most elderly indi
viduals in Pakistan are well taken care of by their off-springs. This is primar
ily due to traditional values and the joint family system that exists in many
parts of the country. The future may see the culture of ‘old people homes’
in our part of the world as well.
The belief that old age is associated with mental and physicaL illness is a
myth. This stereotype image is a source of stigmatisation and discrimina
tion. Most elderly individuals enjoy gooqi health through most part of their
twilight years.
There are several psychologicaL and socia[ factors that have been linked to
increased individual Lite expectancy and quality of tife in older adults. The
majority of attention in the life extension and successful aging fieLd has fo
cused on physical. factors such as exercise, diet, sleep and genetics. There
is a growing body of evidence that suggests that psychological and socio
logical factors also have a significant influence on how well individuals age.
Jt is believed that adjusting to changes that accompany late adulthood and
old age require an individual to be flexible and develop new coping skills.
Aging research has demonstrated a positive correlation between religious
beliefs, social relationships, perceived health, socioeconomic status, cop
ing skills and their ability to age more successfully. The term successful
aging has been defined by three main components: low probability of dis
ease and disease related disability, high cognitive and physical functional
capacity, and active engagement with life.

Development during old age


According to Erikson, old age brings with it a core developmental conflict:
integrity versus despair (see Section C). Integrity can be achieved by inte
grating attitudes, betiefs and experiences so that they fit together comfort
ably and form a coherent whole. This results in a feeling of satisfaction with
a life well-lived. Such integrity is most likely to occur among those who
have cared for people and have adapted themsetves to different triumphs
and disappointments. Without this integrity the elderly individual is filled
with a growing sense of despair and a fear that time is running out on life.
This despair manifests in the form of persistent irritability, disgust or a nag
ging fear of death. Those who do achieve a sense of integrity and whole
ness may develop one of the hallmarks of successful aging: wisdom. This
wisdom forms the basis of seeking advice about complex life problems
from the elderly in cultures such as ours.
Chapter 10
Psychosocial Aspects of Death and Dying
Death is, perhaps, the ultimate test with that we, as patients, relatives and
members of the caring professions must face, No matter how experienced
we become, coping with death is seldom easy. Death is a loss but it can
aLso be a time of peacefut transition. t may represent failure or success,
ending or beginning, disaster or triumph. When patients are dose to death,
heatth professionals may have little or no control over what is happening.
Medication can help us to mitigate some of the pains of dying but, with alt
our knowledge. all of our patients will still die at some point. Despite this, Corn Zombi
patients and their families continue to turn to us for hetp. Death is a social
event, it affects the lives of many people. n this circle of people. the patient
is the centre of care as ton9 as they are alive: but white his/her troubles will
soon be over, thqse of the famity may just be beginning.
The traditional training of doctors and nurses does little to prepare us for
the challenges of terminal and bereavement care. We are so preoccupied
with saving life that we are at a loss to know what to do when life cannot be
saved. Some of us deal with the problem by denying its existence; we insist
on fighting for a cure until the bitter end. Sadly. the weapons that we em
ploy too often impair the quality of the life that is left such that the end,
when it comes, is truly bitter. Others acknowledge to themselves that the
patient is dying but attempt to conceal it from the patient. If they succeed,
the patient may die in blissful’ ignorance. but they often fail. As the disease
4— progresses. the patient looks in the mirror and realises that somebody is
lying. At a time when they most need to trust their medical attendants,
they realise that they have been deceived. In either case, the family who
survive are denied the opportunity to say ‘goodbye’. and to conclude any
unfinished psychological business with the patient. Of course, it is not only
the professional staff who find it hard to cope with people who are dying;
friends. workmates and family members are equally at a loss. They may
deal with their own feelings of inadequacy by putting pressure on us to
continue treatment long after it can do good or to collude with them in
concealing the true situation from the patient. ‘You won’t tell him/her that
they’re dying. will you doctor? It would kilt him/her if they found out’. While
such remarks on the part of the family are said in the spirirt of caring for the
patient. they more likely reflect the informant’s own inability to cope with
the truth rather than that of the patient.

Psychological reactions in a dying patient


Despite the fact that one hundred percent of people die, being given a
terminal prognosis is intensety fear-provoking. Consequently talking about
death is a taboo in many cultures. Reactions to the news can be seen as
an extreme form of psychological stress, which results in common coping
defence mechanisms. These include “denial” in which the fact is deliber
ately blocked from conscious awareness. “Depression and anger” towards
family, friends and health care workers are also common reactions and
demand patience, tolerance and understanding from care givers. Many
people eventually reach a stage of acceptance in which th.e wishes of the
dying person,and plans for those who will continue afterwards can be dis
cussed. All efforts should be made to make such people free of pain and
be given the liberty to make their choices and exercise their will.
Chapter 11
Psychotrauma
Psychotogical trauma or psychotrauma is the result of extraordinarily
stressful events that shatter a person’s sense of security, making him/her
feel helpless and vulnerable in a dangerous world.
Psychological trauma is a wound to the psyche due to an experience
which has endangered one’s life and threatened one’s identity, integrity,
honour and property. The experience is understood as a threat to one’s
physical and psychological well-being and is a sharp confrontation with
death or a challenge to life. A traumatic experience can leave scars on the
mind and body.
Traumatic experiences often invotve a threat to life or safety, but any situ
ation that leaves a person feeling frightened and alone can be traumatic,
even if it does not involve physical harm. Experiences such as verbal abuse.
betrayal or any major loss can be just as traumatizing as a life-threatening
event especially when they happen during childhood. Trauma can result
when an experience is so overwhelming that a person goes numb or
disconnects from what’s happening, whether the threat is psychologicat or
physical, While this automatic response may be protective for a short while.
it also prevents a person from moving on.
Not all potentially traumatic events lead to Lasting psychotrauma. Some
people rebound quickly from even the most tragic and shocking experi
ences. Others are devastated by experiences that on the surface appear
to be less upsetting. ft is not the magnitude of the catastrophe that deter
mines whether an event is traumatic. but a person’s subjective emotional
experience of the event. The more endangered, helpless and unprepared a
person is, the more likely that they will be traumatised.

Causes of psychotrauma
Emotional trauma can be caused by one-time occurrences such as a
house fire, a plane crash, a violent crime or an earthquake. Psychological
and emotional trauma can also be caused by experiences of ongoing and
relenttess stress, such as fighting in a war, living in a dangerous neighbour
hood, enduring chronic abuse or struggling with a life-threatening disease.
Although people respond differently to stressful experiences, a traumatic
event is most likely to result in negative effects if it is:
• Inflicted by humans
• Repeated and ongoing
• Unexpected or unpredictable
• Intentionally cruel
• Experienced in childhood.
People are also more likely to be traumatised as adults if they have a histo
ry of childhood trauma or if they ate already experiencing a lot of stress.

Devetopmentat trauma
Stressful experiences in childhood can be traumatizing whether it is a one
time event such as a car accident or an ongoing situation caused by a neg
ligent or abusive parent or family member. Developmentat or attachment
trauma “esults from anything that disrupts a chitds sense of safety and
security. This can include an unsafe environment, separation from a parent
or a sei Quo iLlness. Developmental trauma is most severe when it involves
betraya or harm at the hands of a cai’egiver. This trauma has a negative
impact on a childs physical, emotional and social development. Children
who have been traumatised see the world as a dangerous and frightening
place. When childhood trauma is i ot resolved, this sense of fear and help
tessness can carry over into adulthood, setting the stage for further trauma.

Normal responses to traumatic events


It is important to distinguish between normal reactions to traumatic events
and symptoms of a more serious and persistent problem Following a
traumatic event, most people experience a variety of emotions, including
shock, fear, anger and reLief to be alive. Often they think or talk of nothing
else except what they went through. Many others feei]umpy, detached or
depressed. Such reactions are neither a sign of weakness nor a positive
indicator of lasting trouble. They represent a normal response to an
abnormal event.
These feelings and symptoms typically last from a few days to a few
months gradually fading as one processes the trauma. Recovering from
trauma takes time and everyone heats at his or her own pace. However if
despite passage of months, the symptoms show no sign of letting up, a
person may be experiencing emotional or psychological trauma.
Professionat help is required if a person has:
Problems at home or work
• Living with persistent fear and anxiety
• Haunted by overwhelming memories or emotions
• Avoiding more and more things that reminds one of trauma

Posttraumatic Stress Disorder


Posttraumatic Stress Disorder tPTSD) is a condition that results from the
most severe kind of trauma. It is characterised by intrusive memories.
flashbacks or nightmares, avoiding things that remind one of the traumatic
event and living in a constant state of red alert, also known as
hyperarousal.

Common reactions to trauma


Guilt and wff-blarne

Mood swings nd lrtftablIftr

Dlstr.sslng memories

Social withdrawal

Feeling sad
Managing Psychotrauma
The process of heating emotional trauma is stow and complex. It involves
facing and resolving unbearable feetings and memories which a person
has tong avoided. The heating journey involves processing the memory of
the trauma through various techniques (see box)
Posttraumatic Growth
Although trauma is most frequently thought of in negative terms, it is also
often seen to have some positive aspects. The term posttraumatic growth
was coined in 1996 by psychologists Richard Iedeschi and Lawrence
Calhoun. Posttraumatic growth describes a posttraumatic change in how
people think of themselves, their relationships with others as well as soci
ety, and profound philosophical, spiritual, or religious changes According
this
to the proponents of this concept. trauma can lead to growth. though
is not always the case. They have found that reports of growth experie nces
in the aftermath of traumatic events far outnum ber reports of psychiatric
disorders. Posttraumatic growth can manifest in the form of improved rela
tionships and new possibilities for ones future. It can also lead to a deeper
appreciation for life, a greater sense of personal strength and spiritual de
velopment. Some tosses can produce valuable gains and individuals may
find themselves becoming more comfbrtable with intimacy and having a
greater sense of compassion for others who experience life difficulties

Techniques to manage trauma


Educate yourself about anxlaty& PSD

Reintegrate yourself Into your life:


do not Isolate or avoid family friends or actMtlesyou previously enjoyed

Know when to seek professional help Visit a mental health


family life
professional If your traumatic experience Is Interfering with your work and

I
Chapter 12
Psychosocial Aspects of Terrorism

It’s hard to fight an enemy who has outposts in your head.


—Satty Kempton

Terrorism has chattenged the physical, psychosocial, occupational. eco


nomic, and spiritual health of Pakistan for more than a decade now. It has
affected all geographic settings and sociocultural sections of the nation.

What is terrorism?
Terrorism is a violent and coercive intimidation strategy. It aims to generate
fear, panic, insecurity, hopelessness and helptessness as wetl as mistrust in
societal institutions. It is employed as a toot to challenge, destabilise, and
destroy a country, or a society in the same way as war. Often, the country’s
reaction to terrorism in the form of excessive use of force, disruptive leg
islation and extensive security measures may add to the fear and distress
of its people. Exhaustive coverage of terrorists and acts of terrorism by the
media can also add to feelings of insecurity and mistrust in the public.

Psychosocial Impact:
Terrorism hasa negative heafth impact on the individual and on society.as
a whole. It isolates individuats, families, communities, cultures, and even
countries from others, It generates strong feelings of mistrust, paranoia,
depression, anxiety, and can even have clinicaL consequences in form of
conditions like posttraumatic stress disorder (PTSD).
At an individual leveL the survivors’ reactions (see table) include changes j’
that may persist for several weeks. months and even years. These include a
preference for isotation, intolerance for noise, marked irritability and hyper
vigilance. Hypervigilance is a state of increased sensory sensitivity and an
increase in the intensity of behaviours that defuse threats. This is the state
when after hearing a gunshot, one starts to become acutely aware of every
little noise, and may hide for cover even when hearing a door banging
loudly. Survivors also experience periods of increased religiosity, followed
by alienation from reLigion, intermittently. Survivors are at an increased
risk for excessive smoking and misuse of tranquiltisers, cannabis, opiates
and atcohot. They may also devetop a tendency to undertake reck[ess
actions, particularly while driving. They also start believing in hearsay, false
attributions and negative propaganda. A higher degree of greed, mistrust,
jealousy, prejudice, need for revenge, paranoia and intolerance towards
4
minorities and certain cultures tends to prevail amongst survivors of terror
ist attacks.
h
These individuals are more prone to develop psychiatric disorders like
PTSD, Dissociative States, Depression, and MedicalLy UnexpLained Symp
toms, and the lowered immunity leaves them vulnerable to autoimmune
disorders in the years to follow. Many survivors may undergo an enhanced
resilience and even Posttraumatic Growth (PTG) after recovering from their
physical and psychological injuries.

I-
Reactions commonly seen In
Survivors of a Terrorist Attack
Why me. Why not him”

- :
I want to bomb them

‘Nothing can be done’ ‘teave us alone’

They are partners with agencies and the West

I do not want to offer prayers anymore”

I w*ft Am

ProfiLe of Suicide Bombers


Suicide bombers are known to share the following sociodemographic
parameters:
Suicide bombers are oorne out of social settings challenged by poverty,
ittiteracy. ignorance. intolers: .e. disease. insecurity and injustice They
come from large families with many children cramped in small houses with
minimal civic amenities in slums andkatchi abaadis. The absence of an
effective authority figure in the house, leaves the younger members of the
family more vulnerable to exploitative forces. The tack of adequate moni
toring and falling out of regular schooling adds to the risks of fatling prey.
High risk factors that add to this risk include:
• homes with a culture of domestic violence
• physical, emotional and sexual abuse,
• history of delinquency.
• conduct disorder,
• timited availability of emohonal outlets like music, sports, gender
interaction.
A traumatic childhood filled with humiliation, feelings of powerlessness
and lack of affection is exploited by charismatic trainers who provide group
identity, strong affinity and ‘brotherhood’ to overcome isolation from family
and community. Young people in their formative years can, thus, be easily
indoctrinated. These individuals are plied with high doses of psychotropic
and street drugs to tower their defenses. The agents of these forces often
use the name of retigion. sectarianism, tribal and political divides to moti
vate individuals. Sometimes even families and communities are manipulat
ed in order to recruit individuals for ulterior motives. Some techniques used
by trainers to ‘create’ terrorists are:
• ritual communion,
• bonding,
psychological investment,
• commitments of reaching heaven,
• stories of futfilment of explicit psychosexual fantasies of adolescent
years
• promises of houses and palaces in heaven, for them and their loved
ones
These are achieved through lectures, sermons, videos, and dramatic reen
actments.
Males between the ages of 10 to 24 are the most common ‘recruits’ of this
indoctrination. Recently, however, females have also begun to be recruited.
A ‘suicide’ bomber is driven mainly by strong feelings of anger, revenge,
and hatred. These feelings are accentuated and exploited by terrorist out
fits through generating apathy, extensive training and use of hallucinogens,
stimulants, and other drugs of abuse. With this extensive work and organ
ised exploitation, it may not be difficult to transform an
adolescent into a suicide bomber.

Characteristic behavior patterns seen amongst terrorists


planning to go on a mission/killing spree include the following:

Visible change in their earlier approach towards


religion, cultural norms, values, and the world in general.

Sodal lsolatlqfl and avoidance of routine


faml!y and societal events, to become a recluse

Reduced interest in loved ones,


family, friends and work.

b.pearrIong pedodswWH

I —
ihoäbo .öften cultural lyäIb
asiaflgoracadef?çEveát’ñ it
tógttfrdtabbwhen asked to explain their abseace,

Giving away personal valuables,


money, belongings.
S
..

ylolent video games.


.v ‘-

Obsession with, and interest in weapons, knives,


chemicals and other potentially dangerous objects etc.
Is a terrorist suffering from mentaL ilLness?
Terrorists are known to experience paranoid anxiety, envy, magical think
ing, omnipotent denial, grandiosity and massive depression. At the same
time they are often highly intelligent, sane, very focused, and have no
identifiable psychopathology. It is, therefore, inappropriate to consider
them sufferers of a clinicat disorder. The traits and behavior patterns of
terriorists are cLoser to those of hardened criminals than patients. To label
them ‘psychotic’ stigmatises patients of psychiatric disorders.
Psychosocial Management of Consequences of Terrorist Acts
Psychotrauma consequent to a terrorist act is often ignored while physi
cal trauma and disabilities draw most attention. Extensive research on the
subject shows that one third of uninjured survivors and two thirds of those
injured in a terrorist attack are likely to develop long-term disabling psy
chosocial consequences. Some key areas to be addressed include:
i. Mobilisation of Psychotrauma Teams: In the same way as health
professionals are trained to respond to injuries, trained teams of
health professionals need to be formed to address psychotrauma.
These teams need to operate as first responders. They can be
formed by training volunteers, rescue workers, ambulance staff,
including ambulance drivers, disaster relief workers, emergency
health personnel social workers, and hospital staff.
2. Integration of Psychosocial Care with Surgical Care. Itis important to
sensitise and train the surgical team members in psychotrauma
and its management. Training in the use of effective communication
skills, empathy. and awareness that fear and numbness can act as
aggravating factors in the perception of pain and reactions to injuries
can help in management.
3. Linking up Survivors and the Dead with Famities and Units/Centres.
The biggest psychosocial support in response following a terrorist
activity comes through help in establishing contact between the
survivors of the act and their families and loved ones. In the chaos
that sets in as the aftermath of such an act, the anguish of not
knowing whether your loved ones are safe leaves a lasting impact.
The longer it takes to reestablish contact. the more traumatizing it is
for the family and friends as well as the survivor. Exact, reliable and
valid information on the injured and dead is recommended from a
behavioural sciences perspective. The strategy to gradually break
bad news by initially announcing that no details are available,
followed by statements like ‘has serious injuries, but will be alright’,
‘we are trying to save them’, for those who have been martyred in the
terrorist attack is counterproductive. Breaking bad news sensitively
and clearly about those injured or martyred is a far more appropriate
approach. The news should, however, always be broken by
somebody responsible and senior in the hierarchy. This helps
the family and the loved ones overcome their initial shock,
numbness and denial reactions.
4. Updating: In case exact information about the Lost, injured or dead is
not available, it is important to not leave the affected families
unattended with crowds, media, and irresponsible people often
lurking outside such settings. Intermittent, updates at regular,
clearly identified intervals and fixed timings by a nominated
representative of the disaster management authority or the
concerned agency can help the confusion that prevails. All
information on the subject should flow out of this single source.
Others involved in the rescue operation and initial response must be
under clear instructions not to give interviews to the media. They
must also refrain from making predictions about the affected
personnel or the identity of the terrorists and the operation.
5. Reconstruction of Dead Bodies: The bodies of the martyrs in
terrorist incidents may sometimes be found in a mutilated and
deformed state. Seeing such bodies can a have a traumatizing
imt5act. Attempts at reconstruction of these bodies by professionals
and presentation of the dead bodies in customised coffin boxes may
be considered. This should be done with utmost sensitivity to the
religious, cultural, and ethnic values and norms of the aggrieved
family.
7. Religious and Cultural Rituals: Sensitivity must be shown to the burial
customs, and funeral rituals that a family or a community prefers.
A genetic approach in this regard that does not cater for indiyidual
needs may cause lasting distress in the members of the affeited
family and community.
8. Ongoing Psychosocial Support for Survivors and Families: The
exhaustive rescue operation following a terrorist attack is often
followed by a period where everything goes quiet. The initial
enthusiastic response and commitment shown must be followed
up by a sustained effort to maintain medical, psychological
surgical follow up after discharge from the hospital. A clearly defined
and well-communicated plan must be formed for this. This must
include plans for physical rehabilitation measures, including
physiotherapy. prostheses, and reconstructive and plastic surgery
needs. This is just as important as the acute response. The social,
economic, occupational, legal, and emotional needs of the injured
survivors as well as of the families of the martyrs also need to be
addressed in both the intermediate and tong term. This continuity
of care and follow-up saves the affected and the responders a great
deal of suffering and improves their resitience. This ensures an early
return to normality of the survivors as well as the affected families
and communities, It also helps build the morale of the community
and the nation in their resolve to fight terrorism.
9. Return to Normal Activity: An early restoration of life in all its
vigour and normality is the key to building resilience and health.
Affected chiLdren who do not have disabling injuries and are not
suffering from any acute medical conditions should be encouraged
to return to the pre-trauma routines of life as early as possible.
• sports.
• drawings,
• supported discussion groups
• mutual support and organised activities
cooking and eating together
Some survvors and membe
5F ffectcd famity deal better with
iltation and long term
their loss by sorkinci towards 5:e :vrhnl
They may also join
support measures of other chest cc
is All these activities help
disaster rnasacement and reter cIes:
F

le mnn qst the aggrieved and


in restoring the esem\’ and aei
help the retu:c. to normality
Ca s. The fir :spc. s terrorist act arc often
ic Care of
-

:a{o rv personnel ft is wrong


traned rescue, potrce. oaramrt:r” ad
:.uidi a imm une to the
to presume that n:embsrc nf such ed, and the dead.
h. .l.rci if iniir
psychotraures cossecisent to ties
ncis iu i’b ‘lanc e perso nnel
fact, alt first resoomfe
the hesn tsl nd th supp ort staff are
drivers health persnase1. it
‘ci’ rn develop
affected. Ti-., coo, the’ef’.ro 2t hira
the char t ter: :fl0 av show cigns of PTSD
psy’:hotr ama in
its p’wi ait tc’ educate all of them
se’.’eral’a’eeks and months itc’.

ws’e strar im_i The members


about the effects 5d Otl\’ :s :f a
.Cflt d’anqes in their
of such teams r,:ev net flfic c.,.

hece E.: -s .; they ‘catch for these


beha’. or the-nestess ts. :‘

cob :‘nn °nm e of the- earliest


sionc in each other and offm :.

acnhnn -nncentrstion, and


sicin inctudo casio ftiqaK,litn,
aT sc’’
r nntH, H hi qc in
short tess: mneoiort-. rmi- am es
:i’ce co’o..e c. .:n’’o escac,t,cs may
5c.boacai.foactonr -

br- nn before the mme. cnsons V


evcrtabitty. ftashoac.kn or h”er.. L. cL-co

es V-c sVci I;cifies and


u. Lecial 5ecurt. and Dperatrcca. ess
.rCH Vs nctn ei 3r more
ndi:duets as vcell es the commcntc--.
,ttl cc .ro,’mm snt tc’ w
secure, comforted, ansi assured
.nrl to ito :c - scia ffest”ely and in an
enforcrng agencres respc
a .oocsotcns ni effer.tr,e
orciacosed fashron. Leeatsupocrt. cc
crnn V ‘ffti’’e and rm
rehabilitation measures when nci erect them adds
ict
relentless pursuts to apprehoec a 50
a: volt as at the
to the resolve and resr’rerce as Vs n.:i .o’es rned
Owl’ cr V.iiy Ly tho
community level Any Vt-um
J of nes des
quarters on the other hen1 iss,ctr so ‘ci: secirsc mcLc
to lsr,q ‘cons
morbidities and rea:[cns. ThiS coca roasi
tt.at cm’ sVrt to surface
such as feeLings of onqe’n:e a, o rvct pc ties niured.
mat” . ar’
amongst ndrvrduaV and the iod aces
:“

qo lflLD effect an the has’s of


12. Miscellaneous measures need to
These ;ncbuci esues
available resources, expertise. and tran;ng
ntem at.onai Ii’s’s,
related to foltowng human rights charters.
rtant n the
and legislation. ihlS s particuLarly impo
• handling ot suspects
ects
• treatment given to FamiLies of susp
ation
• buriaLs of the terrorists killed in the oper
handling of media,
resisonce and rescue operations
• traininq of hum-an resource in
th. ecienr.y and effectiveness
• research and audits evabuatrag
of the process.
Chapter]3
Stress and its Management

Definition
Hans Setye introduced the concept of stress into physiology from phys
ics, where it generally refers to a force acting against some resistance.
He defined stress as “the rate of wear and tear in the body” and “the state
manifested by a specific syndrome which consists of all the non-specifical
ly induced changes within a biological system.” In this General adaptation
syndrome (GAS), glucocotticoids are secreted by the adrenal cortex in re
sponse to adaptation demands placed on the organism by such disparate
stressors as heat, cold, starvation, and other environmental insult. Any such
stimulus to the body results in certain physiological changes in the body
which are cumulatively termed as stress (Lazarus; 1984). Stressors are the
environmental sources of threat to an organism, while stress is what occurs
as a result of this. Richard Lazarus emphasised the potential threat of life
events in causing stress.
Individuals have a personal view of their stress, based on their perception
of the event, past experiences, strengths. biographical assets and social
support. Other factors which influence the outcome of stress upon an
individual include race, gender. age, marital status, socioeconomic sta
tus and early developmental experiences. For example, black people are
more prone to develop hypertension than Caucasians. Females tend to
live tonger and recover quicker from illness than males. They also show
less physiological reactivity to stress as compared to males. The elderly
are more vulnerable to all kinds of stressors. The general observation that
work capacity decreases by 1% every year after 21 years of age, provides
evidence that people.who continue to work with same routines beyond 50
years tend to have higher incidence of coronary heart disease. Marriage
is considered a protective factor against stress and most illnesses in both
genders. Educational and economic attainments provide more resilience
against stress. Early parental loss, quality of love and care received in early
life years and children’s early exposure to socializations shape the re
sponse of people to different types of stress.
Stress is conventionally divided into two types based on the causative
factors:
Physiological stress caused by temperature. noise, hunger, disease, smok
ing, drinking and similar habits are considered generalised life stressors
affecting most people.
Psychosocial stress caused by psychological factors such as low self-es
teem, social factors such as life events (see table), job stability, career satis
faction, economic viability. marriage, children, relationships etc.

Stressors bring about a physiological change in the body of a person.


Some stressors produce impacts in a short span of time; such as loss of
parents or job. Others, such as distressed relationships or care of a
disabled person in the house, inftuence the person gradually over a period
of time. Individual and situational variables mediate the relationship
between life change and illness.
LCU Value
Ranking of the event
life vent
100
Death of spouse

Divorce
65
I
3
Marital separation

all term
5 63
ber
Death of close family mem

Serious personal injury


59
7
Marriage

Dismissal from work


45
9
Marital reconciliation

Retirement 44
aviour U
Change In health and beh
o a family member
ss related disorder e.g.
is prone to devetop a stre
Interpretation: An individuat score in a given year rises above 300.
depression if his carries for the person as as
t. is the impact each event
LCU Value: life change uni ey Adapted from Holmes and Rahe scale, 1994
sessed in the 1994 surv

diate the stress


variables that seem to me
Among the psychotogicat
response are: control of
to which individuats prefer
locus of conttoL the extent ceive the y hav e over
trol they per
their lives and how much con
specific Ufe events,
• need for stimulation,
• openness to change.
stimulus screening.
self-actualization,
use of deniaL
• social support.
ts of stress and stay
echvety combat high teve
People who are able to etf se with high tev of stre
ets ss who devel
in har din ess ” from tho
healthy diff er g the “3 C’s”, in
ess. Ha rdi nes s is a per son al characteristic comprisin
op illn
se of:
which an individual has a sen
t values;
family, and other importan
• Commitment to self, work,
p.

• Control over one’s life; V


her than a threat.
• Change as a challenge rat
nce. Coherence
che rs hav e sug gested a fourth C”: cohere
Recen tly res ear s are predictable
bel ief tha t one ’s int ern at and external environment
is the ed.
as well as can be expect
and that things wilt work out
Job-reLated Stress & Burnout
Work-retated stress can be a source of persistent and intense stress and
strain. Job strain is defined as a combination of high job demands and low
perceived control. Studies have established that job strain is associated
with increased carotid atherosclerosis among men. Burnout is a syndrome
associated with untetenting stress. It includes symptoms of emotional ex
haustion and a decreased sense of personal accomplishment.

Response to stress
The body responds to stress througft.physical, emotional and behavioural
means.
Physical manifestations of stress:
Both sympathetic and parasympathetic nervous systems may go into
overdrive when faced with stress. The sympathetic system comes into
play when stress is acute and manifests in the form of increased heart rate,
dilatation of pupils, dry mouth, pitoerection, increased muscle tone, rapid
breathing and increased blood pressure. When this state is sustained for a
prolonged period of time, three target organs stomach, heart and blood
-

vessels are affected, but none are spared. There is increased acid secre
-

tion in the stomach, increased gastric emptying, nausea, heartburn, upset


stomach, chest pain, hypertension and muscle aches and pains.
Parasympathetic system is stimulated when stress becomes chronic. Some
people are more vulnerable to respond to stress through parasympathetic
system. This causes increased hunger, delayed gut motility, constipation,
increased sweating and disturbed sleep.
Both systems operate through the hypothalamic- pituitary-adrenal axis,
which is a physiological tract invoLved in stress response.
Recent evidence indicates that chronic psychological stress can lead to
increased production of proinflammatory cytokines, particularly interleukin
fIL)-6, which is also triggered by infection and trauma. Proinflammatory cy
tokines have been implicated in a range of diseases in oLder adults that can
be traced to inflammation, including cardiovascular disease, osteoporosis,
arthritis, type 2 diabetes, certain cancers (including multiple myeloma,
non-Hodgkin’s lymphoma, and chronic lymphocytic leukemia), Alzheimer’s
disease, and dental disease. IL-6 promotes the production of C-reactive
protein, which is an important risk factor for MI. The outcomes of stress on
the physical health of a person can be summarised according to its effects
on each organ system:

Cardiovascular System
Stress aggravates all types of heart disease including coronary artery
disease, congestive heart failure and sudden cardiac death. There
is a connection between hostility (common in people with type-A
personality) and cardiac disease. It has been shown that hostility
contributes, independently of other risk factors, to the pathogenesis
of heart disease through lipid accumulation, increased blood
pressure, and heart rate and platelet pathophysiology
Centrat Nervous System
rders often have stress as
Dizziness, vertigo, and other balance diso
the most common
a causative or contributing factor. One of
of stres s in clini cal practice is headache.
neuro[ogic expressions
ache are two common types of
Tension headaches and migraine head
enced by patient stressors.
headaches which are substantially influ
Gastrointestinal. System
on with stress, Hyperacidity
Peptic ulcer disease may have an associati
arks of the sympathetic
and excess enzyme production are hallm
ble Bowet Syndrome (IBS),
arousal that occurs during stress. Irrita
with altered bowel habits
which is characterised by abdominal pain
of a definable lesion or
(constipation or diarrhoea) in the absence
lates with periods of emotional
structural abnormality, frequently corre
stress.
Muscutosketetat System
exacerbations. Complex
Some forms of arthritis show stress-related
have a psychosocial trigger.
syndromes such as fibromyalgia seem to
ion and its related pain
Temporo-mandihular joint (TMJ) dysfunct
mental stress. Chronic pain
syndrome are frequently associated with
n’s life.
tends to correlate with stress in a perso
Respiratory System
y disease appear to be
Asthma and chronic obstructive pulmonar
more powerful self-
worsened by stress Conversely, one of the
ed abdominal breathing.
regulatory stress-control techniques is relax
ction programs have shown
Asthmatics who participate in stress redu
ease d med ical visits.
improved physical activity and decr
Reproductive System
sful times in the lives of
Amenorrhea frequently occurs during stres
s. Dysmenorrhea.
women Fertility also can be affected by stres
tantial stress and strain
dyspareunia and impotence all have subs
connections.
lntegumentary System
associated with the
A variety of skin tosians and rashes have been
threshold by vasodilation
stress response. Stress can tower the itch
many dermatotogic
and may account for the pruritus seen with
are all accompanied by
syndromes Emotional stress, fear, and pain
the fingers.
substantial drops in skin temperature of
Immune System
deficiencies ranging from
Stress can manifest in the form of immune P
Although research on
the common cold to some forms of cancer.
er is still inconclusive, there
the relationship between stress and canc
excision of the primary
is some evidence that stress and surgical
s by suppression of cell
tumour can promote tumour metastasi
patie nts. rapid disease progression
mediated immunity. In HIV positive
ent of one’s sexual
has been associated with greater concealm
life events, and less
preferences, more cumulative stressful
period.
cumulative social support over a 5-year
Emotional Manifestations of Stress:
People who respond with sympathetic reactions tend to show anxiety, fear,
anger and irritability. They try to avoid the stressor by showing the fright
and flight response. Hypervigilance, hyperarousat and increased startle
response are seen. Parasympathetic system responders tend to show guilt.
grief, sadness, depression, feelings of being abandoned and isolated and
tend to avoid seeking support from others.

Behavioural Manifestations of Stress


SociatwithdrawaL poor sleeping and eating habits, poor exercise, ex
cessive drinking and smoking, drug abuse, carelessness. problems with
authority and rules and regulations, insubordination, impaired social rela
tions, break up of old relationships and reckless behaviou are att types of
behaviours that indicate a person is undergoing stress ir Eneir life and is
unable to manage it,

,,Physical conditions with psychological components

Essential hypertension. angina pectorls,


tachycardia, arrhythmia, cardlospasm,
Cardiovascular coronary artery disease, mitral valve
prolapse, myocardial infarction, migraine.
headache.
4
Ji Irritable boI syndrome, gastric ulcer,
G4rolnt.stlnal Uuod.nal u1ce! Cmbn’s disease, -

- ulcratjye colitis.
k
. A
..

Hypoglycemia, diabetes mellitus,


Hormonal hyperthyroidism, hypothyroidism,
hyperparathyroidism, hypoparathyroidasm,
premenstrual syndrome, obesity
[‘W?”. ‘ ‘ ‘‘‘ ‘‘. ‘

-
-..,

• . Neurodermatitis, pruritis. psoriasis,


Integumentary hyperhidrosis, urticaria, alopecia, acne,
• herpes, genital warts

- ChmnkpaIn,tiaadach.sacrofllacpain.
I. Neuromuscular! .
temporomandibularjoint pln,
Skeletal I rheumatoid arthrftli, Peynaud’s diseesa
.‘

Respiratory Asthma, hyperventilation syndrome,


tuberculosis
Stress Management
Managing stress is vital for the health and survival of a person. Poorly
managed stress can result in disease and seriously impairs quality of life, in
addition to undermining the achievements and objectives of a person.
The goal of managing stress is to achieve a state of eustress, a state where
a person is managing stress effectively, has good health, is relaxed and is
improving his capacity and capabilities.
Self-blame, wishful thinking, avoidance and taking things personally are
recognised as negative coping methods and result in poor stress manage
ment and disturbance to health. In contrast, problem sotving. seeking help
and utilizing social support and remaining optimistic are positive coping
styles for handling stress.
Managing the stressor
Managing the environment is the first step towards stress handling. There
is always some aspect of the stressor that can be controlled. That as
pect must be controlled and one thing to be taken at a time. Minimising
the stressors may require imaginative problem solving and help from the
individual’s support group. but it helps to create a sense of control over the
environment
Managing behaviour
It is important to not get fixated on past failures, the past cannot be
changed. Focusing on what can be done now and in future is a more
productive approach. Adopting the can do attitude, practicing anger
management and remembering that you are the person in charge of your
behaviour hetps. Unless you allow yourself to fall down, nothing can make 4
you surrender are important mind sets to get one through a tough situation.
Time management
Making a list of things to do each morning and prioritise the work that
needs to be done goes a tong way towards managing time effectively.
Rather than working tong hours with low productivity, it is far more produc
tive to work for 45 minute stretches with short breaks in between. It is also
important to take time out to evatuate and reflect upon the work done and
efforts made. Learning to say ‘no’ to unreasonable demands on your time
is an important skill. This is especially true of our culture where it is consid
ered rude to say no, so instead we say yes, and avoid being truthiut about
what we can accomplish. It is also important to give ones own self “guilt
fre& time off to enjoy. Learning to delegate responsibility also greatly
improves one’s own productivity.
Managing stress
Accepting the things that ate beyond one’s controt can sometimes be the
first step towards relieving stress. It is also important to learn to deal with
your stress response and adopt effective coping methods.
Exercise and nutrition
Exercising 3-5 times a week conditions the body to deal with stress by us
ing the physical stress response. Keeping caffeine intake to minimum, stop
ping smoking and eating less fatty and sugary food and more fibre, vitamin
B complex and Vitamin C helps to prime your body to deal with stress as
and when it occurs.
Progressive Muscle Relaxation

in progressive reIaxatlon each muscle or group of muscles Is


contracted for 5 to? seconds and then relaxed for
20 to 30 seconds. The cycle is then repeated.

Practice progressive relaxation while lying down or


seated in chair with feet firmly on the floor.

$gl*f

Do the same with the left fist- tensing relaxing,


and noticing the difference.

I Progress through the next major muscle group head, face, throat and shoulders
-
I
Mcw.óthethIrdm*rmuIdemup-ch.st, abdaeean and lowr back.

End with the fourth major muscle group thIghs, buttocks, calves and feet

H
-

areupsthatam
*outd.perr&axatlon -

Relaxation
Practicing progressive muscle relaxation (see box), meditation, deep
breathing exercises and engaging in hobbies provides energyand fuel for
the body to combat stresses of all kinds.

Social support
Developing a circle of friends and family where one can talk openly and
honestly can help one better understand stress and learn to manage it
more effectively. It is also important to utilise resources that can help such
as doctors or senior colleagues.

Communicating With Patients about Stress


p A primary component of stress prevention and treatment involves the way
stress is discussed with patients. A doctor must avoid phrases such as “it’s
all in your head. Equally important is avoiding the message that ‘you are
4 responsible for your illness’ This induces guilt in patients who become ill
and makes them feet that they have failed at being a “better person Avoid
ing negative emphasis, for example, ‘Your hectic pace is going to kitl you,
in favour of a positive framing of the health benefits of stress management
is more likely to have a positive influence. For example, the clinician can
express optimism about the patient’s ability to influence health as follows:
“Fortunately, there are many avenues available for evaluating and changing
your way of responding to life challenges. I’d like to give you an overview of
some strategies that we can discuss now and in future appointments.”
Non-pharmacologic Interventions for reducing Stress

Improve time management

Have a sense of humoAr

Pursue personal and vocational activities

I Engage in spiritual activities such as praying

Clarify valugs
I
Cultivate social support network
.

ro*a assertiveness•

Reduce exposure to unnecessary stressors


.

Haip othati In anyway you cm .


SAMPLE MCQ FOR SECTION E

1. A 35 year old gentleman has been admitted to the hospitat for the Last
10 days. His investigations reveal that his condition is improving, yet
he appears stressed. What could the most important reason for this be?

a) The bad attitude of the nursing staff


b) The cost of the investigations
C) Loss of autonomy and privacy
d) Feeling betrayed at not being visited in the hospitat by the extended
family
e) Being dependent on other people

2. A 23 year oLd man is diagnosed with metastatic pancreatic carcinoma.


The prognosis is poor and the treating doctor gives him the bad news
that he onLy has a few weeks to Live. The first psychoLogicaL reaction
in such a patient wiLt be:

a) Anger.
b) Bargaining
C) Depression.
d) Acceptance.
e) Denial.

4
3. A 75 year otd gentLeman with chronic renaL faiture, suddenty starts to
t - become irritabLe, refuses to recognise his famiLy members, and
appears to be seeing things that aren’t there. His daughter reports
that he does not steep at night and steeps throughout the day instead.
The most Likety diagnosis is:

a) Dementia
b) Delirium
c) Schizophrenia
d) Depression with psychotic features
e) Old age

4. In the case of the 75 yr oLd gentLeman mentioned above, after he is


admitted to the hospitat, which of the fottowing are tikely to make his
condition worse?

a) Unfamiliar medicaL procedures conducted without provision of


informational care.
b) Psychological stress of disease and hospitalization.
c) Lack of loving family members around the patient.
d) Long periods of sensory deprivation as well as sensory overload.
e) Antidepressants
.
Defense mechanisms have all of the following characteristics except

a) They emerge in a deve[opmentat sequence from Less mature to


more mature.
b) They generally can be brought under conscious control to ward off
anxiety.
c) They operate to maintain a sense of welt being and safety.
d) They may be episodic or become more habitual and pervasive.
e) They may contribute towards formation of personality traits

Sampte Short Essay Question For Section E

01. Briefty describe the psychotogicat stages of Grief Reaction


hightighting various defence mechanisms.
Q2. How does one differentiate between post-partum bLues and
post-partum depression?

Answers

i.b
2. e
3. b
4. d
5b
II
APPENDIX
- - - -
U

The Mini-Mntãj _i1MSE)


t - -
L: -

Patient Examiner Date / /

Score Maximum
Orientation
What is the (year) (season) (date) (day) (month)? ( ) 5
( ) 5
Where are we (state) (country) (hospital) (floor)?

Registration
Name 3 objects: I second to say each. Then ask the patient all 3 after
3
you have said them. Give one point for each correct answer. Then repeat
them until he/she learns all 3. Count trials and record.
Trials__________________

6 Attention and Calculation


Serial 7’s 1 point for each correct answer. Stop after 5 answer. 5
Althernatively spell world” backward.

Recall
3
Ask for the 3 objects repeatd above. Give I point for each correct answer.

Language
( ) 2
Name a pencil and watch.
Repeate the following “No ifs, ands: or buts”
(I 3
Follow a 3-stage command:
“Take a paper in your hand, fold it in half, and put it on the floor.”
Read and obey the following: CLOSE YOUR EYES
Write a sentence. t I
Copy the design shown.

Total Score
ASSESS level of consciousness along a continum
Alert Drowsy Stupor Coma

24-30: within normal limits: 23: cognitive impairment


(further formal testing recommended)
\ \ :;j1: 7

(çii) (-.:‘) (-;)) (-3L-) L

.(/3)/) (3) (fl/) (Aa) (re)

(;)a2

(,-L-’i,3

L World “J,L
5

-4
3
;- 5
-JiL5’
2 . rVLI—iLL-i

(‘1?L5LI)

LI( . J
4i
1 (j,y)

c÷_
( f),3 j

‘1

You might also like